warrantless arrests, searches and seizures and privacy of communication

503
1 [GR No. L-824, January 14, 1948] ROAD HILARIO MONCADO, RECURRENT COURT AGAINST THE PEOPLE AND JOHN M. LADAW, AS ATTORNEY SPECIAL, APPEALED. DECISION PAUL, J.: In an original application for certiorari . the appellant, accused of treason in the criminal case No. 3522 of the People's Court, alleges that in April 4, 1945 at about 6 pm, he was arrested by members of the Army CIC U.S. at his home in San Rafael Street, No. 199-A, Manila, without a warrant and was taken to prisons in Muntinglupa, Rizal, a week after his wife, who had moved to his residence in the house Rosario Street, No. 3, Quezon City, was invited by several members of the ICC under the command of Lieutenant Olves to witness the search of his house on Calle San Rafael, who refused to follow them because they had a search warrant, but as assured that even without his presence had to do anyway check, she accompanied them, that on arrival at the house, he saw that several effects were strewn on the ground including several documents, that Lieutenant Olves informed her that was carrying some documents to prove the guilt of her husband, that the June 27, 1946 the appellant filed a motion with the People's Court asking the return of such documents alleging as a reason that has been obtained from his residence without a search warrant, and that court, with grave abuse of discretion or excess of jurisdiction and following the doctrine established in the case of Alveroagainst Dizon (76 Phil., 637) rejected it, that unless this Court order the special prosecutor that appellant returned to , his constitutional rights guaranteed by the Constitution would be violated. And because it has no other choice simple, rapid and appropriate in the ordinary course of law, requests that this Court ( a ) set aside the order of the People's Court of 9 July 1946, ( b ) that the Court is required to order the return of such documents to the plaintiff, ( c ) the issuance of a restraining order prohibiting the Special Prosecutor to present them as evidence against the appellant in the case of treason. These requests show that the documents are relevant evidence, plus admissible because there is no rule that prevents (Model Code of Evidence, 87). This well established appellant's contention that the decision in the case of Alvero against Dizon 76 Phil., 637) is not applicable to the particular case. Documents in the case of Alvero have been seized by members of the CIC when the military government exercised in full swing its occupation army functions. Instead, when they took on April 11, 1945, documents that are the

Upload: pen-malubag

Post on 07-Feb-2016

185 views

Category:

Documents


1 download

DESCRIPTION

constitutional law cases

TRANSCRIPT

Page 1: Warrantless Arrests, Searches and Seizures and Privacy of Communication

1

[GR No. L-824, January 14, 1948]ROAD HILARIO MONCADO, RECURRENT COURT AGAINST THE PEOPLE AND JOHN M. LADAW,

AS ATTORNEY SPECIAL, APPEALED. DECISION

PAUL, J.:In an original application for certiorari . the appellant, accused of treason in the criminal case No. 3522 of

the People's Court, alleges that in April 4, 1945 at about 6 pm, he was arrested by members of the Army

CIC U.S. at his home in San Rafael Street, No. 199-A, Manila, without a warrant and was taken to prisons

in Muntinglupa, Rizal, a week after his wife, who had moved to his residence in the house Rosario Street,

No. 3, Quezon City, was invited by several members of the ICC under the command of Lieutenant Olves

to witness the search of his house on Calle San Rafael, who refused to follow them because they had a

search warrant, but as assured that even without his presence had to do anyway check, she

accompanied them, that on arrival at the house, he saw that several effects were strewn on the ground

including several documents, that Lieutenant Olves informed her that was carrying some documents to

prove the guilt of her husband, that the June 27, 1946 the appellant filed a motion with the People's Court

asking the return of such documents alleging as a reason that has been obtained from his residence

without a search warrant, and that court, with grave abuse of discretion or excess of jurisdiction and

following the doctrine established in the case of Alveroagainst Dizon (76 Phil., 637) rejected it, that unless

this Court order the special prosecutor that appellant returned to , his constitutional rights guaranteed by

the Constitution would be violated. And because it has no other choice simple, rapid and appropriate in

the ordinary course of law, requests that this Court ( a ) set aside the order of the People's Court of 9 July

1946, ( b ) that the Court is required to order the return of such documents to the plaintiff, (  c ) the

issuance of a restraining order prohibiting the Special Prosecutor to present them as evidence against the

appellant in the case of treason. These requests show that the documents are relevant evidence, plus

admissible because there is no rule that prevents (Model Code of Evidence, 87).

This well established appellant's contention that the decision in the case of Alvero against Dizon 76 Phil.,

637) is not applicable to the particular case. Documents in the case of Alvero have been seized by

members of the CIC when the military government exercised in full swing its occupation army

functions. Instead, when they took on April 11, 1945, documents that are the subject of this case, General

MacArthur on behalf of the Government of the United States, and had re-established in February 27 of

that year, the Commonwealth with all its powers and prerogatives (41 Off. Gaz. 86).  The Commonwealth

government was already exerting all his powers and legal eonstitucionales without Iimitación some in the

City of Manila. President Ho talks suspended constitutional guarantees.

Doctrine is well established in the Philippines, United States, England and Canada that the admissibility of

evidence is not affected by the illegality of the means of the party has used to obtain them. [1] is followed

by many years doctrine "until arose - this Court said in Pueblo against Carlos, 47 Phil., 660 - the fatal

majority opinion in the case of Boyd vs . U.S. in 1885, which has had pernicious influence in many states

on subsequent judicial opinions. "

Page 2: Warrantless Arrests, Searches and Seizures and Privacy of Communication

2

"The development of this doctrine of the case of Boyd vs . U.S. was as follows. ( a) The case of Boyd

continued without putting into question in the same court for twenty years, and meanwhile received

frequent disapproval of state courts ( atparagraph 2183). ( b ) Then in case of Adams vs. . New York, in

1904, was implicitly rejected . in the Federal Supreme Court, and the registered Orthodox precedents in

the courts of the State ( at paragraph 2I83) were expressly approved. ( c ) Then, after another twenty

years, in 1914, in the cause of Weeksvs . U.S., the Federal Supreme Court moved at this time not by

history wrong, but by a misplaced sentimentality - back to the original doctrine the cause of Boyd, but

with a condition . namely that the illegality of the search and seizure should first have been directly

litigated and established by a motion made before trial, for the return of things seized, so that, after the

motion and only then, could illegality noted in the main trial and the evidence so obtained should be

excluded. ***. "Under the authority of the Weeks doctrine vs . U.S., and other decisions of the same

school the appellant exercised this action, requesting the return of the documents illegally removed by

ICC members.

We concur with the appellant's complaint that under these constitutional guarantees, had the right to be

respected home, your documents should not be seized by any authority or agent of authority, without a

search warrant duly issued.

These constitutional limitations, however, not go so far as to exclude relevant evidence documents

obtained illegally or improperly E1. The Court Rules, Rule 123, determines what evidence should be

excluded, which is not admissible and competent and incompetent classified as the illegally obtained

evidence. The fundamental law sets the limits far can the executive, legislative and judiciary in the

performance of their duties. The executive should not abuse their power, violating the citizen's home or

improperly seizing their goods and documents, the legislature should not pass laws which frustrate the

sacred home and courts should punish violators of the Constitution, without of whether they are public

servants or not.As President Lumpkin Williams vs . State, 28 SE, 624:

"As we understand it, the main, if not the sole, purpose of our constitutional inhibitions against

unreasonable searches and seizures, was to place a restriction upon the salutary powers of

government.That is to say, we believe the framers of the Constitutions of the United States and of this

and other states Merely Sought to Provide against any attempt, by legislation or Otherwise, to authorize,

justify, or declare lawful, any unreasonable search or seizure. wise This was Intended to Operate

restriction upon legislative bodies, so as to render ineffectual any effort to legalize by statute what the

people Expressly stipulated in no event Could be made lawful; upon executives, so That law not violative

of this constitutional inhibition Should ever be enforced, and upon the judiciary, so as to render it the duty

of the courts to denounce as unlawful search and seizure every unreasonable, without any color Whether

confessedly of authority, or Sought to be Justified under the guise of legislative sanction. For the

misconduct of private persons, acting upon Their Individual Responsibility and of Their Own volition,

surely none of the three divisions of government is responsible. If an official, or a mere agent of the state

petty, exceeds or abuses the authority with Which he is clothed, he is to be Deemed as acting, not for the

Page 3: Warrantless Arrests, Searches and Seizures and Privacy of Communication

3

state, but for himself only, and THEREFORE I alone, and not the state, held accountable Should be for

his acts. If the constitutional rights of a citizen are invaded by a mere individual, The Most That any

branch of government can do is to AFFORD the Such citizen redress as is possible, and bring the

wrongdoer to account for his unlawful conduct. ***. "

We believe that the framers Filipino nan never had the slightest idea of granting criminal immunity to

violating the sanctity of the home, or any lawbreaker criminal for the mere fact that the evidence against

him have been obtained illegally. The procedure healthy, fair and orderly is to be punished according to

Article 128 of the Revised Penal Code to the individual who, under the guise of public officer without a

search warrant, unduly profane a ciudano home and seized his papers and also to punish that citizen is

guilty of a crime, regardless of whether the evidence of his guilt was obtained illegally.  The medium used

in the acquisition of the document does not alter its probative value. Thus in Stevison vs. Earnest, 80,

111. 5L3, said, "It is Contemplated, and such ought ever to be the fact, That the records of Courts

REMAIN permanently in the places Assigned by the law for Their custody. It does not logically follow,

However, That the records, being Obtained, can not be used as instruments of evidence, for the mere fact

of (illegally) does not change them Obtaining That Which is written in them ***. Suppose the Presence of

a witness to Have Been procured by fraud or violence, while the THUS party procuring the attendance of

the witness would be liable to severe punishment, surely That Could not be urged us against the

competency of the witness. If I Could not, why Marshall to record, although illegally taken from its proper

place of custody and Brought before the Court, but otherwise free from suspicion, be held incompetent? "

In Com. vs. . Dana, 2 METC., 329, the Court said: "Admitting That the lottery tickets and matetials Were

Seized illegally, this is still no legal objection to the admission in evidence of them. Were If the search

warrant illegal, or if the officer serving the warrant exceeded his authority, the party on the warrant issued

Whose complaint, or the officer, would be responsible for the wrong done. But this is no good reason for

excluding the papers Seized, as evidence, if They Were pertinent to the issue, As They unquestionably

were. Offered When papers are in evidence the Court can not take notice how They Were Obtained, -

lawfully or unlawfully Whether, - nor would form a collateral issue They determined to question That. "

The appellant cites the case of Burdeau vs . McDowell as follows:

"Certain books, papers, memoranda, etc.., Private property McDowell were stolen by certain people who

were interested in research that would practice the Grand Jury against McDowell for some offense which

said it had made on the use fraudulent email. Such documents and books were delivered to Burdeau after

by the people who had been robbed. Burdeau was the special assistant to the Attorney-General of the

United States, which would have the direction and control of the prosecution before the Grand Jury .

McDowell Burdeau tried to prevent such books and documents used by a motion was filed in this regard.

Burdeau opposed the motion, arguing that he was entitled to use these papers. The Supreme Court of the

United States upheld the contention Burdeau , saying;

"'We knew of no constitutional principle Which Requires the government to surrender the papers under

Such circumstances.

Page 4: Warrantless Arrests, Searches and Seizures and Privacy of Communication

4

"'The papers having eaten into possession of the government without a violation of petitioner's rights by

Governmental authority, we see no reason why the fact That Individuals unconnected With The

Government May have wrongfully taken them, Should Prevent them from being held for use in

prosecuting an offense where the documents are of incriminatory character. ' (Burdeau vs . McDowell.)

"Adopt our Supreme Court doctrine is announced in this decision? Submit that this is a bad rule of law,

and our humble opinion, should not adopt our court."

The recurring appointment after some decisions of state supreme courts nan not adopted this doctrine of

Federal Supreme Court. No wonder. Each court adopts its own discretion. But of the 45 States of the

American Union - by Judge Cardozo in his decision rendered in 1926, in People vs. . Defore, 150 NE, 585

- fourteen adopted heterodox doctrine Weeks and 31 rejected, and as Wigmore, in 1940, fourteen years

later, six more states, 37 in all, including Hawaii and Puerto Rico rejected, maintaining orthodox

doctrine . (8 Wigmore on Evidence, 3. Ed, p. 5-11) and after considering the various decisions of the two

schools, Cardozo made these pertinent observations on the doctrine of Weeks:

"We are confirmed CONCLUSION In This When We Reflect how far-reaching in its effect upon the new

society would be Consequences. The pettiest peace officer would Have It in his power, through overzeal

or indiscretion, to confer immunity upon an offender for crimes the MOST flagitious. A room is searched

against the law, and the body of a murdered man is found. If the place of discovery may not be PROVED,

The Other Circumstances may be Insufficient to connect the defendant With The crime. The privacy of the

home has been infringed, and the murderer goes free. Another search, Once More Against the Law,

discloses counterfeit money or the implements of forgery. The Absence of a warrant means the freedom

of the forger. Multiplied Like instances can be. "

Concretémonos to this case. If documents whose return calls the appellant prove his guilt of the crime of

treason, why the state has to return and save him from the accusation? Is not this consent and validate

the crime? Do not constitute judicial approval of the commission of two crimes, the violation of the

defendant's domicile committed by members of the CIC and the treason committed by the

appellant? Such a practice would encourage crime rather than prevent its commission. Moreover,

obtaining documents not alter its probative value. If he had mediated a search warrant, the documents

would be admissible evidence. There is no constitutional, legal or release the accused from criminal

liability because there was no warrant. Vigilante justice requires that criminal offenders are punished. The

release of the blame for the simple fact that the evidence against E1 has not been legally obtenlda is

judicially punish crime.

Consider a case: John who witnesses a murder, the murderer gets snatch the knife, and with which

orders him arrested and not give you to the presidency of the people. Along the way he meets Peter who

intercedes for the murderer, John, for sentimentality misunderstood returns the dagger and helps the

accused to remove all traces of the crime to avoid being caught. John and Peter, not only commit acts

unworthy of good citizenship, but must be punished for abetting (Art. 19, Cod. Pen. Rev.). The public will

never understand why these two individuals should be punished and instead, a court, under the doctrine

Page 5: Warrantless Arrests, Searches and Seizures and Privacy of Communication

5

of Weeks, may order the return of the stolen document proving the guilt of the accused and to free this

and that stole the document.

Another case. For his suspicious catadura, one Jose is arrested by two police officers to go to the rostrum

where are gathered senior officials of the executive, legislative and judicial Along with diplomatic

representatives of friendly nations to witness stop independence anniversary and in his pocket found a

pump that is capable of flying across the platform. Two other policemen, after learning of the arrest,

confiscate the house of Joseph and found documents showing that he has received orders from a foreign

organization for all of the top polverizar government personel at the first opportunity.  The police have no

warrant, no search warrant. Is it fair that a motion in the criminal José followed against him, is ordered by

the court the return of the documents that prove his crime? Did not give such encouragement to

anarchism in practice? The court would play the sad role of helping those who wish to undermine the

foundations of our institutions. In U.S. vs . Snyder, 278 Fed, 650, the Court said: "To hold that no criminal

can, in any case, be arrested and searched for the evidence and tokens of his crime without a warrant,

would be to leave society, to a large Extent , at the mercy of the shrewdest, The Most expert, and the

MOST depraved of criminals, Facilitating Their escape in many instances. " And in People vs . Mayen,

205 Pac., 435 stated: "Upon what theory can it be held That Such proceeding (for the return of the

articles) is an incident of the trial, in Such a Sense That the ruling thereon goes up on appeal as part of

the record and subject to review by the appellate court? Rather It seems to us an independent civil

proceeding to enforce a right in no way Involved in the criminal case. The right of the defendant is not to

exclude the incriminating documents from evidence, but to recover the possession of articles Which Were

wrongfully taken from him. Entirely That right exists apart from any Proposed use of the property by the

State or its agents. *** The fallacy of the doctrine contended for by appellant is in Assuming That the

constitutional rights of the defendant are Violated by using his private papers as evidence against him,

Whereas it was the invasion of his premises and the taking of his goods That constituted the offense

irrespective of what was taken or what use was made of it, and the law having Declared That the articles

taken are Competent and admissible evidence, notwithstanding the unlawful search and seizure, how can

the circumstance That the court erred in an independent proceeding for the return of the property on

defendant's demand add anything to or detract from the violation of the defendant's constitutional rights in

the unlawful search and seizure?

"The Constitution and the laws of the land are not solicitous to aid persons charged with crime in Their

Efforts to conceal or sequester evidence of Their iniquity." (8 Wig., 37)

Weeks theory vs . U.S. that subverts the rules of evidence is not acceptable in this jurisdiction, is contrary

to the sense of justice and the orderly and sound administration of justice. The orthodox doctrine imposed

by consistency proven through many years. Do not leave if you want that constitutional rights are

respected and not desecrated. The culprits should get their condign punishment, but the evidence against

them have been obtained illegally. [2] and those in breach of the law and the Constitution 

Page 6: Warrantless Arrests, Searches and Seizures and Privacy of Communication

6

improperly seize such evidence should also be punished. This is how the law prevails, majestic and

unscathed.

Rejecting the application with costs.

Moran, CJ, Fair, and Padilla MM. , concur. Tuazon J. , concur in the result. 

[1] See the decisions of England, Canada, the States of Alabama, Arkansas, California, Connecticut,

Georgia, Idaho, Illinois, Iowa, Kansas, Kentucky, Louisiana, Maine, Maryland, Massachusetts, Michigan,

Minnesota, Mississippi, Missouri, Montana, Nebraska, New Hampshire, New Jersey, New York, North

Carolina, Oklahoma, Oregon, South Carolina, South Dakota, Tennessee, Texas, Utah, Vermont,

Washington, West Virginia, Wyoming, Hawaii and Puerto Rico cited by the author in 8 Wigmore on

Evidence, 3. Ed, pages. 5-11. The Constitution guarantees the inviolability of individual rights in the

following terms "seviolará not the right people to be secure in their persons, dwellings, papers and effects

against unreasonable searches and kidnappings, nor be issued search warrants or arrest, unless

probable cause to be determined by the judge after examination under oath or affirmation, the

complainant and the witnesses that arises, and detailed description of the site that is to be recorded and

the persons or nan to grasp things that have to be seized. "  (Title III, Article 1., Paragraph 3.

º.) [2] Barton againstLeyte Asphalt & Mineral Oil Co., 46 Phil., 973.

CONCURRING

YARN, J. :I concur, but I would support the CONCLUSION Further arrived at by the Following additional

considerations:

In April, 1945, When the CIC Detachment of the United States Army made the search at petitioner's

house and effected the seizure of his papers and effects Mentioned in the Majority decision, as is of

general knowledge and Within the legal notice of this Court, fighting continued in Luzon, in fact, as late as

June, 1945, the cannonades and shellings Could Still Be Heard Clearly in this City of Manila, and There

Were Still units of the Japanese Army resisting the liberation forces. Under Such Circumstances,

Continuing the war was not only technically but actually in the island of Luzon, and the military security

and safety of the liberation forces Demanded Measures Such as Were ADOPTED by the CIC

Detachment of the United States Army in making said search and effecting said seizure to the end That

Page 7: Warrantless Arrests, Searches and Seizures and Privacy of Communication

7

the Activities of pro-Japanese elements and Their chances of Effectively aiding the Japanese forces still

continued THUS Which Might Be Brought to resist down to a minimum and, if possible, entirely foiled. The

Difference between this case and the case in L-342, Alvero vs . Dizon, 43 Off. Gaz., 429), is, to my mind,

one of degree-Merely the principle Involved in Both cases is identical.

Dissenting

PERFECT, J. :Accused of treason Petitioner stands before the People's Court, the information having been filed against

him by Prosecutor John M. Ladaw on February 28, 1946.

Almost a year before, on April 4, 1945, at about 6, 00 pm, petitioner was arrested by members of the

Counter Intelligence Corps of the United States army at his residence at 199-A San Rafael St., Manila,

without any warrant of arrest, and taken to the Bilibid Prison at Muntinglupa, Where He Was Detained.

On April 11, 1945, petitioner's wife, who Transferred to Their house at 3 Rosario Drive, Quezon City, was

Approached by several CIC officers, headed by Lt. Olves, and ordered to Accompany them to the house

at San Rafael to witness the taking of documents and things Belonging to petitioner.  Upon hearing from

the officers did not That They Have Any search warrant for the purpose, she refused to go with them, but

after the officers Told Her That With or Without Her Presence They would search the house at San

Rafael, Mrs. Moncado Decided to Accompany them.

Upon arrival at the house, Mrs. Moncado noticed That Their belongings had been ransacked by American

officers and That Which the trunks she had kept in the attic and in the garage When she left the house,

had been ripped open and Their contents scattered on the floor. Lt. Olves Informed That They Were Mrs.

Moncado going to take a bundle of documents and things, Which Were separated from the rest of the

scattered things, Because They PROVED the guilt of her husband. Mrs. Moncado protested in vain. No

receipt was issued to her. Subsequently, after making an inventory of Their belongings at San Rafael,

Mrs. Moncado found the Following things missing:

" (a) Passes issued by Japanese friends for the personal safety of the petitioner and couduct;

"(B) Correspondences of the petitioner as president of the Neighborhood Association in Quezon City

During the Japanese occupation;

"(C) Correspondence of the petitioner with Japanese Certain Officers;

"(D) The personnel file and the love letters of Mrs. Moncado to Dr. Moncado and vice versa;

"(E) Marriage certificate of Dr. Moncado with Mrs. Moncado issued at Reno, Nevada;

"(F) Private correspondence and letters of Dr. Moncado to and from his Filipino Federation of America in

Hawaii and United States;

"(G) Several law books by Guevara, Albert, Francis, Harvard Classics (complete set), books on

diplomacy, international law;

Page 8: Warrantless Arrests, Searches and Seizures and Privacy of Communication

8

"(H) A complete collection of the 'Tribune' During compilation of the same occupation Until the last day of

its Issuance;

"(I) Complete collection of American magazines, from 1940 to 1941 - the Los Angeles Examiner, San

Francisco Chronicle, Los Angeles Evening Herald and newspapers edited and owned by Dr. Moncado

and published in the United States, and National Geographic Society;

"(J) Personal letters of Dr. Moncado with several members of the United States Senate and Congress of

the United States Including a picture of President Hoover dedicated to Dr. Moncado;

"(K) Pictures with personal dedication and autograph to Dr. and Mrs. Moncado by actors and actresses

from Hollywood, Including Mary Astor, Binnie Barnes, Robert Montgomery, Clark Gable, Gary Cooper,

Boris Karloff, Wallace Beery, William and Dick Powell , Myrna Loy, Bette Davis and Ceasar Romero;

"(L) Certificate as first Flighter in the Pan-American Airways and even several stickers Issued by Pan

American Airways for Passengers' baggage;

"(M) A promissory note of Dr. Moncado for Fifty Thousand Pesos (P50, 000) in behalf of Architect Mr.

Igmidio A. Marquez of Quezon City;

"(N) Three (3) volumes of modern ballroom dancing by Arthur MacMurray of New York, pamphlets of

dancing Obtained by Dr. Moncado while I was studying dancing at Waldorf-Astoria, New York;

"(O) Two (2) volumes of rhumba, samba and tango Obtained from Mexico and Argentina by Dr. Moncado.

" (Pages 3 and 4, Petition for Certiorari and Injunction.)

On June 27, 1946, petitioner filed With The People's Court a motion praying That the return of said

documents and things be ordered. The petition was denied on July 9, 1946.

Thereupon, petitioner filed With This Supreme Court on August 10, 1946, a petition praying That the

lower court's order of July 9, 1946, be set-aside, That said court be required to order the return of the

documents and things in question to petitioner , and That the prosecutor be restrained from using and

presenting them as evidence at the trial of the criminal case for treason.

Before proceeding to Consider the questions of law raised In This Case, We Should not ignore three

questions of fact raised in the answers of respondeds: as to the identity of the documents and things, as

to Whether They Were taken from the house at San Rafael or from the house at Rosario Heights, and as

to Whether They Were taken at the time of petitioner's arrest or later.

The Fact That the return of the documents and Things Were Opposed to in the lower court by the

prosecutor, without disputing Their identity, and That in the present proceeding the prosecutor admits To

Have Them In his possession, without disputing or correcting any identity Their mistake of description

made by petitioner, That Convinced us in petitioner's and respondent's minds there is no d isagreement

on the identity in question. There should not be any doubt That the papers and things Described and

CLAIMED by petitioner are the ones in the prosecutor's possession, otherwise, instead of objecting to the

return on legal grounds, I Would Have Alleged That Such things are not in his possession, or I does not

know where They are, or That They did not exist at the all.Whether Things Were taken at San Rafael or at

Page 9: Warrantless Arrests, Searches and Seizures and Privacy of Communication

9

Rosario Heights compeltely is immaterial. The fact Is that the reality and existence of things and

petitioner's ownership thereof, are undisputed, and That They Were taken from a house of petitioner.

That They Were taken not at the time of petitioner's arrest but much later, is indisputably PROVED by

petitioner's and his wife's depositions not contradicted by any other evidence.

This case offers a Conclusive Evidence That key ideas, rules and principles are in constant need of

restatement If They Are Not to Lose Their vitality. So That They may continue radiating the sparks of

Their truth and virtue, They Need the Repeated pounding of intense discussion, as the metal hammered

on the anvil. To make them glow With All Their force, purity and splendor, They need the continuous

smelting analysis and synthesis as the molten iron in a Bessemer furnace. Otherwise, They Become

rusty, decayed or relegated as useless scraps in the dumping ground of oblivion. What is worse, They are

Frequently Replaced by Their antitheses, Which With The deceitful pose dazzle of false gods, clothed in

tinsel and cellophane. The risk, always lurking at every turn of human life, exacts continuous

vigilance. Human minds must always be kept well tempered and sharpened as damask swords, ready to

decapitate the hydra of error and overthrow the idols from the muddy gilded pedestals of pretense and

imposture.

The government may profit from an illegality, an unconstitutional act, or even a crime to serve its AIMS,

Including the loftiest? May justice be Administered by making use of the fruits of a lawless action? If a

private individual, when to Profiting from the fruits of a criminal offeHse, is punished by law as an

accessory after the fact, Why Should the government or an official system of justice be allowed to ignore

and mock the moral principle Which Condemns the individual? Is there a moral standard for the

government different from the one by Which private conduct is Measured? While a private citizen is not

allowed to violate any rule of decency and fair play, the government may follow a procedure Which

shocks the common sense of decency and fair play? If a person can not enrich himself with stolen

property, Why Should a government be allowed to profit and make use of property tainted by theft or

robbery or smeared With The Blood of crime?

The above are among the basic questions answered That Must Be In This case, if we are not lacking the

moral courage to face all the issues raised by the parties.Other questions concern as Affected by

Personal Liberty illegal detention, illegal searches against security personnel and seizures, judicial mind

emancipation from colonial attitude.

Respondents urges us to follow the decision ill Alvero vs . Dizon (L-342) Which, besides having been

rendered by a second Supreme Court, Whose existence is violative of the Constitution, can not merit

Olaim better than a servile adherence to a legal wrong doctrine, decorated by the halo of authority of

courts of a former metropolis. There are minds forget That That duty of thinking by ourselves and of not

sticking to the Teachings of foreign mentors've Become more imperative since July 4, 1946.

The seizure of the papers and effects in question, having been made without any search warrant, was

and is illegal, and was effected in open violation of the Following Provisions of the Constitution:

Page 10: Warrantless Arrests, Searches and Seizures and Privacy of Communication

10

"The right of the people to be secure in Their persons, houses, papers, and effects against unreasonable

searches and seizures Shall Not Be Violated, and no warrants Shall issue but upon probable cause, to be

determined by the Judge after examination under oath or affirmation of the complainant and the

witnesses may have produced, and Particularly Describing the place to be searched, and the persons or

things to be Seized. " (Article III, section 1 [3] of the Constitution.)

"The privacy of communication and correspondence be inviolable except upon Marshall lawful order of

the court or public safety and order When Otherwise require." (Article III, section 1 [5] of the Constitution.)

The seizure was in open violation of Also sections 3, 10, and 11 of Rule 122, Which are as follows:

"SEC. 3. Requirements for Issuing search warrant. -A search warrant issue Shall Not but upon probable

cause to be determined by the judge or justice of the peace after examination under oath or affirmation of

the complainant and the witnesses may have produced, and Particularly Describing the place to be

searched, and the persons or things to be Seized. "

"SEC. 10. Receipt for the property Seized. -The officer seizing property under the warrant must give a

detailed receipt for the same to the person on Whom or in Whose possession it was found, or in the

Absence of any person, must, in the Presence of at least two witnesses, leave a receipt in the place in

Which I found the Seized Property. "

"SEC. 11. Delivery of property and inventory thereof to court. - The officer must forthwith deliver the

property to the justice of the peace or judge of the Municipal Court or of the Court of First Instance issued

the warrant Which, together with a true inventory thereof duly verified by oath. "

Even more, the illegality and unconstitutionality amounted to two.criminal Offenses, one of them heavily

punished with correctional prison . The Offenses are punished by articles 128 and 130 of the Revised

Penal Code, Which read:

"ART. 128. Violation of domicile . - The penalty of prision correctional in its minimum period imposed upon

Shall Be Any public officer or employee who, not being Authorized by court order, Shall enter any dwelling

against the will of the owner thereof, search papers or other effects found therein without the previous

consent of Such owner, or, having surreptitiously Entered said dwelling, and being required to leave the

premises, refuse to do so Marshall.

"If the offense be Committed in the nighttime, or if any papers or effects not Constituting evidence of a

crime be not returned Immediately after the search made by the offender, the penalty Shall Be prision

correctional in its medium and maximum periods. "

"ART. 130. domicile Searching without witnesses .-The penalty of arrest increased in its medium and

maximum periods imposed Shall Be upon a public officer or employee who, in cases where a search is

proper, Shall search the domicile, papers or other belongings of any person, in the Absence of the Latter,

any member of his family, or In Their default. without the Presence of two witnesses Residing in the same

locality. "

The main authority Which Respondents rely upon is the decision of the Supreme Court of the United

States in Bordeau vs . MacDowell (256 U.S., 465), the same Followed in the decision in Alvero vs . Dizon,

Page 11: Warrantless Arrests, Searches and Seizures and Privacy of Communication

11

(L-342). In the case Bordeau, Were Certain documents stolen from MacDowell. Upon finding the

documents Contained That evidence of the fraudulent use of the mails by MacDowell, delivered them to

the robbers Bordeau, in charge of the prosecution against MacDowell. The Latter filed a motion to

Prevent Bordeau from using the documents as evidence against him. The federal Supreme Court denied

the motion on the ground That there is no law or constitutional principle Requiring the government to

surrender papers Which May have eaten into its possession where the government has not Violated the

Constitutional Rights of the petitioner. Two of the greatest American Justices, Justices Holmes and

Brandeis, Whose dissenting opinions, written twenty years ago, are now the guiding beacons of the

Supreme Court of the United States, dissented, the Latter sayings:

"At the foundation of our civil liberty lies the principle to government denies Which Officials exceptional

position before the law, and subjects them to the Which rules of conduct That same commands to the

citizen. And in the development of our liberty insistence upon procedural regularity has been a large

factor. Respect for law will not be advanced by resort, in its enforcement, Which means to shock the

common man's sense of decency and fair play. "

Taking aside the great intellectual, legal and moral prestige of the two dissenters, the poignant sense of

logic and truth rockbottom Expressed by Justice Brandeis is enough to completely discredit the Majority in

the Bordeau case doctrine, a doctrine That in principle and by its evil effects appears to be irretrievably

immoral.

To merit Respect and obedience to government must be just. Justice can not exist where the good is not

distinguished from the wicked. To be just, the government must be good. To be good it must stick to the

principles of decency and fair play As They Are Understood by a common man's sense, by universal

conscience.Good ends do not justify foul means. Should one not profit from crime. Principles are not to be

sacrificed for any purpose. What is bad per se can not be good Because It Is Done to Attain a good

object. No wrong is atoned by good Intention.These are some of the maxims through Which the common

sense of decency and fair play is Manifested.

Reason is a key characteristic of man. There is no miracle Greater than scintillated When its first sparks

in the mind of a child. What before had only the vegetative life of a plant or the animal life of a mollusk or

frog, Suddenly begins to wield the prodigious power of understanding and of intelligent grasping of the

meanings and relations of the Things with Which he is In direct or remote contact through his senses.  The

power of understanding Brings forth freedom of choice. This freedom develops the faculty of

discrimination Between good and evil. That Is Further Developed discrimination into a sense of justice.

While the advent of the astounding miracle of reason has kindled so much the pride of man, to the Extent

of symbolizing It With the fire stolen from the heavens by Prometeus, and of proclaiming himself as the

king of the creation, man had taken millennia of Struggles in order to Develop The Basic Ideas Which Will

insure his survival and allow him to enjoy the greatest measure of well-being and happiness.I soon

discovered That society is an essential condition to Attain his ends. As a Consequence, I Fought against

all anti-social thoughts and conduct and had to discover or invent and then Develop the principles and

Page 12: Warrantless Arrests, Searches and Seizures and Privacy of Communication

12

qualities of sociability. The struggle has been long and it will Have to Continue Until The End of the

centuries. It is the same Eternal Struggle Between truth and error, between right and wrong.

While man, in the multifarious ensemble of the universe, Seems to be the lone and exclusive holder of the

divine fire of reason, I've so far failed to find the key to always correct thinking.  The solution to the failures

of reason is a riddle yet to be unlocked. Man is Easily deceived or led into Committing blunders into The

Most absurd aberrations. The mysterious genes uninterrupted Which keep the chain of heredity, while

permitting the transmission of the best qualities and characteristics, Seems to LACK the power of

checking and staving off the tendencies of atavism.In the moral ctetology, Either kind of Characteristics

and qualities may be Originated and developed. The inconsistency of Respondents Malthus is

explainable.While They would raise Their brows at the mere insinuation That a single private profit may

justifiably by the results or fruits of a criminal offense, They would not measure the government With The

same moral standard. That the inconsistency may be Explained by its genesis is why We Should not

surrender ground to it. To set two moral standards, a strict one for Private Individuals vitiated with laxity

and another for the government, is to throw society into the abyss of legal ataxia.Anarchy and chaos Will

Become inevitable. Such a double standard will be nomoctonous Necessarily.

The idea of moral double standard is incompatible With The temper and idiosyncracy of social order

Established by our Constitution, and Is repugnant to its provisions. All government authority emanates

from the people in Whom Sovereignty resides. The Filipino people ordained and promulgated the

Constitution "in order to Establish That Shall government to embody Their ideals." Among These Ideals

are justice, democracy, the promotion of social justice, equal protection of the laws to everybody. Such

ideals are trampled down by the adoption of the double moral standard Which can only take its place in

the ideology of the supporters of absolute Monarchies. That Theirs is the maxim "the king can do no

wrong." The iniquities and misery Such havocked by maxim would need Hundreds or Thousands of

volumes to record them. The infamy of Japanese occupation Gave our people the bitter taste of the

operation of the double moral standard. It is the antithesis of the golden rule. It would place government in

a category apart from humanity wholly, notwithstanding its being a human institution, unredeemable-an

absurdity.

From "Brandeis, A Free Man's Life" by Alpheus Thomas Mason (pp. 568 and 569), we quote an

analogous legal situmtion:

"In the famous wire-tapping case Chief Justice Taft, delivering the opinion, overruled the defendants'

claim That the Evidence Obtained When government agents tapped telephone wires Violated Their

unreasonable searches and seizures Either or the constitutional protection against self-incrimination.

Tapped No wires Entered Their homes and offices, Taft reasoned, so there was neither search nor

seizure,

"For Justice Brandeis Such a narrow construction degraded our great charter of freedom to the level of a

city ordinance. Quoting Chief Justice Marshall's famous admonition," We must never forget That it is a

Constitution we are expounding'-I pointed out That Just as the power of Congress had been kept by court

Page 13: Warrantless Arrests, Searches and Seizures and Privacy of Communication

13

interpretation abreast of scientific progress, and extended the Fundamental Law to objects of Which the

Founding Fathers never dreamed, so must the Judges Also in construing Limitations on the powers of

Congress be evermindful of Changes Brought About by discovery and invention. To have a living

Constitution, Limitations on power no less than grants of power must ba Broadly construed, 'subtler and.

more far-reaching means of invading privacy Have Become Available to the government,' Observed

Brandeis, *** The progress of science in furnishing the government with means of espionage is not likely

to stop with wire-tapping. Ways may some day be developed by Which the government, without removing

papers from secret drawers, can play them in court, and by Which it will be enabled to expose to a jury

the MOST intimate occurrences of the home. ***

"'Our government is the potent, the omnipresent teacher. For good or ill, it Teaches the whole people by

example, crime is contagious. If the government Becomes a lawbreaker, it breeds contempt for law; it

invites every man to become a law unto himself, it invites anarchy. To declare That in the administration

of the criminal law the end justifies the means - to declare That the government may commit crimes in

order to secure the conviction of a private criminal - would bring dreadful retribution. ***

"'The makers of our Constitution undertook to secure conditions conducive to the pursuit of happiness," I

emphasized. "They Recognized the significance of man's spiritual nature, of his feelings and of his

intellect. They only knew That part of the pain to, pleasure, and Satisfactions of life are to be found in

materials things. They Sought to protect Americans in Their Beliefs, Their Thoughts, Their emotions,

sensations and Their, They conferred, as against the government, the right to be let alone-the most

comprehensive of rights and the right valued by civilized men MOST. ***

"'Experience Should Teach, us to be MOST on our guard to protect liberty When the government's

Purposes are beneficent. Men horn to freedom are naturally alert to repel invasion of Their liberty by evil-

minded rulers. The greatest dangers to liberty lurk in insidious encroachment by men of zeal, well-

meaning, but without understanding. '"(Olmstead vs. . U.S., 277 [U.S.] 438 [1928], p. 473-474, 478, 479,

485.)

The Argument That goods and personal properties illegally taken, stolen, or Snatched from the owner or

possessor without a duly issued search warrant can be Retained by the prosecution for use as evidence

in a criminal case is Instituted Initiated by an original and basic flaw. The argument Rests on the Assumed

existence or commission of a crime as its minor premise. But, under the orderly Processes of law, the

assumption has yet to be proved Please, and it is impossible to be before it can be PROVED of any use

to support and clinch the argument. The prosecution is called upon to make the assumption That the

goods and properties in question are evidence of a crime. To be valid, the assumption has to presuppose

the existence of the commission or crime. That presupposition, in order to be valid, must in turn stand on

an authoritative pronouncement Which can only be made in a final and executory decision rendered by a

court of justice. The prosecution can not make a conclusive pronouncement, as to the existence or

commission of a crime, the basic fact Which, under the argument, will entitle the prosecution to retain and

use the goods and properties in question. the argument Assumes a fact the existence of Which Still

Page 14: Warrantless Arrests, Searches and Seizures and Privacy of Communication

14

Remains to be PROVED and Continues to be enveloped in the mists of the realm of Uncertainties, Which

may lead to the fact disputed right of the prosecution to retain the goods and properties as essential

evidence illegally Seized of the crime. The line of reasoning That build up the argument can be in more

abstract terms Restated as follows: Justify the means by Their Necessity to Attain an end by starting from

the premise That the end was accomplished, Such a reasoning process is Fundamentally subversive to

logic and is naturally incompatible With The workings of the human mind.

The Rules Governing the phenomena of diffusion and osmosis, isotonic equilibrium of permeability and,

of assimilation and waste dislodgment, of development and reproduction, like all laws of life, are uniform

and universal, in the nuclear chromatin Whether or the cytosome of a single protoplasmic of amoeba or

cell in the sinews of the heaviest marsupial, Whether in the formation of the smallest bud or in the display

of color and flavor by the most beautiful flower, Whether in the development of a frog or in the attainment

of the perfect curves and velvety skin of a lovely girl, the uniformity and universality of biological laws are

Manifested unrelentlessly. Any disregard of them is fatal, and will lead to irretrievable disaster and

destruction. Moral standards are the laws of social life. In a different plane and order, They are but

biological laws, Processes and Governing the vital functions of social organism. They are and Should be

uniform and universal and no single unit or organ of human society can disregard them or any one of

them alluring without catastrophic consequences.

Our decision is to grant all the prayers of the petition, and it was so ever since February 24, 1947, When

This Court took the vote for the disposal of this case. In Stating this fact we do not want to put any blame

on the distinguished member who penned the decision now to be promulgated. In justice to him, That we

may record the drafting of the Majority decision was TRANSFERRED and Entrusted to him for many

months after the final vote had been taken on the case. and it did not take him more than a month To

Have Majority opinion. In exposing the fact we mean only to Emphasize the crying need of changing a

situation or a system of procedure That Permits the promulgation of our decisions one year or more after

a case has been submitted to us for final action. It is only part of the crusade to curtail court delay Which

We felt our duty to engage in since it had been our privilege to sit in the supreme Court I, Whose vantage

in the legal field imposes upon the members thereof the role of leadership in legal thought and practice for

The Most Effective administration of justice. 

Dissenting

Bengzon, J. :Sanctity of the home is a by-word anywhere, anytime. The house of man was the first house of God.

In Rome the citizen's dwelling was a safe asylum. Invasion was anathema thereof.Down through the

centuries Respect for men's abodes has Remained a heritage of civilization.

Page 15: Warrantless Arrests, Searches and Seizures and Privacy of Communication

15

In England, the poorest man in his cottage Could, defy all the forces of the Crown."It may be frail, its roof

may shake, the wind may blow through it, the storm may enter, the rain may enter, but the King of

England may not enter; Dare not all his forces cross the threshold of the ruined tenement. '  His home was

indeed his castle.

And in the United States: "The right of the citizen to occupy and enjoy his home, However mean or

humble, free from arbitrary invasion and search, has been protected for centuries With The MOST

solicitous care. ***" The mere fact That a man is an officer, high or low Whether of degree, Gives him no

more right than is possessed by the ordinary private citizen to break in upon the privacy of a home and its

occupants subject to the indignity of a search for the evidence of crime , without a legal warrant procured

for That Purpose. No amount of incriminating evidence, whatever its source, will supply the place of Such

warrant. At the closed door of the home, be it palace or hovel, even bloodhounds must wait till the law, by

authoritative process, bids it open. *** "(McLurg vs . Brenton, 123 Iowa 368, quoted in 20 Phil. 473.)

Logical and practical culmination application of the above principles, embodied in our Organic Laws, is

the ruling we Announced in Alvarez vs . Court of First Instance of Tayabas, 64 Phil. 33 That Seized

documents unlawfully in a man's home must be returned - irrespective of Their evidentiary value -

provided seasonable motions are submitted. We Followed the Federal rule in Boyd vs . U.S. 116 U.S.,

616 and many others. We had said before That "That it is better oftentimes go unpunished crimes Than

That Should The Citizen Should be liable To Have his premises invaded, his desks broken open, his

private books, letters, and papers exposed to prying curiosity, *** under the direction of a mere ministerial

officer "*** Perhaps insensitive to the rights and feelings of others. (U.S. vs . De los Reyes and Esguerra,

20 Phil. 472, citing Cooley, Constitutional Limitations.)

In the Alvarez decision we Reflected That "of all rights of a citizen of Greater Importance are few or more

essential to his peace and happiness than the right of personal security, and That Involves the exemption

of his private affairs, books, and papers from the inspection and scrutiny of others ", and while the power

to search and seize is Necessary to public welfare, still it must be Exercised without transgressing the

constitutional rights of Citizens, Because the enforcement of statutes is never Sufficiently important to

justify violation of the basic principles of government. It is essential Agreed That the rights of the single

guaranteed by the Constitution, Must Be Given Such a liberal construction or strict construction as will be

in his favor, to Prevent gradual or stealthy encroachment Such depreciation of essential rights. (State vs .

Custer County, 198 Pac., 362, State vs. McDaniel 231 Pac., 965, 237 Pac. 373.)

Our Constitution in its Bill of Bights Decrees That "the right pf the people to be secure in Their persons,

houses, papers, and effects against unreasonable searches and seizures Shall Not Be Violated, and no

warrants Shall issue but upon probable cause, to be Determined by the judge after examination under

oath or affirmation of the complainant and the witnesses may have produced, and Particularly Describing

the place to be searched, and the persons or things to be Seized ".(Constitution, article III, section 1 [3].)

Page 16: Warrantless Arrests, Searches and Seizures and Privacy of Communication

16

This is an improvement over the Provisions of the Jones Law Regarding warrants and seizures. It was

designed to make our Constitution "conform entirely" to the Fourth Amendment of the U.S.

Constitution. (Aruego, Framing of the Philippine Constitution, Vol II, p. 1043.)

The Split Between State Supreme Courts several on one side and the Federal Supreme Court on the

other, about the Admissibility of Evidence Obtained through illegal searches and seizures, was familiar to

this Court (People vs . Carlos, 47 Phil. 626, 630) before it voted to adopt the doctrine in Federal

Alvarez vs . Court of First Instance of Tayabas, supra .

This last doctrine, Applied in several subsequent cases (People vs . Sy Juco, 64 Phil. 667; Rodriguez vs .

Villamil, 37 Off. Gaz., 2416) was probably Known to the Constitutional Convention That, in Addition, made

the constitutional mandate on the point more complete and explicit, copying exactly the wording of the

Federal Constitution, a circumstance Which, coupled With The citation of Boyd vs . U.S., adherence to

the Federal Showed That debars evidence doctrine Obtained by illegal search or seizure unlawful.

It is significant the Convention Readily ADOPTED That the recommendation of the Committee on Bill of

Rights after its Chairman had spoken, explaining the meaning and Extent of the provision on searches

and seizures and específicamente Invoking the United States decisions of Boyd vs . U.S. 616 and U.S.

116 Gould vs . U.S. 225 U.S. 298, Which The Majority of this Court would now discard and overrule.

(Aruego op. Cit. Vol I, p. 160, Vol II, p. 1043, 1044.)

THEREFORE, it is submitted, with all due respect, That we are not at liberty now to select Between Two

conflicting theories. The selection has been made by the Constitutional Convention When It impliedly

chose to abide by the Federal decisions, upholding to the limit the inviolability of man's

domicil. Home! The Tie That Binds, the affection That Gives Life, The Pause That soothes, all nestle in an

atmosphere of there security. Remove that security and you destroy the home.

Under This new ruling the "King's forces" may now "cross the threshold of the ruined tenement" seize the

skeleton from the family closet and rattle it in public, in court, to the vexation or shame of the unhappy

occupants. That may be jailed Those forces for trespass, is little consolation. That may be forces Those

pardoned by the King, Their master, Suggests fearful possibilities. The sanctuary, the castle, are gone

with the wind.

An opinion of Mr. Justice Gardozo in the Court of Appeals of New York is cited as authority for the

Majority view (People vs . Defore NE 150, 585). Yet it is markworthy That, in New York, protection against

unreasonable searches and seizures is hot promised by the Constitution of the State but by a mere

statute.(Civil Rights of Law) (See the same case, and 56 CJ, p. 1156.) New York is the only state That

denies this privilege the status of a constitutional prerogative, (supra ). Hence the precedent is obviously

inconclusive.

Moreover, Admitting, for Purposes of argument only, That the decision is legally erroneous Alvarez, I

Maintain That Should the new doctrine apply to future cases - not to HEREIN petitioner who has relied on

it. In Santiago and Flores vs .Valenzuela, No. L-670, April 30, 1947 (44 Off. Gaz., 3291, 3296) I argued

for That proposition as follows:

Page 17: Warrantless Arrests, Searches and Seizures and Privacy of Communication

17

"*** The reserved right to upset previous decisions is likewise qualified by the proposition That Shall have

upsetting Such prospective - not retroactive - effect.

"In Douglass vs. Pike County, 101 U.S. 677 at p. 667, it was declared," The true rule ( of stare decisis ) is

to give a change of judicial construction *** the same effect in its operation '*** as to 'a legislative

amendment, ie, prospective but not make it retroactive.'

"And in Great Northern R. Co. v. . Sunburst Oil & Ref Co., 287 U.S. 358, the Supreme Court, through Mr.

Justice Cardozo, said:

"'A state in defining the limits of adherence to precedent may make a choice for itself Between the

principle of forward operation and That of relation backward. It may Say That decisions of its highest

court, though later overruled, are law none the less for intermediate transactions. Indeed there are cases

intimating, too Broadly (cf. Tidal Oil Co. vs . Flanagan. 263 U.S. 444, 68 Law. ed., 382, 44 S. Ct.

197, supra ), it must give them That That effect, but has never been expresaron doubt That it may so treat

them if it pleases, injustice or hardship Whenever Thereby will be averted. Gelpcke vs . Dubuque, 1 Wall.,

175, 17 Law. ed., 250; Douglass vs . Pike County, 101 U.S. 677, 687, 25 Law. ed., 968, 971; Loeb vs .

Columbia Twp., 179 U. S, 472, 492, 45 Law. ed., 280, 290, 21 S. Ct., 174, etc. '"

"This view is not unanimous, I know. However, inasmuch as one of the main arguments of the opposing

school of thought Is that it makes the decision overruling a mere 'declaratory judgment', and since That

objection is untenable In This jurisdiction where declaratory relief is permitted (Rule 66), the view

advocated HEREIN - future operation only - should be all the more acceptable to our system of

jurisprudence. More about this in the future, if i should happen to agree to an orerruling of previous

decisions and the Should question hinge on its backward or forward application. For the present, enough

to note some of the abundant literature on the point. [1] "

DISSIDENT

BRIONES, M. :I disagree with the paper. I believe that should be granted by the appellant solicitude. I think in this

jurisdiction must adhere to the principle established in the case of Weeks v. . U.S. cited in the majority

decision.

If a demooracia as nortearnericana - ripe and well solidified, strengthened by a centuries-old tradition of

respect for individual freedoms and citizens and for the fair and calm temperament of a breed as

admirable as the Anglo - it was considered necessary to ensure the privileges of the citizen under the

shell of this doctrine, a fortiori we have and ensure these guarantees in a democracy like ours, young,

who are just doing the initial first steps on the road to political independence, and where demagoguery

and anarchy dangerous trends and establishing a regime of force could thwart bendioiones of freedom

won so many coast.

PARAS, M. , as.

Page 18: Warrantless Arrests, Searches and Seizures and Privacy of Communication

18

The request is denied.

[ G.R. No. L-34038, June 18, 1976 ]THE COLLECTOR OF CUSTOMS AIRPORT CUSTOMHOUSE, PASAY CITY, PETITIONER, VS. HON. ONOFRE A. VILLALUZ, AS JUDGE OF THE CIRCUIT CRIMINAL COURT, 7TH JUDICIAL DISTRICT,

STATIONED AT PASIG, RIZAL, AND CESAR T. MAKAPUGAY, RESPONDENTS. 

[G.R. No. L-34243.  June 18, 1976]

NICANOR MARCELO, PETITIONER, VS. HON. ONOFRE A. VILLALUZ, AS JUDGE OF THE CIRCUIT CRIMINAL COURT, 7TH JUDICIAL DISTRICT STATIONED AT PASIG, RIZAL, AND SALVADOR T.

MASCARDO, AS COLLECTOR OF CUSTOMS STATIONED AT THE MIA AIRPORT CUSTOMHOUSE, RESPONDENTS. 

[G.R. No. L-36376.  June 18, 1976]

CALIXTO D.  ENRIQUEZ, REYNALDO REYES AND LUCILA ENRIQUEZ, PETITIONERS, VS. HON. ONOFRE A. VILLALUZ, GREGORIO CONDE AND ANASTACIA TORILLO, RESPONDENTS. 

[G.R. No. L-38688.  June 18, 1976]

FRANCISCO P. FELIX, PETITIONER, VS. THE HON. JUDGE ONOFRE A. VILLALUZ AND FELIX C. HALIMAO, RESPONDENTS.

[G.R. No. L-39525.  June 18, 1976]

PEDRO E. NIEVA, JR., PETITIONER, VS. HON. ONOFRE A. VILLALUZ, IN HIS CAPACITY AS JUDGE OF THE CIRCUIT CRIMINAL COURT, 7TH JUDICIAL DISTRICT, JOSE ARELLANO, AND THE

PEOPLE OF THE PHILIPPINES, RESPONDENTS. 

[G.R. No. L-40031.  June 18, 1976]

PACITA NIEVA, PETITIONER, VS. HON. ONOFRE A. VILLALUZ, IN HIS CAPACITY AS JUDGE OF THE CIRCUIT CRIMINAL COURT, 7TH JUDICIAL DISTRICT, JOSE ARELLANO AND THE PEOPLE OF

THE PHILIPPINES, RESPONDENTS. 

D E C I S I O NMAKASIAR, J.:On July 1, 1971, petitioner Collector of Customs, Salvador T. Mascardo, filed against Cesar T. Makapugay, a letter complaint with respondent Judge of the Circuit Criminal Court for violation of: (a) Section 174 of the National Internal Revenue Code, as amended by Republic Act No. 4713, (b) Central Bank Circular No. 265, in relation to Section 34 of Republic Act No. 265, otherwise known as The Central Bank Act, and (c) Sections 3601 and 3602 of Republic Act No. 1937, in relation to Sections 2505 and 2530 (m) 1 of the same Act, claiming that Cesar T. Makapugay "with malicious intention to defraud the government criminally, willfully and feloniously brought into the country FORTY (40) cartons of 'untaxed blue seal' Salem cigarettes and FIVE (5) bottles of Johnny Walker Scotch Whiskey, also 'untaxed', without the necessary permit from the proper authorities. The respondent submitted a Baggage Declaration Entry which did not declare the said articles. The Customs Examiner assigned further asked him if he has something more to declare but the answer was in the negative. And in utter disregard of existing Central

Page 19: Warrantless Arrests, Searches and Seizures and Privacy of Communication

19

Bank Circulars, particularly C.B. Circular 265, as amended, the respondent brought into the country various Philippine Money in the amount of Two Thousand Two Hundred Eighty (P2,280.00) Pesos cleverly hidden in one of the pieces of baggage examined by the assigned customs examiner, without prior permit from the Central Bank authorities. * * *" (p. 11, rec.).Respondent Judge assumed jurisdiction to conduct and did conduct the preliminary investigation, and on July 6, 1971, issued the challenged order, dismissing "the case with prejudice and ordering the return to private respondent the amount of P2,280.00 his passport No. Ag-2456 FA - No. B103813, and one (1) box of airconditioning evaporator only, as well as the forfeiture of forty (40) cartons of untaxed blue seal Salem cigarettes and five (5) bottles of Johnny Walker Scotch Whiskey" (p. 13, rec.).

Armed with said order, private respondent Makapugay demanded that petitioner release the articles so stated. Petitioner Collector of Customs refused to obey the order due to the "prior institution of seizure proceedings thereon." The refusal prompted respondent Makapugay to file a complaint for "Open Disobedience" under Article 231 of the Revised Penal Code, before the City Fiscal of Pasay City.

Hence, this petition for certiorari with preliminary injunction, seeking to annul and set aside the order dated July 6, 1971 on the ground that respondent Judge has no power to conduct a preliminary investigation of criminal complaints directly filed with him, cannot legally order the dismissal "with prejudice" of a criminal case after conducting a preliminary investigation thereon, and is without authority to order the return of articles subject of seizure proceedings before Customs authorities.In due time, respondents filed their respective answers to the petition and subsequently both parties submitted their respective memoranda in lieu of oral argument.

G. R. No. L-34243On June 22, 1971, respondent Collector of Customs filed a letter-complaint with respondent Judge against petitioner Nicanor Marcelo for an alleged violation of Section 3602 in relation to Section 2505 of Republic Act 1937, otherwise known as the Tariff and Customs Code, supposed to have been committed in the following manner: 

"* * * Mr. Marcelo who is an arriving passenger from Hongkong on board a Philippine Air Lines plane, Flight 307, on June 22, 1971, criminally, feloniously, and with intention to defraud the government did not declare the contents of his pieces of baggage in the Baggage Declaration Entry nor with the assigned Customs Examiner. * * * When his pieces of baggage were examined, instead of personal effects as declared in the Baggage Declaration Entry, what were found were various assorted Watches, Bags, Montagut shirts and Dress materials which are highly taxable. 

"The act of passenger Marcelo in intentionally refusing to declare the said articles in the Baggage Declaration Entry, and before the Customs Examiner despite inquiries made, constitute a criminal offense within the meaning of Section 3602 of the Tariff and Customs Code of the Philippines. * * * (p. 19, rec.).

The criminal complaint having been docketed as Case No. CCC-VII-854-P.C., the respondent Judge assumed jurisdiction over the objection of petitioner's counsel, conducted the preliminary examination and investigation, simultaneously in the manner provided for by Section 13, Rule 112 of the New Rules of Court, and thereafter on October 6, 1971 issued the following order: 

"WHEREFORE, there being a preliminary investigation and examination conducted by the Court and considering that the respondent was given a chance to defend himself let a Warrant of Arrest be issued

Page 20: Warrantless Arrests, Searches and Seizures and Privacy of Communication

20

for his apprehension. The respondent is hereby ordered to post a bond in the amount of P5,000.00 for his provisional release. 

"Pursuant to Section 6, Rule 135 of the New Rules of court, in relation to Section 13, Rule 113 thereto, the City Fiscal of Pasay is hereby ordered to file the corresponding information against the respondent before this court of competent jurisdiction within FORTY EIGHT (48) HOURS from receipt hereof" (p. 23, rec.).

Petitioner Nicanor Marcelo filed this action for certiorari with preliminary injunction, impugning the validity of the order of respondent Judge dated October 6, 1971, on the same ground as the petition in G.R. No. L-34038.On October 20, 1971, the Supreme Court adopted a resolution requiring respondents to file an answer and likewise issued a writ of preliminary injunction, "restraining respondent Judge, his representatives, assigns or persons acting upon his orders, place or stead, from executing, enforcing and implementing his order of October 6, 1971 * * *" (p. 32, rec.).

In compliance therewith, respondent Judge filed a petition for admission of answer on November 29, 1971 (pp. 43-44, rec.), which was granted by this Court in its December 13, 1971 resolution (p. 62, rec.).

On the other hand, respondent Collector of Customs, through the Solicitor General, filed a manifestation on February 1, 1972, adopting as his answer to the petition, the legal grounds averred in the original petition in G.R. No. L-34038, Collector of Customs, etc. versus Hon. Onofre A. Villaluz, etc., et al. (p. 72, rec.).On June 13, 1972, the Supreme Court by resolution resolved to consider the case submitted for decision after noting the failure of petitioner to file his memorandum (p. 94, rec.).

G. R. No. L-36376On February 22, 1973, private respondents Gregorio Conde and Anastacia Torillo, filed a complaint directly with the Circuit Criminal Court, indicting petitioners with violations of the Anti-Graft Law.

The complaint was ultimately docketed and on the same day (February 22, 1973), respondent Judge forthwith issued an order of the following tenor: 

"Considering that the complaint filed * * * * sufficient in form and substance, the same having been filed in accordance with Section 13, Rule 112 of the New Rules of Court, and pursuant to the doctrine laid down by the Supreme Court in the case of "Mateo vs. Villaluz", let the preliminary investigation of this case be set on February 24, 1973 at 8:00 o'clock in the morning" (p. 22, rec.).On the day set, petitioners appeared at the sala of respondent Judge who proceeded to conduct a preliminary investigation of the case. The same was reset on February 26, 1973.

Immediately before the hearing of February 26, 1973, petitioners, through counsel, filed an "Urgent Motion to Suspend Preliminary Investigation" contesting the power of the respondent Judge to conduct the preliminary examination and investigation (p. 23, rec.), which was denied by respondent Judge in his order dated February 27, 1973 (p. 31, rec.). Counsel for petitioners then asked for time to raise the issue before this Court, which respondent Judge granted by giving petitioners a period of just one (1) day to seek relief from this Tribunal.

Accordingly, herein petitioners filed this petition.

Page 21: Warrantless Arrests, Searches and Seizures and Privacy of Communication

21

On March 2, 1973, this Court required respondents to answer the petition and issued a temporary restraining order "enjoining respondent Judge from * * * causing and effecting the arrest of petitioners herein" (p. 39, rec.).

In his answer filed on March 14, 1973, respondent Judge, invoking the same arguments in G.R. No. L-34243, held on to the view that the Circuit Criminal Courts are vested with the power and authority to conduct preliminary investigations. Private respondents conformed thereto.

G. R. No. L-38688On May 23, 1974, private respondent Felix Halimao filed a criminal complaint directly with the Circuit Criminal Court presided over by respondent Judge charging herein petitioner with alleged violations of Republic Act No. 3019, otherwise known as the Anti-Graft and Corrupt Practices Act, which complaint was docketed as Criminal Case No. Prel. Inv. - 116-Rizal.

At the hearing of May 27, 1974, petitioner, through counsel, filed an "Urgent Motion to Suspend Preliminary Investigation" (p. 9, rec.), based on the ground that respondent Judge has no authority to conduct the same.

After arguments by counsels, for both parties, the respondent Judge denied petitioner's motion. An oral motion for reconsideration was likewise denied (pp. 14-15, rec.).

Hence, this petition.

On May 31, 1974, this Court by resolution gave due course to the petition and issued a restraining order, "enjoining respondent Judge, his agents, representatives, and/or any person or persons acting upon his orders or in his place or stead from proceeding further with the preliminary investigation * * *" (p. 24, rec.).

On June 17, 1974, it appearing that the case involved in the petition is criminal in nature, the Court required herein petitioner to IMPLEAD the People of the Philippines as party-respondent (p. 26, rec.). In conformity thereto, petitioner through counsel, filed on June 28, 1974 an amended petition impleading The People (pp. 49-50, rec.).

Except for the Solicitor General who appeared for The People of the Philippines, respondents in answer, frontally met the averments of petitioner.

G. R. No. L-39525On October 24, 1974, petitioner filed this instant petition seeking to annul "any preliminary investigation conducted by respondent Judge in Preliminary Inv. No. 72-Rizal, Circuit Criminal Court, 7th Judicial District, as well as the warrant, if any, that may be issued for the arrest and imprisonment of petitioner" and to enjoin permanently respondent Judge from conducting preliminary investigations and from ordering petitioner's arrest.

On October 30, 1974, the Court required the respondents to file their answer within ten (10) days from notice thereof and issued, effective immediately, a temporary restraining order against respondent Judge (p. 64, rec.).

On November 13, 1974, the Solicitor General filed a manifestation requesting to be excused from filing an answer considering that in three other cases (The Collector of Customs v. Hon. Onofre A. Villaluz, G.R.

Page 22: Warrantless Arrests, Searches and Seizures and Privacy of Communication

22

No. L-34038;Nicanor Marcelo v. Hon. Onofre A. Villaluz, G.R. No. L-34243; and Francisco Felix v. Hon. Onofre A. Villaluz, G.R. No. L-38688) which involve the same legal issue, his office maintains that respondent Judge has no authority to conduct a preliminary investigation of criminal cases which he may try and decide under Republic Act No. 5179 (p. 81, rec.).On November 20, 1974, private respondent filed his answer (pp. 87-104, rec.).

Petitioner, on January 22, 1975, filed a motion praying that the instant case be consolidated and decided jointly with G. R. Nos. L-34038, L-34243, L-36376 and L-38688 as they involve the same issue; and that the memoranda filed for petitioners in said four cases be reproduced and adopted as the memorandum for petitioner in this case, which should be deemed submitted for decision together with the aforementioned cases (pp. 122-124, rec.). Said motion was granted in the resolution of February 10, 1975 (p. 129, rec.).

In his pleading dated February 5, 1975, private respondent (pp. 130-132, rec.) stated that he joins the petitioner in his plea for the consolidation of the instant case with cases Nos. L-34038, L-36376 and L-38688 and prayed that the memorandum filed by respondent in L-38688 be considered reproduced and adopted as the memorandum for private respondent in this case, in addition to the affirmative defenses and arguments contained in private respondent's answer to the petition, and that this case be submitted for decision together with the aforementioned cases (p. 137, rec.).

The records disclosed the following antecedent facts.

On January 11, 1974, herein private respondent Jose Arellano filed a complaint against Pedro E. Nieva, Jr., herein petitioner, together with his wife Pacita and daughter Patricia N. Sacasas, with the Circuit Criminal Court, Seventh Judicial District, Pasig, Rizal, for violation of the Anti-Graft and Corrupt Practices Act (RA No. 3019) in connection with the P230,000.00 industrial loan obtained by the Areson Woodtech Manufacturing Company headed by the complainant, Jose Arellano, from the Development Bank of the Philippines, where herein petitioner holds the position of Auditor. The case was docketed therein as Criminal Case Prel. Inv. CCC-VII-72 Rizal (pp. 1-2, 90-91, pp. 14-16 [Annex "A"] rec.).

On the same day the aforesaid complaint was filed in court, respondent Judge issued an order that reads:

"Pursuant to Section 14, Rule 112 of the New Rules of Court in relation to the doctrine laid down by the Supreme Court in the case of ‘Mateo versus Villaluz', Assistant City Fiscal Teodoro B. Santos is hereby ordered to conduct the preliminary investigation of the above-entitled case within five (5) days from receipt hereof and to file the necessary information in a court of competent jurisdiction if the evidence so warrants.  "* * * *" (pp. 2, 91 [Annex "B"], pp. 21-22, rec.).

On May 22, 1974, investigating Fiscal Teodoro B. Santos endorsed the records of the case back to respondent Judge, because 

“. . . (T)he facts and circumstances which has (sic) been the basis of this instant suit is the same set of facts and circumstances and involving the same parties in a case of ESTAFA THRU FALSIFICATION now pending preliminary investigation and also before this Honorable Court. Hence, this endorsement in order to avoid duplication of effort and time in the resolution and disposition of the same incident."In an urgent ex-parte motion dated May 24, 1974 filed with the Circuit Criminal Court pursuant to paragraph 1 of the Joint Circular of the Department of Justice and the Department of National Defense

Page 23: Warrantless Arrests, Searches and Seizures and Privacy of Communication

23

dated April 29, 1974, herein private respondent prayed that the endorsement of Fiscal Santos be given due course and that the preliminary investigation be conducted by the respondent Judge (pp. 3, 92, 104 [Annex "1"] rec.).Herein petitioner opposed the same in a pleading dated June 1, 1974 (p. 3, pp. 40-49 [Annex "F"], rec.), which was amplified in another pleading dated September 24, 1974 (pp. 3, 50-59 [Annex "G"], rec.).

Under date of June 18, 1974, private respondent filed a motion to strike out herein petitioner's opposition to complainant's ex-parte urgent motion for preliminary investigation in view of the failure of herein petitioner's counsel to comply with the order of the Court to furnish a copy of his opposition to complainant Jose Arellano (pp. 93, 105-106 [Annex "2"], rec.).On September 24, 1974, herein petitioner filed his opposition to the motion to strike out herein respondent's opposition (pp. 7, 55-59 [Annex "G"], rec.). On the same day, a hearing was conducted by the respondent Judge on the urgent motion for preliminary investigation and immediately thereafter, he denied said opposition of herein petitioner (Annex "H", p. 62, pp. 3, 93, rec.).

Hence, this petition.

G. R. No. L-40031On November 2, 1973, Jose Arellano, private respondent herein, filed with the Circuit Criminal Court at Pasig, Rizal, a complaint charging herein petitioner with estafa, allegedly committed under the circumstances provided for in paragraph 4 1(b), Article 315 of the Revised Penal Code (p. 12, rec.). Said complaint was subsequently docketed as CCC Case No. Prel. Inv.-65-Rizal. Thereupon, respondent Judge proceeded to conduct the preliminary investigation in question. After the termination of the proceedings, respondent Judge issued on May 31, 1974 the challenged resolution which reads: 

"Wherefore, pursuant to Section 13, Rule 113 of the New Rules of Court, Assistant City Fiscal Teodoro B. Santos is hereby ordered to file the necessary information for the crime of Estafa against respondent Pacita Nieva, in a court of competent jurisdiction, within forty-eight (48) hours from receipt hereof. 

"Let a warrant of arrest be issued for the immediate apprehension of respondent Mrs. Pacita Nieva, and for her provisional liberty, she is hereby ordered to post a bond in the amount of P20,000.00" (p. 24, rec.).

On July 26, 1974, petitioner's counsel filed an urgent motion to declare the preliminary investigation proceedings null and void ab initio  due to lack of jurisdiction on the part of the court to conduct the same, re-echoing the arguments invoked by petitioners in G.R. Nos. L-34038, L-34243, L-36376 and L-38688 (p. 14, rec.).In an order dated August 8, 1974, respondent Judge denied the same (p. 22, rec.).

On January 28, 1975, this Court by resolution required respondents to file an answer to the petition and not to move for the dismissal of the same. The Court further resolved to consolidate the case with Cases Nos. L-38688, L-34038, L-34243, and L-36376 (p. 26, rec.).

In a manifestation filed on February 10, 1975, the Solicitor General requested that he be excused from filing an answer on the ground that in three cases (G.R. Nos. L-34038, L-34243 and L-38688), which involve the same legal issue, the counsel for the People has taken the position that respondent Judge has no authority or jurisdiction to conduct a preliminary investigation of criminal cases which he may try and decide under Republic Act No. 5179.

Page 24: Warrantless Arrests, Searches and Seizures and Privacy of Communication

24

Private respondent, on the other hand, through the Citizens Legal Assistance Office of the Department of Justice, filed his answer on February 20, 1975, maintaining that respondent Judge has jurisdiction to conduct preliminary investigation, invoking particularly Section 13, Rule 112 of the Revised Rules of Court in relation to Sections 1, 3 and 6 of Republic Act No. 5179.

IThe one common legal issue posed by these six cases is whethser a Circuit Criminal Court possesses the power to conduct preliminary investigations.

Neither the explanatory note to House Bill No. 9801 (now R.A. No. 5179) nor the available Congressional debates intimate that Circuit Criminal Courts are clothed with the authority to conduct preliminary examinations and investigations (Congressional Records of House, March 28, 1967, pp. 41-45; May 15, 1967).

WE therefore examine the law.

Petitioners, in maintaining that respondent Judge has no such power, rest their claim on Section 1 of Republic Act No. 5179, which provides: 

"In each of the sixteen judicial districts for the Court of First Instance as presently constituted, there is hereby created a Circuit Criminal Court with limited jurisdiction, concurrent with the regular Court of First Instance, to try and decide the following criminal cases falling under the original and exclusive jurisdiction of the latter: “a. Crimes committed by public officers, crimes against persons and crimes against property as defined and penalized under the Revised Penal Code, whether simple or complexed with other crimes; 

“b. Violations of Republic Act No. 3019, otherwise known as the Anti-Graft and Corrupt Practices Act, ***; 

“c. Violations of Sections 3601, 3602 and 3604 of the Tariff and Customs Code and Sections 174, 175 and 345 of the National Internal Revenue Code" (italics supplied). 

Petitioners argue that said courts, having been conferred limited jurisdiction, cannot exercise such power of preliminary investigation, the same not being embraced and contemplated within its given function to "try and decide" specific criminal cases.What is limited by Republic Act No. 5179 is the scope of the cases that may be tried by Circuit Criminal Courts.Circuit Criminal Courts are of limited jurisdiction, only because they cannot try and decide all criminal cases falling under the jurisdiction of the Courts of First Instance as courts of general jurisdiction. They can only take cognizance of cases expressly specified in Section 1 of Republic Act No. 5179, as amended by Presidential Decree No. 126. Nevertheless, they have the same powers and functions as those conferred upon regular Courts of First Instance necessary to effectively exercise such special and limited jurisdiction. This is plain and evident from Sections 3 and 6 of their organic law, Republic Act No. 5179: 

"SECTION 3. The provisions of all laws and the Rules of Court relative to the judges of the Courts of First Instance and the trial, and disposition and appeal of criminal cases therein shall be applicable to the circuit judge and the cases cognizable by them insofar as they are not inconsistent with the provisions of this Act. 

Page 25: Warrantless Arrests, Searches and Seizures and Privacy of Communication

25

* * * * * * * * * * * * * * 

"SECTION 6. * * * Unless inconsistent with the provisions of this Act, the Circuit Criminal Courts shall have the same powers as those conferred by the Judiciary Act and the Rules of Court upon regular Courts of First Instance, insofar as may be necessary to carry their jurisdiction into effect.”

Judges of the regular Courts of First Instance are expressly conferred the authority to conduct preliminary examination and investigation by Sections 13 and 14 of Rule 112 of the Revised Rules of Court: 

"SECTION 13. Preliminary examination and investigation by the judge of the Court of First Instance. - Upon complaint filed directly with the Court of First Instance, without previous preliminary examination and investigation conducted by the fiscal, the judge thereof shall either refer the complaint to the justice of the peace referred to in the second paragraph of Section 2, hereof for preliminary examination and investigation, or himself conduct both preliminary examination and investigation simultaneously in the manner provided in the preceding sections, and should he find reasonable ground to believe that the defendant has committed the offense charged, he shall issue a warrant for his arrest, and thereafter refer the case to the fiscal for the filing of the corresponding information" (italics supplied). "SECTION 14. Preliminary examination and investigation by provincial or city fiscal or by state attorney in cases cognizable by the Court of First Instance. - Except where an investigation has been conducted by a judge of first instance, justice of the peace or other officer in accordance with the provisions of the preceding sections, no information for an offense cognizable by the Court of First Instance shall be filed by the provincial or city fiscal, or state attorney, without first giving the accused a chance to be heard in a preliminary investigation conducted by him or by his assistant by issuing a corresponding subpoena. * * *"

The power of preliminary examination and investigation, which may be exercised by judges of the Circuit Criminal Courts, is without doubt, "not inconsistent with the provisions of Republic Act No. 5179," and likewise, "necessary to carry their jurisdiction into effect."

Moreover, Congress further confirmed that the Court of First Instance has the power to conduct preliminary investigation by approving on September 8, 1967 Republic Act No. 5180, prescribing a uniform system of preliminary investigation by all government prosecutors, which provides: 

"SECTION. 1. Notwithstanding any provision of law to the contrary and except when an investigation has been conducted by a Judge of First Instance, city or municipal judge or other officer in accordance with law and the Rules of Court of the Philippines, no information for an offense cognizable by the Court of First Instance shall be filed by the provincial or city fiscal or any of his assistants, or by a state attorney or his assistants, without first giving the accused a chance to be heard in a preliminary investigation conducted by him by issuing a corresponding subpoena. * * * "SECTION. 2. The provisions of Section fifteen, Rule 112, of the New Rules of Court of the Philippines, shall be observed in the investigations of persons in custody."

From the above-quoted provisions, Republic Act No. 5180 likewise continues the procedure prescribed in the Revised Rules of Court of 1964, particularly Rule 112 thereof.

The aforequoted portion of Section 1 of Republic Act No. 5180 was not modified by the amendatory Presidential Decrees Nos. 77 and 911 issued respectively on December 6, 1972 and March 23, 1976.

Page 26: Warrantless Arrests, Searches and Seizures and Privacy of Communication

26

More decisively, the 1935 as well as 1973 Constitutions vests this essential power in all courts to first determine probable cause before ordering the arrest of those charged with a criminal offense (Section 1[3] , Art. III, 1935 Constitution; Sec. 3, Art. IV, 1973 Constitution). The determination of "probable cause" is the sole object of preliminary examinations. Surely, Congress could not have possibly intended to deny the Circuit Criminal Courts such constitutional prerogative, which is part of the basic constitutional right of an individual whose person cannot be legally seized without prior preliminary examination by a judge.WE enunciated that the creation of the Circuit Criminal Courts is for the purpose of alleviating the burden of the regular Courts of First Instance and to accelerate the disposition of criminal cases pending or to be filed therein(People vs. Gutierrez, etc., et al., 36 SCRA 172; Osmeña vs. Sec. of Justice, G.R. No. L-32033, Sept. 30, 1971, 199) or to contribute to the speedy resolution of criminal cases and help curb the progression of criminality in the country (Paraguya vs. Tiro, 41 SCRA 137). As opined by Mr. Justice Barredo in his concurring opinion in the Gutierrez case, supra, "* * * Circuit Criminal Courts are nothing but additional branches of the regular Courts of First Instance in their respective districts * * *", which he reiterated in his concurring opinion in the Osmeña case, thus: "My principal reason for my vote in favor of the judgment in this case is that I cannot find any justification for allowing the Secretary of Justice to have any part at all in the distribution or assignment of cases among the different branches of any Court of First Instance, of which the corresponding Circuit Criminal Court is one. I took this view in my concurring opinion in the case of People v. Gutierrez, cited in the main opinion of Justice Villamor, and I cannot see why I must opine differently now. * * *" (41 SCRA 211).If the main purposes then in creating Circuit Criminal Courts are to alleviate the burden of the regular Courts of First Instance and to accelerate the disposition of the cases therein as well as stem the tide of criminality, it is only logical that such authority vested in the judges of the Courts of First Instance is likewise conferred on Circuit Criminal Courts. Otherwise, the Courts of First Instance would still be carrying the burden of conducting preliminary investigations in those cases where Circuit Criminal Courts have jurisdiction and consequently delaying the trial and disposition of criminal cases pending before such Courts of First Instance.

That Congress, in enacting Republic Act No. 5179 clearly intended, by Sections 3 and 6 thereof, to clothe the Circuit Criminal Court with all the powers vested in regular Courts of First Instance including the authority to conduct preliminary examinations and investigations, is confirmed by the Dangerous Drugs Act of 1972, otherwise known as Republic Act No. 6425, as amended by Presidential Decree No. 44, Section 39 of which confers on Circuit Criminal Courts, Courts of First Instance and Juvenile and Domestic Relations Courts concurrent original jurisdiction over all offenses punishable thereunder and expressly directs that the "preliminary investigation of cases filed under this Act shall be terminated within a period of thirty (30) days from the date of their filing." Before the amendment, the law required only seven (7) days from the date of the commencement of the preliminary investigation. Section 39, as amended, reads: 

"Sec. 39. Jurisdiction. -- The Court of First Instance, Circuit Criminal Court, and Juvenile and Domestic Relations Court shall have concurrent original jurisdiction over all cases involving offenses punishable under this Act: Provided, that in cities or provinces where there are Juvenile and Domestic Relations Courts, the said courts shall take exclusive cognizance of cases where the offenders are under sixteen years of age. 

"The preliminary investigation of cases filed under this Act shall be terminated within a period of thirty (30) days from the date of their filing. 

Page 27: Warrantless Arrests, Searches and Seizures and Privacy of Communication

27

"Where the preliminary investigation is conducted by a prosecuting officer and a prima facie case is established, the corresponding information shall be filed in court within twenty-four (24) hours from the termination of the investigation. If the preliminary investigation is conducted by a judge and a prima facie case is found to exist, the corresponding information shall be filed by the proper prosecuting officer within forty-eight (48) hours from the date of receipt of the records of the case. "Trial of the cases under this section shall be finished by the court not later than ninety (90) days from the date of the filing of the information. Decision on said cases shall be rendered within a period of fifteen (15) days from the date of submission of the case."

It is patent that the aforequoted provision of Section 39 of Republic Act No. 6425 affirms the power of the Circuit Criminal Courts to conduct preliminary examination and investigation in all the cases falling under their jurisdiction and additionally fixes the period for preliminary investigation, the filing of the information and the rendition of decisions in all offenses penalized by the Dangerous Drugs Act of 1972.

Under the amendment, the Circuit Criminal Court no longer has exclusive, but still retains concurrent, jurisdiction with the Courts of First Instance and Juvenile and Domestic Relations Courts under the Dangerous Drugs Act. Its authority to conduct preliminary examination and investigation granted under Section 6 of Republic Act No. 5179, remains intact and undiminished; because the amendatory decree expressly directs that "If the preliminary investigation is conducted by a judge and a prima facie case is found to exist, the corresponding information should be filed by the proper prosecuting officer * * *." There is nothing in the amendatory decree from which it can be reasonably inferred that since the jurisdiction of the Circuit Criminal Court over violations of the Dangerous Drugs Act is no longer exclusive, Circuit Criminal Court Judges no longer possess the authority to conduct preliminary examination and investigation.Recognizing the constitutional power of the courts, including the Courts of First Instance, to conduct preliminary examination, other special laws specifically vest such authority exclusively in the Court of First Instance in cases of violation of the Revised Election Code (Sec. 187, 1947 Revised Election Code, as amended; Sec. 234, 1971 Rev. Election Code) and of the Anti-Subversion Act when the penalty imposable for the offense is prision mayor to death (Sec. 16, Rep. Act No. 1700).It is urged that the word "judge" in the above-quoted section of Presidential Decree No. 44 (and also in the 1935 and 1973 Constitutions) contemplates not the Court of First Instance Judge nor the Circuit Criminal Court Judge but the municipal judge. As heretofore stated, it is an elementary precept in statutory construction that where the law does not distinguish, WE should not distinguish (Colgate Palmolive Philippines, Inc. vs. Gimenez, L-14787, Jan. 28, 1961, 1 SCRA 267). The statute cannot give a restricted meaning to the generic term "judge" used in the constitutional guarantee against reasonable searches and seizures.Furthermore, in People versus Manantan (L-14129, July 31, 1962, 5 SCRA 684), a justice of the peace, accused of violating Section 54 of the Revised Election Code, moved to dismiss the information on the ground that the law refers merely to a justice, judge, or fiscal and that being a justice of the peace, he is beyond the coverage of the said Code. The Supreme Court in denying such contention, held that there was no need of including justices of the peace in the enumeration in said section because the legislature had availed itself of the more generic term "judge". The term "judge", not modified by any word or phrase, is intended to comprehend all kinds of judges, including justices of the peace.The cases of People versus Paderna (22 SCRA 273) and Paraguya versus Tiro (41 SCRA 13) involved not the power of the Circuit Criminal Court to conduct preliminary investigation, but its jurisdiction to try and decide certain cases. They do not at all reveal an iota of any further restriction on the limited jurisdiction of the Circuit Criminal Court other than those delineated in existing laws.

Page 28: Warrantless Arrests, Searches and Seizures and Privacy of Communication

28

Thus, in the Paderna case, supra, involving a violation of Section 174 of the Tax Code, Mr. Chief Justice Castro, then Associate Justice, speaking for the Supreme Court in ruling that the Circuit Criminal Court was without jurisdiction to take cognizance of the case, stated: "* * * [T]he charge is for unlawful possession of untaxed 'blue seal cigarettes' of an appraised value of less than P500.00 x x x and the penalty provided under Republic Act 4713 is a fine of not less than P50.00 nor more than P200.00 and imprisonment of not less than 5 nor more than 30 days because the value of the cigarettes does not exceed P500.00, this case falls within the original and exclusive jurisdiction of the city court * * *." 

"* * * Section 1 of Republic Act 5179, which took effect on September 8, 1967, provides in part that circuit criminal courts shall have - limited jurisdiction concurrent with the regular court of first instance, to try and decide the following criminal cases falling under the original and exclusive jurisdiction of the latter. 

"* * * * * * 

"The jurisdiction of the circuit criminal courts is thus dependent not only on the type of cases but also on the penalties provided for those cases. Inasmuch as the case at bar falls within the exclusive and original jurisdiction of the City Court, it cannot, even if it involves a violation of section 174 of the Tax Code, be taken cognizance of by circuit criminal courts, the jurisdiction of which is concurrent with that of courts of first instance where the latter's jurisdiction is original and exclusive."

The same ruling was substantially reiterated in the more recent Tiro case, supra, involving indirect bribery committed by a public officer. In passing upon the issue of the Circuit Criminal Court's limited jurisdiction, the Supreme Court, through Mr. Justice Jose B.L. Reyes, held: "* * * The law (R.A. 5179) confined the jurisdiction of the circuit criminal courts (which is even made concurrent with the courts of first instance) to crimes committed by public officers; * * * only where they are falling within the original and exclusive jurisdiction of the court of first instance. In short, circuit criminal courts' jurisdiction was limited merely to cases involving crimes specifically enumerated in Section 1 of Republic Act 5179, for which the penalty prescribed by law is imprisonment for more than 3 years (or 6 years in proper cases), or fine of more than P3,000.00 (or P6,000.00 as the case may be), or both such fine and imprisonment (Sec. 44[f] in relation to Sec. 87 [c], Judiciary Act of 1948, as amended; Esperat vs. Avila, L-25922, June 30, 1967, 20 SCRA 596; Mangila vs. Lantin, L-24735, October 31, 1969; 30 SCRA 81; People vs. Tapayan, L-36885, November 28, 1969, 30 SCRA 529; Andico vs. Roan, L-26563, April 16, 1968, 23 SCRA 93)."Since indirect bribery is penalized under the Revised Penal Code with imprisonment for a period not exceeding six months, suspension and public censure (Art. 211, RPC), the case is clearly removed from the competence of the circuit criminal court to pass upon.  It is not denied that the crime of indirect bribery is essentially one committed by public officers. Jurisdiction of the court, however, is determined not only by nature of the offense charged in the information, but also by the penalty imposable thereto. * *" (italics supplied).In these two cases, it was made clear that for the Circuit Criminal Court to acquire jurisdiction, the offense must not only be one of those enumerated under Section 1 of Republic Act No. 5179; it should also be within the original and exclusive jurisdiction of the regular Courts of First Instance. In the aforesaid cases, the Circuit Criminal Court was clearly without jurisdiction to hear and decide the offenses involved, by command of the specific provisions of its charter, the Judiciary Act and the Revised Penal Code; and not by a directive of the Supreme Court, which merely applied in said cited cases the statutory prescriptions.

Page 29: Warrantless Arrests, Searches and Seizures and Privacy of Communication

29

The Supreme Court cannot legally define additional restrictions, which is the sole prerogative of the law-making authority.

The contrary view appears to entertain the mistaken notion that Section 13, Rule 112 of the Revised Rules of Court, being a rule of procedure, the same should be rendered inoperative by reason of the fact that the Supreme Court cannot, by promulgating a rule of procedure, arrogate jurisdiction unto itself or grant any to the lower courts.

It is of course basic that only the Constitution and the law can confer jurisdiction to hear and decide certain cases. But equally true is the fact that both the 1935 and 1973 Constitutions expressly delegated to the Supreme Court the rule-making authority - the power to promulgate rules of pleading, practice and procedure and to amend the existing laws thereon. The law or rule on preliminary investigation is undoubtedly a rule of procedure.

The 1935 Constitution states: 

"The Supreme Court shall have the power to promulgate rules concerning pleading, practice, and procedure in all courts, and the admission to the practice of law. Said rules shall be uniform for all courts of the same grade and shall not diminish, increase or modify, substantive rights. The existing laws on pleading, practice, and procedure are hereby repealed as statutes, and are declared Rules of Courts, subject to the power of the Supreme Court to alter and modify the same. The Congress shall have the power to repeal, alter, or supplement the rules concerning pleading, practice, and procedure, and the admission to the practice of law in the Philippines" (Sec. 13, Art. VIII, 1935 Constitution).

The 1973 Constitution similarly authorizes the Supreme Court to 

"Promulgate rules concerning pleading, practice, and procedure in all courts, the admission to the practice of law, and the integration of the Bar, which, however, may be repealed, altered, or supplemented by the National Assembly. Such rules shall provide a simplified and inexpensive procedure for the speedy disposition of cases, shall be uniform for all courts of the same grade, and shall not diminish, increase or modify substantive rights" (Sec. 5[5] , Art. X, 1973 Constitution).Sections 13 and 14 of Rule 112 of the Revised Rules of Court merely implement Section 3 of Article III of the 1935 Constitution (now Section 3 of Article IV of the 1973 Constitution). Section 13 of Rule 112 of the Revised Rules of Court was not an innovation as it merely restated Section 13 of General Order No. 58, Section 37 of Act No. 1627, and Sections 2 and 4 of Rule 108 of the 1940 Rules of Court, in obedience to its rule-making authority under Section 13, Article VIII of the 1935 Constitution. Rule 112 does not modify substantive rights but continues the procedure already operative prior to the 1935 Constitution.

WE have ruled that Rule 108 of the 1940 Rules of Court, which is the predecessor of Rule 112 of the 1964 Revised Rules of Court, is an adjective or procedural rule (Bustos vs. Lucero, 81 Phil. 640).

While admitting that Courts of First Instance were previously clothed with the power of preliminary investigation by virtue of Section 37 of Act 1627, nevertheless, it is argued that this same section was amended when the Judiciary Act of 1948 was enacted since under Section 99 of said Judiciary Act, "All laws and rules inconsistent with the provisions of this Act" were repealed. The inconsistency, it is claimed, lies in the fact that while the authority of municipal courts and city courts to conduct preliminary investigation was reiterated in said Judiciary Act, there was no mention therein whether Courts of First Instance Judges are still possessed of such authority.

Page 30: Warrantless Arrests, Searches and Seizures and Privacy of Communication

30

If such repeal was intended, it is unconstitutional; because the Constitution of 1935 and 1973 vest in the judge the power to issue a warrant of arrest or search warrant after conducting a preliminary investigation or examination. Congress could not divest the court of such authority as the Constitution does not permit it, for the constitutional guarantee on arrest or search warrant is not qualified by some such phrase as "unless otherwise provided by law." For a clearer appreciation, the Constitutional guarantee on arrest and search warrant reads: 

"(3) The rights of the people to be secure in their persons, houses, papers, and effects against unreasonablesearches and seizures shall not be violated, and no warrants shall issue but upon probable cause, to be determined by the judge after examination under oath or affirmation of the complainant and the witnesses he may produce, and particularly describing the place to be searched, and the persons or things to be seized" (Art. III, 1935 Constitution, italics supplied). "Sec. 3. The right of the people to be secure in their persons, houses, papers, and effects against unreasonablesearches and seizures of whatever nature and for any purpose shall not be violated, and no search warrant orwarrant of arrest shall issue except upon probable cause to be determined by the judge, or such other responsible officer as may be authorized by law, after examination under oath or affirmation of the complainant and the witnesses he may produce, and particularly describing the place to be searched, and the persons or things to be seized” (Art. IV, 1973 Constitution, italics supplied).It is clear from the aforequoted provisions of the 1973 Constitution that until now only the judge can determine the existence of probable cause and can issue the warrant of arrest. No law or presidential decree has been enacted or promulgated vesting the same authority in a particular "responsible officer." Hence, the 1973 Constitution, which was ratified and took effect on January 17, 1973, should govern the last four cases, namely, Nos. L-36376, L-38688, L-39525 and L-40031, which arose after January 17, 1973.

But even under the 1935 Constitution, the term seizures or seized comprehends arrest. Thus, in Vivo versus Montesa (July 29, 1968, 24 SCRA 155), reiterating the doctrines in the cases of Qua Chee Gan, et al. vs. Deportation Board (L-20280, Sept. 30, 1963) and Morano vs. Vivo (L-22196, June 30, 1967, 20 SCRA 462), WE ruled unanimously through Mr. Justice J.B.L. Reyes: "Nevertheless, we are of the opinion that the issuance of warrants of arrest by the Commissioner of Immigration, solely for purposes of investigation and before a final order of deportation is issued, conflicts with paragraph 3, Section 1, of Article III (Bill of Rights) of our Constitution, providing: 

‘3. The right of the people to be secure in their persons, houses, papers and effects against unreasonable searches and seizures shall not be violated, and no warrants shall issue but upon probable cause, to be determined by the judge after examination under oath or affirmation of the complainant and the witnesses he may produce, and particularly describing the place to be searched, and the persons or things to be seized.' 

"It will be noted that the power to determine probable cause for warrants of arrest is limited by the Philippine Constitution to judges exclusively, unlike in previous organic laws and the Federal Constitution of the United States that left undetermined which public officials could determine the existence of probable cause. And in Qua Chee Gan, et al. vs. Deportation Board, L-20280, promulgated on September 30, 1963, this Court pointed out that Executive Order No. 69, of July 29, 1947, issued by President Roxas, in prescribing the procedure for deportation of aliens, only required the filing of a bond by an alien under investigation, but did not authorize his arrest. 

Page 31: Warrantless Arrests, Searches and Seizures and Privacy of Communication

31

"Discussing the implications of the provision of our Bill of Rights on the issuance of administrative warrants of arrest, this Court said in the same case: 

'*** *** *** 

‘Under the express terms of our Constitution it is, therefore, even doubtful whether the arrest of an individual may be ordered by any authority other than the judge if the purpose is merely to determine the existence of probable cause, leading to an administrative investigation. The Constitution does not distinguish between warrants in a criminal case and administrative warrants in administrative proceedings. And if one suspected of having committed a crime is entitled to a determination of the probable cause against him, by a judge, why should one suspected of a violation of an administrative nature deserve less guarantee? Of course it is different if the order of arrest is issued to carry out a final finding of a violation, either by an executive or legislative officer or agency duly authorized for the purpose, as then the warrant is not that mentioned in the Constitution which is issuable only on probable cause. Such, for example, would be a warrant of arrest to carry out a final order of deportation, or to effect compliance of an order of contempt. 

‘The (n) contention of the Solicitor General that the arrest of a foreigner is necessary to carry into effect the power to deportation is valid only when, as already stated, there is already an order of deportation. To carry out the order of deportation, the President obviously has the power to order the arrest of the deportee. But, certainly, during the investigation, it is not indispensable that the alien be arrested. It is enough, as was true before the executive order of President Quirino, that a bond be required to insure the appearance of the alien during the investigation, as was authorized in the executive order of President Roxas.' 

"Following the same trend of thought, this Court, in Morano vs. Vivo  (L-22196, 30 June 1967, 20 SCRA 562; Phil. 1967-B, page 741), distinguished between administrative arrest in the execution of a final deportation order and arrest as preliminary to further administrative proceedings. The Court remarked in said case: ‘Section 1 (3), Article III of the Constitution, we perceive, does not require judicial intervention in the execution of a final order of deportation issued in accordance with law. The constitutional limitation contemplates an order of arrest in the exercise of judicial power as a step preliminary or incidental to prosecution or proceedings for a given offense or administrative action, not as a measure indispensable to carry out a valid decision by a competent official, such as a legal order of deportation, issued by the Commissioner of Immigration, in pursuance of a valid legislation'" (L-24576, pp. 161-162). 

The foregoing doctrine was last reiterated in Ang, et al. versus Galang, etc. (L-21426, Oct. 22, 1975).Under the American Constitution, the aforesaid terms include not only arrest but also invitations for police interview or interrogation as well as stop-and-frisk measures. In the 1968 case of Terry versus Ohio, the United States Supreme Court enunciated: "* * * It is quite plain that the Fourth Amendment governs 'seizures' of the person which do not eventuate in a trip to the station house and prosecution for crime 'arrests' in traditional terminology. It must be recognized that whenever a police officer accosts an individual and restrains his freedom to walk away, he has 'seized' that person (392 U.S. 1, 16 88 S.C.T. 1868, 20 LED. 2d 889; 903 [1968] )."

Page 32: Warrantless Arrests, Searches and Seizures and Privacy of Communication

32

That the aforesaid terms seizures and seized signify arrest was deliberately intended by the founding fathers of the 1935 Constitution, which words are likewise employed in the 1973 Constitution, Delegate Miguel Cuaderno categorically recounted: "An amendment affecting the issuance of an order of arrest and search warrant, to the effect that in each case the order must be supported by the testimony of the complainant and the witnesses he may produce, made before the judge, and also an amendment providing that prisoners charged with capital offenses shall be bailable before conviction unless the evidence of guilt is strong, were approved upon the initiative of Delegate Francisco. It was the prevailing opinion among many delegates that some courts had been rather easy in the issuance of orders of arrest or search warrants, and quite strict in the matter of bail in cases where persons had been charged with capital offenses" (Cuaderno, The Framing of the Philippine Constitution, p. 65, italics supplied).Delegate Jose Aruego added: 

"During the debates on the draft, Delegate Francisco proposed an amendment which was adopted by the Convention, the amendment being the insertion of the words, to be determined by the judge after examination under oath or affirmation of the complainant and the witnesses he may produce. The idea in the Francisco amendment was not new in the Philippines; for it was provided for in the Code of Criminal Procedure of the Philippines. The dignification of the idea into a constitutional provision was zealously insisted upon, in order to make the principle more sacred to the judges and to prosecuting officials. The amendment was intended to be a remedy for the evils pointed out in the debates, caused by the issuance of search warrants, many of which were in blank, upon mere affidavits on facts most of which were generally found afterwards to be false" (Aruego, Framing of the Philippine Constitution, Vol. I, p. 160).The term "judge" employed in both Constitutions cannot be so limited to "municipal judge" as to exclude the judges of the Court of First Instance and Circuit Criminal Court (People vs. Manantan, 5 SCRA 684, 690-695). WE are not justified to create a distinction where the Constitution does not make any.In general, "judge" is a term employed to designate a public officer selected to preside and to administer the law in a court of justice (Ark. - School Dist. No. 18 vs. Grubbs Special School Dist., 43 S.W. 2d 765, 766, 184 Ark. 863, 48 CJS 946).According to intent or context, the term "judge" may include an assistant judge (N.H. - City Bank vs. Young, 43 N.H. 457); a county or court justice (Mo. State vs. O'Gorman, 75 Mo. 370); a justice of the peace (N.Y. People vs. Mann97 N.Y. 530, 49 Am. R. 556).The term "a judge", in Gen. St. C.47, Art. 1 & 22, providing that "a judge" may cause any house or building to be searched for the protection of gambling tables, etc., is equivalent to "any judge" and comprehends an entire class, and cannot, without disturbing its meaning, be restricted in its applications to judges of county, city and police courts and therefore the judge of the Louisville Law and equity court has authority to issue a warrant for such a search (Com. vs. Watzel, 2 S.W. 123, 125, 84 KY 537).Admittedly, Section 99 of the Judiciary Act contains a repealing clause which provides: "All laws and rulesinconsistent  with the provisions of this Act are hereby repealed." The question may now be asked: What is the nature of this repealing clause? It is certainly not an express repealing clause because it fails to identify or designate the Act or Acts that are intended to be repealed (Sutherland, Statutory Construction, [1934], Vol. 1, p. 467). Rather, it is a clause which predicates the intended repeal upon the condition that a substantial and an irreconcilable conflict must be found in existing and prior Acts. Such being the case, the presumption against implied repeals and the rule against strict construction regarding implied repeals apply ex propio vigore; for repeals and amendments by implication are not favored (Jalandoni vs. Andaya, L-23894, Jan. 24, 1974, 55 SCRA 261, 265-6; Villegas vs. Subido, L-31711, Sept. 30, 1971, 41 SCRA 190; Quimseng vs. Lachica, 2 SCRA 182). Indeed, the legislature is presumed to

Page 33: Warrantless Arrests, Searches and Seizures and Privacy of Communication

33

know the existing laws; so that, if a repeal is intended, the proper step is to so express it with specificity (Continental Insurance Co. vs. Simpson, 8 F [2d] 439; Webb vs. Bailey, 151 Ore. 2188, 51 P [2d] 832; State vs. Jackson,  120 W. Va. 521, 199 S.E. 876). The failure to add a specific repealing clause indicates that the intent was not to repeal any existing law (Crawford, Construction of Statute, 1940 ed., p. 631), unless an irreconcilable inconsistency and repugnancy exist between the terms of the new and of the old statutes (Iloilo Palay and Corn Planters Association, Inc. vs. Feliciano, 13 SCRA 377). Here, there is no such inconsistency.To begin with, the two laws, although with a common objective, refer to different persons and different methods applicable under different circumstances. Thus, while Section 87 of the Judiciary Act provides that municipal judges and judges of city courts may also conduct preliminary investigation for any offense alleged to have been committed within their respective municipalities and cities * * *; Section 37 of Act 1627 reads in part that such power of "every justice of the peace including the justice of Manila, * * * shall not exclude the proper judge of the Court of First Instance * * * from exercising such jurisdiction."

WE should not, and cannot, adopt the theory of implied repeal except upon a clear and unequivocal expression of the will of Congress, which is not manifest from the language of Section 99 of the Judiciary Act, apart from the fact that Congress by itself alone had no power to amend the Constitution.

The opposite view likewise denies that the jurisdiction of our courts to conduct preliminary investigation could be traced to the Constitution, adding that the Charter of Manila and other cities confer upon the respective fiscals of said cities the power to conduct preliminary investigations.

The organic acts prior to the 1935 Constitution did not prohibit the conferment of such a power to conduct preliminary examination or investigation on quasi-judicial officers like the city fiscals of chartered cities (see the instructions of President McKinley to First Philippine Commission, the Philippine Bill of 1902, Jones Law of 1916, and the Revised Administrative Code of 1917).

But the power thus granted to the Manila City Fiscals (and later to City Fiscals and City Attorneys of other chartered cities) to conduct preliminary investigations did not and does not include the authority to issue warrants of arrest and search warrants, which warrants the courts alone can issue then as now. The constitutional guarantee against unreasonable searches and seizures under the 1935 Constitution provides that only a judge can issue a search warrant or warrant of arrest after he has by himself personally determined the existence of probable cause upon his examination under oath of the complainant and his witnesses; although as ruled in one case, he may rely on the investigation conducted by the fiscal or prosecutor (Amarga vs. Abbas, 98 Phil. 739, 741-42).It is patent that under the 1935 Constitution, only the "judge" is directed to conduct a preliminary examination for the issuance of the warrant of arrest by express constitutional conferment.

But the 1973 Constitution empowers the National Assembly to grant the power to issue search warrants or warrants of arrest after conducting the necessary preliminary examination to "other responsible officer." Until such a law is enacted by the National Assembly, only the judge can validly conduct a preliminary examination for the issuance of a warrant of arrest or search warrant.

Even when the fiscal or prosecutor conducts the preliminary investigation, only the judge can validly issue the warrant of arrest. This is confirmed by Section 6 of Rule 112 of the 1964 Revised Rules of Court, which directs the judge to issue the warrant of arrest when he is "satisfied from the preliminary examination conducted by him or by the investigating officer (referring to the fiscal or the municipal mayor

Page 34: Warrantless Arrests, Searches and Seizures and Privacy of Communication

34

under Sec. 5) that the offense complained of has been committed and that there is reasonable ground to believe that the accused has committed it, * * *."

Thus, the power of the city prosecutors to conduct preliminary examination and investigation (minus the authority to issue warrants of arrest or search warrant) is purely statutory. On the other hand, the judge derives his authority not only from the Rules of Court, but also - and originally - from the fundamental law to which all other laws are subordinate. If an objection must be raised, it should be against the authority of the fiscal to exercise such power of preliminary investigation, which, as has been stated, is merely statutory. No less than the Constitution confers upon the judge the power to conduct such examination and investigation.

The case of Albano vs. Alvarez (December 22, 1965, 15 SCRA 518) is authority for the proposition that Sec. 13 of Rule 112 of the 1964 Revised Rules of Court contains an innovation, which requires that, when the Court of First Instance itself conducts the preliminary investigation, it must not only conduct the preliminary examination proper but the preliminary investigation as well since Section 13 commands the Court of First Instance to conduct both the preliminary examination and investigation simultaneously (523-524). Said Albano case does not negate, but recognizes the authority of the judge of the Court of First Instance to conduct such preliminary investigation.It is true that this COURT held expressly and impliedly that under the charters of the cities of Manila, Bacolod and Cebu, the power to conduct preliminary investigation is exclusively lodged in the city prosecutor (Sayo vs. Chief of Police, 80 Phil. 859, 868-869, May 12, 1948; Espiritu vs. De la Rosa, 45 OG 196; Montelibano vs. Ferrer, 97 Phil. 228, June 23, 1955; and Balite vs. People,  18 SCRA 280, 285-286, Sept. 30, 1966). But the charters of the cities of Manila, Bacolod and Cebu do not contain any provision making such grant of power to city prosecutors exclusive of the courts (Kapunan, Criminal Procedure, 3rd Edition, 1960), which cannot be deprived of such authority to conduct preliminary examination because said prerogative of the courts emanates from the Constitution itself. Unless the Constitution is amended, the Judge cannot be divested of such a power, which is an essential element of the cardinal right of an individual against unreasonable searches and seizures. If the present city charters conferred on city fiscals or city prosecutors the power to issue warrants of arrest, it would be an unconstitutional grant of power under the 1935 Constitution. As heretofore intimated, the present practice or rule of court authorizing the judge to issue warrants of arrest based on the preliminary investigation conducted by the city fiscal, seems to violate the 1935 Constitution, which requires the judge himself to conduct the preliminary examination. Neither the judge nor the law can delegate such an authority to another public officer without trenching upon this constitutional guarantee against unreasonable searches and seizures.The theory that Courts of First Instance and Circuit Criminal Courts Judges cannot exercise the power of preliminary examination and investigation, and that as a necessary consequence, they cannot also issue warrants of arrest, obviously collides with the 1935 and 1973 Constitutions.

Moreover, the theory tolerates an unthinkable - because anomalous - situation wherein the Court of First Instance and the Circuit Criminal Court must wait for prosecutors and courts inferior to them to conduct the preliminary examination and/or to issue the needed warrants of arrest before they could effectively exercise their power to try and decide the cases falling under their respective jurisdiction. This situation would make the Courts of First Instance and Circuit Criminal Courts totally dependent upon state prosecutors and municipal courts, which are inferior to them, for their proper functioning. The possibility that the administration of criminal justice might stand still will not be very remote.

Page 35: Warrantless Arrests, Searches and Seizures and Privacy of Communication

35

The two-fold purpose for which the Circuit Criminal Courts were created was to alleviate the burden of the regular Courts of First Instance and accelerate the disposition of criminal cases filed therein (Osmeña vs. Secretary of Justice, supra; People vs. Gutierez, supra). Such being the admitted purpose, the power to conduct preliminary examination must necessarily attach to the duties of a Circuit Criminal Court Judge; for aside from being one of the instruments by which a case may be accelerated and disposed of, it is a duty which truly lies within the scope of the office, essential to the accomplishment of the main purpose for which the office was created (Sec. 3, Art. III, 1935 Constitution; Sec. 3, Art. IV, 1973 Constitution), even if regarded as incidental and collateral, is germane to, and serves to promote the accomplishment of the principal purpose (Lo Cham vs. Ocampo, 77 Phil. 635).WE RULE that both Section 1(3), Article III of the 1935 Constitution and Section 3, Article IV of the 1973 Constitution provide the source of the power of all Judges, including Judges of the Court of First Instance, the Circuit Criminal Courts, and other courts of equivalent rank, to conduct the examination to determine probable cause before the issuance of the warrant of arrest and therefore sustain the proceedings conducted by respondent Judge leading to the issuance of the warrants of arrest and his referral of the cases to the fiscal or other government prosecutor for the filing of the corresponding information.

IIIt may be well to trace briefly the historical background of our law on criminal procedure.

During the Spanish regime, the rules on criminal procedure were found in the Provisional Law on Criminal Procedure which accompanied the Spanish Penal Code. These two laws were published in the Official Gazette in Manila on March 13 and 14, 1887 and became effective four (4) months thereafter (U.S. vs. Tamparong, 31 Phil. 32-33; Francisco, Criminal Procedure, 1969 ed., p. 8).While the Provisional Law on Criminal Procedure provided for a preliminary summary oral trial by the justice of the peace or gobernadorcillo, it did not require any preliminary examination or investigation before trial. The sumariowas abolished by General Order No. 58 (U.W. vs. Tamparong, supra; Navarro, Criminal Procedure, 1960 ed., pp. 171, 174; Revilla, Vol. 2, Philippine Penal Code and Procedure, 1930 ed., pp. 1134-35).When the Philippines came under American Sovereignty, General Order No. 58 was promulgated by the U.S. Military Governor in the exercise of his legislative powers as commander-in-chief of the occupation army and took effect on April 13, 1900. General Order No. 58 was amended by Act No. 194 of August 10, 1901, the Philippine Bill of 1902, Act No. 590 of January 9, 1903, Act No. 1627 of July 1, 1907, the Jones Law of 1916, Section 2474 of the Revised Administrative Code of 1917, Act No. 3042 of March 10, 1922, and Act No. 4178 of December 5, 1934.

General Order No. 58 amended (Sec. 1) the Criminal Code of Procedure enforced during the Spanish regime and vested in the magistrate  "the authority to conduct preliminary investigation (Sec. 13) for the issuance of the warrant of arrest" and authorized "a judge or a justice of the peace" to issue a search warrant upon his determination of the existence of probable cause therefor "particularly describing the place to be searched and the person or thing to be seized" (Secs. 95 and 97). The term "magistrate" comprehended the Court of First Instance (Temporosa vs. Yatco, 79 Phil. 225, 226 [1947] ; Marcos vs. Cruz, 68 Phil. 96, 104-107 [1939] ; People vs. Red, 55 Phil. 706, 710 [1931] ; People vs. Solon, 47 Phil. 443, 441 [1925] ; Navarro, Criminal Procedure, 1960 ed., 1973; Padilla, Criminal Procedure, 1965 ed., p. 270). "A 'magistrate' is an 'officer having power to issue a warrant for the arrest of a person charged with a public offense.' People vs. Swain, 90 P. 720, 722, 5 Cal. App. 421, citing Pen. Code, S807. 

Page 36: Warrantless Arrests, Searches and Seizures and Privacy of Communication

36

"A 'magistrate' is an officer having power to issue a warrant for the arrest of a person charged with the commission of a crime. The following persons are magistrates: (1) the justices of the Supreme Court; (2) the judges of the Circuit Court; (3) the county judges and justices of the peace; (4) all municipal officers authorized to exercise the powers and perform the duties of a justice of the peace. Wallowa County vs. Oakes, 78 P. 892, 46 Or. 33" (26 Words and Phrases, pp. 44, 45).Act No. 194 of August 10, 1901 amended General Order No. 58 by empowering "every justice of the peace * * * to make preliminary investigation of any crime alleged to have been committed within his municipality, jurisdiction to hear and determine which is by law now vested in the judges of the Courts of First Instance" (Italics supplied).The obvious inference from the aforequoted provision of Act No. 194 is that before its passage, the justice of the peace had no power to conduct preliminary investigation of any offense triable by the Court of First Instance, which alone can conduct such preliminary investigation of a crime under its original jurisdiction pursuant to General Order No. 58. But its enactment did not divest the Court of First Instance of such authority.

In the 1939 case of Marcos, et al. vs. Cruz, the Supreme Court, through Justice Imperial, sustained the power of the Court of First Instance to conduct preliminary investigations under Sections 13 and 14 of General Order No. 58 (68 Phil. 96, 106-107), which was impliedly followed in the 1947 case of Temporosa vs. Yatco, et al., supra.While General Order No. 58 vested the authority in a magistrate, a generic term which includes judges of the Courts of First Instance and justices of the peace; Section 1 of Act No. 194 is less categorical by employing the clause "jurisdiction to hear and determine which is by law now vested in the judges of the Courts of First Instance."

The Philippine Bill of 1902 in a similar ambiguous vein contained such authority when it merely provided that the "Supreme Court and the Courts of First Instance of the Philippine Islands shall possess and exercise jurisdiction asheretofore provided and such additional jurisdiction as shall hereafter be prescribed by the Government of said Islands, subject to the power of said Government to change the practice and method of procedure.  The municipal courts of said Islands shall possess and exercise jurisdiction as heretofore provided by the Philippine Commission, subject in all matters to such alteration and amendment as maybe hereafter enacted by law; ***" (Sec. 9, italics supplied).Act No. 590 of January 9, 1903 further amended Act No. 194 by extending the power to conduct preliminary investigation to the justice of the peace of the provincial capital or of the town wherein the provincial jail is situated of crimes committed anywhere within the province but again utilized and equivocal clause "jurisdiction to hear and determine which is by law now vested in the Courts of First Instance; * * *" (Sec. 7, Act 590, italics supplied).Act No. 1627 of July 1, 1907 had the virtue of greater clarity when it authorized expressly every justice of the peace, including the justice of the peace of Manila, to "conduct preliminary investigation of all crimes and offenses alleged to have been committed within his municipality and cognizable by Courts of First Instance, but this shall not exclude the proper judge of the Court of First Instance or of a municipal court fromexercising such jurisdiction.  The justice of the peace of a capital or of a municipality in which the provincial jail is located, when directed byan order from the judge of First Instance, shall have jurisdiction to conduct investigation  at the expense of the municipality wherein the crime or offense was committed, although alleged to have been committed anywhere within the province, to issue orders of arrest, * * *" (Sec. 37, Act No. 1627, italics supplied).

Page 37: Warrantless Arrests, Searches and Seizures and Privacy of Communication

37

The Jones Law of 1916, like the Philippine Bill of 1902, merely provides "that the Supreme Court and the Courts of First Instance of the Philippine Islands shall possess and exercise jurisdiction as heretofore provided and such additional jurisdiction as shall hereafter be prescribed by law" (Sec. 26, Jones Law).

Section 2474 of the Revised Administrative Code of 1917 re-affirms the power of the Court of First Instance of Manila to conduct preliminary examination - 

"Sec. 2474.  Persons arrested to be promptly brought before a court - Preliminary examinations in municipal court and Court of FirstInstance.  - Everyperson arrested shall, without unnecessary delay, be brought before the municipal court, or the Court of First Instance forpreliminary hearing, release on bail, or trial.  In cases triable in the municipal court the defendant shall not be entitled as of right to a preliminary examination, except a summary one to enable the court to fix the bail, in any case where the prosecution announces itself ready and is ready for trial within three days, not including Sundays, after the request for an examination is presented.  In cases triable only in the Court of FirstInstance the defendant shall not be entitled as of right to a preliminary examination in any case where the fiscal of the city, after a dueinvestigation of the facts, shall have presented an information against him in proper form.  But the Court of First Instance may make suchsummary investigation into the case as it may deem necessary to enable it to fix the bail or to determine whether the offense isbailable." (Italics supplied).It is clear that both the Manila Court of First Instance and municipal court can conduct a preliminary hearing or examination.  Section 2474 aforequoted, adds, however, that the City Fiscal impliedly may conduct such preliminary examination; because it provides that in "cases triable only in the Court of First Instance the defendant shall not be entitled as of right to a preliminary examination in any case where the fiscal of the city, after a due investigation of the facts, shall have presented an information against him in proper form." It will be noted, however, that it is only after the City Fiscal has conducted a preliminary examination that the accused ceases to "be entitled as of right" to a preliminary examination by the Judge of the Court of First Instance who, however, retains inferentially the discretion to conduct another preliminary investigation because the Court of First Instance Judge is not foreclosed by the preliminary examination conducted by the City Fiscal.  But, when the City Fiscal has not conducted any preliminary examination, the Court of First Instance Judge himself certainly can proceed with such preliminary exami-nation, which the defendant can demand as a matter of right.

Act No. 3042 of March 10, 1922, while amending Section 13 of General Order No. 58, re-states the power of the magistrate to conduct the preliminary examination for the issuance of the warrant of arrest.Act No. 4178 of December 5, 1934 further amended Section 13 of General Order No. 58 but still retained the authority of the magistrate  to conduct the preliminary examination.  As heretofore stated, Sections 13 and 14 of General Order No. 58, as amended, were applied by the Supreme Court in Marcos, et al. versus Cruz (68 Phil. 96, 99, 106-107).Under the jurisprudence then or prior to the 1935 Constitution, the preliminary investigation before the justice of the peace or municipal court consisted of two stages, namely, preliminary examination for the issuance of the warrant of arrest where only the complainant and his witnesses are heard by the justice of the peace; and the second stage where the accused and his witnesses are heard.  The Judge of the Court of First Instance conducts only the first stage, that is, preliminary examination for purposes of the issuance of the warrant of arrest, to be followed by the actual trial (Marcos vs. Cruz, supra; People vs. Moreno, 77 Phil. 548, 555 [1946]).The basic source of the power of the Courts of First Instance to conduct preliminary examination or investigation from May 14, 1935 to January 17, 1973, is paragraph 3 of Section 1 of Article III of the 1935 Constitution, which guarantees "the right of the people to be secure in their persons * * * against

Page 38: Warrantless Arrests, Searches and Seizures and Privacy of Communication

38

unreasonable * * * seizures * * * and no warrants shall issue but upon probable cause, to be determined by the judge after an examination under oath or affirmation of the complainant and the witnesses he may produce, and particularly describing * * * the persons * * * to be seized." Construing the forgoing constitutional right against unreasonable searches and seizures, the Supreme Court, through then Chief Justice Ricardo Paras, pronounced that the determination of the existence of "probable cause must depend upon the judgment and discretion of the judge * * * issuing the warrant.  * * *.  His conclusion as to whether ‘probable cause' existed or not is final and conclusive.  If he is satisfied that 'probable cause' exists from the facts stated in the complaint, made upon the investigation by the prosecuting attorney, then his conclusion is sufficient upon which to issue a warrant of arrest.  He may, however, if he is not satisfied, call such witnesses as he may deem necessary before issuing the warrant.  * * *.  There is no law which prohibits him from reaching the conclusion that `probable cause' exists from the statement of the prosecuting attorney alone, or any other persons whose statement or affidavit is entitled to credit in the opinion of the judge * * *.  The preliminary investigation conducted by the petitioner (Provincial Fiscal) under Republic Act No. 732 * * * does not, as correctly contended by the respondent Judge, dispense with the latter's duty to exercise his judicial power of determining, before issuing the corresponding warrant of arrest, whether or not probable cause exists therefor.  The Constitution vests such power in the respondent judge who, however, may rely on the facts stated in the information filed after preliminary investigation by the prosecuting attorney" (Amarga vs. Abbas, March 28, 1956, 98 Phil. 739, 741-742).While the power to conduct preliminary examination may be delegated by law to government prosecutors, only the judge can issue the warrant of arrest under the 1935 Constitution and prior thereto (Sayo, et al. vs. Chief of Police, et al. 80 Phil. 859; Lino vs. Fugoso, 77 Phil. 933; Hashim vs.  Boncan, 71 Phil. 216).  The valid seizure of a person can only be executed through a lawful warrant of arrest.  Arrest without a warrant can only be legally effected by a police officer or private individual a) when the person to be arrested has committed, is actually committing, or is about to commit an offense in his presence; b) when an offense has in fact been committed, and he has reasonable ground to believe that the person to be arrested has committed it; and c) when the person to be arrested is a prisoner who has escaped from a penal establishment or place where he is serving final judgment or temporarily confined while his case is pending, or has escaped while being transferred from one confinement to another (Sec. 6, Rule 113, 1964 Revised Rules of Court).In all other cases, there must be a valid warrant of arrest.  When the seizure of a person is made without a warrant of arrest or with a warrant of arrest which is not based on a determination by the judge of the existence ofprobable cause, the arrest becomes unreasonable and therefore unconstitutional.Sections 2 and 4 of Rule 108 of the 1940 Rules of Court expressly confer on the municipal or city judge, the City Fiscal and the Judge of the Court of First Instance the power to conduct preliminary examination or investigation.

On June 20, 1957, Republic Act No. 1700, otherwise known as the Anti-Subversion Law, was approved.  The proviso of Section 5 thereof expressly provides that the preliminary investigation of offenses defined and penalized therein by prision mayor to death shall be conducted by the proper Court of First Instance.  This grant obviously is exclusive of the provincial or city fiscal or other government prosecutors whose power to conduct preliminary investigation in all other cases is affirmed in the first clause of Section 5 thereof.Sections 13 and 14 of the 1964 Revised Rules of Court re-state Sections 2 and 4 of Rule 108 of the 1940 Rules of Court.

Page 39: Warrantless Arrests, Searches and Seizures and Privacy of Communication

39

As aforestated, aside from the challenged Sections 3 and 6 of Republic Act No. 5179 creating the Circuit Criminal Courts, Republic Act 5180 was approved on September 8, 1967, which affirms the prerogative of the Courts of First Instance to conduct preliminary investigation of offenses punishable by said courts.

Presidential Decrees Nos. 77 and 911 promulgated respectively on December 6, 1972 and March 23, 1976, amending Republic Act No. 5180, did not modify the opening clause of Section 1 of said Republic Act 5180 affirming the power of the Court of First Instance to conduct preliminary investigation in accordance with law and the Rules of Court.

Section 234 of the 1971 Revised Election Code, otherwise known as Republic Act No. 6388, vests in the Court of First Instance "exclusive original jurisdiction to make preliminary investigations, issue warrants of arrest and try and decide any criminal case or proceeding for violation of" the Election Law.  This provision was a reiteration of the previous election laws (Act No. 1582 of 1907; Com. Act No. 357 of 1938; and Republic Act No. 180 of 1947, as amended).

After the ratification of the 1973 Constitution on January 17, 1973, the source of the authority of the judge to conduct preliminary examination for purposes of issuing a warrant of arrest, is still the Constitution, this time the 1973 Constitution, which likewise guarantees "the right of the people to be secure in their persons * * * against unreasonable * * * seizures for whatever nature and for any purpose * * * and no search warrant or warrant of arrest shall issue except upon probable cause to be determined by the judge, or such other responsible officer as may be authorized by law, after examination under oath or affirmation of the complainant and the witnesses he may produce, and particularly describing * * * the persons * * * to be seized" (Sec. 3 of Art. IV, 1973 Constitution).  The 1973 Constitution, instead of employing the generic term warrants to comprehend both search warrants and warrants of arrest, as did the 1935 Constitution, expressly specifies "search warrants or warrants of arrest." The purpose of such specification was apparently to clarify the doubt raised by the dissenting opinion of Mr. Justice Montemayor in the Amarga case, supra, that the 1935 Constitution merely guarantees against unreasonable searches but not against unreasonable arrests, despite the fact that the constitutional guarantee expressly affirms "the right of the people to be secure in their persons * * * against unreasonable * * * seizures * * * and no warrant shall issue but upon probable cause, to be determined by the judge * * * particularly describing * * * the persons * * * to be seized" (Par. 3, Sec. 1, Art. III, 1935 Constitution).In passing, the dissent of Justice Montemayor in the Amarga case seems to deny equal, if not greater, importance to individual freedom from illegal arrest or arbitrary detention vis-a-vis property rights and right against self-incrimination.  It will also likewise be noted that the 1973 Constitution also authorizes the law-making authority to empower other responsible officers to conduct such preliminary examination for purposes of the issuance of a warrant of arrest.  As enunciated in the Amarga case and in U.S. versus Ocampo (18 Phil. 1, 41-42), the government prosecutors may be authorized to conduct such preliminary examination and their determination of the existence of probable cause may be relied upon by the judge, who may, as a consequence, issue the warrant of arrest; although the judge himself is not precluded from conducting his own preliminary examination despite the conclusion of the prosecuting attorney as to the existence or non-existence of probable cause.

III1.  The challenged order of July 6, 1971 issued by the respondent Judge in G.R. No. L-34038 (Collector of Customs, etc. vs. Hon. Onofre Villaluz, et al.) dismissed the criminal complaint filed by petitioners therein against private respondent with prejudice, obviously meaning that the case may not be re-filed

Page 40: Warrantless Arrests, Searches and Seizures and Privacy of Communication

40

without exposing the accused to double jeopardy.  The respondent Judge seriously erred in so issuing said order, contravening as it does a basic legal principle on double jeopardy, and committing thereby a grave abuse of discretion.  The constitutional right against double jeopardy exists, not after the first preliminary examination of investigation, but only after the first trial which results either in conviction or acquittal or in the dismissal or termination of the case without the express consent of the accused by a court of competent jurisdiction upon a valid complaint or information and after the accused had pleaded to the charge (Sec. 9, Rule 117, Revised Rules of Court; Taladuavs. Ochotorena, et al., L-25595, February 15, 1974; Republic vs. Agoncillo, L-27257, August 31, 1971; 40 SCRA 579; People vs. Obsania, L-24447, June 29, 1968, 23 SCRA 1249; People vs. Ylagan, 58 Phil. 851).As correctly stated by the Solicitor General, petitioner's counsel, "dismissal at preliminary investigation is never with prejudice.  Re-filing of the same is allowed if evidence has become sufficient to warrant conviction of private respondent." There has been no deviation from such established jurisprudence exemplified in People vs.Bagsican (6 SCRA 400), wherein the Court held that "the finding in the preliminaryinvestigation that no prima facie case existed against the accused does not bar subsequent prosecution and conviction.  - Such finding isnot final acquittal as would preclude further proceedings" (italics supplied).2.  Aggravating his grave mistake and misapprehension of the law, respondent Judge also directed through the same order the return of the articles allegedly seized from the person of respondent Makapugay.  This portion of the questioned order is fraught with undesirable consequences.

As stated heretofore, the dismissal of a case, even with prejudice, during the stage of preliminary investigation does not bar subsequent prosecution and conviction if the evidence warrants the re-filing of the same.  But with the challenged order commanding the return of the articles subject matter of the complaint, the re-filing of the same becomes next to impossible.  For the enforcement of such order would virtually deprive herein petitioner Collector of Customs of the evidence indispensable to a successful prosecution of the case against the private respondent.  Worse, the order nullified the power of seizure of the customs official.

Respondent Judge ignored the established principle that from the moment imported goods are actually in the possession or control of the Customs authorities, even if no warrant of seizure had previously been issued by the Collector of Customs in connection with seizure and forfeiture proceedings, the Bureau of Customs acquires exclusive jurisdiction over such imported goods for the purpose of enforcing the Customs laws, subject to an appeal only to the Court of Tax Appeals and to final review by the Supreme Court (Sections 2205 and 2303, Tariff and Customs Code; Papa, et al. vs. Mago, et al., Feb. 28, 1968, 22 SCRA 857; Virata, et al. vs. Aquino, et al., Sept. 30, 1973, 53 SCRA, 24; see also Vierneza vs. Commissioner, July 30, 1968, 24 SCRA 394; Farm Implement & Machinery vs. Commissioner, August 30, 1968, 24 SCRA 905; Lazatin vs. Commissioner, et al., July 30, 1969, 28 SCRA 1016; Asaali, et al. vs. Commissioner, December 16, 1968, 26 SCRA 382; Sare Enterprises vs. Commissioner, Aug. 28, 1969, 29 SCRA 112; Geotina, etc. vs. Court of Tax Appeals, et al., August 30, 1971, 40 SCRA 362; Commissioner vs. Court of Tax Appeals, et al., January 31, 1972; Lopez vs. Commissioner, et al., January 30, 1971, 37 SCRA 327; Geotina vs. Broadway, etc., et al., January 30, 1971, 37 SCRA 410; Auyong Hianvs. Court of Tax Appeals, et al., September 12, 1974, 59 SCRA 110; and Pacis, et al. vs. Pamaran, etc., et al., March 15, 1974, 56 SCRA 16).  Such exclusive jurisdiction precludes the Court of First Instance as well as the Circuit Criminal Court from assuming cognizance of the subject matter (Enrile, et al., vs. Venuya, et al., January 30, 1971, 37 SCRA 381) and divests such courts of the prerogative to replevin properties subject to seizure and forfeiture proceedings for violation of the Tariff and Customs Code (Diosamito, et al. vs. Balanque, et al., July 28, 1969, 28 SCRA 836; Señeris vs. Frias,

Page 41: Warrantless Arrests, Searches and Seizures and Privacy of Communication

41

June 10, 1971, 39 SCRA 533); because proceedings for the forfeiture of goods illegally imported are not criminal in nature since they do not result in the conviction of the wrongdoer nor in the imposition upon him of a penalty (Lazatin vs. Commissioner, et al., July 30, 1969, 28 SCRA 1016).Respondent Judge claims that the pendency of a seizure proceeding was never brought to his attention (p. 038, rec.) and that he could not have foreseen the possibility that petitioner would be instituting seizure proceedings * * * and besides, it is understood that the order of the court commanding the release of the subject articles was on a premise that herein petitioner was not holding or withholding the same for some other lawful reasons (p. 039, rec.).

The questioned order of respondent Judge is unqualified and contains no intimation that the "release * * * was on a premise that herein petitioner was not holding or withholding the same for some other lawful reason." On the contrary, the tenor of the order is so absolute and so emphatic that it really leaves no alternative for petitioner Collector of Customs except to return the articles.

The records of the case, moreover, reveal that a report of seizure (p. 14, rec.) and a warrant of seizure and detention  (p. 15, rec.) were made by petitioner Collector of Customs on June 30, 1971 and on July 9, 1971 respectively.  It is patent that respondent Judge knew actually of the existence at least of the report of seizure of June 30, 1971, which is six days prior to his order of dismissal dated July 6, 1971.  He should have anticipated that a warrant of seizure and detention will logically be issued as in fact it was issued on July 9, 1971, because it was the petitioner Collector of Customs who filed the criminal complaint directly with him on July 1, 1971.  Respondent Judge chose to ignore the presence of the report of seizure dated June 30, 1971, six days before his order of dismissal and the filing of the criminal complaint on July 1, 1971.  Prudence should have counselled him, so as not to frustrate the petitioner Collector of Customs in enforcing the tariff and customs laws, against ordering the release of the seized articles without first ascertaining from the petitioner Collector of Customs whether the latter intended to institute or had instituted seizure proceedings.As aptly expressed by Mr. Justice Barredo in his Concurring Opinion in People vs. Gutierez, supra, "It is not enough that a judge trusts himself or can be trusted as capable of acting in good faith, it is equally important that no circumstance attendant to the proceedings should mar that quality of trustworthiness." We have enjoined judges to apply the law as interpreted by the Supreme Court and not to dispose of a case according to their personal views (Albert vs. Court of First Instance, 23 SCRA 948).

IVIn G.R. No. L-36376 (Enriquez, et al. vs. Hon. Onofre Villaluz, et al.), the arbitrary denials displayed by respondent Judge of motions presented before him likewise invite some cautionary reminders from this Court.In this case, petitioners were given an unreasonable period of one (1) day within which to elevate the matter before this Tribunal.  But considering the novelty of the issue, a grant of twenty-four hours to prepare a petition for certiorari is a virtual denial of the motion.  And petitioners' motion for an extension of at least one (1) day was peremptorily brushed aside by respondent Judge with one single word - DENIED.

The fact that petitioners succeeded in bringing the matter before the Supreme Court within the constricted period of time granted them is beside the point.  More important is the consideration by this Court of the dangers posed by respondent Judge's peremptory denial of a reasonable time.

Indeed, it is commendable to see judges hasten the disposition of cases pending before them.  But more commendable would be for judges to contribute their share in maintaining the unswerving faith of litigants

Page 42: Warrantless Arrests, Searches and Seizures and Privacy of Communication

42

in the courts of justice.  WE once again stress that "One important judicial norm is that a judge's official conduct should be free from appearance of impropriety" (Luque vs. Kayanan, 29 SCRA 165).

VBut while we sustain the power of the Circuit Criminal Courts to conduct preliminary examination (p. 36), pursuant to OUR constitutional power of administrative supervision over all courts (Sec. 6, Art. X, 1973 Constitution) as a matter of policy, WE enjoin the respondent Judge and other Circuit Criminal Court Judges to concentrate on hearing and deciding criminal cases filed before their courts (see Mateo vs. Villaluz, 50 SCRA 18, 28-29, March 31, 1973).  The primary purpose of the creation of the Circuit Criminal Courts in addition to the existing Courts of First Instance, as above intimated, is to mitigate the case load of the Courts of First Instance as well as to expedite the disposition of criminal cases involving serious offenses specified in Section 1 of Republic Act 5179, as amended.  Circuit Criminal Judges therefore, should not encumber themselves with the preliminary examination and investigation of criminal complaints, which they should refer to the municipal judge or provincial or city fiscal, who in turn can utilize the assistance of the state prosecutor to conduct such preliminary examination and investigation.  Or the Judge of the Circuit Criminal Court can directly request the Secretary of Justice to assign a state prosecutor for the same purpose (Sec. 3, Republic Act No. 5184).Moreover, it seems that respondent Judge does not have adequate time to hear and dispose of the 34 criminal cases with detention prisoners pending in his sala, aside from the 479 pending cases of voluntary submission by drug addicts, as of January 31, 1975) (A.M. No. 230-CCC, Item 42, Agenda of March 31, 1975), as revealed by his letter dated February 26, 1975, wherein he requested the Supreme Court to renew the temporary detail in his sala of Municipal Judge Hermenegildo C. Cruz of Mandaluyong, Rizal, to assist him.  This significant fact should further dissuade him from actively conducting the preliminary investigation of criminal cases directly filed with him.

Furthermore, Judges of the Circuit Criminal Courts whose dockets permit, may be assigned by the Supreme Court for a period not exceeding 6 months, unless with their consent, to assist Judges of regular Courts of First Instance with clogged dockets (Sec. 5 [3], Art. X, 1973 Constitution).

WHEREFORE , IN G.R. NOS. L-34243, 36376, 38688 AND 39525, THE PETITIONS ARE HEREBY DISMISSED AND THE WRITS OF PRELIMINARY INJUNCTION AND/OR RESTRAINING ORDERS ISSUED THEREIN ARE HEREBY LIFTED; IN G.R. NO. L-40031, THE PETITION IS HEREBY DISMISSED; AND IN G.R. NO. L34038, THE ORDER OF RESPONDENT JUDGE DATED JULY 6, 1971 IS HEREBY SET ASIDE AS NULL AND VOID INSOFAR AS THE SAME DISMISSED THE CRIMINAL CASE WITH PREJUDICE AND INSOFAR AS THE SAME DIRECTED THE RETURN TO PRIVATE RESPONDENT THEREIN OF THE ARTICLES SEIZED FROM HIM WHICH ARE NOW SUBJECT OF SEIZURE PROCEEDINGS BEFORE THE CUSTOMS AUTHORITIES, AND THE WRIT OF PRELIMINARY INJUNCTION ISSUED THEREIN IS HEREBY MADE PERMANENT. NO COSTS.Castro, C.J., Teehankee, Antonio, Esguerra, Muñoz Palma, Aquino, and Martin, JJ., concur.Fernando, J., concurs and submits a brief opinion.Barredo, J., concurs in a separate opinion.

CONCURRING OPINIONFERNANDO, J.:The opinion of the Court, both thorough and comprehensive, penned by Justice Makasiar, is impressive for its analytical skill and scholarly attributes. On the whole then, especially so where reference is made to our previous decisions, there is no impediment to full concurrence. This is particularly true where it concerns the ruling announced by this Court, i.e., "that both Section 1(3), Article III of the 1935

Page 43: Warrantless Arrests, Searches and Seizures and Privacy of Communication

43

Constitution and Section 3, Article IV of the 1973 Constitution provide the source of the power of all Judges, including Judges of the Court of First Instance, the Circuit Criminal Courts, and other courts of equivalent rank, to conduct the examination to determine probable cause before the issuance of the warrant of arrest and therefore sustain the proceedings conducted by respondent Judge leading to the issuance of the warrants of arrest and his referral of the cases to the fiscal or other government prosecutor for the filing of the corresponding information."[1] At that, there is still need, it seems to me, for a few words not only to set forth the extent of my agreement with my brethren but also to indicate what for me are the precise limits of our holding. The full and exhaustive treatment of the specific issue dealing with the power of the circuit criminal courts to conduct preliminary examination, with historical and textual allusions to the previous judicial pronouncements and comparable statutory provisions, certainly a virtue to be commended, may for those not sufficiently discerning, yield implications which, for me, go further than is intended by us. It is my understanding then that the decision reached is at most an affirmation that the present Constitution, as did the 1935 Constitution, confers the power to conduct preliminary examination preparatory to issuing a warrant of arrest, to a circuit criminal court judge. Even then, however, he should for sound policy reasons curb any eagerness or propensity to make use of such competence.1. To repeat, it is solely the first stage in the criminal process that may lead to the apprehension of the accused that has been passed upon by this Court. It has not considered the second stage, that of preliminary investigation proper, one of equal significance. As far back as 1910, its importance was stressed in United States vs. Grant and Kennedy.[2]  Thus: "The object or purpose of a preliminary investigation, or a previous inquiry of some kind, before an accused person is placed upon trial, is to secure the innocent against hasty, malicious, and oppressive prosecutions, and to protect him from an open and public accusation of crime, from the trouble, expense, and anxiety of a public trial, and also to protect the State from useless and expensive trials."[3]  It is of the essence then that the accused should be heard. There are overtones in the opinion of the Court susceptible to being misinterpreted in this regard, if it be assumed that upon the termination of the preliminary examination the arraignment and trial could then proceed. I would dissociate myself from such a view. I am gratified therefore that it is made explicit therein that our ruling is limited to the power of a judge under the Circuit Criminal Court Act [4]  to conduct a preliminary examination. As to his competence regarding a preliminary investigation, it is my understanding that the question has been left open.2. Respondent Judge was likewise admonished "to concentrate on hearing and deciding criminal cases filed before their courts (see Mateo vs. Villaluz, 50 SCRA 18, 28-29, March 31, 1973)."[5]  That is as it should be. It is well that it is so. The occasion for its exercise should be minimized. That is the teaching of Mateo v. Villaluz, the same respondent Judge in these petitions. The facts could be differentiated, but the principle announced holds true. The load to be shouldered by a trial judge is heavy enough for him to attend to matters which could be looked after by municipal judges. So this excerpt from Mateo would indicate: ‘To avoid any further controversies of this nature, lower court judges are well-advised to limit themselves to the task of adjudication and to leave to others the role of notarizing declarations. The less an occupant of the bench fritters away his time and energy in tasks [that could be left to other hands], the less the danger of his being a participant in any event that might lend itself to the interpretation that his impartiality has been compromised. There is much to be said for displaying zeal and eagerness in stamping out criminality, but that role is hardly fit for a judge who must bide his time until the case is before him. He must ever be on guard lest what is done by him, even from the best of motives, may be thought of as eroding that objectivity and sobriety which are the hallmarks of judicial conduct. Thus should he attend to the performance of the sacred trust that is his."[6]  For me, the fact that a judge had listened to

Page 44: Warrantless Arrests, Searches and Seizures and Privacy of Communication

44

testimony damaging to a prospective accused, without his being given the opportunity to refute the same, may lead to a subconscious prejudice difficult to erase at the stage of trial.

[1] Opinion, I (36).[2] 18 Phil. 122.[3] Ibid, 147. The United States vs. Grant decision was cited with approval in United States vs. Laban, 21 Phil. 297 (1912); United States vs. Carlos, 21 Phil. 553 (1911); United States vs. Go Chanco, 23 Phil. 641 (1912); United States vs. Ipil, 27 Phil. 530 (1914); United States vs. Remegio, 37 Phil. 599 (1918); United States vs. Alabot, 38 Phil. 698 (1918); Uy Kheytin vs. Villareal, 42 Phil. 886 (1920); People vs. Solon, 47 Phil. 443 (1925); People vs. Villegas, 55 Phil. 567 (1931); People vs. Cariñgan, 61 Phil. 416 (1935); People vs. Castillo, 76 Phil. 72 (1946);People vs. Dizon, 76 Phil. 265 (1946); People vs. Zapanta, 79 Phil. 308 (1947); Sayo vs. Chief of Police of Manila,80 Phil. 859 (1948); Bustos vs. Lucero, 81 Phil. 640 (1948); Lozada vs. Hernandez, 92 Phil. 1051 (1953);Rodriguez vs. Arellano, 96 Phil. 954 (1955); Santos, Jr. vs. Flores, L-18251, Aug. 31, 1952, 5 SCRA 1136;Molinyawe vs. Flores, L-18256, Aug. 31, 1962, 5 SCRA 1137; People vs. Figueroa, L-24273, April 30, 1969, 27 SCRA 1239; Sausi vs. Querubin, L-24122, Jan. 29, 1975, 62 SCRA 154.[4] Republic Act No. 5179 (1967).[5] Opinion, V.[6] L-34756-59, March 31, 1973, 50 SCRA 18, 28-29.

CONCURRINGBARREDO,J.:I concur in the result of the judgment in these cases, for although the main opinion sustains the authority of Circuit Criminal Courts to conduct preliminary investigations, it strictly ordains, however, that "as a matter of policy (sic)We enjoin the respondent Judge and other Circuit Criminal Court Judges to concentrate on hearing and deciding criminal cases filed before their courts." With such an imperious mandate, I am satisfied that Circuit Criminal Courts will not anymore do what I am fully convinced they are not legally permitted to do. I am certain no Criminal Court Judge will dare deviate from the "policy" announced in the main opinion, which, of course, I say is the policy of Republic Act 5179 itself. Indeed, my uncompromising position is that it is the policy of the law itself, rather than that of this Court alone as the main opinion would seem to imply, that Circuit Criminal Courts should strictly confine themselves to merely trying and deciding the cases assigned to them, and I have always insisted that it should be on the basis of that very policy of the law itself informed in public interest that this Court should construe the statutory provision here in issue, Section 1 of Republic Act 5179 which provides as follows: "In each of the sixteen judicial districts for the Court of First Instance as presently constituted, there is hereby created a Circuit Criminal Court with limited jurisdiction, concurrent with the regular Court of First Instance, to try and decide the following criminal cases falling under the original and exclusive jurisdiction of the latter: 

'a. Crimes committed by public officers, crimes against persons and crimes against property as defined and penalized under the Revised Penal Code, whether simple or complexed with other crimes; 

'b. Violations of Republic Act No. 3019, otherwise known as the Anti-Graft and Corrupt Practices Act, x x x; 

' c. Violations of Sections 3601, 3602 and 3604 of the Tariff and Customs Code and Sections 174, 175 and 345 of the National Internal Revenue Code.'" 

Page 45: Warrantless Arrests, Searches and Seizures and Privacy of Communication

45

Thus, the judgment of the Court in these cases will after all effectively effectuate what I maintain is the spirit of the Act, notwithstanding the considerations predicating the main opinion which, with due respect to my learned brethren in majority, I find it impossible to agree with. And so, I can give my assent to the judgment in these cases without my having to sacrifice my conviction regarding the question of statutory construction herein involved, which I am explaining in this separate opinion. Frankly, I will never be able to comprehend why the majority can give the above provision a construction contrary to what plainly appears to be policy that underlies it, only for them to just the same "enjoin" all Circuit Criminal Courts "as a matter of policy," (of the Court) that they should not conduct preliminary investigations, which I say the statute, as a matter of policy, never intended to allow them to do anyway.Notwithstanding the scholarly and extended main opinion. I am not persuaded that the legislature ever intended to confer upon Circuit Criminal Courts the power to conduct preliminary investigations. Not only the specific words of the above provision, but the development of the law on preliminary investigations and the circumstances obtaining at the Republic Act 5 179 was enacted point unmistakably, in my considered opinion, to this conclusion.

There are already two earlier cases in which this Court had to dwell on the extent of the jurisdiction of the circuit criminal courts. In both of them, the approach was restrictive. Way back in 1968, in the case of People vs. Paderna, 22 SCRA 273, the Court was confronted with the question of whether or not the mere fact that under Section 1 (c) of Republic Act 5179, the organic act of the circuit criminal courts, mentions violations of Section 174 of the National Internal Revenue Code to be among the cases under the jurisdiction of said courts, is enough justification for disregarding the penalty provided in the Revenue Code of fine of not less than P50 nor more than £200 and imprisonment of not less than 5 nor more than 30 days when the value of the cigarettes involved does not exceed £500, which ordinarily would make such violation fall within the original jurisdiction of the City Court of La Carlota City and considering such violations to be within the jurisdiction of the corresponding Circuit Criminal Court. The Court, thru Justice Fred Ruiz Castro, resolved the problem this wise: "The jurisdiction of the circuit criminal courts is thus dependent not only on the type of cases but also on the penalties provided for those cases. Inasmuch as the case at bar falls within the exclusive and original jurisdiction of the city court, it cannot, even if it involves a violation of Section 174 of the Tax Code, be taken cognizance of by the circuit criminal courts the jurisdiction of which is concurrent with that of courts of first instance in criminal cases where the latter's jurisdiction is original and exclusive." (At p. 279.)

Almost four years later, in Paraguya vs. Tiro, 41 SCRA 137, the issue was whether or not a case of indirect bribery, a crime committed by a public officer included in Section l(a) of the Act, but punishable under Article 211 of the Revised Penal Code with arresto mayor, suspension and public censure, penalties which are imposable by the city of municipal courts concurrently with the courts of first instance, may be considered as within the jurisdiction of the Circuit Criminal Courts. We held that the fact alone that the crime involved was one committed by a public officer did not suffice to place the case within the jurisdiction of said courts. Reiterating the predicate of adherence to the letter of the statute adopted in Parenda, supra, Justice J.B.L. Reyes, emphasized the reason therefor thus: "In fact, the intention of the legislature to bestow unto these special criminal courts limited jurisdiction is clear not only from the provision of the law itself; it was so stated that this limited jurisdiction of the circuit courts would enable them to act with dispatch on the cases cognizable by said tribunals. And, this is precisely the purpose for which the circuit criminal courts were created - — to contribute to the speedy resolution of criminal cases and help curb the progression of criminality in the country (Explanatory Note to Senate Bill No. 388, which became Republic Act No. 5179)" (At p. 142.)

Page 46: Warrantless Arrests, Searches and Seizures and Privacy of Communication

46

In the cases at bar, it is admitted in the main opinion that because "the primary purpose of the creation of the Circuit Criminal Courts in addition to the existing Courts of First Instance, as above intimated, is to expedite the disposition of criminal cases involving serious offenses specified in Section 1 of Republic Act 5179, . . . Circuit Criminal Judges, therefore, should not encumber themselves with attending to the preliminary examination and investigation of criminal complaints, which they should refer to the Provincial or City Fiscals, who, in turn can utilize the assistance of the state prosecutor to conduct the requisite preliminary examination and investigation. Or the Judge of the Circuit Criminal Court can directly request the Secretary of Justice to assign a state prosecutor for the same purpose." What is more, as if to predicate such observations on actuality and project them in the context of what is happening in the very court of respondent judge, the main opinion invites attention to the number of pending cases and matters therein which compelled respondent judge, according to the opinion, to seek from this Court the detail of a municipal judge to assist him. It further points out that under Section 5(3) Article X of the Constitution, Criminal Court Judges may be temporarily assigned by the Supreme Court to other stations, provided that, without the consent of the judges concerned, such assignment may not last longer than six months.[1] And to these very apt observations, it may be added that unlike in the regular courts of first instance, in circuit criminal courts "the trial of cases ... once commenced, shall be continuous until terminated and the judgment shall be rendered within thirty days from the time the case is submitted for decision." (Sec. 6, R.A. 5179).To my mind, all these considerations were precisely what the Congress had in mind when it enacted the law creating the circuit criminal courts. As may be seen, all of these considerations point to the necessity of freeing the said courts from all functions other than "to try and decide" the cases enumerated in the Act. It is inconceivable that with said considerations in view, Congress could have meant by omitting mention of preliminary investigations in the statute that it should nevertheless be construed in the sense of "encumbering", to borrow the language of the main opinion, the circuit criminal courts with the burden of "attending to preliminary examination and investigation of criminal complaints," which the main opinion emphasizes and the legislature must be presumed to have known can be better performed by the multitudinous other offices in the prosecution staff government already referred to above.

It is important to note that the conferment in the Judiciary Act of jurisdiction upon the regular courts of first instance is worded thus: 

"SECTION 44. Original jurisdiction. — Courts of First Instance shall have original jurisdiction: (a) In all civil actions in which the subject of the litigation is not capable of pecuniary estimation;  (b) In all civil actions which involve the title to or possession of real property, or any interest therein, or the legality of any tax, impost or assessment, except actions of forcible entry into and detainer on lands or buildings, original jurisdiction of which is conferred by this Act upon city and municipal courts; 

(c) In all cases in which the demand, exclusive of interest, or the value of property in controversy, amounts to more than ten thousand pesos; (RA Nos. 2613 & 3828.) 

(d) In all actions in admiralty and maritime jurisdiction, irrespective of the value of the property in controversy or the amount of the demand; 

(e) In all matters of probate, both of testate and intestate estates, appointment of guardians, (See also Section 90, and note thereof.) trustees and receivers, and in all actions for annulment of marriage, and in all such special cases and proceedings as are not otherwise provided for; 

Page 47: Warrantless Arrests, Searches and Seizures and Privacy of Communication

47

(f) In all criminal cases in which the penalty provided by law is imprisonment for more than six months, or a fine of more than two hundred pesos; 

(g) Over all crimes and offenses committed on the high seas or beyond the jurisdiction of any country, or within any of the navigable waters of the Philippines, on board a ship or watercraft of any kind registered or licensed in the Philippines in accordance with the laws thereof. The jurisdiction herein conferred may be exercised by the Court of First Instance in any province into which the ship or watercraft upon which the crime or offense was committed shall come after the commission thereof: Provided, That the court first lawfully taking cognizance thereof shall have jurisdiction of the same to the exclusion of all other courts in the Philippines, and 

(h) Said court and their judges, or any of them, shall have the power to issue writ of injunction, mandamus, certiorari, prohibition quo warranto and habeas corpus in their respective provinces districts, in the manner provided in the Rules of Court." Significantly, unlike Section 1 ofRepublic Act 5179, this provision does not say that the Courts of First Instance shall "try and decide" the cases therein enumerated. Rather, it simply says they shall have original jurisdiction "in" and "over" the respective cases mentioned. In other words, Section 1 of Republic Act 5179 does not grant the circuit criminal courts jurisdiction "in" or "over" the cases listed, but, as may be plainly seen in the above-quoted tenor of its pertinent provision, only "the limited jurisdiction ... to try and decide" them. To my mind, this difference in phraseology must have been intentional in order to emphasize the restricted and limited prerogatives of Circuit Criminal Courts, not only as to the nature of the cases that can be filed with them but also as to the extent of their functions and powers relative to said cases.

I maintain that consonant with the need to make of the Circuit Criminal Courts the courts of special and limited jurisdiction designed to attend with utmost expeditiousness to the cases assigned to them, as undoubtedly the law intends them to be, Section 1 of the Act should be construed, even in case of doubt, in the sense not only that the jurisdiction of said courts is limited to the cases which they may take cognizance of, but also in that any other work not strictly part of the functions to "try and decide" said cases, is not contemplated to be performed by them.

It is a familiar rule that the jurisdiction of a court, may not be deemed granted by mere implication, unless perhaps in instances when this is indubitably clear. Whenever there is reason to doubt, as in the case before Us, precisely because of the considerations expounded in the main opinion as to why, as a matter or Court's policy, at least, the circuit criminal courts should retrain from holding preliminary investigations the rule, as ' kn ° .J is to deny the existence of power. In this connection, it sh0 be borne in mind that the power to conduct preliminary 'investigations has never been deemed as a mere incidental prerogative of any court. It exists only when duly granted.

It should be noted from the provisions of Section 44 of the Judiciary Act aforequoted that even the authority of the regular courts of first instance to grant writs of injunction, mandamus, certiorari, prohibition, quo warranto andhabeas corpus, which by their nature could reasonably be deemed inferable from the grant of general jurisdiction, had still to be granted expressly to said courts, and only within their respective provinces and districts. And this Court has been very restrictive in construing this particular grant of jurisdiction. (See Director of Forestry vs. Ruiz,L-24882, April 30, 1971, 38 SCRA 559, and cases therein cited.) To repeat, such authority would seem to be implicit from the grant of general jurisdiction, and yet We always insist that it should be specifically

Page 48: Warrantless Arrests, Searches and Seizures and Privacy of Communication

48

conferred. Now, is there anything in the conduct of preliminary investigations that makes it more inherent or inseparable from the expressed power "to try and decide" that necessarily, We must consider the same as included in said power or as something that must indispensably be added thereto, such that the authority therefor need not be spelled out in black and white? Withal, if in the case of inferior courts, which everyone knows have always conducted preliminary investigations since the enactment of Act 194 in 1901, the Judiciary Act had to expressly provide for the grant of such authority to them, what special reason is there why the conferment upon circuit criminal courts of the faculty to "try and decide" certain types of criminal cases should be deemed as necessarily including the authority to conduct preliminary investigations related thereto, when according to what the main opinion emphasizes, such function can be better performed by the prosecution staff of the government?The main opinion holds that it "is plain and evident from Sections 3 and 6 of their organic act, Republic Act 5179" that circuit criminal courts "have the same powers and functions as those conferred upon regular Courts of First Instance necessary to effectively exercise (their) special and limited jurisdiction." But I am afraid this reasoning ignores that "the powers and Actions (of) the regular Courts of First Instance" conferred upon the circuit criminal courts are only those "necessary (for them) to effectively exercise (their) special and limited jurisdiction " and the issue precisely is what is the extent of that special and limited jurisdiction. As I have already pointed out, that "special and limited jurisdiction" is "try and decide" the cases enumerated and this power does not have to be accompanied, whether by logical implication or by the reasons behind the organization of the courts, by the authority to conduct preliminary investigations. I dare say, in connection with the provisions of Section 3 of the Act, that the provisions of laws and Rules of Court, if any, granting jurisdiction to regular courts of first instance to conduct preliminary investigations are inconsistent with the provisions of the Act, considering that these latter provisions contemplate circuit criminal courts which should not undertake the functions of conducting preliminary investigations, as found factually by the main opinion, albeit surprisingly the majority would give weight to such factual finding only to serve as basis for a policy only of the Court, instead of utilizing the same as premise for the proper construction of the Act in order that such policy may be legally effectuated, since it is indeed the policy underlying the law itself. And besides, a careful reading of Section 3 should make it clear to everyone that its phraseology studiously refers not to all the powers of the judges of the Courts of First Instance, but only to "the provisions of the laws and the Rules of Court relative to the Judges of the Courts of First Instance," meaning their qualifications, salaries, transfer etc. and to their powers and prerogatives in "the trial, and disposition and appeal of criminal cases" in the circuit criminal courts, which is plainly consistent with the scope of the power granted to them under Section 1 ""to try and decide."The main opinion quotes from my concurring opinion in People vs. Gutierrez,   36 SCRA 172, apparently to show that in my view, "circuit criminal courts are nothing but additional branches of the regular Courts of First Instance in their respective districts . But the portion quoted from my opinion is not complete, wna I said was this: "I take it that under Republic Act 5179, Circuit Criminal Courts are nothing but additional branches of the regular Courts of First Instance in their respective districts with the limited concurrent jurisdiction to take cognizance of, try and decide only those cases enumerated in Section 1 of the Act. This is readily implied from Section 3 of the Act which says: 

"'SECTION 3. The provisions of all laws and the Rules of Court relative to the judges of the Courts of First Instance and the trial, disposition and appeal of criminal cases therein shall be applicable to the circuit judge and the cases cognizable by them insofar as they are not inconsistent with the provisions of this Act.' 

Page 49: Warrantless Arrests, Searches and Seizures and Privacy of Communication

49

"xxx xxx xxx"

In other words, I adhered closely to the language of the statute and referred to the jurisdiction of the criminal courts as comprising of the power "to take cognizance of, try and decide" only the cases therein enumerated. I did not concede that the authority was broadly "over" those cases, as in Section 44 of Judiciary Act, but strictly "to take cognizance of, try and decide" them.

There is another point which is more transcendental. The main opinion assumes the correctness of the generally prevailing impression that courts of first instance continue to possess the jurisdiction to conduct preliminary investigations. It cites the Rules of Court as the source of such authority. For my part, 1 am not sure, to put it mildly, that the Supreme Court can arrogate jurisdiction unto itself or grant any to the lower courts by merely  promulgating a rule to such effect. I believe it is safer to hold that jurisdiction to act on any given matter may be granted only by statute or legislative enactment, for the simple reason that jurisdiction is substantive and not adjective in nature. And so, the question in my mind is simply this, assuming arguendo that circuit criminal courts have all the powers of the regular courts of first instance, which I dispute, is it clear that the latter courts  continued to possess, after the Judiciary Act of 1948 went into effect, the power to conduct preliminary investigations? In other words, are the provisions of the Rules of Court invoked in the main opinion, Section 13 of Rule 112, predicated on any law or statute?According to former Chief Justice Moran, this section was "taken, with amendments, from Section 4 of former Rule 108, which was a substantial re-statement of the ruling of the Supreme Court in one case," citing People vs. Solon, supra. (See 4 Moran, Rules of Court,   p. 117, 1970 ed.) But Sec. 4 of Rule 108 was part of the Rules of Court of 1940, when Act 1627 was still in force. Apparently, when Rule 108 was revised in the 1964 Rules, it was overlooked that under Section 99 of the Judiciary Act, "all laws and rules inconsistent with the provisions of this Act" was repealed thereby wiping away Section 37 of Act 1627.No matter how many times one may read the provisions of the whole Judiciary Act of 1948 and particularly those that refer to the jurisdiction of the Courts of First Instance, one will never find any word therein that directly or indirectly confers upon said courts the authority to conduct preliminary investigations. In pointing out this patent omission, I am of course assuming that the jurisdiction to conduct preliminary investigations, while sometimes given to courts in spite of its being basically an executive function per Orendain,[2] is not inherent in every court.For instance, in the Judiciary Act itself, it can be clearly seen that as in the case of Act 194, seventy-five years ago, by Section 87 of the Act, the legislature had to expressly vest upon inferior courts the power to conduct such preliminary investigations. Thus, Section 87 provides in unmistakable terms:

xxx xxx xxx 

"Said municipal judges and judges of city courts may also conduct preliminary investigation for any offense alleged to have been committed within their respective municipalities and cities which are cognizable by Courts of First Instance and the information filed with their courts without regard to the limits of punishment, and may release, or commit and bind over any person charged with such offense to secure his appearance before the proper court."

xxx xxx xxx

If, as the majority maintain, the power to conduct preliminary investigation is vested in all our courts by the Bill of Rights in the Constitution, of what need is there for the provision just quoted? Upon the other hand,

Page 50: Warrantless Arrests, Searches and Seizures and Privacy of Communication

50

if such conferment is merely confirmatory of an existing constitutionally based authority, I see no reason at all why there should be such an express confirmation of the power of inferior courts alone and none at all of that of the Courts of First Instance.

My position is that the silence of the pertinent provisions of the Judiciary Act on the matter, taken together with the fact that Section 99 of the Act repeals all laws and rules inconsistent with the provisions of this Act, indicates an unmistakable legislative intention to remove from the Courts of First Instance the prerogative under discussion.

It is argued that to thus argue is to rely on repeal by implication which is not favored. I contend, however, that such pose overlooks the fact that the Judiciary Act of 1948 is indisputably in the nature of a codification of all laws existing at the time of its passage related to the judiciary, the judges, the courts and their respective jurisdictions. Such being the case, the applicable rule of statutory construction is that to the effect that when scattered statutes and provisions relative to the same subject matter are embodied subsequently in a single comprehensive legislation, any particular provision incorporated therein and germane to the main subject matter is deemed to be repeated. (Sutherland Statutory Construction, Vol. 1, Sec. 2019, pp. 480-481.) Which is but logical, as otherwise, of what use is the integration?The main opinion points to certain legislations subsequent to' 1948 which it contends constitute recognition on the part of Congress of the continued authority of Courts of First Instance to conduct preliminary investigations, such as, the Dangerous Drugs Act of 1972 or Republic Act 6425, and Republic Act 5180 governing preliminary investigations by fiscals.

As regards Republic Act 5180, the main opinion claims that because Section I thereof makes mention of "investigation . . . conducted by a Court of First Instance ... in accordance with law," said provision is proof of a legislative assumption that said courts can exercise such power. To start with, I have never denied that there are instances when by specific provision of the pertinent laws, preliminary investigations in prosecutions under said laws have to be done by the Courts of First Instance, such as, in violations of the Election Law, the Anti-Subversion Act, Republic Act 1700 and the Dangerous Drugs Act, as amended by Presidential Decree No. 44.

But as I see it, if Republic Act 5180 is of any materiality in this discussion, it is in that it makes more patent that the policy of the law on preliminary investigations is to make them as expeditious as possible but without depriving the accused of the opportunity to be heard, which is likely to happen in a preliminary investigation in a Court of First Instance, following Solon[3] and Marcos,[4] unless, of course, the procedure provided for in Section 13, Rule 112 is followed pursuant to Albano vs. Arranz.[5]  It sounds to me rather anachronistic for a law to emphasize the right of an accused to be heard before he is arrested, while it perpetuates in the same breadth as a general rule a procedure which denies that right. Besides, why did not Republic Act 5180 which was approved on the same day as Republic Act 5179, mention preliminary investigation by Circuit Criminal Courts, just as the other later law, Republic Act 6425, cited in the main opinion expressly treated and referred to said courts separately from the Courts of First Instance and Domestic Relations Courts, if really Congress intended to confer the power in issue on them?The reference to Republic Act 6425 is even more revealing of the insistence of the majority to cling to any drifting straw in their effort to prove their point. Republic Act 6425 originally granted to the Circuit Criminal Courts exclusively jurisdiction over cases for violation thereof. Of course, it also contained provisions about preliminary investigations, but these did not in any manner indicate whether expressly or impliedly that the same courts would have authority to conduct such investigations. Here is the pertinent provision, before it was amended by Presidential Decree No. 44: 

Page 51: Warrantless Arrests, Searches and Seizures and Privacy of Communication

51

"SECTION 39. Jurisdiction of the Circuit Criminal Court. — The Circuit Criminal Court shall have exclusive original jurisdiction over all cases involving offenses punishable under this Act.The preliminary investigation of cases filed under this Act shall be resolved within a period of seven (7) days from the date of termination of the preliminary investigation. Where aprimafacie case is established, the corresponding information shall be filed in court within twenty-four (24) hours. Decision on said cases shall be rendered within a period of fifteen (15) days from the date of submission of the case."It is to be noted that there is here a requirement that the corresponding information should be filed in court within 24 hours. Does not this show that the preliminary investigation is not to be conducted by the court itself? But, as if to make it more patent that it is better that the investigation is undertaken by another authority, Presidential Decree 44 amended the above provision as follows: "SECTION 39. Jurisdiction. — The Court of First Instance, Circuit Criminal Court, and Juvenile and Domestic Relations Court shall have concurrent original jurisdiction over ail cases involving offenses punishable under this Act: Provided, That in cities or provinces where there are Juvenile and Domestic Relations Courts, the said courts shall take exclusive cognizance of cases where the offenders are under sixteen years of age.The preliminary investigation of cases filed under this Act shall be terminated within a period of thirty (30) days from the date of their filing.

Where the preliminary investigation is conducted by a prosecuting officer and a prima facie case is established, the corresponding information shall be filed in court within twenty-four (24) hours from the termination of the investigation. If the preliminary investigation is conducted by a judge and a. prima facie case is found to exist, the corresponding information shall be filed by the proper prosecuting officer within forty-eight (48) hours from the date of receipt of the records of the case.Trial of the cases under this section shall be finished by the court not later than ninety (90) days from the date of the tiling of the information. Decision on said cases shall be rendered within a period of fifteen (15) days from the date of submission of the case."

That the foregoing provision does not vest any preliminary investigation authority in any of the courts mentioned is best proven by the fact that the Juvenile and Domestic Relations Courts have never conducted any preliminary investigation whether under its original charter or under this provision. I am not aware that any Court of First Instance has ever done so. The mention of "the preliminary investigation (being) conducted by a judge" in the above provision contemplates, to my mind, not the judges of the courts specified therein, but the proper municipal judges, bearing in mind the considerations already discussed above relative to the tendency of the every new law to remove from superior courts the power to conduct preliminary investigations. Indeed, in this connection, it is to me a mystery how easily my brethren have forgotten that when in another case the very same respondent judge here did nothing more than act as the officer before whom the accused swore a confession which the said accused later on repudiated as having been secured thru violence and intimidation, We disqualified respondent from trying the case for fear that he might not be able to maintain "the cold neutrality of an impartial judge." Quite inconsistently, they now hold that the law in question allows a judge to conduct the preliminary examination of the witnesses of the prosecution to issue a warrant of arrest and to subsequently try the main case on the merits, even if the language of said law in issue is not really clear and the existence of the pretended power is just being gathered from inference of doubtful logic, while, on the other hand, there is a multitude of reasons strongly justifying the contrary construction.In what I consider, with the pardon I hope of my learned colleagues, to be a desperate but vain effort to provide substantive law basis for Section 13 of Rule 112, the main opinion falls back on of all things the provision of the Bill of Rights of the Constitutions of 1935 and 1973 enjoining that no warrant (of arrest)

Page 52: Warrantless Arrests, Searches and Seizures and Privacy of Communication

52

"may issue but upon probable cause, to be determined by the judge[6]  after examination under oath or affirmation of the complainant and the witnesses he may produce." It is posited that this constitutional mandate is the ultimate source of the authority of the Courts of First Instance, assumingthe absence of any statutory basis, to conduct preliminary investigation. As I understand it, the theory is that under the Constitution, warrants of arrests may be issued only by judges (under the 1935 Constitution), and since before doing so, they must examine the complainant and his witnesses under oath, ergo, judges, and I presume that would mean all judges, are constitutionally vested with jurisdiction to conduct preliminary examinations, if not investigations. But as I will demonstrate anon, I sense some kind of non sequitur here. At this point, however, I will just make the observation that if it were true that all judges may conduct preliminary examinations by virtue of the above provision of the Bill of Rights, why did the majority have to go thru all the trouble of a lengthy and laborious, if scholarly, desertation of why Circuit Criminal Courts have all the powers of the Courts of First Instance to prove that they can like the latter courts conduct preliminary examinations, when all they had to say is that Circuit Criminal Court Judges are among the judges the Constitution contemplates. Moreover, if the theory of the majority is to be pursued to its logical conclusion, then the jurisdiction of judges in the matter in issue cannot but be exclusive, for the Constitution mentions no other officer who may issue warrants of arrest. But then the question would arise, from where did our municipal mayors derive their authority under existing rules to perform such function?I have carefully perused with deep interest the elaborate statement in the main opinion of the "historical background of our law on criminal procedure." I regret to state, however, that even after such a very refreshing intellectual excursion, I still cannot see that such historical background traced by my scholarly brethren necessarily leads to the conclusion that the power of our courts to conduct preliminary investigation springs from the Constitution or that after the Judiciary Act of 1948 repealed all laws and rules inconsistent with its provisions, the statutory authority of Courts of First Instance to conduct preliminary examinations and investigations still continued to exist. Quite to the contrary, my reading of the history of the law on preliminary investigations in this jurisdiction indicates that this Court has been consistently holding that the right to a preliminary investigation is not a constitutional right, at least in so far as the so-called second stage thereof is concerned. In Marcos vs. Cruz.68 Phil. 96, this Court unanimously held: "In this jurisdiction, the preliminary investigation in criminal cases is not a creation of the Constitution; its origin is statutory   and the right thereto can be invoked when so established and granted by law." (at p. 104) According to the same decision, it is only when there is a statute granting such right and slill it is denied to the accused in spite of his demand therefor that there is a violation of the due process clause of the Constitution. More authoritatively, in my opinion, in Hashim vs. Boncan, 71 Phil. 216, no less than Justice Laurel took occasion to say: "Viewed in the light of fundamental principles, the right to preliminary investigation is statutory, not constitutional." (at p. 225.)Of course, I am not overlooking the fact that seemingly what the main opinion contends to be constitutionally based is the power of judges to issue warrants of arrest, which corresponds only to the first stage of the prosecution known as preliminary examination, and for this reason, it is maintained the purported ruling can stand together with the Marcos and Hashim doctrines which relate to the second stage known as the preliminary investigation. 1 do not see it that way.

My understanding of the Bill of Rights provision pertinent to this discussion, which reads thus: 

"Sec. 3. The right of the people to be secure in their persons, houses, papers, and effects against unreasonable searches and seizures of whatever nature and for any purpose shall not be violated, and no search warrant or warrant of arrest shall issue except upon probable cause to be determined by the judge, or such other responsible officer as may be authorized by law, after examination under oath or

Page 53: Warrantless Arrests, Searches and Seizures and Privacy of Communication

53

affirmation of the complainant and the witnesses he may produce, and particularly describing the place to be searched, and the persons or things to be seized." (Art. IV, 1973 Constitution.)

is that it is a prohibition against anyjudge issuing a warrant of arrest without complying with the requirements set forth therein. In fact, an arrest may even be made without a warrant, and it is only when a warrant is needed that the judge who is to issue the same is constitutionally bound to adhere to the conditions therein laid down. Literally, the provision does not refer to all judges, but only to "the judge" who will issue the warrant and that to me is presumably only the judge who by statute is authorized to act in the premises. In fine, the Constitution does not vest uponjust anyjudge, much less upon all judges, jurisdiction to issue warrants of arrests; it merely limits and lays down conditions before anyjudge authorized by law to issue warrants may do so. In like manner, it cannot be argued that because Section 4( 1) of the Bill of Rights provides that privacy of communication and correspondence shall be inviolable except upon lawful order of the court, just any court in the Philippines, even a municipal court can grant such authority or that because the liberty of abode and of travel shall not be impaired except upon lawful order of the court, according to Section 5, also of the Bill, it follows that all courts in the Philippines may act in the premises, regardless of the definition and allocation of jurisdiction by the National Assembly or the legislature, who, after all is constitutionally endowed with authority to precisely make such allocation. (Sec. 1, Art. X, 1973 Constitution.) Indeed, this provision which reads thus: "SECTION l.The Judicial power shall be vested in one Supreme Court and in such inferior courts as may be established by law. The National Assembly shall have the power to define, prescribe, and apportion the jurisdiction of the various courts, but may not deprive the Supreme Court of its jurisdiction over cases enumerated in Section five hereof."

readily connotes that except in the case of the jurisdiction of the Supreme Court, it is not the Constitution but the statutes that are the sources of the jurisdiction of all the various courts of the country.

Moreover, to my mind, the development of the law on preliminary investigations in this jurisdiction evinces a clear tendency not only to give the accused in all such investigations the opportunity to be present, to cross-examine the witnesses of the prosecution and to present his own evidence, until lately when the right to cross-examine was eliminated by Presidential Decree 77 as amended by Presidential Decree 911. but also (2) to transfer the function of conducting preliminary investigations, sans the power to issue warrants of arrest, to prosecuting officers belonging to the Executive Department to which the prerogative to prosecute or not to prosecute properly belongs in the exercise of the President's duty to see to it that the laws are properly executed. (Estrella vs. Orenclain, 37 SCRA 640.) Of course, in special cases wherein it is required by what in the legislature's judgment is the public interest, the particular statute concerned expressly provides that the preliminary investigation be conducted by the Court of First Instance, such as, in cases of violation of Election Code and cases of violations of the Anti-Subversion Law (Act 1700). Indeed, with the broad control given to the Secretary of Justice over crime prosecution by Presidential Decree 911, not to mention Our own ruling in Estrella recognizing his power of supervision and control over fiscals, as long as the case has not passed to the jurisdiction of the court, it does not sound realistic and in keeping with the trend of recent developments in the pertinent laws to further allow prosecutions to be initiated in the Courts of First Instance.[7]

At this juncture. I would like to address myself to the separate concurring opinion of Mr. Justice Fernando, whose specialization in matters of constitutional law has won recognition not only for him but also for our country from no less than the organizers of the constitutional aspect of the bicentennial celebration of the American. I do not mind saying that whenever I want to be comprehensive in my study of constitutional issues, I always find his views illuminating. But on the point now in controversy, I find it difficult to see his

Page 54: Warrantless Arrests, Searches and Seizures and Privacy of Communication

54

point. Thus, he particularly underlines his conformity to the ruling in the main opinion that the 1935 as well as the 3973 Constitution "provide the source of the power of all Judges, including Judges of the Courts of First Instance, the Circuit Criminal Courts, and other courts of equivalent rank to determine probable cause before the issuance of arrest and therefore sustain the proceedings conducted by respondent Judge leading to the issuance of the warrants of arrest and his referral of the cases to the fiscal or other government prosecutor for the filing of the corresponding information," not without hastening to clarify, however, that "it is (his) understanding . . . that the decision reached is at most an affirmation that the present Constitution, as did the 1935 Constitution, confers the power to conduct (the) preliminary examination preparatory to issuing a warrant of arrest, to a circuit criminal court judge" and that "it is only the first stage in the criminal process that may lead to the apprehension of the accused that has been passed upon by the Court," such that as to the judges' competence regarding a preliminary investigation," or "the second stage, (the) preliminary investigation proper," . . . "that question has been left open."Of course, that such specifically was what the members of the Court were made to understand during the deliberations by the distinguished writer of the main opinion is clear and distinct in my recollection. As a matter of fact, for a moment I entertained earnestly the thought that I could probably join my brethren in the formulation of such a ruling. I had in mind then Mr. Justice Fernando's pose in his book on the Bill of Rights (1970 Edition) that the significance of entrusting the responsibility of determining the existence of probable cause exclusively to judiciary (under the 1935 Constitution) in the defense of freedom cannot be overestimated, (p. 177) But after trying very hard to see it his way, I have to confess, I have arrived at the conclusion that such a proposition cannot stand close scrutiny, if only because even if none but judges may issue warrants of arrest, it is not indispensable that all judges be vested with such power, so it is really up to the legislature to determine which court or judge should be endowed with it.My knowledge, if limited, of the origin of the competence of judges to issue warrants of arrest is to the effect that it is a prerogative that antedates both the 1935 and the 1973 Constitutions. It was in fact recognized by the American military occupation authorities from the very inception of their rule over the Philippine Islands in 1901, as evidenced by General Orders No. 58, our first code of criminal procedure of American vintage. Surely, such military order cannot in any sense be deemed to be a mandate of constitutional stature. No doubt, Section 13 of Rule 112 appears to be a mere reiteration, if with substantial modifications, of similar provisions of General Orders No. 58 and Section 37 of Act 1627, but I regret I cannot accept the hypothesis emphasized in the main opinion that because said provision of the rules is supposed to be an implementation of the Bill of Rights provision against unreasonable searches and seizures, We must perforce conclude that the Bill of Rights is the source of the jurisdiction of the judges to act in the manner provided in said rule. There can be no dispute about the imperative need to make the safeguards against unreasonable arrests, searches and seizures as air tight as possible, but it is equally undeniable that giving the power to determine the existence of probable cause exclusively to judges is not the only guarantee that can ensure that end. Not only fiscals but even municipal mayors have for decades exercised said power in this jurisdiction, and instead of condemning such practice, this Court has expressly sanctioned the same as being conducive to a more efficient system of prosecution of offenses. (See Hashim vs. Boncan, supra.) What is more, the 1973 Constitution has given the practice explicit constitutional basis by providing that probable cause may also be determined by "such other responsible officer (not necessarily a judge) as may be authorized by law."More than ever before, I now hold that the Bill of Rights provision under discussion has not been designed to confer the power to determine probable cause to every judge in the courts of the Philippines; rather said provision lays down the conditions and limitations which the particular judges authorized by law to perform such function must observe. I feel I am supported in this view by the following excerpts

Page 55: Warrantless Arrests, Searches and Seizures and Privacy of Communication

55

from the records of the Constitutional Convention of 1934 containing the apt observations of no less than Senator Vicente J, Francisco and Justice Jose P. Laurel: 

"EL PRESIDENTS. Tiene la palabra el Delegado por Cavite. EL SR. FRANCISCO RAZONA SV ENMIENDA SR. FRANCISCO. Senor Presidente y Caballeros de la Convencion: bajo el proyecto del Comite de 7, se puede expedir, mandamientos de registro, con tal de que la peticion vaya acompanada de un affidavit en el que aparezcan hechos y circunstancias que demuestren causas probables. Bajo mi enmienda, un juez puede expedir un mandamiento de registro   sino solo despues de haber examinado at denunciante y a sus testigos bajo juramento. Parece ser que la diferencia es grande. El texto en ingles del projecto dice: 

'xxx and no warrants shall issue but upon probable cause, supported by oath or affirmation and particularly describing the place to be searched, and the persons or things to be seized.' 

Esta expresion ha sido interpretadapor los tribunates de America en el sentido de que el juez tiene dos medios: o puede tomar en cuenta para la expedicion de un mandamiento de registro un affidavit en el que consten hechos y demuestren la causa probable, o mediante examen del denunciante. Someto a la consideracion de esta Asamblea que es completamente peligroso permitir que un juez expida mandamiento de registro, atendiendoseexclusivamenie a lo que consta en un affidavit. Esta idea de que se puede expedir mandamiento de registro mediante affidavit, o sea, solamente mediante un documento jurado en el que aparezcan hechos probables, no ha sido acepfada por esta jurisdiction, no solamente en la practica sino tambien por la orden general Num. 58. Esta convencion, creo, se habra jijado que en dicha Orden General Num. 58 esta disposicion que aparece en el proyecto del Comite de 7 que es una reproduccion o copia del precepto que aparece en el bill de Filipinos y luego en la Ley Jones, aparece reproducida, como ya he dicho, en la Orden General Num. 58, coma articulo 2 7. Este articulo 2 7, dice lo siguiente: 'No se expedir a   mandamiento de registro sino por causa probable y por falta de peticion apoyada por juramento'. Como ya he dicho, 'peticion apoyada por juramento 'puede ser testimonio del festigo o affidavit. Considerandose, sin embargo, que estos es verdaderamente peligroso para el derecho que tiene un individuo a la seguridad de sus bienes y papeles, nuestro mismo Codigo de Procedimiento Civil inserta en su Articulo 28 una disposicion que exige como requisito 'sine quanon' el que el Juez no pueda expedir mandamiento de registro sino mediante el examen de testigos, especialmente del denunciante. Este articulo viene a ser el Articulo 28 del Codigo de Procedimiento Civil que dice lo siguiente: 'El Juez de Primera Instancia o el Juez de paz debera, antes de expedir el mandamiento, examinara bajo juramento al denunciante o al testigo presente, consignando sus declaraciones por escrito.' De modo que mi enmienda es a tenor o en consonancia con esta disposicion legal. Como ya he dicho, si mantuvieramos el precepto del proyecto de constitucion, esta disposicion de la Orden General Num. 58 podra en cierto modo ser contradictoria al precepto del proyecto de constitucion, y nosotros sabemos muy bien que si se aprobara una constitucion en la forma como esta el precepto, cuya enmienda pido, y si encontrara una discrepancia sustancial entre dicho precepto y el Codigo de Procedimiento Civil, creo que este ultimo tendria que quedarse derogado, o al menos no puede mantenerse este precepto por anti-constitucional. Pero yo creo que ninguno de los miembros de esta Asamblea vera que mi enmienda no responde a una razon fundamentaly a una necesidad que se ha sentido en la practica. Los abogados que estamos en el ejercicio de la profesion hemos visto muchas veces casos en que agent es secretos consiguen mandamientos de registro solamente mediante la presentacion de un affidavit que reune los requisitos de la Ley. Pero que expedido el registro e impugnados despues los terminos del affidavit se descubre que los hechos que aparecen en el mismo son completamente falsos. De ahi que si queremos salvaguardar en todo lo posible el derecho de unindividuo a arrestos o registros arbitrarios; si queremos que el derecho

Page 56: Warrantless Arrests, Searches and Seizures and Privacy of Communication

56

de! individuo a la seguridad de sus bienes o papeles este rodeado de todas las garantias que puedan impedir o que   impidan la expedicion de registros inmotivados o infundados que pueden dar lugar a molestias o vejaciones injustas e irreparables, creo que debemos hacer que en nuestra constitucion se consigne el precepto tal como yo propongo que se enmienda. " (Pp. 750- 752, Vol. III.) "EL PRESIDENTE. El Delegado por Batangas (Senor Laurel) acepta la enmienda?I cannot close this separate opinion without inviting attention to certain specific points of procedure which the main opinion seems not to have bothered to pass upon, notwithstanding what I consider to be their importance. In G .R. No. L-34038,1 notice that respondent judge conducted a preliminary investigation on the basis of nothing more than a letter-complaint of the petitioner Collector of Customs. It is not stated whether or not it was in due form or under oath. While as Mr. Justice Fernando stresses, this decision recognizes only the power of respondent judge to conduct the first stage or the preliminary examination, in G.R. No L--34038, L-34243, L- 39525 and L-40031, what are actually involved are preliminary investigations, both the first and second stages. It is only in G.R. Nos. L-36376 and L-38688 that respondent judge has not been able to conduct even the preliminary examination. Frankly, I am at a loss as to how the dispositive portion of Our judgment is to be understood, considering that the consensus among the members of the Court, as attested to by Mr. Justice Fernando, reaches only preliminary examinations and not preliminary investigations, in order precisely to avoid having to overrule Hashim vs. Boncan and Marcos vs. Cruz, which I understand some members of the Court are not ready to do.Regarding G.R. No. L-34038,1 agree that respondent judge exceeded his authority in providing that his order of dismissal is with prejudice and in ordering the return of the articles seized by the customs authorities to his co-respondent Makapugay. Of course, anyway, in legal contemplation the qualification "with prejudice" thus made by respondent judge means nothing. In no way can it have the effect of jeopardy, since what was conducted by him was only a preliminary investigation, which in my opinion is unauthorized and void. And assuming it to be valid, there would be no need of setting aside the order of dismissal itself; it is enough to say that it is a dismissal before arraignment and jeopardy has not thereby attached, the express qualification therein of "with prejudice" notwithstanding.

IN VIEW OF ALL THE FOREGOING, I vote to grant the petitions in G.R. No. L-34038 insofar as the respondent judge's impugned order of July 6, 1971 orders the return of the articles seized to his co-respondent Makapugay, and insofar as G.R. Nos. L-34243, L-36376, L-39525, L-38688 and L-40031 are concerned, I am giving my concurrence to the judgment therein subject to the qualifications I have discussed in this separate opinion.SR. LAUREL. No, senor Presidente, y quisiera dec'tr dos palabras.MR. LAUREL. Mr. President and Gentlemen of the Convention: The anomalies pointed out by the Honorable gentleman from Cavite, Mr. Francisco, if they ever occur at the present time, it is because of the irregularities committed by some justices. The amendment introduced by the distinguished Delegate from Cavite is already covered by existing legislation, and if those irregularities pointed out by him really occurred, it is because some justices have not enforced and adhered to the specific provision of the General Order. The General Order, or the Code of Criminal Procedure, now provides that the judge, before issuing a search warrant, must examine the complainant and his witnesses and that he must take their depositions in writing. The reason why we are in favor of this amendment is because we are incorporating in our constitution something of a fundamental character. Now, before a judge could issue a search warrant, he must be under the obligation to examine personally under oath the complainant and if he has any witness, the witnesses that he may produce. It is not necessary for me to recall here one of the grievances of the early settlers in America which was one of the causes of the revolution against the mother country, England; the issuing of the so-called general search warrant. It is, therefore, quite important that we impose this obligation upon the judge, so that he will not be issuing search warrant in

Page 57: Warrantless Arrests, Searches and Seizures and Privacy of Communication

57

blank, or simply accompanied by affidavits, but that he must consider the sanctity of the home. It is necessary that we surround that power with the necessary constitutional guaranty. You might say that as this amendment is already in the general legislation, what is the necessity of incorporating this in the constitution. The necessity consists in that the constitution is something permanent for the protection of the individual citizen. It is proper that we incorporate that provision rather than general legislation in this constitution that we shall adopt. For this reason, the committee accepts and approves of the amendment as suggested by Delegate Francisco" (Pp. 757-758, Vol. III.)

And so, since there is neither any constitutional provision nor statute that presently confers on Judges of the Courts of First Instance the power to conduct preliminary examinations, and the trend of our laws is to leave such function to other responsible officers, except the very act of issuing the warrant of arrest, I have no alternative but to deny to Circuit Criminal Courts such power.

[A.M. No. RTJ-01-1639.  November 29, 2002]

CONCERNED CITIZEN OF MADDELA and JUDITH B. ERMITANIO, complainants, vs. JUDGE MA. THERESA DELA TORRE-YADAO, Regional Trial Court, Branch 38, Maddela, Quirino, respondent.

[A.M. No. 00-9-427-RTC.  November 29, 2002]

JUDITH ERMITANIO, complainant, vs. JUDGE MA. THERESA DELA TORRE-YADAO, Regional Trial Court, Branch 38, Maddela, Quirino, respondent.

D E C I S I O N

SANDOVAL-GUTIERREZ, J.:

The present administrative cases stemmed from two letter-complaints dated February 23, 2000 and March 1, 2000 filed respectively by a “concerned citizen of Maddela” and Judith B. Ermitanio against Judge Ma. Theresa dela Torre-Yadao,[1] Regional Trial Court (RTC), Branch 38, Maddela, Quirino.

In the anonymous letter-complaint, respondent judge is being denounced for: (1) sleeping with a female employee in her chambers; (2) collecting gasoline allowance from politicians; (3) directing her court personnel to render services in her house at Quezon City; (4) buying narra logs from a litigant and storing them in her chambers; (5) bringing to her house books, furnishings and equipment intended for office use; and (6) reporting at her sala only three days a month.

In her letter-complaint, Judith Ermitanio alleged that despite the filing on March 18, 1999 of the Information in Criminal Case No. 38-034 for murder involving the death of her husband, respondent judge failed to issue the corresponding warrant of arrest against accused Michael Badangngayon and Peter Guinannoy.  It was only on March 7, 2000, or after one (1) year, when respondent judge came to know that an administrative case had been filed against her, that she issued a warrant of arrest dated March 26, 1999.

Page 58: Warrantless Arrests, Searches and Seizures and Privacy of Communication

58

In her comment on the anonymous letter, respondent judge denied the allegations therein for being false and without basis.  She stated that the writer is actually Marilou Cabanatan, a court stenographer in her sala.  Respondent judge explained that she sent Marilou memoranda and show-cause letters for her habitual absenteeism and tardiness, disobedience and neglect of duty.  Obviously, the latter was motivated by ill will in resorting to an anonymous letter-complaint.

On the complaint of Judith Ermitanio, respondent judge explained that after the filing of the Information in Criminal Case No. 38-034, or on March 26, 1999, she issued the corresponding warrant of arrest against the accused, a copy of which was received by the Maddela Police Station on the same day.  Considering that the accused could not be apprehended, she issued an order on March 2, 2000 directing that the case be archived and that an alias warrant of arrest be issued.   On March 8, 2000, the PNP of Maddela received a copy of the alias warrant.  On March 15, 2000, respondent judge received a request for the issuance of an alias warrant of arrest from the Maddela Police Station, but she only noted it in view of the March 2, 2000 order.

Due to the seriousness of the allegations in the letter-complaints against respondent judge, the Office of the Court Administrator referred the matter to the National Bureau of Investigation (NBI) for a discreet investigation.

In his report dated September 18, 2000, NBI Special Investigator Gerard L. Butale stated that after conducting an investigation, he found that there is reason to believe that respondent judge committed the acts complained of.   He, therefore, recommended that an administrative complaint for serious misconduct and gross inefficiency be filed against her.

In a Resolution dated July 11, 2001, this Court referred the case to Justice Eloy R. Bello, Jr. of the Court of Appeals for investigation, report and recommendation.

In his Report and Recommendation, Justice Bello stated that no evidence was presented to support the allegations in the anonymous letter-complaint dated February 23, 2000 against respondent judge.  Hence, his Report and Recommendation deals mainly with the complaint of Judith Ermitanio, thus:

“Amidst the conflicting claims of the opposing parties and after a careful consideration and scrutiny of the evidence, particularly the testimonial evidence presented by both parties, this Court finds more reasons to believe the version of the complainant.

“If indeed, a warrant of arrest had been issued and was served on the PNP Maddela on the date being claimed by the respondent judge, the complainant would not have filed the present administrative case in the first place, since her only complaint is the non-issuance of a warrant of arrest in the case of her husband.  If a warrant of arrest had been issued as early as 26 March 1999 and was received by the PNP Maddela on the same date, why was not the complainant told about the said warrant during the times she went to the court to follow up whether a warrant of arrest regarding her husband’s case had already been issued?  The testimony of Crisostomo Molina, one of the witnesses for the respondent judge, that the complainant never went to the court to follow up the case is not at all believable.  That the complainant has been following up the case since April 1999 until March 2000 has been corroborated by the testimony of all the other witnesses for the complainant.  Moreover, the fact that the complainant was able to write to Mayor Fred Lim and even went as far as the Supreme Court would show how persistent complainant is in fighting for the cause of her late husband.  It would be highly improbable for the complainant not to have gone to the court to follod

“It has also been established that the complainant had been following up the warrant with the PNP Maddela and the Fiscal’s Office.  If a warrant had already been issued and served on the PNP Maddela, why did the latter seem not to know about the said warrant when the complainant went to their office to follow up the case?   If a warrant of arrest has already been issued and delivered to the PNP Maddela as early as 26 March 1999, we do not find any reason or motive on the part of the PNP Maddela, to hide the fact of the issuance of the said warrant from the complainant.  Also, why was not Fiscal Orias informed about the said warrant when he also went to the court to follow up the case and why did he not see a copy of the said warrant in the records of the case when he inspected the said records?

Page 59: Warrantless Arrests, Searches and Seizures and Privacy of Communication

59

“The logical conclusion that could be drawn from all these is that, maybe, there was no warrant of arrest issued on the date being claimed by the respondent judge, or a warrant of arrest had been issued by the respondent judge on the date being claimed by her, but the same was not timely served on the PNP Maddela.

“The document being presented by the respondent judge, purporting to be the warrant of arrest dated 26 March 1999, and which was received by the Maddela Police on the same date, deserves scant consideration since the due execution of the same is being refuted by the testimonies of the witnesses for the complainant.

“Although the respondent judge enjoys the presumption of regularity in the performance of her official duty, this presumption is not, however, conclusive.  It is only a disputable presumption, meaning, it is satisfactory only if uncontradicted and may be overcome by other evidence to the contrary.  The testimonies presented by the complainant dispelling the regularity of the issuance of the warrant of arrest are more than enough to dispute this legal presumption of regularity in the performance of official duties.

“According to the police, they have not received any warrant of arrest regarding the case of People of Philippines vs. Michael Badangayon and Peter Guinannoy until March 2000. P/Chief Insp. Bernardo Baui testified that on 07 March 2000, a warrant of arrest dated 26 March 1999 was received by their office, particularly by SPO4 Librado Raquipiso.  He further testified that on 14 March 2000, the respondent judge, together with her staff, went to the police station asking that another warrant of arrest dated 26 March 1999 be received by his office without indicating the date of receipt of the same.  He claimed that he refused at first, but consented eventually since they have earlier received the same warrant of arrest on 07 March 2000 indicating the date of receipt, and so he called on SPO1 Honofre Reolalas to receive the same without indicating the same of receipt.  The aforesaid testimony was further corroborated by the testimony of SPO1 Honofre Reolalas.

“What is more telling is the testimony given by Norman Ruabaro, one of the staff of the respondent judge working as docket clerk in Branch 38, RTC, Maddela.  He attested to the fact that complainant has been following up her husband’s case with the court monthly, dispelling Crisostomo Molina’s testimony that the complainant never went to court to follow up the case.  But that part of his testimony which is most damaging to the claims of respondent judge and which at the same time further bolstered the allegations of complainant, is that part where he claimed that no warrant of arrest was issued on March 1999 and narrated how three warrants of arrest were issued all in all on March 2000.  He claimed that he typed a warrant of arrest in 1999 but the judge refused to sign the same.  On March 2000, he prepared a warrant of arrest dated 26 March 1999, he delivered the same to the PNP Maddela on 07 March 2000 and the same was received by SPO4 Raquipiso on the same date.  He also testified that OIC Molina was able to retrieve the said warrant from the PNP Maddela as per instructions of the respondent judge and claimed that he hid the same afterwards.  He further testified that subsequently, he, together with the respondent judge and the other staff, went to the PNP Maddela to have another warrant of arrest dated 26 March 1999 received by the PNP Maddela without indicating the date and that they succeeded in having the same received by Sr. Police Officer Onofre Reolalas without indicating the date of receipt.  Lastly, an alias warrant dated 02 March 2000 was issued.

“Since the date of receipt by the PNP Maddela of the said warrant of arrest is being refuted by evidence to the contrary, the said warrant of arrest being presented by the judge could at most prove the issuance of the same on 26 March 1999, but not the receipt of the PNP Maddela on the same date.  Hence, the logical probability is that the respondent judge issued the subject warrant of arrest, only that the same was not served to the PNP Maddela on time.   But being the presiding judge, it is her duty to monitor the due service of legal processes in her court.  Thus, she is guilty of simple negligence on the principle of command responsibility.”[2]

In determining the appropriate penalty, Justice Bello considered the presence of the following circumstances which he believed could mitigate respondent’s liability: a) this is her first offense since her appointment to the judiciary; b) it was probable that she issued a warrant of arrest on March 26, 1999 but

Page 60: Warrantless Arrests, Searches and Seizures and Privacy of Communication

60

the PNP at Maddela was not furnished with a copy on time; and c) her failure to monitor the service of the warrant of arrest was an oversight on her part considering that she was designated Presiding Judge of RTC, Branch 37 in Bayombong, Nueva Viscaya and RTC, Branch 81 in Quezon City, in addition to her regular duties as Presiding Judge of RTC, Branch 38, Maddela, Quirino.  Justice Bello then recommended that respondent be merely reprimanded and warned that a repetition of the same or similar offense will be dealt with more severely. 

While the findings of the Investigating Justice are well-taken, we do not, however, agree with his conclusion and recommendation.

The issue here is whether respondent judge actually issued a warrant for the arrest of the accused in Criminal Case No. 38-034 on March 26, 1999 or after the filing of the Information on March 18, 1999.

Chief Inspector Bernardo Baui and Senior Police Officer Onofre Riolalas, both of the Maddela Police Station, Norman Ruaboro, a docket clerk at the RTC, Branch 38, Maddela, and Ferdinand Orias, Provincial Prosecutor of Quirino, all testified that respondent judge did not issue a warrant of arrest on March 26, 1999, nor did the Maddela Police receive any warrant on that day.  We note that they even confirmed the fact that complainant was persistent and assiduous in following up the issuance of the warrant of arrest.  If indeed respondent judge issued the warrant on that date, complainant should have been informed about it by the court personnel when she was consistently following it up from April 1999 to March 2000, or a span of one (1) year.  Instead, she was always told by the OIC of the court that the judge was not around.

Moreover, it did not escape this Court’s attention that respondent judge even attempted to cover up her inaction when she issued a warrant of arrest dated March 26, 1999 almost a year after, or on March 7, 2000.  Norman Ruaboro, RTC docket clerk, attested to the fact that sometime in March 2000, he prepared a warrant of arrest dated March 26, 1999 and delivered the same on March 7, 2000 to the Maddela PNP.  This was received by SPO4 Librado Raquipiso on the same date. However, the warrant was retrieved by OIC Crisostomo Molina upon instructions of respondent judge.  Norman further testified that he, together with respondent judge and the other members of her staff, went to Maddela Police Station to file another warrant of arrest dated March 26, 1999 with her instruction that the date of receipt should not be specified.  Senior Police Officer Riolalas received the same without indicating the date.  Chief Inspector Baui and Senior Police Officer Riolalas corroborated Norman’s testimony.

At this point it bears stressing that it is within the discretion of the judge to issue a warrant for the arrest of an accused in a criminal case.[3] A judge is required to personally evaluate the resolution of the prosecutor and its supporting evidence.  He may immediately dismiss the case if the evidence on record clearly fails to establish probable cause.  However, if he finds probable cause, then he is mandated by law to issue such warrant.[4] In such case, the issuance of the warrant of arrest is a matter of extreme urgency to abate the possibility of flight of the accused.

Here, while respondent judge found probable cause which justified the issuance of warrant of arrest in Criminal Case No. 38-034, she did not issue the same as mandated by law.  As a result, the accused could no longer be apprehended.

While we understand that respondent judge at that time was designated Presiding Judge of two other RTC salas (Bayombong, Nueva Viscaya and Quezon City), such fact does not justify her failure to issue a warrant of arrest.[5] She herself admitted that Branch 38 of RTC, Maddela has only a few pending cases.  Clearly, she could have acted with dispatch.  Her inaction obviously delayed the proceedings in Criminal Case No. 38-034 and undermined complainant’s trust in the judiciary.

We thus hold that respondent judge violated Rule 3.05, Canon 3 of the Code of Judicial Conduct admonishing all judges, among others, to dispose of the court’s business promptly.

It is the sworn duty of judges to administer justice without undue delay under the time-honored precept that justice delayed is justice denied.[6] The present clogged dockets on all levels of our judicial system cannot be cleared, unless each and every magistrate earnestly, painstakingly and faithfully complies with the mandate of the law. Undue delay in the disposition of cases amounts to a denial of justice which, in turn, brings the courts into disrepute and ultimately erodes the faith and confidence of the

Page 61: Warrantless Arrests, Searches and Seizures and Privacy of Communication

61

public in the judiciary.[7] Hence, failure of judges to promptly dispose of the court’s business constitutes gross inefficiency and warrants the imposition of administrative sanctions against them.[8]

Time and again, we remind judges of the importance of high sense of duty in the administration of justice.  They should dispose of the court’s business within the prescribed period, as delay reinforces in the people’s minds that the wheels of justice grind ever so slowly.[9] As this Court eloquently stated in one case:[10]

“On the whole, judges ought to be mindful of the crucial role they play in keeping the flames of justice alive and forever burning.  Cognizant of this sacred task, judges are duty-bound to vigilantly and conscientiously man the wheels of justice as it grinds though eternity.  In a sense, judges are revered as modern-day sentinels, who, like their erudite forerunners, must never slumber, so to speak, in the hour of service to their countrymen.

“For as lady justice never sleeps, so must the gallant men tasked to guard her domain.”

Section 9(1), Rule 140 of the Rules of Court, as amended, classifies violation of Supreme Court rules as a less serious charge which, under Section 11(B) of the same Rule, is penalized with either suspension from office without salary and other benefits for not less than one (1) nor more than three (3) months, or a fine of more than P10,000.00 but not exceeding P20,000.00.

WHEREFORE, this Court finds Judge Ma. Theresa dela Torre-Yadao liable for violation of Rule 3.05, Canon 3, Code of Judicial Conduct.  She is ordered to pay a FINE of TWENTY THOUSAND PESOS (P20,000.00), with a stern warning that a repetition of the same offense will be dealt with more severely.

SO ORDERED.

Panganiban, (Acting Chairman), Corona, and Carpio-Morales, JJ., concur.Puno, (Chairman), J., on official business.

[ G.R. No. 71782, April 14, 1988 ]HADJI IBRAHIM SOLAY PANGANDAMAN, MAGAMBAAN PANGANDAMAN, MACARIAN

PANGANDAMAN, MAMINTAL PANGANDAMAN, PACALUNDO PANGANDAMAN, MANGORAMAS PANGANDAMAN, MACADAOB P. PANGORANGAN, KILATUN PANGANDAMAN, MARIO

PANGANDAMAN, MACABIDAR PANGANDAMAN, PUYAT P. ROMAMPAT, SANTORANI P. DIMAPENGEN, NASSER P. DIMAPENGEN AND DIAMA OPAO, PETITIONERS, VS. DIMAPORO T. CASAR, AS MUNICIPAL CIRCUIT TRIAL JUDGE OF POONABAYABAO, TAMPARAN AND MASIU,

LANAO DEL SUR AND THE PEOPLE OF THE PHILIPPINES, RESPONDENTS. 

D E C I S I O NNARVASA, J.:The petitioners ask this Court:1) to annul the warrant for their arrest issued by respondent Judge Dimaporo T. Casar of the Municipal

Circuit Court of Masiu, Lanao del Sur, in Criminal Case No. 1748 entitled “People vs. Hadji Ibrahim Solay Pangandaman, et al.”;

 

2) to prohibit the Judge from taking further cognizance of said Criminal Case No. 1748; and 

3) to compel the Judge to forward the entire record of Criminal Case No. 1748 to the Provincial Fiscal of Lanao del Sur for proper disposition.[1]

Page 62: Warrantless Arrests, Searches and Seizures and Privacy of Communication

62

Their plea is essentially grounded on the claim that the warrant for their arrest was issued by the respondent Judge without a proper preliminary investigation.[2] The Solicitor General agrees and recommends that their petition be granted and the warrant of arrest voided.[3]

On July 27, 1985, a shooting incident occurred in Pantao, Masiu, Lanao del Sur, which left at least five persons dead and two others wounded. What in fact transpired is still unclear. According to one version, armed men had attacked a residence in Pantao, Masiu, with both attackers and defenders suffering casualties.[4] Another version has it that a group that was on its way to another place, Lalabuan, also in Masiu, had been ambushed.[5]

On the following day, Atty. Mangurun Batuampar, claiming to represent the widow of one of the victims, filed a letter-complaint with the Provincial Fiscal at Marawi City, asking for a “full blast preliminary investigation” of the incident.[6] The letter adverted to the possibility of innocent persons being implicated by the parties involved on both sides -- none of whom was, however, identified -- and promised that supporting affidavits would shortly be filed. Immediately the Provincial Fiscal addressed a “1st indorsement” to the respondent Judge, transmitting Atty. Batuampar’s letter and requesting that “all cases that may be filed relative ** (to the incident) that happened in the afternoon of July 27, 1985,” be forwarded to his office, which “has first taken cognizance of said cases.”[7]

No case relative to the incident was, however, presented to the respondent Judge until Saturday, August 10, 1985, when a criminal complaint for multiple murder was filed before him by P.C. Sgt. Jose L. Laru-an, which was docketed as Case No. 1748.[8] On that same day, the respondent Judge “examined personally all (three) witnesses (brought by the sergeant) under oath thru * * (his) closed and direct supervision,” reducing to writing the questions to the witnesses and the latter’s answers.[9] Thereafter the Judge “approved the complaint and issued the corresponding warrant of arrest” against the fourteen (14) petitioners (who were named by the witnesses) and fifty (50) “John Does.”[10]

An “ex-parte” motion for reconsideration was filed on August 14, 1985 by Atty. Batuampar (joined by Atty. Pama L. Muti), seeking recall of the warrant of arrest and subsequent holding of a “thorough investigation” on the ground that the Judge’s initial investigation had been “hasty and manifestly haphazard” with “no searching questions” having been propounded.[11] The respondent Judge denied the motion for “lack of basis”;[12] hence the present petition.

While they concede the authority of the respondent Judge to conduct a preliminary investigation of the offenses involved, which are cognizable by Regional Trial Courts, the petitioners and the Solicitor General argue that the Judge in the case at bar failed to conduct the investigation in accordance with the procedure prescribed in Section 3, Rule 112 of the Rules of Court;[13] and that that failure constituted a denial to petitioners of due process which nullified the proceedings leading to the issuance of the warrant for the petitioners’ arrest.[14] It is further contended that August 10, 1985 was a Saturday during which “Municipal Trial Courts are open from 8:00 a.m. to 1:00 p.m. only, * * *” and “* * * it would hardly have been possible for respondent Judge to determine the existence of probable cause against sixty-four (64) persons whose participations were of varying nature and degree in a matter of hours and issue the warrant of arrest in the same day;”[15] and that there was undue haste and an omission to ask searching questions by the Judge who relied “mainly on the supporting affidavits which were obviously prepared already when presented to him by an enlisted PC personnel as investigator.”[16]

Page 63: Warrantless Arrests, Searches and Seizures and Privacy of Communication

63

The petitioners further assert that the respondent Judge conducted the preliminary investigation of the charges “* * * in total disregard of the Provincial Fiscal * * *” who, as said respondent well knew, had already taken cognizance of the matter twelve (12) days earlier and was poised to conduct his own investigation of the same;[17]and that issuance of a warrant of arrest against fifty (50) “John Does” transgressed the Constitutional provision requiring that such warrants should particularly describe the persons or things to be seized.[18]

There can be no debate about the proposition that in conducting a preliminary investigation of any crime cognizable by the Regional Trial Courts, a judge of an inferior court (other than in Metro-Manila or the chartered cities, where no authority to conduct preliminary investigation is vested in such officials) must observe the procedure prescribed in Section 3 of Rule 112, 1985 Rules on Criminal Procedure. And although not specifically so declared, the procedure mandated by the Rule actually consists of two phases or stages.

The first phase consists of an ex-parte inquiry into the sufficiency of the complaint and the affidavits and other documents offered in support thereof. And it ends with the determination by the Judge either: (1) that there is no ground to continue with the inquiry, in which case he dismisses the complaint and transmits the order of dismissal, together with the records of the case, to the provincial fiscal; or (2) that the complaint and the supporting documents show sufficient cause to continue with the inquiry and this ushers in the second phase.

This second phase is designed to give the respondent notice of the complaint, access to the complainant’s evidence and an opportunity to submit counter-affidavits and supporting documents. At this stage also, the Judge may conduct a hearing and propound to the parties and their witnesses questions on matters that, in his view, need to be clarified. The second phase concludes with the Judge rendering his resolution, either for dismissal of the complaint or holding the respondent for trial, which shall be transmitted, together with the record, to the provincial fiscal for appropriate action.

The procedure above described must be followed before the complaint or information is filed in the Regional Trial Court. Failure to do so will result in a denial of due process. [19]

Here, no information has as yet been filed with the Regional Trial Court. There is no pretense that the preliminary investigation has been completed, insofar as the respondent Judge is concerned, and that he does not intend to undertake the second phase. In this situation, it cannot be said that he has failed to observe the prescribed procedure. What has happened is simply that after receiving the complaint and examining the complainant’s witnesses, and having come to believe, on the basis thereof, that the offenses charged had been committed, the respondent Judge issued the warrant now complained of against the fourteen (14) respondents (now petitioners) named and identified by the witnesses as the perpetrators of the killings and injuries, as well as against 50 “John Does.”

The real question, therefore, is whether or not the respondent Judge had the power to issue the warrant of arrest without completing the entire prescribed procedure for preliminary investigation. Stated otherwise, is completion of the procedure laid down in Section 3 of Rule 112 a condition sine qua non for the issuance of a warrant of arrest?

There is no requirement that the entire procedure for preliminary investigation must be completed before

Page 64: Warrantless Arrests, Searches and Seizures and Privacy of Communication

64

a warrant of arrest may be issued. What the Rule[20] provides is that no complaint or information for an offense cognizable by the Regional Trial Court may be filed without completing that procedure. But nowhere is it provided that the procedure must be completed before a warrant of arrest may issue. Indeed, it is the contrary that is true. The present Section 6 of the same Rule 112 clearly authorizes the municipal trial court to order the respondent’s arrest even before opening the second phase of the investigation if said court is satisfied that a probable cause exists and there is a necessity to place the respondent under immediate custody in order not to frustrate the ends of justice.

“Sec. 6. When warrant of arrest may issue. --

***(b) By the Municipal Trial Court. -- If the municipal trial judge conducting the preliminary investigation is satisfied after an examination in writing and under oath of the complainant and his witnesses in the form of searching questions and answers, that a probable cause exists and that there is a necessity of placing the respondent under immediate custody in order not to frustrate the ends of justice, he shall issue a warrant of arrest.”[21]

This was equally true under the former rules, where the first phase of the investigation was expressly denominated “preliminary examination” to distinguish it from the second phase, or preliminary investigation proper. Thus, the former Section 6 of Rule 112 provided:“SEC. 6. Warrant of arrest, when issued. -- If the judge be satisfied from the preliminary examination conducted by him or by the investigating officer that the offense complained of has been committed and that there is reasonable ground to believe that the accused has committed it, he must issue a warrant or order for his arrest.”In Mayuga vs. Maravilla,[22] this Court found occasion to dwell in some detail on the process of preliminary investigation and, incidentally, to affirm the power of a justice of the peace or municipal judge conducting a preliminary investigation to order the arrest of the accused after the first stage (preliminary examination), saying:“Appellant should bear in mind that a preliminary investigation such as was conducted by the Justice of the Peace has for its purpose only the determination of whether a crime has been committed and whether there is probable cause to believe the accused guilty thereof, and if so, the issuance of a warrant of arrest. And it should not be forgotten that a preliminary investigation has two stages: First, a preliminary examination of the complainant and his witnesses prior to the arrest of the accused; and, second, the reading to the accused after his arrest of the complaint or information filed against him, and his being informed of the substance of the evidence against him, after which he is allowed to present evidence in his favor, if he so desires. Probable cause, in regard to the first stage of preliminary investigation, depends on the discretion of the judge or magistrate empowered to issue the warrant of arrest. It suffices that facts are presented to him to convince him, not that a person has committed the crime, but that there is probable cause to believe that such person committed the crime charged. The proceeding is generally ex parte unless the defendant desires to be present and while under the old Rules the Justice of the Peace or investigating officer must take the testimony of the complainant and the latter’s witnesses under oath, only the testimony of the complainant shall be in writing and only an abstract of the testimony of the other is required. Regarding preliminary investigation, it has thus been ruled that ‘the occasion is not for the full and exhaustive display of the parties’ evidence; it is for the presentation of such evidence only as may engender well-grounded belief that an offense has been committed and that the accused is probably guilty thereof.’ * * *”[23]

Page 65: Warrantless Arrests, Searches and Seizures and Privacy of Communication

65

The rule on arrest after preliminary examination has, of course, been modified somewhat since the occurrence of the facts upon which Mayuga was decided, but not to abrogate the authority of the investigating judge to order such arrest, and only to prescribe the requirement that before he may do so, he must examine the witnesses to the complaint, the examination to be under oath and reduced to writing in the form of searching questions and answers. This modification was introduced by Republic Act 3838, approved June 22, 1963, amending Section 87 of the Judiciary Act of 1948, and the “searching questions and answers” requirement is incorporated in the present Section 6 of Rule 112 already quoted.

The argument, therefore, must be rejected that the respondent Judge acted with grave abuse of discretion in issuing the warrant of arrest against petitioners without first completing the preliminary investigation in accordance with the prescribed procedure. The rule is and has always been that such issuance need only await a finding of probable cause, not the completion of the entire procedure of preliminary investigation.

Also without appreciable merit is petitioners’ other argument that there was scarcely time to determine probable cause against sixty-four persons (the fourteen petitioners and fifty “Does”) within a matter of hours on a Saturday when municipal trial courts are open only from 8:00 a.m. to 1:00 p.m. That argument founders upon the respondent Judge’s positive affirmations that he had personally and closely examined under oath the three witnesses to the complaint[24] and that he had issued the warrant of arrest “believing that the offense thus filed had been committed.”[25] Nothing in the record before this Court belies or discredits those affirmations which have, besides, the benefit of the legal presumption that official duty has been regularly performed.[26] The contention that the witnesses to the complaint had merely sworn before the respondent Judge to statements prepared beforehand and submitted by a military investigator[27] must, in view of the foregoing considerations and for lack of any support in the record, be dismissed as mere speculation.

The same argument also unwarrantedly assumes that the respondent Judge limited the proceedings on preliminary examination to the usual Saturday office hours of 8:00 a.m. to 1:00 p.m., in addition to not making any persuasive showing that such proceedings could not have been completed within that time-frame. For all that appears, said respondent could have put off the 1:00 p.m. adjournment until he had finished interrogating the witnesses to his satisfaction. And there is really nothing unusual in completing within a three-hour period the questioning of three witnesses in a preliminary examination to determine the existence of probable cause.

The record which, lacking proof to the contrary, must be accepted as an accurate chronicle of the questioned proceedings, shows prima facie that the respondent Judge had personally examined the witnesses to the complaint, and a consideration of the latter’s sworn answers to his questions satisfies this Court that the finding of probable cause against the petitioners was neither arbitrary nor unfounded.

The three witnesses to the complaint, Misandoning Monasprang, a student, Lawandato Ripors, an engineering graduate, and Sanny Monib, a farmer, gave mutually corroborative accounts of the incident. Under separate questioning, they declared that they were members of a party that was passing by Pantao on its way to Lalabuan from Talaguian, all in Masiu, Lanao del Sur, at about 10:00 a.m. on July 27, 1985, when they were ambushed and fired upon by an armed group which included the petitioners and about fifty other unidentified persons; that five of the party had been killed and two (the witnesses Lawandato Ripors and Sanny Monib) wounded; that even after they had killed their victims, the ambushers had

Page 66: Warrantless Arrests, Searches and Seizures and Privacy of Communication

66

continued to fire at the dead bodies; that the witnesses managed to escape their attackers and return to Talaguian, where they informed their relatives about what had happened, and thence went to the municipal hall in Masiu to report to the authorities; that the dead victims were recovered only late in the afternoon of that day because the authorities could not “penetrate” the area and the ambushers refused to release the bodies; and that the ambush was an offshoot of a grudge between the families of the ambushers and those of the victims.[28]

The witnesses named and identified the dead victims as Cadar Monasprang, Macacrao Guiling, Macrang Hadji Alawi, Alicman Ripors and Malabato Diator. All of them also identified by name each of the fourteen petitioners as members of the ambush group. The respondent Judge can hardly be faulted for finding enough cause to hold the petitioners named in the statements of three eyewitnesses to killings perpetrated in broad daylight.

In Luna vs. Plaza,[29] this Court ruled that the term “searching questions and answers” means -“* * * only, taking into consideration the purpose of the preliminary examination which is to determine “whether there is a reasonable ground to believe that an offense has been committed and the accused is probably guilty thereof so that a warrant of arrest may be issued and the accused held for trial,” such questions as have tendency to show the commission of a crime and the perpetuator thereof. What would be searching questions would depend on what is sought to be inquired into, such as: the nature of the offense, the date, time, and place of its commission, the possible motives for its commission; the subject, his age, education, status, financial and social circumstances, his attitude toward the investigation, social attitudes, opportunities to commit the offense; the victim, his age, status, family responsibilities, financial and social circumstances, characteristics, etc. The points that are the subject of inquiry may differ than case to case. The questions, therefore must to a great degree depend upon the Judge making the investigation. * * *”Upon this authority, and considering what has already been stated above, this Court is not prepared to question the propriety of the respondent Judge’s finding of probable cause or substitute its judgment for his in the matter of what questions to put to the witnesses during the preliminary examination.

Upon the facts and the law, therefore, the warrant of arrest in question validly issued against the petitioners, such issuance having been ordered after proceedings, to which no irregularity has been shown to attach, in which the respondent Judge found sufficient cause to commit the petitioners to answer for the crime complained of.

Insofar, however, as said warrant is issued against fifty (50) “John Does” not one of whom the witnesses to the complaint could or would identify, it is of the nature of a general warrant, one of a class of writs long proscribed as unconstitutional and once anathematized as “totally subversive of the liberty of the subject.”[30] Clearly violative of the constitutional injunction that warrants of arrest should particularly describe the person or persons to be seized,[31] the warrant must, as regards its unidentified subjects, be voided.

The fact that the Provincial Fiscal may have announced his intention of investigating the incident himself did not, in the view of the Court, legally inhibit the respondent Judge from conducting his own inquiry into the matter if, as is made to appear here, it was regularly brought before him and no formal complaint was filed before the Fiscal. Courtesy may have dictated that in those circumstances he leave the investigation to the Fiscal and simply endorse to the latter the complaint filed with him; duty did not, and if he

Page 67: Warrantless Arrests, Searches and Seizures and Privacy of Communication

67

nonetheless chose to conduct his own investigation, nothing in the rules states or implies that he could not do so.

Be that as it may, since the action and final resolution of the respondent Judge after completing the second stage of the preliminary investigation are subject to review by the Provincial Fiscal, practical considerations of expediency and the avoidance of duplication of work dictate that the latter official be permitted to take over the investigation even in its present stage.

WHEREFORE, the warrant complained of is upheld and declared valid insofar as it orders the arrest of the petitioners. Said warrant is voided to the extent that it is issued against fifty (50) “John Does.” The respondent Judge is directed to forward to the Provincial Fiscal of Lanao del Sur the record of the preliminary investigation of the complaint in Criminal Case No. 1748 of his court for further appropriate action. Without pronouncement as to costs.

SO ORDERED.

Teehankee, C.J., Cruz, Gancayco, and Griño-Aquino, JJ., concur.

Pangandaman v. Casar

Facts:

The case originated in Lanao. The offended party was ambushed in Lanao, but hesurvived. Based on his description, there were around 50 persons who staged theambush from both sides of the hill. However, he could not recognize anyone of the50. But he filed a case against all 50 ambushers, all “JOHN DOES”. So the courtissued a warrant of arrest against the 50 John Does.

Issue:

W/N the warrant of arrest is valid? Can a court issue a warrant of arrestagainst an unknown accused?

Held:

NO it is not valid. It is of the nature of a general warrant, one of a call of writs longprescribed as unconstitutional and once anathematized as totally subversive of the liberty of the subject. Clearly violative of the constitutional injunction thatwarrants of arrest should particularly describe the person or persons to be seized. The warrant as against unidentified subjects will be considered as null and void.

[ G.R. No. 129651, October 20, 2000 ]FRANK UY AND UNIFISH PACKING CORPORATION, PETITIONERS, VS. BUREAU OF INTERNAL

REVENUE AND HON. MERCEDES GOZO-DADOLE, RESPONDENTS.

D E C I S I O NKAPUNAN, J.:

Page 68: Warrantless Arrests, Searches and Seizures and Privacy of Communication

68

Petitioners assail the validity of the warrants issued for the search of the premises of the Unifish Packing Corporation, and pray for the return of the items seized by virtue thereof.

On 30 September 1993, a certain Rodrigo Abos reported to the Bureau of Internal Revenue (BIR) that petitioners Unifish Packing Corporation and Uy Chin Ho alias Frank Uy were engaged in activities constituting violations of the National Internal Revenue Code. Abos, who claimed to be a former employee of Unifish, executed an Affidavit[1] stating:

1. He has personal knowledge that UNIFISH PACKING CORPORATION (hereinafter referred to as UNIFISH), a canning factory located at Hernan Cortes Street, under the active management of UY CHIN HO alias Frank Uy [,] is selling by the thousands of [sic] cartons of canned sardines without issuing receipt. This is in violation of Sections 253 and 263 of the Internal Revenue Code.

2. This grand scale tax fraud is perpetrated through the following scheme:(1) Uy Chin Ho a director of UNIFISH buys in bulk from the company;

(2) Being a director, Uy Chin Ho has a lot of clout in the distribution of the canned sardines processed by UNIFISH;

(3) Uy Chin Ho dictates the value of canned sardines that he orders and buys from UNIFISH without any receipt of his purchases;

(4) The moment he has the quantity he wants, UNIFISH through Uy Chin Ho delivers to the different supermarkets such as White Gold, Gaisano, etc.;

(5) Payments made by these tax evading establishments are made by checks drawn payable to cash and delivered to Uy Chin Ho; These payments are also not receipted (sic);

(6) Uy Chin Ho will then pay UNIFISH for the quantity of sardines he had withdrawn from the corporation;

3. Another fraudulent practice perpetrated by UNIFISH through Uy Chin Ho's direction is the sale of imported oil locally to different customers. This is a case of smuggling in the sense that UNIFISH, being an export company registered with the Board of Investments, is enjoying certain exemptions in their importation of oil as one of the raw materials in its processing of canned tuna for export. These tax exemptions are granted by the government on the condition that the oil is to be used only in the processing of tuna for export and that it is not to be sold unprocessed as is to local customers.

4. Another fraudulent practice involves the sales of unused cans; UNIFISH also enjoys tax exemptions in its purchases of tin cans subject to the condition that these are to be used as containers for its processed tuna for export. These cans are never intended to be sold locally to other food processing companies.

5. Prior to 1990, that is from 1980 to 1990, the factory of the UNIFISH PACKING CORPORATION was then run by the PREMIER INDUSTRIAL & DEVELOPMENT CORPORATION (hereinafter referred to as PREMIER) [,] which corporation was being controlled by the same majority stockholders as those now running and controlling UNIFISH; [a]t that time, PREMIER was also committing the same fraudulent acts as what is being perpetrated by UNIFISH at present.

Page 69: Warrantless Arrests, Searches and Seizures and Privacy of Communication

69

6. The records containing entries of actual volume of production and sales, of both UNIFISH AND PREMIER, are found in the office of the corporation at its factory site at H. Cortes Street, Mandaue City. The particular place or spot where these records [official receipts, sales invoices, delivery receipts, sales records or sales books, stock cards, accounting records (such as ledgers, journals, cash receipts books, and check disbursements books)] are kept and may be found is best described in the herein attached sketch of the arrangement of the office's furniture and fixture of the corporation which is made an integral part hereof and marked as Annex "A",

7. He is executing this affidavit to attest under oath the veracity of the foregoing allegations and he is reserving his right to claim for reward under the provisions of Republic Act No. 2338.

On 1 October 1993, Nestor N. Labaria, Assistant Chief of the Special Investigation Branch of the BIR, applied for search warrants from Branch 28 of the Regional Trial Court of Cebu. The application sought permission to search the premises of Unifish.

After hearing the depositions of Labaria and Abos, Judge Mercedes Gozo-Dadole issued the disputed search warrants. The first[2] is docketed as "SEARCH WARRANT NO. 93-10-79 FOR: VIOLATION OF SECTION 253" ("Search Warrant A-1"), and consists of two pages. A verbatim reproduction of Search Warrant A-1 appears below:

REPUBLIC OF THE PHILIPPINES REGIONAL TRIAL COURT OF CEBU

7th Judicial RegionBranch 28 

Mandaue CityTHE PEOPLE OF THE PHILIPPINES,    

Plaintiff,   

    SEARCH WARRANT NO. 93-10-79 FOR:

    VIOLATION OF SEC. 253

- versus -    

     

UY CHIN HO alias FRANK UY,    

Unifish Packing Corporation    

Hernan Cortes St., Cebu City    x - - - - - - - - - - - - - - - - - - - - - - - - - /    

(with sketch)    

SEARCH WARRANTTO ANY PEACE OFFICER:

G R E E T I N G S:

It appearing to the satisfaction of the undersigned, after examination underoath (sic), Nestor N. Labaria, Asst. Chief, Special Investigation Branch, BIR and witness Rodrigo Abos that there is a (sic) probable cause to believe that the crime of violation of Section 253 - attempt to evade or defeat the tax has been

Page 70: Warrantless Arrests, Searches and Seizures and Privacy of Communication

70

committed and there is good and sufficient reason to believe that Uy Chin Ho c/o Unifish Packing Corporation, Hernan Cortes St., Mandaue City has in his possession, care and control, the following:

1. Multiple sets of Books of Accounts; Ledgers, Journals, Columnar Books, Cash Register Books, Sales Books or Records; Provisional & Official Receipts;

2. Production Record Books/Inventory Lists [,] Stock Cards;

3. Unregistered Delivery Receipts;

4. Unregistered Purchase & Sales Invoices;

5. Sales Records, Job Order;

6. Corporate Financial Records; and

7. Bank Statements/Cancelled ChecksYou are hereby commanded to make an immediate search at any time of day or night of said premises and its immediate vicinity and to forthwith seize and take possession of the articles above-mentioned and other properties relative to such violation and bring said properties to the undersigned to be dealt with as the law directs.

WITNESS MY HAND this 1 st  day of October, 1993.(sgd.)

    MERCEDES GOZO-DADOLE

    Judge

The second warrant[3]is similarly docketed as "SEARCH WARRANT 93-10-79 FOR: VIOLATION OF SEC. 253" ("Search Warrant A-2"). Search Warrant A-2, reproduced below, is almost identical in content to Search Warrant A-1, save for the portions indicated in bold print. It consisted of only one page.

REPUBLIC OF THE PHILIPPINES REGIONAL TRIAL COURT OF CEBU 

7th Judicial Region Branch 28 

Mandaue CityTHE PEOPLE OF THE PHILIPPINES,    

Plaintiff,   

    SEARCH WARRANT NO. 93-10-79 FOR:

- versus -   VIOLATION OF SEC. 253UY CHIN HO alias FRANK UY, and    

Unifish Packing Corporation    

Hernan Cortes St., Mandaue City    x - - - - - - - - - - - - - - - - - - - - - - - - - /    

Page 71: Warrantless Arrests, Searches and Seizures and Privacy of Communication

71

(with sketch)    

SEARCH WARRANTTO ANY PEACE OFFICER:

G R E E T I N G S:

It appearing to the satisfaction of the undersigned, after examination underoath [sic], Nestor N. Labaria, Asst. Chief, Special Investigation Branch, BIR and witness Rodrigo Abos that there is a [sic] probable cause to believe that the crime of violation of Section 253 - attempt to evade or defeat the tax has been committed and there is good and sufficient reason to believe that Uy Chin Ho alias Frank Uy and Unifish Packing Corporation, Hernan Cortes St., Mandaue City has in his possession, care and control, the following:

1. Multiple sets of Books of Accounts; Ledgers, Journals, Columnar Books, Cash Register Books, Sales Books or Records; Provisional & Official Receipts;

2. Production Record Books/Inventory Lists [,] Stock Cards;

3. Unregistered Delivery Receipts;

4. Unregistered Purchase & Sales Invoices;

5. Sales Records, Job Order;

6. Corporate Financial Records; and

7. Bank Statements/Cancelled ChecksYou are hereby commanded to make an immediate search at any time of day or night of said premises and its immediate vicinity and to forthwith seize and take possession of the articles above-mentioned and other properties relative to such violation and bring said properties to the undersigned to be dealt with as the law directs.

WITNESS MY HAND this 1st day of October, 1993.(sgd.)

    MERCEDES GOZO-DADOLE

    Judge

Judge Gozo-Dadole issued a third warrant,[4] which was docketed as "SEARCH WARRANT 93-10-80 FOR: VIOLATION OF SEC. 238 in relation to SEC. 263" (hereinafter, "Search Warrant B"). Except for the docket number and the designation of the crime in the body of the warrant ("Section 238 in relation to Sec. 263 - non-issuance of sales invoice and use and possession of unregistered delivery receipts and/or sales invoices"), Search Warrant B is a verbatim reproduction of Search Warrant A-2.On the strength of these warrants, agents of the BIR, accompanied by members of the Philippine National Police, on 2 October 1993, searched the premises of the Unifish Packing Corporation. They seized, among other things, the records and documents of petitioner corporation. A return of said search was duly made by Nestor Labaria with the RTC of Cebu , Branch 28.

Page 72: Warrantless Arrests, Searches and Seizures and Privacy of Communication

72

On 8 February 1995, the BIR filed against petitioners a case before the Department of Justice. The records, however, do not reveal the nature of this case.

On 31 March 1995, petitioners filed motions to quash the subject search warrants with Branch 28 of the Cebu RTC.

The RTC, however, denied petitioners' motions to quash as well as their subsequent motion for reconsideration, prompting petitioners to file a petition for certiorari with the Court of Appeals (CA). The CA dismissed their petition, holding that petitioners failed to comply with Section 2(a), Rule 6 of the Revised Internal Rules of the Court of Appeals (RIRCA), which states:a. What Should be Filed. - The petition shall be filed in seven (7) legible copies and a copy thereof shall be served on each of the respondents, and must be accompanied by a certified true copy of the decision or order complained of and true copies of the pleadings and other pertinent documents and papers. (As amended by S.Ct. Res., dated November 24, 1992).

The CA found that petitioners did not submit certified true copies of (1) the Motions to Quash, (2) the Motion for Reconsideration, and (3) the Affidavit of Rodrigo Abos.

The CA also held that certiorari was not the proper remedy to question the resolution denying the motion to quash.In this case now before us, the available remedies to the petitioners, assuming that the Department of Justice will eventually file the case, are: a petition for reinvestigation; the right to post bail; a Motion to Quash the Information; and in case of denial, an appeal, after judgment on the merits, or after the case shall have been tried. This brings us to the case of Lai vs. Intermediate 220 SCRA 149 and the pronouncement, thus:Criminal Procedure: Certiorari: Certiorari should not be allowed where petitioner has other remedies available. -- Anent the remedy resorted to by petitioners (referring to the petition for certiorari) from the Regional Trial Court of Negros Oriental presided by Judge Diez, the same should not have been granted. Petitioners were not without plain, speedy and adequate remedies in the ordinary course of law against Judge Lomeda's order for their arrest. These remedies are as enumerated by respondent appellate court in its decision: "1. they can post bail for their provisional release; 2. They can ask the Provincial Fiscal for a reinvestigation of the charge against them. If unsatisfied with the fiscal's resolution they can ask for a review by the Minister of Justice; (Sec. 1(), RA 5180 as amended by P.D. 911); 3. if their petition for review does not prosper, they can file a motion to quash the information in the trial court. (Rule 117, Rules of Court). 4. If the motion is denied, they can appeal the judgment of the court after the case shall have been tried on the merits. x x xWhere motion to quash is denied, remedy is not certiorari, but to go to trial.-- Moreover, in the case of Acharon vs. Purisima, this Court held that when a motion to quash a criminal case is denied, the remedy is not certiorari but to go to trial without prejudice to reiterating the special defenses involved in said Motion. In the event that an adverse decision is rendered after trial on the merits, an appeal therefrom should be the next legal step.

xxx

In this case now before Us, there is no pretention [sic] that the Court issued the Search Warrants without jurisdiction. On the contrary, it had jurisdiction. The argument therefore that the Court committed an error in not describing the persons or things to be searched; that the Search Warrants did not describe with

Page 73: Warrantless Arrests, Searches and Seizures and Privacy of Communication

73

particularity the things to be seized/taken; the absence of probable cause; and for having allegedly condoned the discriminating manner in which the properties were taken, to us, are merely errors in the Court's finding, certainly not correctible by certiorari, but instead thru an appeal.[5]

In any event, the CA ruled, no grave abuse of discretion amounting to lack of jurisdiction was committed by the RTC in the issuance of the warrants.

As petitioners' motion for reconsideration proved futile, petitioners filed the instant petition for review.

Petitioners claim that they did submit to the CA certified true copies of the pleadings and documents listed above along with their Petition, as well as in their Motion for Reconsideration. An examination of the CA Rollo, however, reveals that petitioners first submitted the same in their Reply, after respondents, in their Comment, pointed out petitioners' failure to attach them to the Petition.Nevertheless, the CA should not have dismissed the petition on this ground although, to its credit, it did touch upon the merits of the case. First, it appears that the case could have been decided without these pleadings and documents. Second, even if the CA deemed them essential to the resolution of the case, it could have asked for the records from the RTC. Third, in a similar case,[6] we held that the submission of a document together with the motion for reconsideration constitutes substantial compliance with Section 3, Rule 46 of the Rules of Court, requiring the submission of a certified true copy of "material portions of the record as are referred to [in the petition], and other documents relevant or pertinent thereto" along with the petition. So should it be in this case, especially considering that it involves an alleged violation of a constitutionally guaranteed right. The rules of procedure are not to be applied in a very rigid, technical sense; rules of procedure are used only to help secure substantial justice. If a technical and rigid enforcement of the rules is made, their aim could be defeated.[7]

The CA likewise erred in holding that petitioners cannot avail of certiorari to question the resolution denying their motions to quash the subject search warrants. We note that the case of "Lai vs. Intermediate," cited by the appellate court as authority for its ruling does not appear in "220 SCRA 149." The excerpt of the syllabus quoted by the court, as observed by petitioners,[8] appears to have been taken from the case of Yap vs. Intermediate Appellate Court, 220 SCRA 245 (1993). Yap, however, is inapplicable since that case involved a motion to quash a complaint for qualified theft, not a motion to quash a search warrant.The applicable case is Marcelo vs. De Guzman,[9] where we held that the issuing judge's disregard of the requirements for the issuance of a search warrant constitutes grave abuse of discretion, which may be remedied by certiorari:Expressly announced in Section 1, Rule 65 of the Rules of Court is the general rule that certiorari is available where a tribunal or officer exercising judicial functions "has acted without or in excess of its or his jurisdiction, or with grave abuse of discretion and there is no appeal, nor any plain, speedy, and adequate remedy in the ordinary course of law."

In the light of the findings of the lower court, herein above quoted, it is indisputable that Judge de Guzman gravely abused his discretion in issuing the said search warrant. Indeed, he acted whimsically and capriciously when he ignored the explicit mandate of Section 3, Rule 126 of the Rules of Court that "a search warrant shall not issue but upon probable cause in connection with one specific offense to be determined by the municipal or city judge after examination under oath or affirmation of the complainant and the witnesses he may produce, and particularly describing the place to be searched and the persons or things to be seized; and that "no search warrant shall issue for more than one specific offense."

Page 74: Warrantless Arrests, Searches and Seizures and Privacy of Communication

74

The utter disregard by Judge de Guzman of the requirements laid down by the said rule renders the warrant in question absolutely null and void. It has been held that where the order complained of is a patent nullity, a petition for certiorari and mandamus may properly be entertained despite the existence of the remedy of appeal.

Moreover, an appeal from the order of Judge de Guzman would neither be an adequate nor speedy remedy to relieve appellee of the injurious effects of the warrant. The seizure of her personal property had resulted in the total paralization of the articles and documents which had been improperly seized. Where the remedy of appeal cannot afford an adequate and expeditious relief, certiorari can be allowed as a mode of redress to prevent irreparable damage and injury to a party.

This Court had occasion to reiterate the above pronouncement in Silva vs. Presiding Judge, RTC of Negros Oriental, Br. XXXIII,[10] which also involved a special civil action for certiorari:[11]

Thus, in issuing a search warrant, the judge must strictly comply with the constitutional requirement that he must determine the existence of probable cause by examining the applicant and his witnesses in the form of searching questions and answers. His failure to comply with this requirement constitutes grave abuse of discretion. As declared in Marcelo vs. De Guzman, G.R. No. L-29077, June 29, 1982, 114 SCRA 657, "the capricious disregard by the judge in not complying with the requirements before issuance of search warrants constitutes grave abuse of discretion".In this case, petitioners alleged in their petition before the CA that the issuing judge violated the pertinent provisions of the Constitution and the Rules of Court in issuing the disputed search warrants, which, if true, would have constituted grave abuse of discretion. Petitioners also alleged that the enforcers of the warrants seized almost all the records and documents of the corporation thus resulting in the paralysis of its business. Appeal, therefore, would not be an adequate remedy that would afford petitioners expeditious relief.

We now proceed to the merits of the case.

Section 2, Article III of the Constitution guarantees the right of the people against unreasonable searches and seizures:

The right of the people to be secure in their persons, houses, papers, and effects against unreasonable searches and seizures of whatever nature and for any purpose shall be inviolable, and no search warrant or warrant of arrest shall issue except upon probable cause to be determined personally by the judge after examination under oath or affirmation of the complainant and the witnesses he may produce, and particularly describing the place to be searched and the persons or things to be seized.

In relation to the above provision, Rule 126 of the Rules of Court provides:

SEC. 3. Requisite for issuing search warrant. - A search warrant shall not issue but upon probable cause in connection with one specific offense to be determined personally by the judge after examination under oath or affirmation of the complainant and the witnesses he may produce, and particularly describing the place to be searched and the things to be seized.SEC. 4. Examination of complainant; record. - The judge must, before issuing the warrant, personally examine in the form of searching questions and answers, in writing and under oath the complainant and any witnesses he may produce on facts personally known to them and attach to the record their sworn statements together with any affidavits submitted.

Page 75: Warrantless Arrests, Searches and Seizures and Privacy of Communication

75

A search warrant must conform strictly to the requirements of the foregoing constitutional and statutory provisions. These requirements, in outline form, are:

(1) the warrant must be issued upon probable cause;

(2) the probable cause must be determined by the judge himself and not by the applicant or any other person;

(3) in the determination of probable cause, the judge must examine, under oath or affirmation, the complainant and such witnesses as the latter may produce; and

(4) the warrant issued must particularly describe the place to be searched and persons or things to be seized.[12]

The absence of any of these requisites will cause the downright nullification of the search warrants. [13] The proceedings upon search warrants must be absolutely legal, "for there is not a description of process known to the law, the execution of which is more distressing to the citizen. Perhaps there is none which excites such intense feeling in consequence of its humiliating and degrading effect." The warrants will always be construed strictly without, however, going the full length of requiring technical accuracy. No presumptions of regularity are to be invoked in aid of the process when an officer undertakes to justify under it.[14]

Petitioners contend that there are several defects in the subject warrants that command their nullification. They point out inconsistencies in the description of the place to be searched in Search Warrant A-1, as well as inconsistencies in the names of the persons against whom Search Warrants A-1 and A-2 were issued. That two search warrants (Search Warrants A-1 and A-2) were issued for the same crime, for the same place, at a single occasion is cited as another irregularity. Petitioners also dispute the existence of probable cause that would justify the issuance of the warrants. Finally, they claim that the things to be seized were not described with particularity. These defects, according to petitioners, render the objects seized inadmissible in evidence.[15]

Inconsistencies in the   description of the   place to be searchedPetitioners observe that the caption of Search Warrant A-1 indicates the address of Uy Chin Ho alias Frank Uy as "Hernan Cortes St., Cebu City" while the body of the same warrant states the address as "Hernan Cortes St.,Mandaue City." Parenthetically, Search Warrants A-2 and B consistently state the address of petitioner as "Hernan Cortes St., Mandaue City."The Constitution requires, for the validity of a search warrant, that there be a particular description of "the place to be searched and the persons of things to be seized."[16] The rule is that a description of a place to be searched is sufficient if the officer with the warrant can, with reasonable effort, ascertain and identify the place intended[17]and distinguish it from other places in the community.[18] Any designation or description known to the locality that points out the place to the exclusion of all others, and on inquiry leads the officers unerringly to it, satisfies the constitutional requirement.[19] Thus, in Castro vs. Pabalan,[20] where the search warrant mistakenly identified the residence of the petitioners therein as Barrio Padasil instead of the adjoining Barrio Maria Cristina, this Court "admitted that the deficiency in the writ is not of sufficient gravity to call for its invalidation."In this case, it was not shown that a street similarly named Hernan Cortes could be found in Cebu City. Nor was it established that the enforcing officers had any difficulty in locating the premises of petitioner

Page 76: Warrantless Arrests, Searches and Seizures and Privacy of Communication

76

corporation. That Search Warrant A-1, therefore, inconsistently identified the city where the premises to be searched is not a defect that would spell the warrant's invalidation in this case.

Inconsistencies in the   description of the   persons named in the   two warrantsPetitioners also find fault in the description of the names of the persons in Search Warrants A-1 and A-2. Search Warrant A-1 was issued solely against "Uy Chin Ho alias Frank Uy." Search Warrant A-2, on the other hand, was directed against "UY CHIN HO alias FRANK UY, and Unifish Packing Corporation."These discrepancies are hardly relevant.

In Miller v. Sigler,[21] it was held that the Fourth Amendment of the United States Constitution, from which Section 2, Article III of our own Constitution is historically derived, does not require the warrant to name the person who occupies the described premises. Where the search warrant is issued for the search of specifically described premises only and not for the search of a person, the failure to name the owner or occupant of such property in the affidavit and search warrant does not invalidate the warrant; and where the name of the owner of the premises sought to be searched is incorrectly inserted in the search warrant, it is not a fatal defect if the legal description of the premises to be searched is otherwise correct so that no discretion is left to the officer making the search as to the place to be searched.[22]

Since, in the case at bar, the warrant was issued not for search of the persons owning or occupying the premises, but only a search of the premises occupied by them, the search could not be declared unlawful or in violation of the constitutional rights of the owner or occupants of the premises, because of inconsistencies in stating their names.[23]

Two warrants issued   at one time for one   crime and one placeIn any event, Search Warrant A-1 should be deemed superseded by Search Warrant A-2.

Two warrants, Search Warrants A-1 and A-2, were actually issued by the trial court for the same crime (violation of "SEC. 253" of the National Internal Revenue Code). It appears, however, that Search Warrant A-2 was issued merely to correct the inconsistencies in the address in Search Warrant A-1, as well as to include Unifish Packing Corporation as a party against whom the warrant was issued. Search Warrant A-2 was evidently an attempt by the issuing judge to be more precise in the names of the persons against whom the warrant was issued and in the description of the place to be searched. Indeed, it would be absurd for the judge to issue on a single occasion two warrants authorizing the search of a single place for a single offense. Inasmuch as the apparent intent in issuing Search Warrant A-2 was to supersede Search Warrant A-1, the latter should be deemed revoked by the former.

The alleged absence ofprobable causePetitioners claim there was no probable cause for Judge Gozo-Dadole to issue the subject search warrants.

Probable cause is defined as such facts and circumstances which would lead a reasonably discreet and prudent man to believe that an offense has been committed and that the objects sought in connection with the offense are in the place sought to be searched.[24]

Page 77: Warrantless Arrests, Searches and Seizures and Privacy of Communication

77

In the determination of probable cause, the Constitution and the Rules of Court require an examination of the witnesses under oath. The examination must be probing and exhaustive, not merely routine or pro forma. The examining magistrate must not simply rehash the contents of the affidavit but must make his own inquiry on the intent and justification of the application.[25] Asking of leading questions to the deponent in an application for search warrant, and conducting of examination in a general manner, would not satisfy the requirements for issuance of a valid search warrant.[26]

The witnesses, in turn, must testify under oath to facts of their own personal knowledge. The oath required must refer to the truth of the facts within the personal knowledge of the petitioner or his witnesses, because the purpose thereof is to convince the committing magistrate, not the individual making the affidavit and seeking the issuance of the warrant, of the existence of probable cause.[27] Search warrants are not issued on loose, vague or doubtful basis of fact, nor on mere suspicion or belief.[28]

It may be recalled that before issuing the warrants, the judge deposed two witnesses, namely, Nestor Labaria of the BIR, and Rodrigo Abos, who claimed to be an old employee of Unifish. Petitioners claim that the testimonies of Labaria and Abos are hearsay. We agree with this contention, but only as to the testimony of Labaria, who stated during the examination:

Q. Do you know of a certain Uy Chin Ho alias Frank Uy?

A. No.

Q. Do you know his establishment known as Unifish Packing Corporation?

A. I have only heard of that thru the affidavit of our informer, Mr. Abos.

Q. Why are you applying for search warrant in the premises of Unifish Packing Corporation?

A. Because of that information we received that they are using only delivery receipts instead of the legal sales invoices. It is highly indicative of fraud.

Q. From where did you get that information?

A. From our informer, the former employee of that establishment.[29]

The above portion of the transcript shows that Labaria's knowledge of the alleged illegal activities of petitioners was acquired not through his own perception but was merely supplied by Abos. Therefore, the deposition of Labaria, which is based on hearsay, standing alone, cannot justify the issuance of the search warrants.[30]

The application for the warrants, however, is not based solely on Labaria's deposition but is supported by that of Abos, whose knowledge of petitioners' alleged illegal practices was apparently obtained during his employment with Unifish. In his deposition, Abos detailed the schemes employed by Frank Uy and Unifish to evade the payment of taxes, and described the place where the documents supposedly evidencing these schemes were located:

Q Do you know Frank Uy?

A Yes.

Q Why do you know him?

A Because I were (sic) an employee of his from 1980 until August of 1993.

Q Where is this Unifish Packing Corporation located?

Page 78: Warrantless Arrests, Searches and Seizures and Privacy of Communication

78

A Hernan Cortes St.

Q What is it being engaged of?

A It is engaged in canning of fish.Q You have executed an affidavit here to the effect that

it seems that in his business dealings that he is actually doing something that perpetrated tax evasion. Is that correct?

A Yes.

Q How is it done?A As an officer, he is an active member of the

corporation who is at the same time making his authority as appointing himself as the distributor of the company's products. He sells these products thru supermarkets in Visayas and Mindanao, in fact, the whole Philippines. He makes it appear that it is the company which is selling when actually it is him selling the goods and he does not issue any invoices.

Q Since he does not issue any invoices, how is it done?

A Thru delivery receipts.

Q Is the delivery receipt official?

A No. It is unregistered.

Q For how long has this been going on?

A As far as I know, it is still in 1986 since we started producing the sardines.

Q When was the last time that you observed that that is what he is doing?

A August, 1993, last month.

Q How did you happen to know about this last month?A Because he delivered to certain supermarkets and

the payments of that supermarket did not go directly to the company. It went to him and he is the one who paid the company for the goods that he sold.

Q Can you tell this Court the name of that certain supermarkets?

A White Gold and Gaisano.

Q How did you know this fact?

AAs a manager of the company I have access to all the records of that company for the last three years. I was the Operating Chief.

Q Until now?

A No. I was separated already.

Q When?

A August, 1993. Q How does he do this manipulation?A He sells the goods to the supermarkets afterwhich

the company, Unifish will deliver to his customers, then his customers will pay directly to him and in turn, he pays to the company.

Page 79: Warrantless Arrests, Searches and Seizures and Privacy of Communication

79

Q And these transactions, were they reflected in their books of account or ledger or whatever?

A It is written but it is supposed to be a secret transaction. It is not for the public, not for the BIR but it is only for the purpose of keeping the transactions between the company and him. It is not made to be shown to the BIR.

Q In that books of account, is it reflected that they have made some deliveries to certain supermarkets?

A Yes.

Q For the consumption of the BIR what are the papers that they show? .

A It is the private accounting firm that prepares everything.

Q Based on what?

A Based on some fictitious records just as they wish to declare.

Q In your affidavit you stated that there are sales invoices, official receipts, delivery receipts, sales records, etc. These documents are records that you have stated, in your affidavit, which are only for the consumption of the company?

A Yes, not for the BIR

Q Where are they kept now?A They are kept on the table which I have drawn in the

sketch. This is the bird's eyeview (sic) of the whole office. When you enter thru the door this Gina Tan is the one recording all the confidential transactions of the company. In this table you can find all the ledgers and notebooks.

Q This sketch is a blow-up of this portion, Exh. "A"?A Yes. Exh. "B" is the blow-up of Exh. "A" inside the

office. In this blow-up there are four personnel plus one new personnel. Gina Tan collects all the records from this girl and this girl makes the statements. This first girl delivers the receipts. The second girl prepares the bill of lading. The third girl keeps the inventory of all the stocks. This sketch here is the bodega where the records are kept. The records from these people are stored in this place which is marked as "C".

Q So what you want to impress on that now is that only current records are kept by Gina because according to you the whole records are already placed in the bodega?

A Yes.

Q But how can you enter the bodega?A Here, from the main entrance there is a door which

will lead to this part here. If you go straight there is a bodega there and there is also a guard from this exit right after opening the door.

Q The problem is that, when actually in August have

Page 80: Warrantless Arrests, Searches and Seizures and Privacy of Communication

80

you seen the current records kept by Gina?

A I cannot exactly recall but I have the xerox copies of the records.

Q Where are they now?A They are in my possession (witness handling [sic] to

the Court a bunch of records).Q The transactions that are reflected in these xerox

copies that you have given me, especially this one which seems to be pages of a ledger, they show that these are for the months of January, February, March, April and May. Are these transactions reflected in these xerox copies which appear in the ledger being shown to the BIR?

A As far as I know, it did not appear.

QWhat about this one which says Columnar Book Cash Receipt for the month of January, what does it show?

A It shows that Frank Uy is the one purchasing from the company and these are his customers.

Q Do these entries appear in the columnar books which are the basis for the report to the BIR?

A As far as I know, it does not reflect.

Q What are these xerox copies of checks?A I think we cannot trace it up. These ones are the

memos received by Unifish for payment of sardines. This is the statement of the company given to Uy Chin Ho for collection.

Q It is also stated in your affidavit that the company imported soya oil. How is it done?

A The company imports soya oil to be used as a component in the processing of canned tuna for export. The company enjoys certain BOI privilege and so it is tax free. As far as I know, they profit more to dispose the product locally. Whatever excess of this soya oil are sold to another company.

Q Is that fact reflected in the xerox copies?

A No. I have the actual delivery receipt.Q In other words, the company imports soya oil

supposedly to be used as a raw material but instead they are selling it locally?

A Yes. ([W]itness showing DR No. 3053 dated November 13, 1991.) This delivery receipt was the delivery receipt to Celebes Canning Corp. of the 90 grams soya oil.

Q In other words, this soya oil should have to be used by Unifish but instead they are seeling (sic) it?

A Yes, at a profit.

Q You also said that there is tax evasion in the selling of cans. What do you mean by this?

A There is another privileged [sic] by the BOI for a special price given to packaging materials. When you

Page 81: Warrantless Arrests, Searches and Seizures and Privacy of Communication

81

export the product there is a 50% price difference. Now, taking that advantage of that exemption, they sold it to certain company here, again to Virginia Farms.

Q Do you have proof to that effect?

A No, but we can get it there.Q Will that fact be shown in any listed articles in the

application for search warrant since according to you, you have seen this manipulation reflected on the books of account kept by Gina? Are you sure that these documents are still there?

A Yes. I have received information.

COURT: Alright.[31]

Abos stated that, as former Operating Chief of Unifish, he had access to the company records, and even showed the issuing judge photocopies thereof. Thus, we reject the contention that this witness did not have personal knowledge of the facts to which he testified. The contents of the deposition clearly demonstrate otherwise.

The deposition also shows that, contrary to petitioners' submission, the inquiries made by the judge were far from leading or being a rehash of the witness' affidavit. We find such inquiries to be sufficiently probing.

Alleged lack ofparticularity in the   description of thethings seizedPetitioners note the similarities in the description of the things to be seized in the subject warrants and those inStonehill vs. Diokno,[32] Bache & Co. (Phil.), Inc. vs. Ruiz,[33] and Asian Surety & Insurance Co., Inc. vs. Herrera.[34]

In Stonehill, the effects to be searched and seized were described as:"Books of accounts, financial records, vouchers, journals correspondence, receipts, ledgers, portfolios, credit journals, typewriters, and other documents and/or papers showing all business transactions including disbursement receipts, balance sheets and related profit and loss statements."

This Court found that the foregoing description failed to conform to the requirements set forth by the Constitution since:

x x x the warrants authorized the search for and seizure of records pertaining to all business transactions of petitioners herein, regardless of whether the transactions were legal or illegal. The warrants sanctioned the seizure of all records of the petitioners and the aforementioned corporations, whatever their nature, thus openly contravening the explicit command of our Bill of Rights - that the things to be seized be particularly described - as well as tending to defeat its major object: the elimination of general warrants.In Bache & Co., this Court struck down a warrant containing a similar description as those in Stonehill:The documents, papers, and effects sought to be seized are described in Search Warrant No. 2-M-70 in this manner:

Page 82: Warrantless Arrests, Searches and Seizures and Privacy of Communication

82

"Unregistered and private books of accounts (ledgers, journals, columnars, receipts and disbursements books, customers' ledgers); receipts for payments received; certificates of stocks and securities; contracts, promissory notes and deeds of sale; telex and coded messages; business communications; accounting and business records; checks and check stubs; records of bank deposits and withdrawals; and records of foreign remittances, covering the years 1966 to 1970."

The description does not meet the requirement in Art. III, Sec. 1, of the Constitution, and of Sec. 3, Rule 126 of the Revised Rules of Court, that the warrant should particularly describe the things to be seized.

x x x

In Uy Kheytin, et al. vs. Villareal, etc., et al., 42 Phil. 886, 896, this Court had occasion to explain the purpose of the requirement that the warrant should particularly describe the place to be searched and the things to be seized, to wit: "x x x Both the Jones Law (sec. 3) and General Orders No. 68 (sec. 97) specifically require that a search warrant should particularly describe the place to be searched and the things to be seized. The evident purpose and intent of this requirement is to limit the things to be seized to those, and only those, particularly described in the search warrant - to leave the officers of the law with no discretion regarding what articles they shall seize, to the end that `unreasonable searches and seizures' may not be made, - that abuses may not be committed. That is the correct interpretation of this constitutional provision borne out by the American authorities."The purpose as thus explained could, surely and effectively, be defeated under the search warrant issued in this case.

A search warrant may be said to particularly describe the things to be seized when the description therein is as specific as the circumstances will ordinarily allow (People vs. Rubio, 57 Phil, 384); or when the description expresses a conclusion of fact - not of law - by which the warrant officer may be guided in making the search and seizure (idem., dissent of Abad Santos, J.,); or when the things described are limited to those which bear direct relation to the offense for which the warrant is being issued (Sec. 2, Rule 126, Revised Rules of Court). The herein search warrant does not conform to any of the foregoing tests. If the articles desired to be seized have any direct relation to an offense committed, the applicant must necessarily have some evidence, other than those articles, to prove the said offense; and the articles subject of search and seizure should come in handy merely to strengthen such evidence. In this event, the description contained in the herein disputed warrant should have mentioned, at least, the dates, amounts, persons, and other pertinent data regarding the receipts of payments, certificates of stocks and securities, contracts, promissory notes, deeds of sale, messages and communications, checks, bank deposits and withdrawals, records of foreign remittances, among others, enumerated in the warrant.In Asian Surety & Insurance Co., Inc. vs. Herrera, the description of the things to be seized, i.e., "Fire Registers, Loss, Bordereau, Adjusters' Report, including subrogation receipts and proof of loss, Loss Registers, Book of Accounts including cash receipts and disbursements and general ledger, etc." was held to be "an omnibus description" and, therefore, invalid:x x x Because of this all embracing description which includes all conceivable records of petitioner corporation, which if seized x x x, could paralyze its business, petitioner in several motions filed for early resolution of this case, manifested that the seizure of TWO carloads of their papers has paralyzed their business to the grave prejudice of not only the company, its workers, agents, employees but also of its numerous insured and beneficiaries of bonds issued by it, including the government itself, and of the

Page 83: Warrantless Arrests, Searches and Seizures and Privacy of Communication

83

general public. And correlating the same to the charges for which the warrant was issued, We have before Us the infamous general warrants of old.

In the case at bar, the things to be seized were described in the following manner:

1. Multiple sets of Books of Accounts; Ledgers, Journals, Columnar Books, Cash Register Books, Sales Books or Records; Provisional & Official Receipts;

2. Production Record Books/Inventory Lists [,] Stock Cards;

3. Unregistered Delivery Receipts;

4. Unregistered Purchase & Sales Invoices;

5. Sales Records, Job Order;

6. Corporate Financial Records; and

7. Bank Statements/Cancelled ChecksWe agree that most of the items listed in the warrants fail to meet the test of particularity, especially since witness Abos had furnished the judge photocopies of the documents sought to be seized. The issuing judge could have formed a more specific description of these documents from said photocopies instead of merely employing a generic description thereof. The use of a generic term or a general description in a warrant is acceptable only when a more specific description of the things to be seized is unavailable. The failure to employ the specificity available will invalidate a general description in a warrant. [35] The use by the issuing judge of the terms "multiple sets of books of accounts, ledgers, journals, columnar books, cash register books, sales books or records, provisional & official receipts," "production record books/inventory lists, stock cards," "sales records, job order," "corporate financial records," and "bank statements/cancelled checks" is therefore unacceptable considering the circumstances of this case.As regards the terms "unregistered delivery receipts" and "unregistered purchase & sales invoices," however, we hold otherwise. The Solicitor General correctly argues that the serial markings of these documents need not be specified as it is not possible to do so precisely because they are unregistered.[36] Where, by the nature of the goods to be seized, their description must be rather general, it is not required that a technical description be given, as this would mean that no warrant could issue. Taking into consideration the nature of the articles so described, it is clear that no other more adequate and detailed description could have been given, particularly because it is difficult to give a particular description of the contents thereof.[37] Although it appears that photocopies of these unregistered documents were among those handed by Abos to the issuing judge, it would be impractical to require the latter to specify each and every receipt and invoice, and the contents thereof, to the minutest detail.The general description of most of the documents listed in the warrants does not render the entire warrant void. Insofar as the warrants authorize the search and seizure of unregistered delivery receipts and unregistered purchase and sales invoices, the warrants remain valid. The search warrant is severable, and those items not particularly described may be cut off without destroying the whole warrant. In United States v. Cook,[38] the United States Court of Appeals (Fifth Circuit) made the following pronouncement:x x x. The leading decision is Aday v. Superior Court, 53 Cal.2d 789, 362 P.2d 47, 13 Cal.Rptr. 415 (1961). In Aday, a warrant was issued authorizing the seizure of two particularly described books and myriad other generally described items. On appeal, the California Supreme Court held that only the books were particularly described in the warrant and lawfully seized. The court acknowledged that the warrant

Page 84: Warrantless Arrests, Searches and Seizures and Privacy of Communication

84

was flawed, but rather than suppress everything seized, the court chose to sever the defective portions of the warrant and suppress only those items that were not particularly described.Although the warrant was defective x x x it does not follow that it was invalid as a whole. Such a conclusion would mean that the seizure of certain articles, even though proper if viewed separately, must be condemned merely because the warrant was defective with respect to other articles. The invalid portions of the warrant are severable from the authorization relating to the named books x x x.

The search for and seizure of these books, if otherwise valid, were not rendered illegal by the defects concerning other articles.

x x x

x x x We agree with the reasoning of the Supreme Court of California and the majority of state courts that have considered this question and hold that in the usual case the district judge should sever the infirm portion of the search warrant as passes constitutional muster. See United States v. Giresi, 488 F.Supp. 445, 459-60 (D.N.J.1980). Items that were not described with the requisite particularity in the warrant should be suppressed, but suppression of all of the fruits of the search is hardly consistent with the purposes underlying exclusion. Suppression of only the items improperly described prohibits the Government from profiting from its own wrong and removes the court from considering illegally obtained evidence. Moreover, suppression of only those items that were not particularly described serves as an effective deterrent to those in the Government who would be tempted to secure a warrant without the necessary description. As the leading commentator has observed, "it would be harsh medicine indeed if a warrant which was issued on probable cause and which did particularly describe certain items were to be invalidated in toto merely because the affiant and the magistrate erred in seeking and permitting a search for other items as well." 2 W. LaFave, Search and Seizure: A Treatise on the Fourth Amendment §4.6(f) (1978).Accordingly, the items not particularly described in the warrants ought to be returned to petitioners.

Petitioners allege that the following articles, though not listed in the warrants, were also taken by the enforcing officers:

1. One (1) composition notebook containing Chinese characters,"

2. Two (2) pages writing with Chinese characters,

3. Two (2) pages Chinese character writing,

4. Two (2) packs of chemicals,

5. One (1) bound gate pass,

6. Surety Agreement.[39]

In addition, the searching party also seized items belonging to the Premier Industrial and Development Corporation (PIDC), which shares an office with petitioner Unifish.

The things belonging to petitioner not specifically mentioned in the warrants, like those not particularly described, must be ordered returned to petitioners. In order to comply with the constitutional provisions regulating the issuance of search warrants, the property to be seized under a warrant must be particularly

Page 85: Warrantless Arrests, Searches and Seizures and Privacy of Communication

85

described therein and no other property can be taken thereunder.[40] In Tambasen vs. People,[41] it was held:Moreover, by their seizure of articles not described in the search warrant, the police acted beyond the parameters of their authority under the search warrant. Section 2, Article III of the 1987 Constitution requires that a search warrant should particularly describe the things to be seized. "The evident purpose and intent of the requirement is to limit the things to be seized to those, and only those, particularly described in the search warrant, to leave the officers of the law with no discretion regarding what articles they should seize, to the end that unreasonable searches and seizures may not be made and that abuses may not be committed" (Corro v. Lising, 137 SCRA 541, 547 [1985]); Bache & Co. [Phil.], Inc. v. Ruiz, 37 SCRA 823 [1971]; Uy Kheytin v. Villareal, 42 Phil. 886 [1920]). The same constitutional provision is also aimed at preventing violations of security in person and property and unlawful invasions of the sanctity of the home, and giving remedy against such usurpations when attempted (People v. Damaso, 212 SCRA 547 [1992] citing Alvero v. Dizon, 76 Phil. 637, 646 [1946]).

Clearly then, the money which was not indicated in the search warrant, had been illegally seized from petitioner. The fact that the members of the police team were doing their task of pursuing subversives is not a valid excuse for the illegal seizure. The presumption juris tantum of regularity in the performance of official duty cannot by itself prevail against the constitutionally protected right of an individual (People v. Cruz, 231 SCRA 759 [1994]; People v. Veloso, 48 Phil. 169, 176 [1925]). Although public welfare is the foundation of the power to search and seize, such power must be exercised and the law enforced without transgressing the constitutional rights of the citizens (People v. Damaso, supra, citing Rodriguez v. Evangelista, 65 Phil. 230, 235 [1937]). As the Court aptly puts it inBagahilog v. Fernandez, 198 SCRA 614 (1991), "[z]eal in the pursuit of criminals cannot ennoble the use of arbitrary methods that the Constitution itself abhors."The seizure of the items not specified in the warrants cannot be justified by the directive in the penultimate paragraph thereof to "seize and take possession of other properties relative to such violation," which in no way can be characterized as a particular description of the things to be seized.

As regards the articles supposedly belonging to PIDC, we cannot order their return in the present proceedings. The legality of a seizure can be contested only by the party whose rights have been impaired thereby, and the objection to an unlawful search and seizure is purely personal and cannot be availed of by third parties.[42]

WHEREFORE, the Resolutions of respondent Court of Appeals dated 27 June 1996 and 14 May 1987, affirming the Order of the Regional Trial Court dated 17 July 1995, are hereby AFFIRMED insofar as said Resolutions upheld the validity of the subject Search Warrants authorizing the seizure of the unregistered delivery receipts and unregistered purchase and sales invoices, but REVERSED with respect to the rest of the articles subject of said warrants. The respondent Bureau of Internal Revenue is hereby ordered to return to petitioners all items seized from the subject premises and belonging to petitioners, except the unregistered delivery receipts and unregistered purchase and sales invoices.SO ORDERED.Davide, Jr., C.J., (Chairman), Puno, Pardo, and Ynares-Santiago, JJ., concur. 

Frank Uy & Unifish Packing Corp. vs Bureau of Internal Revenue et al

Search and Seizure – Requisites of a Valid Search Warrant

Page 86: Warrantless Arrests, Searches and Seizures and Privacy of Communication

86

In Sept 1993, Rodrigo Abos, a former employee of UPC reported to the BIR that Uy Chin Ho aka Frank

Uy, manager of UPC, was selling thousands of cartons of canned cartons without issuing a report. This is

a violation of Sec 253 & 263 of the Internal Revenue Code. In Oct 1993, the BIR requested before RTC

Cebu to issue a search warrant. Judge Gozo-Dadole issued a warrant on the same day. A second

warrant was issued which contains the same substance but has only one page, the same was dated Oct

1st 2003. These warrants were issued for the alleged violation by Uy of Sec 253. A third warrant was

issued on the same day for the alleged violation of Uy of Sec 238 in relation to sec 263. On the strength

of these warrants, agents of the BIR, accompanied by members of the PNP, on 2 Oct 1993, searched the

premises of the UPC. They seized, among other things, the records and documents of UPC. A return of

said search was duly made by Labaria with the RTC of Cebu. UPC filed a motion to quash the warrants

which was denied by the RTC. They appealed before the CA via certiorari. The CA dismissed the appeal

for a certiorari is not the proper remedy.

ISSUE: Whether or not there was a valid search warrant issued.

HELD: The SC ruled in favor of UPC and Uy in a way for it ordered the return of the seized items but

sustained the validity of the warrant. The SC ruled that the search warrant issued has not met some basic

requisites of validity. A search warrant must conform strictly to the requirements of the foregoing

constitutional and statutory provisions. These requirements, in outline form, are:

(1) the warrant must be issued upon probable cause;

(2) the probable cause must be determined by the judge himself and not by the applicant or any other

person;

(3) in the determination of probable cause, the judge must examine, under oath or affirmation, the

complainant and such witnesses as the latter may produce; and

(4) the warrant issued must particularly describe the place to be searched and persons or things to be

seized.

The SC noted that there has been inconsistencies in the description of the place to be searched as

indicated in the said warrants. Also the thing to be seized was not clearly defined by the judge. He used

generic itineraries. The warrants were also inconsistent as to who should be searched. One warrant was

directed only against Uy and the other was against Uy and UPC. The SC however noted that the

inconsistencies wered cured by the issuance of the latter warrant as it has revoked the two others.

Page 87: Warrantless Arrests, Searches and Seizures and Privacy of Communication

87

Section 2, Article III of the Constitution guarantees the right of the people against unreasonable searches

and seizures:

The right of the people to be secure in their persons, houses, papers, and effects against unreasonable

searches and seizures of whatever nature and for any purpose shall be inviolable, and no search warrant

or warrant of arrest shall issue except upon probable cause to be determined personally by the judge

after examination under oath or affirmation of the complainant and the witnesses he may produce, and

particularly describing the place to be searched and the persons or things to be seized.

[ G.R. No. 122092, May 19, 1999 ]

PAPER INDUSTRIES CORPORATION OF THE PHILIPPINES, EVARISTO M. NARVAEZ JR., RICARDO G. SANTIAGO, ROBERTO A. DORMENDO, REYDANDE D. AZUCENA, NICEFORO V. AVILA, FLORENTINO M. MULA, FELIX O. BAITO, HAROLD B. CELESTIAL, ELMEDENCIO C. CALIXTRO, CARLITO S. LEGACION, ALBINO T. LUBANG, JEREMIAS I. ABAD AND HERMINIO V. VILLAMIL, PETITIONERS, VS. JUDGE MAXIMIANO C. ASUNCION, PRESIDING JUDGE, BRANCH 104, REGIONAL TRIAL COURT OF QUEZON CITY; STATE PROSECUTOR LEO B. DACERA III; AND THE SPECIAL OPERATIONS UNIT OF THE PNP TRAFFIC MANAGEMENT COMMAND, RESPONDENTS.

D E C I S I O N

PANGANIBAN, J.:

To preserve and to uphold the constitutional right against unreasonable searches and seizures, the requisites for the issuance of a search warrant must be followed strictly. Where the judge fails to personally examine the applicant for a search warrant and the latter's witnesses, or where the witnesses testify on matters not of their own personal knowledge, the search warrant must be struck down.

The Case

Before us is a Petition for Certiorari and Prohibition[1] praying for (1) the nullification of Search Warrant No. 799 (95) and the Orders dated March 23, 1993 and August 3, 1995, issued by the Regional Trial Court (RTC), Branch 104, of Quezon City;[2] and (2) the issuance of a temporary restraining order (TRO) or an injunction against State Prosecutor Leo B. Dacera III, ordering him to desist from proceeding with IS No. 95-167.

Page 88: Warrantless Arrests, Searches and Seizures and Privacy of Communication

88

In its October 23, 1995 Resolution,[3] this Court issued the TRO prayed for and required the respondents to comment on the said Petition. On December 20, 1995, Respondent PNP Traffic Management Command filed its 31-page Opposition[4] to the Petition, together with 90 pages of annexes.[5] On February 22, 1996, the Office of the Solicitor General filed its Comment[6] agreeing with petitioners that the writs prayed for must be granted. After petitioners filed a Reply to the Opposition, the Court gave due course to the Petition and required the parties to submit their respective memoranda.

In view of the contrary opinion of the Office of the Solicitor General, the Court, in its February 5, 1997 Resolution,[7] required State Prosecutor Leo B. Dacera to prepare the memorandum for the public respondents. After issuing a show-cause order to Dacera on June 23, 1997,[8] the Court in its September 24, 1997 Resolution gave him a non-extendible period ending on October 31, 1997 within which to file the required memorandum. In view of Dacera's manifestation that he was only a nominal party and that he had yet to receive the records of the case from the PNP, the Court, in its December 8, 1999 Resolution, ordered the Special Operations Unit (SOU) of the PNP Traffic Management Command to file its memorandum within thirty days from notice; "otherwise, the petition will be deemed submitted for decision."[9] Even after the expiration of the said period, the required pleading was not yet received by this Court.

Hence, this Court considered Respondent SOU's refusal/failure to submit its memorandum as a waiver of its privilege to do so.

The Facts

On January 25, 1995, Police Chief Inspector Napoleon B. Pascua applied for a search warrant before the said RTC of Quezon City, stating:[10]

"1. That the management of Paper Industries Corporation of the Philippines, located at PICOP compound, Barangay Tabon, Bislig, Surigao del Sur, represented by its Sr. Vice President Ricardo G[.] Santiago, is in possession or ha[s] in [its] control high powered firearms, ammunitions, explosives, which are the subject of the offense, or used or intended to be used in committing the offense, and which xxx are [being kept] and conceal[ed] in the premises herein described.

"2. That a Search Warrant should be issued to enable any agent of the law to take possession and bring to this Honorable Court the following described properties:

'Seventy (70) M16 Armalite rifles cal. 5.56, ten (10) M16 US rifles, two (2) AK-47 rifle[s], two (2) UZI submachinegun[s], two (2) M203 Grenade Launcher[s] cal.40mm, ten (10) cal.45 pistol[s], ten (10)

Page 89: Warrantless Arrests, Searches and Seizures and Privacy of Communication

89

cal.38 revolver[s], two (2) ammunition reloading machine[s], assorted ammunitions for said calibers of firearms and ten (10) handgrenades.'

Attached to the application[11] were the joint Deposition of SPO3 Cicero S. Bacolod and SPO2 Cecilio T. Morito,[12] as well as a summary of the information and the supplementary statements of Mario Enad and Felipe Moreno.

After propounding several questions to Bacolod, Judge Maximiano C. Asuncion issued the contested search warrant,[13] the pertinent portion of which reads:

"It appearing to the satisfaction of the undersigned, after examining under oath, SPO3 Cicero S. Bacolod, that there is probable cause to believe that the management of Paper Industries Corporation of the Philippines, located at PICOP Compound, Barangay Tabon, Bislig, Surigao del Sur, represented by its Sr. Vice President Ricardo G. Santiago, has in its possession or control the following:

Seventy (70) M16 Armalite rifles cal. 5.56

Ten (10) M14 US rifles

Two (2) AK-47 rifle[s]

Two (2) UZI submachinegun[s]

Two (2) M203 Grenade Launcher[s] cal. 40mm.

Ten (10) cal 45 pistol[s]

Ten (10) cal 38 revolver[s]

Two (2) ammunition reloading machine[s]

Assorted ammunitions for said calibers of firearms

Ten (10) handgrenades

in violation of the Provisions of PD 1866 (Illegal Possession of Firearms, Ammunition and Explosives), and the same should be seized and brought before this Court.

"NOW, THEREFORE, you are hereby authorized to make an immediate search daytime between 8:00 a.m. [and] 4:00 p.m. of the aforementioned premises and to seize and bring the articles above-described and make an immediate return there[of]"[14]

Page 90: Warrantless Arrests, Searches and Seizures and Privacy of Communication

90

On February 4, 1995, the police enforced the search warrant at the PICOP compound and seized the following:[15]

MAKE/TYPE

CALIBER

SERIAL NUMBER

BRAND

01

M16 Rifle

5.56

RP 175636

Elisco

02

M16 Rifle

5.56

RP 175636 (Tampered)

Elisco

03

M16 Rifle

5.56

RP 171702

Elisco

04

M16 Rifle

5.56

Defaced

Page 91: Warrantless Arrests, Searches and Seizures and Privacy of Communication

91

Elisco

05

M16 Rifle

5.56

RP174253 (Tampered)

Elisco

06

M16 Rifle

5.56

RP173627 (Tampered)

Elisco

07

M16 Rifle

5.56

RP171337

Elisco

08

M16 Rifle

5.56

RP171114

Elisco

09

M16 Rifle

5.56

RP171114 (Tampered)

Page 92: Warrantless Arrests, Searches and Seizures and Privacy of Communication

92

Elisco

10

M16 Rifle

5.56

RP171167 (Tampered)

Elisco

11

M16 Rifle

5.56

170881 (Tampered)

Elisco

12

M16 Rifle

5.56

RP170897

Elisco

13

M16 Rifle

5.56

RP171509

Elisco

(With pending case-Casaway Case)

14

M16 Rifle

5.56

Page 93: Warrantless Arrests, Searches and Seizures and Privacy of Communication

93

RP 171754

Elisco

15

M16 Rifle

5.56

RP170881 (Tampered)

Elisco

16

M16 Rifle

5.56

RP174637

Elisco

17

M16 Rifle

5.56

RP171366

Elisco

18

M16 Rifle

5.56

RP174637 (Tampered)

Elisco

19

M16 Rifle

5.56

Page 94: Warrantless Arrests, Searches and Seizures and Privacy of Communication

94

RP174610

Elisco

20

M16 Rifle

5.56

RP171367 (Tampered)

Elisco

01

M14Rifle

7.62

1499694

Elisco

02

M14Rifle

7.62

889163

Elisco

01

BAR

Cal. 30

865975

Royal

01

Carbine M1

Cal. 30

Page 95: Warrantless Arrests, Searches and Seizures and Privacy of Communication

95

384181

US Carbin

02

Carbine M1

Cal. 30

998201

US Carbin

01

Garand M1

Cal. 30

1194008

Springfield

02

Garand M1

Cal. 30

3123784

Springfield

01

Shotgun

12 Gauge

H359704

Omega

02

Shotgun

Page 96: Warrantless Arrests, Searches and Seizures and Privacy of Communication

96

12 Gauge

9211

Homemade

(Paltik)

MAGAZINE ASSEMBLY

QTY.

01 M16 (long)

29 pcs.

02 M16 (short)

48 pcs.

03 Carbine M1

171 pcs.

04 BAR

19 pcs.

LIVE AMMUNITION

QTY.

Page 97: Warrantless Arrests, Searches and Seizures and Privacy of Communication

97

01 M16 2,023

rounds

03 Carbine M1

276 rounds

04 M-60 Cal. 7.62

1,800 rounds

05 M1 Garand

1,278 rounds

06 Rifle Grenade

11 rounds

07 Hand Grenade

4 pcs.

AMMO DAM POST NO. 24

MAKE/TYPE

Page 98: Warrantless Arrests, Searches and Seizures and Privacy of Communication

98

CALIBER

SERIAL NUMBER

BRAND

01.

M16 Rifle

5.56

171425 (Tampered)

Gyno Corp.

02.

Machine Pistol

22

651 (Tampered)

Landmann

MAGAZINE ASSEMBLY

QTY.

01. M16 (short)

3 pcs.

02. M16 (long)

1 pc.

Page 99: Warrantless Arrests, Searches and Seizures and Privacy of Communication

99

03. M14

8 pcs.

04. Clip M1 Garand

3 pcs.

05. Mag Assy. Cal .22

1 pc.

LIVE AMMUNITION

QTY.

01. M16

73 rounds

02. M14

160 rounds

03. M1 Garand Cal .30

30 rounds

04. Rifle Grenade

Page 100: Warrantless Arrests, Searches and Seizures and Privacy of Communication

100

1 round

MANAGEMENT INTEL/INVEST UNIT

MAKE/TYPE

CALIBER

SERIAL NUMBER

BRAND

01.

M16 Rifle

5.56

RP 171725

Elisco

02.

M16 Rifle

5.56

RP 170799 (Tampered)

Elisco

03.

M16 Rifle

5.56

RP 132320

Page 101: Warrantless Arrests, Searches and Seizures and Privacy of Communication

101

Elisco

04.

Machine

9 MM

54887

Intratec

Pistol

05.

Three (3)

12 Gauge

Surit-Surit (H)

Shotguns

MAGAZINE ASSEMBLY

QTY.

01. M16 (long)

3 pcs.

02. M16 (short)

4 pcs.

03. Intratec

Page 102: Warrantless Arrests, Searches and Seizures and Privacy of Communication

102

1 pc.

04. US Carbine (defective)

2 pcs.

LIVE AMMUNITION

QTY.

01. M16

147 rds.

02. Cal. 30

5 rounds

03. 12 gauge Shotgun

7 rounds

04. Carbine

5 rounds

05. Rifle grenade (AVA-0051-84/0056-84)

2 rounds

Page 103: Warrantless Arrests, Searches and Seizures and Privacy of Communication

103

06. 9MM

30 rounds

NEW ARMORY POST NO. 16

MAKE/TYPE

CALIBER

SERIAL NUMBER

BRAND

01.

Shotgun

12 Gauge

A359910

Armscor

02.

Shotgun

12 Gauge

Page 104: Warrantless Arrests, Searches and Seizures and Privacy of Communication

104

A359716

Armscor

03.

Shotgun

12 Gauge

A359706

Armscor

04.

Shotgun

12 Gauge

A359707

Armscor

05.

Shotgun

12 Gauge

1036847

Armscor

06.

Shotgun

12 Gauge

A359702

Armscor

07.

Shotgun

12 Gauge

Page 105: Warrantless Arrests, Searches and Seizures and Privacy of Communication

105

A359732

Armscor

08.

Shotgun

12 Gauge

A359728

Armscor

09.

Shotgun

12 Gauge

A359708

Armscor

10.

Shotgun

12 Gauge

A359711

Armscor

11.

Shotgun

12 Gauge

A359723

Armscor

12.

Shotgun

Page 106: Warrantless Arrests, Searches and Seizures and Privacy of Communication

106

12 Gauge

A359713

Armscor

13.

Shotgun

12 Gauge

1031271

Armscor

14.

Shotgun

12 Gauge

A262338

SB

15.

Shotgun

12 Gauge

A261619

SB

16.

Shotgun

12 Gauge

Defaced

Not Indicated

LIVE AMMUNITION

Page 107: Warrantless Arrests, Searches and Seizures and Privacy of Communication

107

QTY.

01. 12 GAUGE shotgun

306 rds.

02. M16

2,349 rds.

MAGAZINE ASSEMBLY

QTY.

01. Carbine (defective)

76 pcs.

02. Cal. 22 -do-

16 pcs.

03. M16 (long-defective)

2 pcs.

04. M16 (short-defective)

2 pcs.

Page 108: Warrantless Arrests, Searches and Seizures and Privacy of Communication

108

05. Thompson (defective)

8 pcs.

06. Shotgun 12 Gauge (defective)

17 pcs.

07. BAR (defective)

2 pcs.

Believing that the warrant was invalid and the search unreasonable, the petitioners filed a "Motion to Quash"[16] before the trial court. Subsequently, they also filed a "Supplemental Pleading to the Motion to Quash" and a "Motion to Suppress Evidence."[17]

On March 23, 1995, the RTC issued the first contested Order which denied petitioners' motions.[18] On August 3, 1995, the trial court rendered its second contested Order[19] denying petitioners' Motion for Reconsideration.[20]

Hence, this recourse to this Court on pure questions of law.

Issues

In their Memorandum, petitioners submit the following grounds in support of their cause:[21]

"I

Petitioners respectfully submit that Judge Asuncion has committed grave abuse of discretion or has exceeded his jurisdiction in refusing to quash Search Warrant No. 799(95). Probable cause

Page 109: Warrantless Arrests, Searches and Seizures and Privacy of Communication

109

[has] not xxx been sufficiently established and partaking as it does of the nature of a general warrant.

"II

Petitioners respectfully submit that Judge Asuncion has committed grave abuse of discretion or has exceeded his jurisdiction in refusing to quash Search Warrant No. 799(95) on the ground that it was unlawfully served or implemented.

"III

Petitioners respectfully submit that State Prosecutor Dacera is acting with grave abuse of discretion or exceeding his jurisdiction in continuing with the proceedings in IS No. 95-167 on the basis of illegally seized evidence."

In the main, petitioners question the validity of the search warrant. As a preliminary matter, we shall also discuss respondents' argument that the Petition should be dismissed for raising factual questions.

This Court's Ruling

The petition is meritorious.

Preliminary Issue:

Alleged Factual Questions

In their Opposition, respondents argue that the Petition should be dismissed for raising questions of fact, which are not proper in a petition for certiorari under Rule 65. They maintain that the Petition merely assails the "factual basis for the issuance of the warrant and the regularity of its implementation."[22]

This argument is not convincing. It is settled that "there is a question of fact when the doubt arises as to the truth or the falsity of alleged facts."[23] In the present case, petitioners do not

Page 110: Warrantless Arrests, Searches and Seizures and Privacy of Communication

110

question the truth of the facts as found by the judge; rather, they are assailing the way in which those findings were arrived at, a procedure which they contend was violative of the Constitution and the Rules of Court. We agree that the Petition raises only questions of law, which may be resolved in the present case.

Main Issue:

Validity of the Search Warrant

The fundamental right against unreasonable searches and seizures and the basic conditions for the issuance of a search warrant are laid down in Section 2, Article III of the 1987 Constitution, which reads:

"The right of the people to be secure in their persons, houses, papers and effects against unreasonable searches and seizures of whatever nature and for any purpose shall be inviolable, and no search warrant or warrant of arrest shall issue except upon probable cause to be determined personally by the judge after examination under oath or affirmation of the complainant and the witnesses he may produce, and particularly describing the place to be searched and the persons or things to be seized." (Emphasis supplied)

Consistent with the foregoing constitutional provision, Sections 3 and 4, Rule 126 of the Rules of Court,[24] detail the requisites for the issuance of a valid search warrant as follows:

"SEC. 3. Requisite for issuing search warrant. -- A search warrant shall not issue but upon probable cause in connection with one specific offense to be determined personally by the judge after examination under oath or affirmation of the complainant and the witnesses he may produce, and particularly describing the place to be searched and the things to be seized."

"SEC. 4. Examination of complainant; record. -- The judge must, before issuing the warrant, personally examine in the form of searching questions and answers, in writing and under oath the complainant and any witnesses he may produce on facts personally known to them and attach to the record their sworn statements together with any affidavits submitted."

More simply stated, the requisites of a valid search warrant are: (1) probable cause is present; (2) such presence is determined personally by the judge; (3) the complainant and the witnesses he or she may produce are personally examined by the judge, in writing and under oath or affirmation; (4) the applicant and the witnesses testify on facts personally known to them; and (5) the warrant specifically describes the place to be searched and the things to be seized.[25] In the present case, the search warrant is invalid because (1) the trial court failed to examine personally the complainant and the other deponents; (2) SPO3 Cicero Bacolod, who appeared during the hearing for the issuance of the search warrant, had no personal knowledge that petitioners were not licensed to possess the subject firearms; and (3) the place to be searched was not described with particularity.

No Personal Examination of the Witnesses

Page 111: Warrantless Arrests, Searches and Seizures and Privacy of Communication

111

In his Order dated March 23, 1995, the trial judge insisted that the search warrant was valid, stating that "before issuing the subject warrant, the court propounded searching questions to the applicant and the witnesses in order to determine whether there was probable cause x x x."[26] (Emphasis supplied.) This was supported by the Opposition to the Motion to Quash, which argued that "it is erroneous for PICOP to allege that the Honorable Court did not propound searching questions upon applicant P/Chief Inspector Napoleon Pascua and the witnesses he produced."[27] The records, however, proclaim otherwise.

As earlier stated, Chief Inspector Pascua's application for a search warrant was supported by (1) the joint Deposition of SPO3 Cicero S. Bacolod and SPO2 Cecilio T. Morito, (2) a summary of information and (3) supplementary statements of Mario Enad and Felipe Moreno. Except for Pascua and Bacolod, however, none of the aforementioned witnesses and policemen appeared before the trial court. Moreover, the applicant's participation in the hearing for the issuance of the search warrant consisted only of introducing Witness Bacolod:[28]

"COURT:

Where is the witness for this application for search warrant?

P/Chief Insp. NAPOLEON PASCUA:

SPO3 CICERO S. BACOLOD, Your Honor.

COURT:

Swear the witness.

STENOGRAPHER: (To the witness)

Please raise your right hand, sir. Do you swear to tell the truth, the whole truth and nothing but the truth before this Court?

WITNESS:

Yes Ma'am.

STENOGRAPHER:

Page 112: Warrantless Arrests, Searches and Seizures and Privacy of Communication

112

Please state your name, age, civil status, occupation, address and other personal circumstances.

WITNESS:

SPO3 Cicero S. Bacolod, 42 years old, married, policeman, c/o Camp Crame, Quezon City, SOU, TMC.

x x x x x x x x x"

Chief Inspector Pascua was asked nothing else, and he said nothing more. In fact, he failed even to affirm his application. Contrary to his statement, the trial judge failed to propound questions, let alone probing questions, to the applicant and to his witnesses other than Bacolod (whose testimony, as will later be shown, is also improper). Obviously, His Honor relied mainly on their affidavits. This Court has frowned on this practice in this language:

"Mere affidavits of the complainant and his witnesses are thus not sufficient. The examining Judge has to take depositions in writing of the complainant and the witnesses he may produce and attach them to the record. Such written deposition is necessary in order that the Judge may be able to properly determine the existence or non-existence of the probable cause, to hold liable for perjury the person giving it if it will be found later that his declarations are false.

x x x x x x x x x

"It is axiomatic that the examination must be probing and exhaustive, not merely routinary or pro-forma, if the claimed probable cause is to be established. The examining magistrate must not simply rehash the contents of the affidavit but must make his own inquiry on the intent and justification of the application."[29]

Bacolod's Testimony Pertained Not to

Facts Personally Known to Him

Bacolod appeared during the hearing and was extensively examined by the judge. But his testimony showed that he did not have personal knowledge that the petitioners, in violation of PD 1866, were not licensed to possess firearms, ammunitions or explosives. In his Deposition, he stated:

"Q How do you know that said properties were subject of the offense?

A

Page 113: Warrantless Arrests, Searches and Seizures and Privacy of Communication

113

Sir, as a result of our intensified surveillance and case build up for several days, we gathered informations from reliable sources that subject properties [which] are in their possession and control [are] the herein described properties subject of the offense. (Summary of Information dtd Oct `94, SS's of Mario Enad and Felipe Moreno both dtd 30 Nov `94 are hereto attached)."[30]

When questioned by the judge, Bacolod stated merely that he believed that the PICOP security guards had no license to possess the subject firearms. This, however, does not meet the requirement that a witness must testify on his personal knowledge, not belief. He declared:

"Q

This is an application for Search Warrant against Paper Industries Corporation located at PICOP Compound, Barangay Tabon, Bislig, Surigao del Sur. How come that you have knowledge that there are illegal firearms in that place?

A

At Camp Crame, Quezon City, I was dispatched by our Commander to investigate the alleged assassination plot of Congressman Amante.

Q

In the course of your investigation, what happened?

A

We found out that some of the suspects in the alleged assassination plot are employees of PICOP.

Q

Know[ing] that the suspects are employees of PICOP, what did you do?

A

We conducted the surveillance in that area inside the compound of PICOP in Tabon.

Q

What did you find xxx?

A

I found xxx several high-powered firearms.

Page 114: Warrantless Arrests, Searches and Seizures and Privacy of Communication

114

Q

How were you able to investigate the compound of PICOP?

A

I exerted effort to enter the said compound.

Q

By what means?

A

By pretending to have some official business with the company.

Q

So, in that aspect, you were able to investigate the compound of PICOP?

A

Yes, sir.

Q

What did you f[i]nd xxxt?

A

I found xxx several high-powered firearms being kept in the compound of PICOP.

Q

Where are those located?

A

Sir, there are firearms kept inside the ammo dam.

Page 115: Warrantless Arrests, Searches and Seizures and Privacy of Communication

115

Q

Inside the compound?

A

Located inside the compound.

Q

Then what?

A

Others, sir, were kept in the security headquarters or office.

Q

You mean to say that this Paper Industries Corporation has its own security guards?

A

Yes, they call it Blue Guards.

Q

You mean to say that their own security guards guarded the PICOP?

A

Yes, sir.

Q

So, it is possible that the firearms used by the security guards are illegally obtained?

A

I believe they have no license to possess high-powered firearms. As far as the verification at FEU, Camp Crame, [is concerned,] they have no license. (Emphasis supplied.)

Page 116: Warrantless Arrests, Searches and Seizures and Privacy of Communication

116

Q

Have you investigated the Blue Guards Security Agency?

A

I conducted the inquiry.

Q

What did you find out?

A

They are using firearms owned by PICOP.

Q

Using firearms owned by PICOP?

A

Yes, sir.

Q

You mean to say that this Blue Guard Security Agency has no firearms of their own?

A

No high-powered firearms.

Q

By the way, Mr. Witness, what kind of firearms have you seen inside the compound of PICOP?

A

There are M-16 armalite rifles.

Page 117: Warrantless Arrests, Searches and Seizures and Privacy of Communication

117

Q

What else?

A

AK-47, armalites, M-203 Grenade Launcher, M-14 US rifles, .38 caliber revolvers, .45 caliber pistols, several handgrenades and ammos."[31] (Emphasis supplied)

Moreover, Bacolod failed to affirm that none of the firearms seen inside the PICOP compound was licensed. Bacolod merely declared that the security agency and its guards were not licensed. He also said that some of the firearms were owned by PICOP. Yet, he made no statement before the trial court that PICOP, aside from the security agency, had no license to possess those firearms. Worse, the applicant and his witnesses inexplicably failed to attach to the application a copy of the aforementioned "no license" certification from the Firearms and Explosives Office (FEO) of the PNP, or to present it during the hearing. Such certification could have been easily obtained, considering that the FEO was located in Camp Crame where the unit of Bacolod was also based. In People v. Judge Estrada,[32] the Court held:

"The facts and circumstances that would show probable cause must be the best evidence that could be obtained under the circumstances. The introduction of such evidence is necessary in cases where the issue is the existence of the negative ingredient of the offense charged - for instance, the absence of a license required by law, as in the present case - and such evidence is within the knowledge and control of the applicant who could easily produce the same. But if the best evidence could not be secured at the time of the application, the applicant must show a justifiable reason therefor during the examination by the judge."

Particularity of the

Place to Be Searched

In view of the manifest objective of the constitutional safeguard against unreasonable search, the Constitution and the Rules limit the place to be searched only to those described in the warrant.[33] Thus, this Court has held that "this constitutional right [i]s the embodiment of a spiritual concept: the belief that to value the privacy of home and person and to afford its constitutional protection against the long reach of government is no less than to value human dignity, and that his privacy must not be disturbed except in case of overriding social need, and then only under stringent procedural safeguards."[34] Additionally, the requisite of particularity is related to the probable cause requirement in that, at least under some circumstances, the lack of a more specific description will make it apparent that there has not been a sufficient showing to the magistrate that the described items are to be found in a particular place.[35]

In the present case, the assailed search warrant failed to describe the place with particularity. It simply authorizes a search of "the aforementioned premises," but it did not specify such premises. The warrant identifies only one place, and that is the "Paper Industries Corporation of the Philippines, located at PICOP Compound, Barangay Tabon, Bislig[,] Surigao del Sur." The PICOP compound, however, is made up of "200 offices/buildings, 15 plants, 84 staff houses, 1

Page 118: Warrantless Arrests, Searches and Seizures and Privacy of Communication

118

airstrip, 3 piers/wharves, 23 warehouses, 6 POL depots/quick service outlets and some 800 miscellaneous structures, all of which are spread out over some one hundred fifty-five hectares."[36] Obviously, the warrant gives the police officers unbridled and thus illegal authority to search all the structures found inside the PICOP compound.[37]

In their Opposition, the police state that they complied with the constitutional requirement, because they submitted sketches of the premises to be searched when they applied for the warrant. They add that not one of the PICOP Compound housing units was searched, because they were not among those identified during the hearing.[38]

These arguments are not convincing. The sketches allegedly submitted by the police were not made integral parts of the search warrant issued by Judge Asuncion. Moreover, the fact that the raiding police team knew which of the buildings or structures in the PICOP Compound housed firearms and ammunitions did not justify the lack of particulars of the place to be searched.[39] Otherwise, confusion would arise regarding the subject of the warrant - the place indicated in the warrant or the place identified by the police. Such conflict invites uncalled for mischief or abuse of discretion on the part of law enforcers.

Thus, in People v. Court of Appeals,[40] this Court ruled that the police had no authority to search the apartment behind the store, which was the place indicated in the warrant, even if they really intended it to be the subject of their application. Indeed, the place to be searched cannot be changed, enlarged or amplified by the police, viz.:

"x x x. In the instant case, there is no ambiguity at all in the warrant. The ambiguity lies outside the instrument, arising from the absence of a meeting of the minds as to the place to be searched between the applicants for the warrant and the Judge issuing the same; and what was done was to substitute for the place that the Judge had written down in the warrant, the premises that the executing officers had in their mind. This should not have been done. It [was] neither fair nor licit to allow police officers to search a place different from that stated in the warrant on the claim that the place actually searched - although not that specified in the warrant - [was] exactly what they had in view when they applied for the warrant and had demarcated in their supporting evidence. What is material in determining the validity of a search is the place stated in the warrant itself, not what the applicants had in their thoughts, or had represented in the proofs they submitted to the court issuing the warrant. Indeed, following the officers' theory, in the context of the facts of this case, all four (4) apartment units at the rear of Abigail's Variety Store would have been fair game for a search.

"The place to be searched, as set out in the warrant, cannot be amplified or modified by the officers' own personal knowledge of the premises, or the evidence they adduced in support of their application for the warrant. Such a change is proscribed by the Constitution which requires inter alia the search warrant to particularly describe the place to be searched as well as the persons or things to be seized. It would concede to police officers the power of choosing the place to be searched, even if it not be that delineated in the warrant. It would open wide the door to abuse of the search process, and grant to officers executing a search warrant that discretion which the Constitution has precisely removed from them. The particularization of the description of the place to be searched may properly be done only by the Judge, and only in the warrant itself;

Page 119: Warrantless Arrests, Searches and Seizures and Privacy of Communication

119

it cannot be left to the discretion of the police officers conducting the search." (Emphasis supplied.)

Seized Firearms and Explosives

Inadmissible in Evidence

As a result of the seizure of the firearms, effected pursuant to Search Warrant No. 799 (95) issued by the respondent judge, the PNP filed with the Department of Justice a complaint docketed as IS No. 95-167 against herein petitioners for illegal possession of firearms. State Prosecutor Dacera, to whom the Complaint was assigned for preliminary investigation, issued a subpoena requiring petitioners to file their counter-affidavits.

Instead of complying with the subpoena, petitioners asked for the suspension of the preliminary investigation, pending the resolution of their motion to quash the search warrant. They argued, as they do now, that the illegally obtained firearms could not be the basis of the criminal Complaint. Their motion was denied. A subsequent Motion for Reconsideration met the same fate. In the present Petition for Certiorari and Prohibition, petitioners assert that "State Prosecutor Dacera cannot have any tenable basis for continuing with the proceedings in IS No. 95-167."[41]

Because the search warrant was procured in violation of the Constitution and the Rules of Court, all the firearms, explosives and other materials seized were "inadmissible for any purpose in any proceeding."[42] As the Court noted in an earlier case, the exclusion of unlawfully seized evidence was "the only practical means of enforcing the constitutional injunction against unreasonable searches and seizures."[43] Verily, they are the "fruits of the poisonous tree." Without this exclusionary rule, the constitutional right "would be so ephemeral and so neatly severed from its conceptual nexus with the freedom from all brutish means of coercing evidence xxx."[44]

In the present case, the complaint for illegal possession of firearms is based on the firearms and other materials seized pursuant to Search Warrant No. 799 (95). Since these illegally obtained pieces of evidence are inadmissible, the Complaint and the proceedings before State Prosecutor Dacera have no more leg to stand on.

This Court sympathizes with the police effort to stamp out criminality and to maintain peace and order in the country; however, it reminds the law enforcement authorities that they must do so only upon strict observance of the constitutional and statutory rights of our people. Indeed, "there is a right way to do the right thing at the right time for the right reason."[45]

WHEREFORE, the instant petition for certiorari and prohibition is hereby GRANTED and Search Warrant No. 799 (95) accordingly declared NULL and VOID. The temporary restraining order

Page 120: Warrantless Arrests, Searches and Seizures and Privacy of Communication

120

issued by this Court on October 23, 1995 is hereby MADE PERMANENT. No pronouncement as to costs.

SO ORDERED.

Romero (Chairman), Vitug, and Gonzaga-Reyes, JJ., concur.

Purisima, J., did not participate in the deliberations.

PICOP v. Asuncion, 307 SCRA 253) (1999)

FACTS: On January 25, 1995, Police Chief Inspector Napoleon B. Pascua applied for a search warrant before the RTC of Quezon City, stating: 1. That the management of Paper Industries Corporation of the Philippines, located at PICOP compound, is in possession or ha[s] in [its] control high powered firearms, ammunitions, explosives, which are the subject of the offense, or used or intended to be used in committing the offense, and which . . . are [being kept] and conceal[ed] in the premises described; 2. That a Search Warrant should be issued to enable any agent of the law to take possession and bring to the described properties. After propounding several questions to Bacolod, Judge Maximiano C. Asuncion issued the contested search warrant. On February 4, 1995, the police enforced the search warrant at the PICOP compound and seized a number of firearms and explosives. Believing that the warrant was invalid and the search unreasonable, the petitioners filed a “Motion to Quash” before the trial court. Subsequently, they also filed a “Supplemental Pleading to the Motion to Quash” and a “Motion to SuppressEvidence.” On March 23, 1995, the RTC issued the first contested Order which denied petitioners’ motions. On August 3, 1995, the trial court rendered its second contested Order denying petitioners’ Motion for Reconsideration.

ISSUE: WON the search warrant issued was valid

HELD:

The requisites of a valid search warrant are: (1) probable cause is present; (2) such presence is determined personally by the judge; (3) the complainant and the witnesses he or she may produce are personally examined by the judge, in writing and under oath or affirmation; (4) the applicant and the witnesses testify on facts personally known to them; and (5) the warrant specifically describes the place to be searched and the things to be seized. In the present case, the search warrant is invalid because (1) the trial court failed to examine personally the complainant and the other deponents; (2) SPO3 Cicero Bacolod, who appeared during the hearing for the issuance of the search warrant, had no personal knowledge that petitioners were not licensed to possess the subject firearms; and (3) the place to be searched was not described with particularity.

Page 121: Warrantless Arrests, Searches and Seizures and Privacy of Communication

121

[ G.R. No. 149878, July 01, 2003 ]

PEOPLE OF THE PHILIPPINES, PLAINTIFF-APPELLEE, VS. TIU WON CHUA A.K.A. "TIMOTHY TIU" AND QUI YALING Y CHUA A.K.A. "SUN TEE SY Y CHUA," ACCUSED-APPELLANT.

D E C I S I O N

PUNO, J.:

This is an appeal from the decision of the Regional Trial Court (RTC) of Manila, Branch 27, convicting appellants Tiu Won Chua a.k.a. Timothy Tiu (Tiu Won) and Qui Yaling y Chua a.k.a. Sun Tee Sy y Chua (Qui Yaling) for violation of Section 16, Article III of Republic Act No. 6425, otherwise known as the Dangerous Drugs Act of 1972, as amended by Republic Act No. 7659.

Appellants were charged with the crime of illegal possession of a regulated drug, i.e., methamphetamine hydrochloride, otherwise known as "shabu," in an information which reads:

The undersigned accuses TIU WON CHUA aka "Timothy Tiu" and QUI YALING Y CHUA aka "Sun Tee Sy Y Chua" of violation of Section 16, Article III in relation to Section 2 (e-2), Article I of Republic Act No. 6425, as amended by Batas Pambansa Blg. 179 and as further amended by Republic Act No. 7659, committed as follows:

That on or about the 3rd day of October 1998, in the City of Manila, Philippines, the said accused without being authorized by law to possess or use any regulated drug, did then and there wilfully, unlawfully, knowingly and jointly have in their possession and under their custody and control the following, to wit:

A sealed plastic bag containing two three four point five (234.5) grams of white crystalline substance;

Four (4) separate sealed plastic bags containing six point two two four three (6.2243) grams of white crystalline substance;

Sixteen (16) separate sealed plastic bags containing twenty point three six seven three (20.3673) grams of white crystalline substance; or a total of 261.0916 grams, and;

Page 122: Warrantless Arrests, Searches and Seizures and Privacy of Communication

122

An improvised tooter with traces of crystalline substance

known as "SHABU" containing methamphetamine hydrochloride, a regulated drug, without the corresponding license or prescription thereof.

Contrary to law.[1]

During arraignment, a plea of not guilty was entered. Appellants, with the assistance of counsel, and the prosecution stipulated on the following facts:

The authenticity of the following documents:

The letter of Police Senior Inspector Angelo Martin of WPD, District Intelligence Division, United Nations Avenue, Ermita, Manila, dated October 12, 1998, to the Director of the NBI requesting the latter to conduct a laboratory examination of the specimen mentioned therein;

The Certification issued by Forensic Chemist Loreto Bravo of the NBI, dated October 13, 1998, to the effect that the specimen mentioned and enumerated therein gave positive results for methamphetamine hydrochloride, Exhibit "B"; and

Dangerous Drug Report No. 98-1200 issued by Forensic Chemist Bravo, dated October 13, 1998, to the effect that the specimen mentioned therein gave positive results for methamphetamine hydrochloride;

The existence of one plastic bag containing 234.5 grams of methamphetamine hydrochloride, Exhibit "D"; four (4) plastic sachets also containing methamphetamine hydrochloride with a total net weight of 6.2243 grams, Exhibits "E", "E-1", "E-2" and "E-3"; additional 16 plastic sachets containing methamphetamine hydrochloride with a total net weight of 20.3673 grams, Exhibits "F", "F-1" to "F-15", and one improvised tooter with a length of 8 inches more or less and with a red plastic band, Exhibit "G";

Forensic Chemist Loreto Bravo has no personal knowledge as to the source of the regulated drug in question; and

Tiu Won Chua and Qui Yaling y Chua as stated in the information are the true and correct names of the two accused.[2]

Page 123: Warrantless Arrests, Searches and Seizures and Privacy of Communication

123

The witnesses presented by the prosecution were SPO1 Anthony de Leon, PO2 Artemio Santillan and PO3 Albert Amurao. Their testimonies show that the police authorities, acting on an information that drug-related activities were going on at the HCL Building, 1025 Masangkay St., Binondo, Manila, surveyed the place on October 2, 3, 4 and 5, 1998. At about 10 p.m. of October 6, they conducted a test-buy operation, together with a Chinese-speaking asset. They were able to buy P2,000.00 worth of substance from appellants, which, upon examination by the PNP crime laboratory, proved positive for methamphetamine hydrochloride.[3] Nonetheless, they did not immediately arrest the suspects but applied for a warrant to search Unit 4-B of HCL Building, 1025 Masangkay St., Binondo, Manila. Their application to search the unit supposedly owned by "Timothy Tiu" was granted by Judge Ramon Makasiar of Branch 35 of the RTC of Manila on October 9.[4] Armed with the warrant, they proceeded to the place and learned that Tiu Won was not inside the building. They waited outside but Tiu Won did not come. After several stakeouts, they were able to implement the warrant on October 12. Failing to get the cooperation of the barangay officials, they requested the presence of the building coordinator, Noel Olarte, and his wife, Joji, who acted as witnesses.

During the enforcement of the warrant, there were three (3) persons inside the apartment, namely, appellants Tiu Won and Qui Yaling, and a housemaid. The search was conducted on the sala and in the three (3) bedrooms of Unit 4-B. On top of a table inside the master's bedroom, one (1) big pack, containing 234.5 grams of shabu, was found inside a black leather man's handbag supposedly owned by Tiu Won, while sixteen (16) small packs of shabu weighing 20.3673 grams were found inside a lady's handbag allegedly owned by Qui Yaling. Also contained in the inventory were the following items: an improvised tooter, a weighing scale, an improvised burner and one rolled tissue paper.[5] The authorities also searched a Honda Civic car bearing Plate No. WCP 157, parked along Masangkay Street, registered in the name of the wife of Tiu Won and found four (4) plastic bags containing 6.2243 grams of shabu, which were likewise confiscated. A gun in the possession of Tiu Won was also seized and made subject of a separate criminal case.

The defense presented appellants Tiu Won and Qui Yaling. They denied that Timothy Tiu and Tiu Won Chua are one and the same person. They presented papers and documents to prove that appellant is Tiu Won Chua and not Timothy Tiu, as stated in the search warrant. Tiu Won also claimed that he does not live in the apartment subject of the search warrant, alleging that he is married to a certain Emily Tan and is a resident of No. 864 Alvarado St., Binondo, Manila. Nonetheless, he admitted that his co-appellant, Qui Yaling, is his mistress with whom he has two children. Qui Yaling admitted being the occupant of the apartment, but alleged that she only occupied one room, while two other persons, a certain Lim and a certain Uy, occupied the other rooms. Both appellants denied that they were engaged in the sale or possession of shabu. They asserted that they are in the jewelry business and that at the time the search and arrest were made, the third person, whom the prosecution identified as a housemaid, was actually a certain Chin, who was there to look at some of the pieces of jewelry sold by Tiu Won. They also denied that a gun was found in the possession of Tiu Won.

Qui Yaling recalled that upon asking who was it knocking at the door of her apartment on October 12, the police authorities represented that they were electric bill collectors. She let them in. She was surprised when upon opening the door, around ten (10) policemen barged inside her unit. She, together with Tiu Won and Chin, was asked to remain seated in the sofa while the men searched each room. Tiu Won alleged that after a fruitless search, some of the policemen went out, but came back a few minutes later with another person. Afterwards, he was made to sign a piece of paper. Appellants also claimed that the policemen took their bags which contained

Page 124: Warrantless Arrests, Searches and Seizures and Privacy of Communication

124

money, the pieces of jewelry they were selling and even Qui Yaling's cell phone. They both denied that shabu was discovered in the apartment during the search. Appellants were arrested and brought to the police station.

In a decision, dated August 15, 2001, the RTC found proof beyond reasonable doubt of the guilt of the appellants and sentenced them to suffer the penalty of reclusion perpetua and a fine of P500,000.00 each.[6]

Thus, appellants interpose this appeal raising the following assignment of errors:

I

THE TRIAL COURT ERRED IN DISREGARDING THE LEGAL DEFECTS OF THE SEARCH WARRANT USED BY THE POLICE OPERATIVES AGAINST BOTH ACCUSED.

II

THE TRIAL COURT ERRED IN TAKING INTO CONSIDERATION EVIDENCES (sic) WHICH SHOULD HAVE BEEN EXCLUDED AND DISREGARDED WHICH RESULTED IN THE ERRONEOUS CONVICTION OF BOTH ACCUSED.

III

THE TRIAL COURT ERRED IN CONVICTING BOTH ACCUSED DESPITE THE ABSENCE OF PROOF BEYOND REASONABLE DOUBT.

IV

THE TRIAL COURT ERRED IN DISREGARDING THE FACT THAT THE CONSTITUTIONAL RIGHTS OF BOTH ACCUSED WERE SERIOUSLY VIOLATED BY THE POLICE OPERATIVES.[7]

These issues can be trimmed down to two i.e., the legality of the search warrant and the search and arrest conducted pursuant thereto, and the correctness of the judgment of conviction imposed by the RTC.

Page 125: Warrantless Arrests, Searches and Seizures and Privacy of Communication

125

As regards the propriety of the search warrant issued in the name of Timothy Tiu, which did not include appellant Qui Yaling, appellants contend that because of this defect, the search conducted and consequently, the arrest, are illegal. Being fruits of an illegal search, the evidence presented cannot serve as basis for their conviction.

We beg to disagree. There are only four requisites for a valid warrant, i.e,: (1) it must be issued upon "probable cause"; (2) probable cause must be determined personally by the judge; (3) such judge must examine under oath or affirmation the complainant and the witnesses he may produce; and (4) the warrant must particularly describe the place to be searched and the persons or things to be seized.[8] As correctly argued by the Solicitor General, a mistake in the name of the person to be searched does not invalidate the warrant,[9] especially since in this case, the authorities had personal knowledge of the drug-related activities of the accused. In fact, a "John Doe" warrant satisfies the requirements so long as it contains a descriptio personae such as will enable the officer to identify the accused.[10] We have also held that a mistake in the identification of the owner of the place does not invalidate the warrant provided the place to be searched is properly described.[11]

Thus, even if the search warrant used by the police authorities did not contain the correct name of Tiu Won or the name of Qui Yaling, that defect did not invalidate it because the place to be searched was described properly. Besides, the authorities conducted surveillance and a test-buy operation before obtaining the search warrant and subsequently implementing it. They can therefore be presumed to have personal knowledge of the identity of the persons and the place to be searched although they may not have specifically known the names of the accused. Armed with the warrant, a valid search of Unit 4-B was conducted.

We affirm, however, the illegality of the search conducted on the car, on the ground that it was not part of the description of the place to be searched mentioned in the warrant. It is mandatory that for the search to be valid, it must be directed at the place particularly described in the warrant.[12] Moreover, the search of the car was not incidental to a lawful arrest. To be valid, such warrantless search must be limited to that point within the reach or control of the person arrested, or that which may furnish him with the means of committing violence or of escaping.[13] In this case, appellants were arrested inside the apartment, whereas the car was parked a few meters away from the building.

In a prosecution for illegal possession of a dangerous drug, it must be shown that (1) appellants were in possession of an item or an object identified to be a prohibited or regulated drug, (2) such possession is not authorized by law, and (3) the appellants were freely and consciously aware of being in possession of the drug.[14] We also note that the crime under consideration is malum prohibitum, hence, lack of criminal intent or good faith does not exempt appellants from criminal liability. Mere possession of a regulated drug without legal authority is punishable under the Dangerous Drugs Act.[15]

Page 126: Warrantless Arrests, Searches and Seizures and Privacy of Communication

126

In the case at bar, the prosecution has sufficiently proved that the packs of shabu were found inside Unit 4-B, HCL Building, 1025 Masangkay St., Binondo, Manila. Surveillance was previously conducted. Though no arrest was made after the successful test-buy operation, this does not destroy the fact that in a subsequent search, appellants were found in possession of shabu. The testimonies of the prosecution witnesses are consistent in that after the test-buy operation, they obtained a search warrant from Judge Makasiar, pursuant to which, they were able to confiscate, among others, several packs of shabu from a man's handbag and a ladies' handbag inside a room in the unit subject of the warrant. Furthermore, the seizure of the regulated drug from Unit 4-B is proven by the "Receipt for Property Seized"[16] signed by SPO1 de Leon, the seizing officer, Noel, the building administrator, and Joji Olarte, his wife, who were also present. De Leon attested to the truth and genuineness of the receipt which was not contradicted by the defense.

Be that as it may, we cannot sustain the trial court's decision attributing to both appellants the illegal possession of the same amount of shabu. We note that nowhere in the information is conspiracy alleged. Neither had it been proven during the trial. As such, we need to look at the individual amounts possessed by each appellant.

In his testimony, Tiu Won admitted ownership of the man's handbag where 234.5 grams of shabu were found, viz:

Q: During those ten to 20 minutes, what were those policemen doing inside that unit?

A:

They went inside the rooms and started ransacking the drawers and everything. As a matter of fact, even handbags were searched by them.

Q: Whose handbags were searched?

A: My bag, the one I was carrying that day, with jewelry and checks and others were taken by them.[17] (emphasis supplied)

Qui Yaling, in her appellant's brief, denied owning the handbag where 20.3673 grams of shabu were discovered. However, during her testimony, she admitted its ownership, viz:

Q:

Now, the police testified before this court that you has (sic) a bag and when they searched this bag, it yielded some sachets of shabu(.) (W)hat can you say about that?

A:

That is an absolute lie, sir. What they saw in my bag were my cosmetics.[18] (emphasis supplied)

Page 127: Warrantless Arrests, Searches and Seizures and Privacy of Communication

127

An admission is an act or declaration of a party as to the existence of a relevant fact which may be used in evidence against him.[19] These admissions, provided they are voluntary, can be used against appellants because it is fair to presume that they correspond with the truth, and it is their fault if they do not.[20]

Qui Yaling likewise argues that the lower court erred in attributing ownership of the handbag to her considering that there was another girl present at the apartment during the search. She contends that since the prosecution was not able to establish the ownership of the bag, then such could have also been owned by Chin.

We do not subscribe to this argument. The defense failed to bring Chin to court, although during the course of the presentation of their evidence, they manifested their intention to present her testimony. Furthermore, a visitor does not normally leave her bag lying anywhere, much more in the master's bedroom. Being the occupant of the apartment, it is more logical to presume that the handbag belongs to Qui Yaling. The failure of the prosecution to present the bags and proofs that the bags belong to the appellants is immaterial because the bags, the license of Tiu Won found inside the man's handbag and the passport of Qui Yaling found inside the ladies' handbag are not illegal. Having no relation to the use or possession of shabu, the authorities could not confiscate them for they did not have the authority to do so since the warrant authorized them to seize only articles in relation to the illegal possession of shabu.[21] Not within their control, they could not have been presented in court.

We now come to the penalties of the appellants. R. A. No. 6425, as amended by R. A. No. 7659, applies. Thus, since 234.5 grams of shabu were found inside the man's handbag, deemed to be owned by Tiu Won, he is guilty of violating Section 16, Article III of R.A. No. 6425, while Qui Yaling, whose handbag contained only 20.3673 grams of shabu is guilty of violating Section 20 thereof. Section 16, in connection with Section 20 (1st paragraph), provides the penalty of reclusion perpetua to death and a fine ranging from five hundred thousand pesos to ten million pesos where the amount of shabu involved is 200 grams or more. Where the amount is less than 200 grams, Section 20 punishes the offender with the penalty ranging from prision correccional to reclusion perpetua.

IN VIEW WHEREOF, the decision of RTC Br. 27, Manila as to the penalty of appellant Tiu Won is affirmed, while that of appellant Qui Yaling is modified. Tiu Won Chua is sentenced to a penalty of reclusion perpetua, and a fine of five hundred thousand pesos (P500,000.00) in accordance with Section 16 and Section 20 (1st paragraph) of R.A. No. 6425, as amended by R.A. No. 7659. Qui Yaling y Chua is sentenced to an indeterminate sentence of prision correccional as minimum to prision mayor as maximum, there being no mitigating or aggravating circumstances.

SO ORDERED.

Page 128: Warrantless Arrests, Searches and Seizures and Privacy of Communication

128

Panganiban, Sandoval-Gutierrez, Corona, and Carpio-Morales, JJ., concur.

G.R. No. 81567 October 3, 1991

IN THE MATTER OF THE PETITION FOR HABEAS CORPUS OF ROBERTO UMIL, ROLANDO DURAL and RENATO VILLANUEVA, MANOLITA O. UMIL and NICANOR P. DURAL, FELICITAS V. SESE, petitioners, vs.FIDEL V. RAMOS, MAJ. GEN. RENATO DE VILLA, BRIG. GEN. RAMON MONTANO, BRIG. GEN. ALEXANDER AGUIRRE, respondents.

G.R. Nos. 84581-82 October 3, 1991

AMELIA ROQUE and WILFREDO BUENAOBRA, petitioners, vs.GEN. RENATO DE VILLA and GEN, RAMON MONTANO, respondents.

G.R. Nos. 84583-84 October 3, 1991

IN THE MATTER OF THE PETITION FOR HABEAS CORPUS OF ATTY. DOMINGO T. ANONUEVO and RAMON CASIPLE: DOMINGO T. ANONUEVO and RAMON CASIPLE, petitioners, vs.HON. FIDEL V. RAMOS, GEN. RENATO S. DE VILLA, COL. EVARISTO CARIÑO, LT. COL. REX D. PIAD, T/SGT. CONRADO DE TORRES, S/SGT. ARNOLD DURIAN, and Commanding Officer, PC-INP Detention Center, Camp Crame, Quezon City, respondents.

G.R. No. 83162 October 3, 1991

IN THE MATTER OF THE APPLICATION FOR HABEAS CORPUS OF VICKY A. OCAYA AND DANNY RIVERA: VIRGILIO A. OCAYA, petitioners, vs.BRIG. GEN. ALEXANDER AGUIRRE, COL. HERCULES CATALUNA, COL. NESTOR MARIANO,respondents.

G.R. No. 85727 October 3, 1991

IN THE MATTER OF APPLICATION FOR HABEAS CORPUS OF DEOGRACIAS ESPIRITU, petitioner, vs.BRIG. GEN.ALFREDO S. LIM, COL. RICARDO REYES, respondents.

G.R. No. 86332 October 3, 1991

IN THE MATTER OF THE PETITION FOR HABEAS CORPUS OF NARCISO B. NAZARENO: ALFREDO NAZARENO,petitioner, 

Page 129: Warrantless Arrests, Searches and Seizures and Privacy of Communication

129

vs.THE STATION COMMANDER OF THE MUNTINGLUPA POLICE STATION, Muntinglupa, Metro Manila, P/SGT. JACINTO MEDINA, P/SGT. ELADIO TAGLE, P/SGT. LEVI SOLEDAD, and P/SGT. MALTRO AROJADO, respondents.

Efren H. Mercado for petitioners in G.R. No. 81567 and G. R. No. 83162.

Ricardo C. Valmonte for petitioners in G.R. Nos. 84581-82

Josefina G. Campbell-Castillo for petitioners in G.R. Nos. 84583-84.

Potenciano A. Flores, Jr. for petitioner in G.R. No. 85727.

The Solicitor General for the respondents.

R E S O L U T I O N

 

PER CURIAM:p

Before the Court are separate motions filed by the petitioners in the above-entitled petitions, seeking reconsideration of the Court's decision promulgated on 9 July 1990 (the decision, for brevity) which dismissed the petitions, with the following dispositive part:

WHEREFORE, the petitions are hereby DISMISSED, except that in G.R. No. 85727 (Espiritu vs. Lim), the bail bond for petitioner's provisional liberty is hereby ordered reduced from P60,000.00 to P10,000.00. No costs.

The Court avails of this opportunity to clarify its ruling a begins with the statement that the decision did not rule — as many misunderstood it to do — that mere suspicion that one is Communist Party or New People's Army member is a valid ground for his arrest without warrant. Moreover, the decision merely applied long existing lawsto the factual situations obtaining in the several petitions. Among these laws are th outlawing the Communist Party of the Philippines (CPP) similar organizations and penalizing membership therein be dealt with shortly). It is elementary, in this connection, if these laws no longer reflect the thinking or sentiment of the people, it is Congress as the elected representative of the people — not the Court — that should repeal, change or modify them.

In their separate motions for reconsideration, petitioners, in sum, maintain:

1. That the assailed decision, in upholding the validity of the questioned arrests made without warrant, and in relying on the provisions of the Rules of Court, particularly Section 5 of Rule 113 (Arrest), disregards the fact that such arrests violated the constitutional rights of the persons arrested;

2. That the doctrine laid down in Garcia vs. Enrile 1 and Ilagan vs. Enrile 2 should be abandoned;

3. That the decision erred in considering the admissions made by the persons arrested as to their membership in the Communist Party of the Philippines/New People's Army, and their ownership of the unlicensed firearms, ammunitions and subversive documents found in their possession at the time of arrest, inasmuch as those confessions do not comply with the requirements on admissibility of extrajudicial admissions;

Page 130: Warrantless Arrests, Searches and Seizures and Privacy of Communication

130

4. That the assailed decision is based on a misappreciation of facts;

5. That G.R. No. 81567 (the Umil case) should not be deemed moot and academic.

We find no merit in the motions for reconsideration.

It can not be overlooked that these are petitions for the issuance of the writ of habeas corpus, filed by petitioners under the Rules of Court. 3 The writ of habeas corpus exists as a speedy and effective remedy to relieve persons from unlawful restraint. 4 Therefore, the function of the special proceedings of habeas corpus is to inquire into the legality of one's detention, 5 so that if detention is illegal, the detainee may be ordered forthwit released.

In the petitions at bar, to ascertain whether the detention petitioners was illegal or not, the Court before rendering decision dated 9 July 1990, looked into whether their questioned arrests without warrant were made in accordance with law. For, if the arrests were made in accordance with law, would follow that the detention resulting from such arrests also in accordance with law.

There can be no dispute that, as a general rule, no peace officer or person has the power or authority to arrest anyo without a warrant of arrest, except in those cases express authorized by law. 6 The law expressly allowing arrests witho warrant is found in Section 5, Rule 113 of the Rules of Court which states the grounds upon which a valid arrest, without warrant, can be conducted.

In the present cases, the focus is understandably on Section 5, paragraphs (a) and (b) of the said Rule 113, which read:

Sec. 5. Arrest without warrant; when lawful. — A peace officer or a private person may, without a warrant, arrest a person:

(a) When, in his presence, the person to he arrested has committed, is actually committing, or is attempting to commit an offense;

(b) When an offense has in fact just been committed, and he has personal knowledge of facts indicating that the person to be arrest has committed it; and

. . . (Emphasis supplied).

The Court's decision of 9 July 1990 rules that the arrest Rolando Dural (G.R. No. 81567) without warrant is justified it can be said that, within the contemplation of Section 5 Rule 113, he (Dural) was committing an offense, when arrested because Dural was arrested for being a member of the New People's Army, an outlawed organization, where membership penalized, 7 and for subversion which, like rebellion is, under the doctrine ofGarcia vs. Enrile, 8 a continuing offense, thus:

The crimes of insurrection or rebellion, subversion, conspiracy or proposal to commit such crimes, and other crimes and offenses committed in the furtherance (sic) on the occasion thereof, or incident thereto, or in connection therewith under Presidential Proclamation No. 2045, are all in the nature of continuing offenses which set them apart from the common offenses, aside from their essentially involving a massive conspiracy of nationwide magnitude. . . .

Given the ideological content of membership in the CPP/NPA which includes armed struggle for the overthrow of organized government, Dural did not cease to be, or became less of a subversive, FOR PURPOSES OF ARREST, simply because he was, at the time of arrest, confined in the St. Agnes Hospital. Dural was identified as one of several persons who the day before his arrest, without warrant, at

Page 131: Warrantless Arrests, Searches and Seizures and Privacy of Communication

131

the St. Agnes Hospital, had shot two (2) CAPCOM policemen in their patrol car. That Dural had shot the two (2) policemen in Caloocan City as part of his mission as a "sparrow" (NPA member) did not end there and then. Dural, given another opportunity, would have shot or would shoot other policemen anywhere as agents or representatives of organized government. It is in this sense that subversion like rebellion (or insurrection) is perceived here as a continuing offense. Unlike other so-called "common" offenses, i.e. adultery, murder, arson, etc., which generally end upon their commission, subversion and rebellion are anchored on an ideological base which compels the repetition of the same acts of lawlessness and violence until the overriding objective of overthrowing organized government is attained.

Nor can it be said that Dural's arrest was grounded on mere suspicion by the arresting officers of his membership in the CPP/NPA. His arrest was based on "probable cause," as supported by actual facts that will be shown hereafter.

Viewed from another but related perspective, it may also be said, under the facts of the Umil case, that the arrest of Dural falls under Section 5, paragraph (b), Rule 113 of the Rules of Court, which requires two (2) conditions for a valid arrestt without warrant: first, that the person to be arrested has just committed an offense, and second, that the arresting peace officer or private person has personal knowledge of facts indicating that the person to be arrested is the one who committed the offense. Section 5(b), Rule 113, it will be noted, refers to arrests without warrant, based on "personal knowledge of facts" acquired by the arresting officer or private person.

It has been ruled that "personal knowledge of facts," in arrests without warrant must be based upon probable cause, which means an actual belief or reasonable grounds of suspicion 9

The grounds of suspicion are reasonable when, in the absence of actual belief of the arresting officers, the suspicion that the person to be arrested is probably guilty of committing the offense, is based on actual facts, i.e., supported by circumstances sufficiently strong in themselves to create the probable cause of guilt of the person to be arrested. 10 A reasonable suspicion therefore must be founded on probable cause, coupled with good faith on the part of the peace officers making the arrest. 11

These requisites were complied with in the Umil case and in the other cases at bar.

In G.R. No. 81567 (Umil case), military agents, on 1 February 1988, were dispatched to the St. Agnes Hospital, Roosevelt Avenue, Quezon City, to verify a confidential information which was received by their office, about a "sparrow man" (NPA member) who had been admitted to the said hospital with a gunshot wound; that the information further disclosed that the wounded man in the said hospital was among the five (5) male "sparrows" who murdered two (2) Capcom mobile patrols the day before, or on 31 January 1988 at about 12:00 o'clock noon, before a road hump along Macanining St., Bagong Barrio, Caloocan City; that based on the same information, the wounded man's name was listed by the hospital management as "Ronnie Javellon," twenty-two (22) years old of Block 10, Lot 4, South City Homes, Biñan, Laguna. 12

Said confidential information received by the arresting officers, to the effect that an NPA member ("sparrow unit") was being treated for a gunshot wound in the named hospital, is deemed reasonable and with cause as it was based on actual facts and supported by circumstances sufficient to engender a belief that an NPA member was truly in the said hospital. The actual facts supported by circumstances are: first — the day before, or on 31 January 1988, two (2) CAPCOM soldiers were actually killed in Bagong Bario, Caloocan City by five (5) "sparrows" including Dural; second — a wounded person listed in the hospital records as "Ronnie Javellon" was actually then being treated in St. Agnes Hospital for a gunshot wound; third — as the records of this case disclosed later, "Ronnie Javellon" and his address entered in the hospital records were fictitious and the wounded man was in reality Rolando Dural.

In fine, the confidential information received by the arresting officers merited their immediate attention and action and, in fact, it was found to be true. Even the petitioners in their motion for

Page 132: Warrantless Arrests, Searches and Seizures and Privacy of Communication

132

reconsideration, 13 believe that the confidential information of the arresting officers to the effect that Dural was then being treated in St. Agnes Hospital was actually received from the attending doctor and hospital management in compliance with the directives of the law, 14 and, therefore, came from reliable sources.

As to the condition that "probable cause" must also be coupled with acts done in good faith by the officers who make the arrest, the Court notes that the peace officers wno arrested Dural are deemed to have conducted the same in good faith, considering that law enforcers are presumed to regularly perform their official duties. The records show that the arresting officers did not appear to have been ill-motivated in arresting Dural. 15 It is therefore clear that the arrest, without warrant, of Dural was made in compliance with the requirements of paragraphs (a) and (b) of Section 5, Rule 113.

Parenthetically, it should be mentioned here that a few day after Dural's arrest, without warrant, an information charging double murder with assault against agents of persons in authority was filed against Dural in the Regional Trial Court of Caloocan City (Criminal Case No. C-30112). He was thus promptly placed under judicial custody (as distinguished fro custody of the arresting officers). On 31 August 1988, he wa convicted of the crime charged and sentenced to reclusion perpetua. The judgment of conviction is now on appeal before this Court in G.R. No. 84921.

As to Amelia Roque and Wilfredo Buenaobra (G.R. Nos. 84581-82), Domingo Anonuevo and Ramon Casiple(G.R. Nos. 84583-84) and Vicky Ocaya (G.R. No. 83162), their arrests, without warrant, are also justified. They were searched pursuant to search warrants issued by a court of law and were found wit unlicensed firearms, explosives and/or ammunition in their persons. They were, therefore, caught in flagrante delicto which justified their outright arrests without warrant, under Sec 5(a), Rule 113, Rules of Court. Parenthetically, it should be mentioned here that a few davs after their arrests without warrant, informations were filed in court against said petitioners, thereby placing them within judicial custody and disposition. Furthermore, Buenaobra mooted his own petition fo habeas corpus by announcing to this Court during the hearing of these petitions that he had chosen to remain in detention in the custody of the authorities.

More specifically, the antecedent facts in the "in flagrante" cases are:

1. On 27 June 1988, the military agents received information imparted by a former NPA about the operations of the CPP and NPA in Metro Manila and that a certain house occupied by one Renato Constantine, located in the Villaluz Compound, Molave St., Marikina Heights, Marikina, Metro Manila was being used as their safehouse; that in view of this information, the said house was placed under military surveillance and on 12 August 1988, pursuant to a search warrant duly issued by court, a search of the house was conducted; that when Renato Constantine was then confronted he could not produce any permit to possess the firearms, ammunitions, radio and other communications equipment, and he admitted that he was a ranking member of the CPP. 16

2. In the case of Wilfredo Buenaobra, he arrived at the house of Renato Constantino in the evening of 12 August 1988, and admitted that he was an NPA courier and he had with him letters to Renato Constantine and other members of the rebel group.

3. On the other hand, the arrest of Amelia Roque was a consequence of the arrest of Buenaobra who had in his possession papers leading to the whereabouts of Roque; 17 that, at the time of her arrest, the military agents found subversive documents and live ammunitions, and she admitted then that the documents belonged to her. 18

4. As regards Domingo Anonuevo and Ramon Casiple they were arrested without warrant on 13 August 1988, when they arrived at the said house of Renato Constantine in the evening of said date; that when the agents frisked them, subversive documents,

Page 133: Warrantless Arrests, Searches and Seizures and Privacy of Communication

133

and loaded guns were found in the latter's possession but failing to show a permit to possess them. 19

5. With regard to Vicky Ocaya, she was arrested, without warrant when she arrived (on 12 May 1988) at the premises ofthe house of one Benito Tiamzon who was believed to be the head of the CPP/NPA, and whose house was subject of a search warrant duly issued by the court. At the time of her arrest without warrant the agents of the PC-Intelligence and Investigation found ammunitions and subversive documents in the car of Ocaya. 20

It is to be noted in the above cases (Roque, Buenaobra, Anonuevo, Casiple and Ocaya) that the reason which compelled the military agents to make the arrests without warrant was the information given to the military authorities that two (2) safehouses (one occupied by Renato Constantine and the other by Benito Tiamzon) were being used by the CPP/NPA for their operations, with information as to their exact location and the names of Renato Constantine and Benito Tiamzon as residents or occupants thereof.

And at the time of the actual arrests, the following circumstances surrounded said arrests (of Roque, Buenaobra, Anonuevo and Casiple), which confirmed the belief of the military agents that the information they had received was true and the persons to be arrested were probably guilty of the commission of certain crimes: first: search warrant was duly issued to effect the search of the Constantine safehouse; second: found in the safehouse was a person named Renato Constantine, who admitted that he was a ranking member of the CPP, and found in his possession were unlicensed firearms and communications equipment; third: at the time of their arrests, in their possession were unlicensed firearms, ammunitions and/or subversive documents, and they admitted ownership thereof as well as their membership in the CPP/NPA. And then, shortly after their arrests, they were positively identified by their former comrades in the organization as CPP/NPA members. In view of these circumstances, the corresponding informations were filed in court against said arrested persons. The records also show that, as in the case of Dural, the arrests without warrant made by the military agents in the Constantino safehouse and later in the Amelia Roque house, do not appear to have been ill-motivated or irregularly performed.

With all these facts and circumstances existing before, during and after the arrest of the afore-named persons (Dural, Buenaobra, Roque, Anonuevo, Casiple and Ocaya), no prudent an can say that it would have been better for the military agents not to have acted at all and made any arrest. That would have been an unpardonable neglect of official duty and a cause for disciplinary action against the peace officers involved.

For, one of the duties of law enforcers is to arrest lawbreakers in order to place them in the hands of executive and judicial authorities upon whom devolves the duty to investigate the acts constituting the alleged violation of law and to prosecute and secure the punishment therefor. 21 An arrest is therefore in the nature of an administrative measure. The power to arrest without warrant is without limitation as long as the requirements of Section 5, Rule 113 are met. This rule is founded on an overwhelming public interest in peace and order in our communities.

In ascertaining whether the arrest without warrant is conducted in accordance with the conditions set forth in Section 5, Rule 113, this Court determines not whether the persons arrested are indeed guilty of committing the crime for which they were arrested. 22 Not evidence of guilt, but "probable cause" is the reason that can validly compel the peace officers, in the performance of their duties and in the interest of public order, to conduct an arrest without warrant. 23

The courts should not expect of law-enforcers more than what the law requires of them. Under the conditions set forth in Section 5, Rule 113, particularly paragraph (b) thereof, even if the arrested persons are later found to be innocent and acquitted, the arresting officers are not liable. 24 But if they do not

Page 134: Warrantless Arrests, Searches and Seizures and Privacy of Communication

134

strictly comply with the said conditions, the arresting officers can be held liable for the crime of arbitrary detention, 25 for damages under Article 32 of the Civil Code 26 and/or for other administrative sanctions.

In G.R. No. 85727, Espiritu, on 23 November 1988, was arrested without warrant, on the basis of the attestation of certain witnesses: that about 5:00 o'clock in the afternoon of 22 November 1988, at the corner of Magsaysay Boulevard and Velencia St., Sta. Mesa, Manila, Espiritu spoke at a gathering of drivers and sympathizers, where he said, among other things:

Bukas tuloy ang welga natin . . . hanggang sa magkagulona. 27 (Emphasis supplied)

and that the police authorities were present during the press conference held at the National Press Club (NPC) on 22 November 1988 where Espiritu called for a nationwide strike (of jeepney and bus drivers) on 23 November 1988. 28 Espiritu was arrested without warrant, not for subversion or any "continuing offense," but for uttering the above-quoted language which, in the perception of the arresting officers, was inciting to sedition.

Many persons may differ as to the validity of such perception and regard the language as falling within free speech guaranteed by the Constitution. But, then, Espiritu had not lost the right to insist, during the pre-trial or trial on the merits, that he was just exercising his right to free speech regardless of the charged atmosphere in which it was uttered. But, the authority of the peace officers to make the arrest, without warrant, at the time the words were uttered, or soon thereafter, is still another thing. In the balancing of authority and freedom, which obviously becomes difficult at times, the Court has, in this case, tilted the scale in favor of authority but only for purposes of the arrest (not conviction). Let it be noted that the Court has ordered the bail for Espiritu's release to be reduced from P60,000.00 to P10,000.00.

Let it also be noted that supervening events have made the Espiritu case moot and academic. For Espiritu had before arraignment asked the court a quo for re-investigation, the peace officers did not appear. Because of this development, the defense asked the court a quo at the resumption of the hearings to dismiss the case. Case against Espiritu (Criminal Case No. 88-68385) has been provisionally dismissed and his bail bond cancelled.

In G.R. No. 86332 (Nazareno), the records show that in the morning of 14 December 1988, Romulo Bunye II was killed by a group of men in Alabang, Muntinlupa, Metro Manila; that at about 5:00 o'clock in the morning of 28 December 1988, Ramil Regala, one of the suspects in the said killing, was arrested and he pointed to Narciso Nazareno as one of his companions during the killing of Bunye II; that at 7:20 of the same morning (28 December 1988), the police agents arrested Nazareno, without warrant, for investigation. 29

Although the killing of Bunye II occurred on 14 December 1988, while Nazareno's arrest without warrant was made only on 28 December 1988, or 14 days later, the arrest fans under Section 5(b) of Rule 113, since it was only on 28 December 1988 that the police authorities came to know that Nazareno was probably one of those guilty in the killing of Bunye II and the arrest had to be made promptly, even without warrant, (after the police were alerted) and despite the lapse of fourteen (14) days to prevent possible flight.

As shown in the decision under consideration, this Court, in upholding the arrest without warrant of Nazareno noted several facts and events surrounding his arrest and detention, as follows:

. . . on 3 January 1989 (or six (6) days after his arrest without warrant), an information charging Narciso Nazareno, Ramil Regala and two (2) others, with the killing of Romulo Bunye II was filed wit the Regional Trial Court of Makati, Metro Manila. The case is dock eted therein as Criminal Case No. 731.

Page 135: Warrantless Arrests, Searches and Seizures and Privacy of Communication

135

On 7 January 1989, Narciso Nazareno filed a motion to post bail but the motion was denied by the trial court in an order dated 10 January 1989, even as the motion to post bail, earlier filed by his co-accused, Manuel Laureaga, was granted by the same trial court.

On 13 January 1989, a petition for habeas corpus was filed with this Court on behalf of Narciso Nazareno and on 13 January 1989, the Court issued the writ of habeas corpus, retumable to the Presiding Judge of the Regional Trial Court of Bifian, Laguna, Branch 24, ordering said court to hear the case on 30 January 1989 and thereafter resolve the petition.

At the conclusion of the hearing, or on 1 February 1989, the Presiding Judge of the Regional Trial Court of Biñan, Laguna issued a resolution denying the petition for habeas corpus, it appearing that the said Narciso Nazareno is in the custody of the respondents by reason of an information filed against him with the Regional Trial Court of Makati, Metro Manila which liad taken cognizance of said case and had, in fact, denied the motion for bail filed by said Narciso Nazareno (presumably because of the strength of the evidence against him).

This Court reiterates that shortly after the arrests of Espiritu and Nazareno, the corresponding informations against them were filed in court. The arrests of Espiritu and Nazareno were based on probable cause and supported by factual circumstances. They complied with conditions set forth in Section 5(b) of Rule 113. They were not arbitrary or whimsical arrests.

Parenthetically, it should be here stated that Nazareno has since been convicted by the court a quo for murder and sentenced to reclusion perpetua. He has appealed the judgment of conviction to the Court of Appeals where it is pending as of this date ( CA-G.R. No. still undocketed).

Petitioners contend that the decision of 9 July 1990 ignored the contitution requisiteds for admissibility of an extrajudicial admission.

In the case of Buenaobra (G.R. Nos. 84581-82), he admitted 30 that he was an NPA courier. On the other hand, in the case of Amelia Roque, she admitted 31 that the unlicensed firearms, ammunition and subversive documents found in her possession during her arrest, belonged to her.

The Court, it is true, took into account the admissions of the arrested persons of their membership in the CPP/NPA, as well as their ownership of the unlicensed firearms, ammunitions and documents in their possession. But again, these admissions, as revealed by the records, strengthen the Court's perception that truly the grounds upon which the arresting officers based their arrests without warrant, are supported by probable cause, i.e. that the persons arrested were probably guilty of the commission of certain offenses, in compliance with Section 5, Rule 113 of the Rules of Court. To note these admissions, on the other hand, is not to rule that the persons arrested are already guilty of the offenses upon which their warrantless arrests were predicated. The task of determining the guilt or innocence of persons arrested without warrant is not proper in a petition for habeas corpus. It pertains to the trial of the case on the merits.

As to the argument that the doctrines in Garcia vs. Enrile, and Ilagan vs. Enrile should be abandoned, this Court finds no compelling reason at this time to disturb the same, particularly ln the light of prevailing conditions where national security and liability are still directly challenged perhaps with greater vigor from the communist rebels. What is important is that everv arrest without warrant be tested as to its legality via habeas corpus proceeding. This Court. will promptly look into — and all other appropriate courts are enjoined to do the same — the legality of the arrest without warrant so that if the conditions under Sec. 5 of Rule 113, Rules of Court, as elucidated in this Resolution, are not met, then the detainee shall forthwith be ordered released; but if such conditions are met, then the detainee shall not be made to

Page 136: Warrantless Arrests, Searches and Seizures and Privacy of Communication

136

languish in his detention but must be promptly tried to the end that he may be either acquitted or convicted, with the least delay, as warranted by the evidence.

A Final Word

This Resolution ends as it began, reiterating that mere suspicion of being a Communist Party member or a subversive is absolutely not a ground for the arrest without warrant of the suspect. The Court predicated the validity of the questioned arrests without warrant in these petitions, not on mere unsubstantiated suspicion, but on compliance with the conditions set forth in Section 5, Rule 113, Rules of Court, a long existing law, and which, for stress, are probable cause and good faith of the arresting peace officers, and, further, on the basis of, as the records show, the actual facts and circumstances supporting the arrests. More than the allure of popularity or palatability to some groups, what is important is that the Court be right.

ACCORDINGLY, the motions for reconsideration of the decision dated 9 July 1990, are DENIED. This denial is FINAL.

SO ORDERED.

Narvasa, Melencio-Herrera, Paras, Padilla, Bidin, Griño-Aquino, Medialdea and Davide, Jr., JJ., concur.

Separate Opinions

FERNAN, C.J., concurring and dissenting:

After a deep and thorough reexamination of the decision of Julv 9, 1990 and an exhaustive evaluation of the motions for reconsideration of the said decision, I am inclined to agree with the, majority's resolution on said motions for reconsideration except for the legality of the warrantless arrests of petitioner Deogracias Espiritu for the crime of inciting to sedition and petitioner Alfredo Nazareno for the crime of murder.

In the words of the resolution, Espiritu "was arrested without warrant, not for subversion or any 'continuing offense,' but for uttering" the following: "Bukas tuloy ang welga natin . . . hanggang sa magkagulo na." Apparently, such statement was, in the perception of the arresting officers, inciting to sedition. While not conceding the validity of such perception, realizing that it is indeed possible that Espiritu was merely exercising his right to free speech, the resolution nonetheless supports the authority of peace officers "only for purposes of the arrest."

I find this position to be adverse to the very essence of the resolution which sanctions warrantless arrests provided they are made in accordance with law. In the first place, Espiritu mav not be considered as having "just committed" the crime charged. He allegedly first uttered seditious remarks at the National Press Club in the afternoon of November 12, 1988. The second allegedly seditious remark aforequoted was made at around 5:00 o'clock in the same afternoon (Decision, pp. 23-24). Under these circumstances, the law enforcement agents had time, short though it might seem, to secure a warrant for his arrest. Espiritu's apprehension may not therefore be considered as covered by Section 5(b) of Rule 113 which allows warrantless arrests "when an offense has in fact just been committed."

The same observation applies with greater force in the case of Nazareno who was arrested 14 days after the commission of the crime imputed to him.

Page 137: Warrantless Arrests, Searches and Seizures and Privacy of Communication

137

Secondly, warrantless arrests may not be allowed if the arresting officer are not sure what particular provision of law had beeri violated by the person arrested. True it is that law en.orcement agents and even prosecutors are not all adept at the However, errneous perception, not to mention ineptitude among their ranks, especially if it would result in the violation of any right of a person, may not be tolerated. That the arrested person has the "right to insist during the pre-trial or trial on the merits" (Resolution., p. 18) that he was exercising a right which the arresting officer considered as contrary to law, is beside the point. No person should be subjected to the ordeal of a trial just because the law enforcers wrongly perceived his action.

Thirdly, inciting to sedition is not a continuous crime for which the offender may be arrested without a warrant duly issued by the proper authority. By its nature, a single act of urging others to commit any of the acts enumerated in Article 142 of the Revised Penal Code may suffice to hold anyone liable for inciting to sedition. While the crime is aimed at anarchy and radicalism and presents largely a question of policy (Espuelas vs. People, 90 Phil, 524 [1951]), it should be remembered that any of the prohibited acts in Article 142 may infringe upon the fundamental freedoms of speech and expression. There arises, therefore, the necessity of balancing interests; those of the State as against those of its individual citizen. Here lies the urgency of judicial intervention before an arrest is made. Added to this is the subjectivity of the determination of what may incite other people to sedition. Hence, while the police should act swiftly when a seditious statement has been uttered in view of the jeopardy it may cause the government, speedy action should consist not in warrantless arrests but in securing warrants for such arrests.

On the legality of warrantless arrests of violators of the Anti-Subversion Law, it should be underscored that anyone who undertakes such arrest must see to it that the alleged violator is knowing member of a subversive organization as distinguished from a nominal one (People vs. Ferrer, L-32613-14, December 27, 1972, 48 SCRA 382). Thus, a subversive may be arrested even if has not committed overt act of overthrowing the government such as bombing of government offices trie assassination of government officials provided there is probable cause to believe that he is in the roll of members of a subversive organization. It devolves upon the accused to prove membership by force or ciorcion. Certainly, one may not be in such a roll without undergoing the concious act of enlistment.

It bears repeating theat warrantless arrests are governed by law and subject to stringent application. Section 5, Rule 113 of the Rules on Criminal Procedure now requires that an offense "has in fact just been committed. "connotes immediacy in point of time and excludes cases under the old rule where an offense 'has in fact been committed' no how long ago. Similarly, the arrestor must have 'personal knowledge of the facts indicating that the [arrestee] has committed it' (instead of just 'reasonable ground believe that the [arrestee] has committed it' under the old rule)." (Dissenting opinion in Ilagan vs. Enrile, G.R. No. 70748, October 21, 1985, 139 SCRA 349, 408).

I deem it aptherein to recall other Court rulings provide guidelines in effecting arrests without warrants. In People vs. Burgos (G.R. No. 68955, September 4, 1986,144 SCRA 1), the Court considered as illegal the warrantless arrest of a subversive not based on the arresting officer's personal knowledge such subversion and held that any rule on arrests witho warrants must be strictly construed. We categorically state therein that warrantless arrests should "clearly fall within the situations when securing a warrant be absurd or is manifestly unnecessary was provided by the Rules" (144 SCRA at 14). Moreover. "it is not enough that there is reasonable ground to believe that the person to be arrested has committed a crime. A crime must in fact or actually (has just) been committed first. That crime has actually been committed is an essential precondition. It is not enough to suspect that a crime may have been committed. The fact of the commission of the offense must be undisputed. The test of reasonable ground applies only to the identity of the perpetrator. (Supra, at p. 15).

Earlier, in Morales, Jr. vs. Enrile (G.R. No. 61016, April 26, 1983, 121 SCRA 538), the Court laid out the procedure to be observed the moment a person is arrested:

At the time a person is arrested, it shall be the duty of the arresting officer to imform him of the reason for the arrest and he must be shown the warrant of arrest, if any. He shall

Page 138: Warrantless Arrests, Searches and Seizures and Privacy of Communication

138

be informed of his constitutional rights to remain silent and to counsel, and that any statement he might make could be used against him. The person shall have the right to communicate with his lawyer, a relative, or anyone he chooses by the most expedient means — by telephone if possible — or by letter or messenger. It shall be the responsibility of the arresting officer to see to it that this is accomplished. No custodial investigation shall be conducted unless it be in the presence of counsel engaged by the person arressted, by any person on his behalf, or appointed by the court upon petition on his behalf, or appointed the court upon the petition either of the detainee himself or by anyone on his behalf. The right to counsel may be waived but the waiver shall not be valid unless made with the assistance of counsel. Any statement obtained in violation of the procedure herein laid down, whether exculpatory or inculpatory, in whole or in part shall be inadmissible evidence. (121 SCRA at 554).

These judicial pronouncements must be observed by everyone concerned: the military and civilian components of the government tasked with law enforcement as well as the ordinary citizen who faces a situation wherein civic duty demands his intervention to preserve peace in the community.

I am not unmindful of the fact that abuses occur in arrests especially of offenders of crimes with a political or ideological element. Such abuses are more often than not, triggered by the difficulty in finding evidence that could stand judicial scrutiny — to pinpoint a subversive, police officers usually have to make long persistent surveillance. However, for the orderly administration of government and the maintenance of peace and order in the country, good faith should be reposed on the officials implementing the law. After all, we are not wanting in laws to hold any offending peace officer liable both administratively and criminally for abuses in the performance of their duties. Victims of abuses should resort to legal remedies to redress their grievances.

If existing laws are inadequate, the policy-determining branches of the government may be exhorted peacefully by the citizenry to effect positive changes. This Court, mandated b the Constitution to uphold the law, can only go as far as inter pruting existing laws and the spirit behind them. Otherwise, we hail be entering the dangerous ground of judicial legislation.

GUTIERREZ, JR., J., concurring and dissenting:

The philosophy adopted in our Constitution is that liberty is an essential condition for order, It is disturbing whenever the Court leans in the direction of order instead of liberty in har cases coming before us.

People all over the world are fast accepting the theory that only as a society encourages freedom and permits dissent can it have lasting security and real progress, the theory that enhancing order through constraints on freedom is deceptive because restrictions on liberty corrode the very values Govenment pretends to promote. I believe we should move with the people of the world who are fast liberating themselves.

I, therefore, vote for the strict application of Section 5 (a) and (b) of Rule 113 on arrests without warrant, to wit:

Sec. 5. Arrest without warrant; when lawful. — A peace officer or a private person may, without a warrant, arrest a person:

(a) When, in his presence, the person to be arrested has committed, is actually committing, or is attempting to commit an offense;

(b) When an offense has in fact just been committed, and he has personal knowledge of facts indicating that the person to be arrested has committed it.

Page 139: Warrantless Arrests, Searches and Seizures and Privacy of Communication

139

xxx xxx xxx

Only in the cases found in the Rule should we allow arrests without warrants. In case of doubt, the tendency should be to declare the warrantless arrest illegal.

Insofar as G.R. Nos, 84581-82, G.R. Nos. 84583-84 and G.R. No. 83162 involving Amelia Roque, Wilfredo Buenaobra, Domingo Anonuevo, Ramon Casiple, and Vicky Ocaya are concerned, the petitioners were arrested after having been apprehended while in possession of illegal firearms and ammunitions. They were actually committing a crime when arrested. I concur in the denial of their motions for reconsideration.

I vote to grant the motion for reconsideration in G.R. No. 85727 where Deogracias Espiritu was arrested while urging jeepnev and bus drivers to join a strike of transport workers on the ground that that was inciting to sedition.

This impresses me as Court validation of a clear infringement of an individual's freedom of speech. "Inciting to sedition" is a term over which the most learned writers and jurists will differ when applied to actual cases. I doubt if there are more than a handful of policemen in the whole country who would know the full dimensions of the fine distinctions which separate the nation's interest in the liberty to fully anfd freely discuss matters of national importance on one hand and the application of the clear and present danger rule as the test when claims of national security and public safety are asserted, on the other. In fact, the percentage of knowledgeability would go down further if we consider that "inciting to sedition" requires the ability to define, among other (1) what kinds of speeches or writings fall lander the term "inciting" (2) the meaning of rising publicly and tumultously; (3,) when does a certain effort amount to force, intimidation. or illegal method; (4) what constitute the five objects or ends of sedition; and (5) what is a scurrilous libel against the Philippines. If we allow public speakers to be picked up simply because what they say is irritating or obnoxious to the ears of a peace officer or critical of government policy and action, we will undermine all pronouncements of this Court on the need to protect that matrix of all freedoms, which is freedom of expression. At the very least, a warrant of arrest after a preliminary examination by a Judge is essential in this type of offense.

Insofar as G.R. No. 81567 is concemed, I join the other dissenting Justices in their observations regarding "continuing oftenses." To base warrantless arrests on the doctrine of continuing offense is to give a license for the illegal detention of persons on pure suspicion. Rebellion, insurrection, or sedition are political offenses where the line between overt acts and simple advocacy or adherence to a belief is extremely thin. If a court has convicted an accused of rebellion and he is found roaming around, he may be arrested. But until a person is proved guilty, I fail to see how anybody can jump to a personal conclusion that the suspect is indeed a rebel and must be picked up on sight whenever seen. The grant of authority in the majority opinion is too broad. If warrantless searches are to be validated, it should be Congress and not this Court which should draw strict and narrow standards. Otherwise, the non-rebels who are critical, noisy, or obnoxious will be indiscriminately lumped up with those actually taking up arms against the Government.

The belief of law enforcement authorities, no matter how well grounded on past events, that the petitioner would probably shoot other policemen whom he may meet does not validate warrantless arrests. I cannot understand why the authorities preferred to bide their time, await the petitioner's surfacing from underground, and pounce on him with no legal authority instead of securing warrants of arrest for his apprehension. The subsequent conviction of a person arrested illegally does not the warrantless arrest.

In G.R. No. 86332, Romulo Bunye was killed on December 14, 1988. The information that Narciso Nazareno was one of the killers came to the attention of peace officers only on December 28, 1988 or fourteen (14) days later. To say that the offense "has in fact just been committed" even if 14 days have lapsed is to stretch Rule 11 3 on warrantless arrests into ridiculous limits. A warrant of arrest is essential in this case. I vote to grant the motion for reconsideration.

Page 140: Warrantless Arrests, Searches and Seizures and Privacy of Communication

140

The subsequent conviction of a person arrested illegally does not reach back into the past and render legal what was illegal. The violation of the constitutional right against illegal seizures is not cured by the fact that the arrested person is indeed guilty of the offense for which he was seized. A government of laws must abide by its own Constitution.

CONSIDERING THE FOREGOING, I VOTE TO:

(1) DENY the motions for reconsideration in G.R. Nos. 84581-82; G.R. No. 84583-84; and G.R. No. 83162;

(2) GRANT the motion for reconsideration in G.R. No. 85727;

(3) GRANT the motion for reconsideration in G.R. No. 86332;and

(4) GRANT the motion for reconsideration in G.R. No. 81567.

CRUZ, J., Separate Opinion:

I reiterate my concurrence with the ponencia insofar as it dismissed the petitions of those who were arrested inflagrante, or subsequently posted bail or chose to remain in the custody of the military, or voluntarily permitted the search of the house without warrant. I do not think that under the applicable circumstances the petitioners can validly complain that they are being unlawfully detained.

But I must again express may dissent to the continued observance of Garcia-Padilla vs. Enrile, 121 SCRA 472, to justify the warrantless arrest and detention of the other petitioners on the ground that they were apprehended for the continuing offenses of rebellion and other allied crimes.

We find in the said decision this partltularly disturbing observation, which was quoted with approval in the originalponencia:

The arrest of persons involved in the rebellion, whether as its fighting armed elements, or for committing non-violent acts but in furtherance of the rebellion, is more an act of capturing them in the course of an armed conflict, to quell the rebellion, than for the purpose of immediately prosecuting them in court for a statutory offense. The arrest, therefore, need not follow the usual procedure in the prosecution of offenses which requires the determination by a judge of the existence of probable cause before the issuance of arrest and the granting of bail of the offense is bailable. Obviously, the absence of a judicial warrant is no legal impediment to arresting or capturing persons committing overt acts of violence against govenment forces, or any other milder acts but equally in pursuance of the rebellious movement. (Emphasis supplied.)

The treatment suggested envisions an actual state of war and is justified only when a recognition of beuigerency is accorded by the legitimate government to the rebels, resulting in the application of the laws of war in the regulation of their relations. The rebels are then considered alien enemies-to be treated as prisoners of war when captured-and cannot invoke the municipal law of the legitimate government they have disowned. It is in such a situation that the processes of the local courts are not observed and the rebels cannot demand the protection of the Bill of Rights that they are deemed to have renounced by their defiance of the government.

But as long as that recognition has not yet been extended, the legitimate govenment must treat the rebels as its citizens, subject to its municipal law and entitled to all the rights provided thereunder, including and especially those guaranteed by the Constitution. Principal among these — in our country — are whose embodied in the Bill of Rights, particularly those guaranteeing due process, prohibiting unreasonable

Page 141: Warrantless Arrests, Searches and Seizures and Privacy of Communication

141

searches and seizures, allowing bail, and presuming the innocence of the accused. The legitimate government cannot excuse the suppression of these rights by the "exigencies" of an armed conflict that at this time remains an intemal matter governed exclusively by the laws of the Republic of the Philippines.

Treatment of the rebels as if they were foreign invaders — or combatants — is not justified in the present situation as our government continues to prosecute them as violators of our own laws. Under the doctrine announced in Garcia-Padilla, however, all persons suspected as rebels are by such suspicion alone made subject to summary arrest no different from the unceremonious capture of an enemy soldier in the course of a battle. The decision itself says that the arrest "need not follow the usual procedure in the prosecution of offenses" and "the absence of a judicial warrant is no impediment" as long as the person arrested is suspected by the authorities of the "continuing offense" of subversion or rebellion or other related crimes. International law is thus substituted for municipal law in regulating the relations of the Republic with its own citizens in a purely domestic matter.

As for the duration of the offenses, the decision contained the following pronouncement which this Court has also adopted as its own:

. . . The crimes of insurrection or rebellion, subversion, conspiracy or proposal to commit such crimes, and other crimes and offenses committed in the furtherance on the occasion thereof, or incident thereto, or in connection therewith under Presidential Proclamation No. 2045, are all in the nature of continuing offenses which set them apart from the common offenses, aside front their essentially involving a massive conspiracy of nationwide manitude. (Emphasis supplied.)

The beginning of the "continuing offense" may be arbitrarily fixed by the authorities, usually by simply placing the suspect "under surveillance," to lay the basis for his eventual apprehension. Once so placed, he may at any time be arrested without warrant on the specious pretext that he is in the process of committing the "continuing offense," no matter that what he may be actuallly doing at the time is a perfectly innocent act.

In the case of Dural. the arrest was made while he was engaged in the passive and innocuous act of undergoing medical treatment. The fiction was indulged that he was even then, as he lay supine in his sickbed, engaged in the continuing offense of rebellion against the State. In further justification, the Court says that the arresting officers acted on "confidential information" that he was in the hospital, which information "was found to be true." This is supposed to have validated the determination of the officers that there was "probable cause" that excused the absence of a warrant.

My own impression is that probable cause must be established precisely to justify the issuance of a warrant, not to dispense with it; moreover, probable cause must be determined by the judge issuing the warrant, not the arresting officer who says it is not necessary.

In the case of Espiritu, the arrest was made while he was actually sleeping, and for allegedly seditious remarks made by him the day before. The Court says his case is not covered by the Garcia-Padilla doctrine but approves the arrest just the same because the remarks were supposed to continue their effects even to the following day. The offense was considered as having been just committed (to make it come under Rule 113, Section 5, of the Rules of Court) despite the considerable time lapse.

It was worse in the case of Nazareno, who was also arrested without warrant, and no less than fourteen days after the killing. In sustaining this act, the Court says that it was only on the day of his arrest that he was identified as one of the probable killers, thus suggesting that the validity of a warrantless arrest is reckoned not from the time of the commission of an offense but from the time of the Identification of the suspect.

Page 142: Warrantless Arrests, Searches and Seizures and Privacy of Communication

142

Section 5 of Rule 113 says that a peace officer may arrest a person without a warrant if the latter "has committed, is actually committing, or is attempting to commit an offense" or when an offense "has in fact just been committed." The requirement of immediacy is obvious from the word "just," which, according to Webster, means "a very short time ago." The arrest must be made almost immediately or soon after these acts, not at any time after the suspicion of the arresting officer begins, no matter how long ago the offense was committed.

I am also uneasy over the following observations in the present resolution which I hope will not be the start of another dangerous doctrine:

The Court, it is true, took into account the admissions of the arrested persons of their membership in the CPP/NPA, as well as their ownership of the unlicensed firearms, ammunitions and documents in their possession. But again, these admissions, as revealed by the records, strengthen the Court's perception that truly the grounds upon wmch the arresting officers based their arrests without warrant, are supported by probable cause, i.e., that the persons arrested were probably guilty of the commission of certain offenses, in compliance with Section 5, Rule 113 of the Rules of Court.

I can only repeat my own misgivings when I dissented in the recent case of People vs. Malmstedt, G.R. No. 91107, June 19, 1991, where I noted: "The conclusion that there was probable cause may have been influenced by the subsequent discovery that the accused was carrying a prohibited drug. This is supposed to justify the soldier's suspicion. In other words, it was the fact of illegal possession that retroactively established the probable cause that validated the illegal search and seizure. It was the fruit of the poisonous tree that washed clean the tree itself."

I submit that the affirmation by this Court of the Garcia-Padilla decision to justify the illegal arrests made in the cases before us is a step back to that shameful past when individual rights were wantonly and systematically violated by the Marcos dictatorship. It seems some of us have short memories of that repressive regime, but I for one am not one to forget so soon. As the ultimate defender of the Constitution, this Court should not gloss over the abuses of those who, out of mistaken zeal, would violate individual liberty in the dubious name of national security. Whatever their ideology and even if it be hostile to ours, the petitioners are entitled to the protection of the Bill of Rights, no more and no less than any other person in this country. That is what democracy is all about.

FELICIANO, J., concurring and dissenting:

I concur in the result reached by the majority in the Resolution disposing of the Motion for Reconsideration.

At the same time, however, I feel compelled to dissent from certain statements made by the majority principally concerning the applicability of the "continuing crimes" doctrine to the problem of arrests without warrants. It seems clear that these statements are really obiter dicta, since they are quite unnecessary for sustaining the actual results reached in the majority Resolution. This was summarily pointed out in my very brief statement concurring in the result reached in the original Decision of the Court dated 9 July 1990. The subsequent developments in several of the cases here consolidated, which are carefully detailed in the majority Resolution, make this even clearer. Nonetheless, the majority Resolution has taken the time and trouble expressly to reiterate the "continuing crimes" doctrine as applicable in respect of warrantless arrests. Although the above statements are obiter, they have been made and, I believe, need to be addressed to some extent and the inter-relation of the "continuing crimes" doctrine with constitutional rights explored.

1. We start at the beginning, that is, the constitutional guarantee against unreasonable seizures of persons. Article III Section 2 of the Constitution reads:

Page 143: Warrantless Arrests, Searches and Seizures and Privacy of Communication

143

Sec. 2. The right of the people to be secure in their persons, houses, papers, and effects against unreasonable searches and seizures of whatever nature and for any purpose shall be inviolable, and no search warrant or warrant of arrest shall issue except upon probable cause to be determined personally by the judge after examination under oath or affirmation of the complainant and the witnesses he may produce, and particularly describing the place to be searched and the persons or things to be seized. (Emphais supplied)

Under the above provision, arrests, i.e., the constraint and seizure of the persons of individual members of society, must, as a general rule, be preceded by the securing of a warrant of arrest, the rendition of which complies with the constitutional procedure specified in Article III Section 2. Arrests made without a warrant issued by a judge after complying with the constitutional procedure, are prima facie unreasonable seizures of persons within the meaning of Article III Section 2.

2. There are, however, certain well-recognized exceptions to the norm that warrantless arrests are unreasonable seizures of persons. Those exceptions are, in our day, essentially found in Section 5(a) and (b) of Rule 113 of the Rules of Court. Section 5(a) and (b) mark out the situations where an officer of the law, or a private person for that matter, may lawfully arrest a person without previously securing a warrant of arrest. The full text of Section 5, Rule 113 follows:

Sec. 5. Arrest without warrant, when lawful. — A peace officer or a private person may, without a warrant, arrest a person:

(a) When, in his presence, the person to be arrested has committed, is actually committing, or is attempting to commit an offense;

(b) When an offense has in fact just been committed, and he has personal knowledge of facts indicating that the person to be arrested has committed it; and

(c) When the person to be arrested is a prisoner who has escaped from a penal establishment or place where he is serving final judgment or temporarily confined while his case is pending, or has escaped while being transferred from one confinement to another.

In cases falling under paragraphs (a) and (b) hereof, the person arrested without a warrant shall be forthwith delivered to the nearest police station or jail, and he shall be proceeded against in accordance with Rule 112, Section 7.

3. Before examining the scope and implications of Section 5(a) and (b), it is important to recall that judicial interpretation and application of Section 5(a) and (b) must take those provision for what they are: they areexceptions to a vital constitutional norm enshrined in the Bill of Rights. Exceptions to such a norm must be strictly construed so as not to render futile and meaningless the constitutional rule requiring warrants of arrests before the persons of individuals may be lawfully constrained and seized. The ordinary rule generally applicable to statutory provisions is that exceptions to such provisions must not be stretched beyond what the language in which they are cast fairly warrants, and all doubts should be resolved in favor of the general provision, rather than the exception. 1 This rule must apply with special exigency and cogency where we deal, not with an ordinary statutory provision, but with a constitutional guarantee. 2 Exceptions to such a guarantee must be read with especial care and sensitivity and kept within the limits of their language so to keep vital and significant the general constitutional norms warrantless arrests. In Alvarez vs. Court of First Instance, 3 this Court, stressing that:

II. As the protection of the citizen and the maintenance of his constitutional rights is one of the highest duties and privileges of the court. these constitutional guaranties should be given a liberal construction or a strict construction in favor of the individual, to prevent

Page 144: Warrantless Arrests, Searches and Seizures and Privacy of Communication

144

stealthy encroachment upon, or gradual depreciation of, the rights secured by them (State vs. Custer County, 198 Pac., 362; State vs. McDaniel, 231 Pac., 965; 237 Pac., 373). Since the proceeding is a drastic one, it is the general rule that statutes authorizing searches and seizures or search warrants must be strictly construed (Rose vs. St. Clair, 28 Fed. [2d], 189; Leonard vs. U.S., 6 Fed. [2d], 353; Perry vs. U.S., 14 Fed. [2d], 88; Cofer vs. State, 118 So., 613. (emphasis supplied)

held that:

. . . All illegal searches and seizures are unreasonable whith lawful ones are reasonable. 4

In People vs. Burgos, 5 this Court reiterated the above rule in the following terms:

There is no such personal knowledge in this case. Whatever knowledge was possessed by the arresting officers, it came in its entirety from the information furnished by Cesar Masamlok. The location of the firearm was given by the appellant's wife.

At the time of the appellant's arrest, he was not in actual possession of any firearm or subversive document. Neither was he commit ting any act which could be described as subversive. He was, in fact plowing his field at the time of the arrest.

The right of a person to be secure against any unreasonable seizure of his body and any deprivation of his liberty is a most basic and fundamental one. The statute or rule which allows exceptions the requirement of warrants of arrest is strictly construed. Any exception must clearly fall within the situations when securing a warrant would be absurd or is manifestly unnecessary as provided by the Rule. We cannot liberally construe the rule on arrests without warrant or extend its application beyond the cases specifically provided by law. To do so would infringe upon personal liberty and set back a basic right so often vilated and so deserving of full protection. 6 (emphasis supplied)

4. Section 5(a) relates to situations where a crime is committed or attempted to be committed in the presence of the arresting officer. The fact of the occurrence of the offense, or of the attempt to commit an offense, in the presence of the arresting officer, may be seen to be the substitute, under the circumstances, for the securing of a warrant of arrest. In such situation, there is an obvious need for immediate, even instantaneous, action on the part of the arresting officer to suppress the breach of public order and to prevent further breaches then and there. Section 5(a) may, moreover, be seen to refer to overt acts constitutive of a crime taking place in the presence of the arresting officer. The term "presence" in this connection is properly and restrictively construed to relate to acts taking place within the optical or perhaps auditory perception of the arresting officer. 7 If no overt, recognizably criminal, acts occur which are perceptible through the senses of the arresting officer, such officer could not, of course, become aware at all that a crime is being committed or attempted to be committed in his presence. 8 It is elementary that purely mental or psychological phenomena, not externalized in overt physical acts of a human person, cannot constitute a crime in our legal system. For a crime to exist in our legal law, it is not enough that mens rea be shown; there must also be an actus reus. If no such overt acts are actually taking place in the presence or within the sensor perception of the arresting officer, there would, in principle, be ample time to go to a magistrate and ask for a warrant of arrest. There would, in other words, not be that imperious necessity for instant action to prevent an attempted crime, to repress the crime being committed, or to capture the doer of the perceive criminal act, the necessity which serves as the justification in law of warrantless arrests under Section 5(a).

5. Turning to Section 5 (b), two (2) elements must be coincide before a warrantless arrest may be sustained under this subsection: 1) the offense must have "just been committed" when the arresting officer arrived in the scene; and 2) the officer must have "personal knowledge" of facts indicating tha the

Page 145: Warrantless Arrests, Searches and Seizures and Privacy of Communication

145

person to be arrested has committed the offense. In somewhat different terms, the first requirement imports that th effects or corpus of the offense which has just been committed are still visible: e.g. a person sprawled on the ground, dead of gunshot wound; or a person staggering around bleeding profusely from stab wounds. The arresting officer may not ha seen the actual shooting or stabbing of the victim, and thereto the offense can not be said to have been committed "in [his] presence." The requirement of "personal knowledge" on the part of the arresting officer is a requirement that such knowledge must have been obtained directly from sense perception the arresting officer. That requirement would exclude informtion conveyed by another person, no matter what his reputation for, truth and reliability might be. 9 Thus, where the arresting officer comes upon a person dead on the street and sees a person running away with a knife from where the victim is sprawled the ground, he has personal knowledge of facts which render it highly probable that the person fleeing was the doer of the criminal deed. The arresting officer must, in other words, perceive through his own senses some act which directly connects the person to be arrested with the visible effects or corpus of a crime which has "just been committed."

6. The use of the words "has in fact just been committed" underscores the requirement that the time interval between the actual commission of the crime and the arrival of the arresting officer must be brief indeed. In the first place, the word "just" was fairly recently inserted in Section 5(b) by the 1985 Rules on Criminal Procedures, no doubt in order to underscore the point here being made. In the second place, a latitudinarian view of the phrase "has in fact just been committed" would obviously render pointless the requirement in Section 5(a) that the crime must have been committed "[in] the presence" of the arresting officer. In G.R. No. 86332, the warrantless arrest of Alfredo Nazareno 14-days after the occurrence of the killing with which he was charged along with other persons, cannot by any standard be justified under Section 5(b). In G.R. No. 81567, Dural was arrested without warrant while being treated in a hospital the day after the shooting of the policemen in which he was suspected to have been a participant. While 1-day may be substantially different from 14-days, still it must be pointed out that at the time Dural was arrested in the hospital, the killing of the two (2) policemen in Caloocan City far away from the St. Agnes Hospital in Quezon City could not reasonably be said to have been just committed. There was no showing, nor did the Court require it, that the arresting officers had been in "hot pursuit" of Dural beginning at the scene of the killing and ending the next day in the hospital.

7. It is worth noting that the requisite of "personal knowledge" on the part of the arresting officer who is determining "probable cause" right at the scene of the crime, is in a sense more exacting than the standard imposed by the Constitution upon the judge who, in the seclusion of his chambers, ascertains "probable cause" by examining the evidence submitted before him. The arresting officer must himself have "personal knowledge"; the magistrate may rely upon the personal knowledge of the witnesses examined by or for him in issuing a warrant of arrest. In the present Resolution, the majority begins with noting the requirement of "personal knowledge" in Section 5(b), but winds up in the next page with a very diluted standard of "reasonable belief and "good faith" on the part of the arresting officers. The stricter standard is properly applicable to the officers seizing a person without a warrant of arrest, for they are acting in derogation of a constitutional right. That the person unlawfully arrested without a warrant may later turn out to be guilty of the offense he was suspected of in the first place is, course, quite beside the point. Even a person secretly guilty some earlier crime is constitutionally entitled to be secure from warrantless arrest, unless he has in fact committed physically observable criminal acts in the presence of the arresting officer or hadjust committed such acts when the arresting officer burst upon the scene.

8. Examination of the utilization in the majotity Resolution of the doctrine of "continuing crimes," shows that doctrine is here being used as a substitute for the requirement under Section 5(a) that the offense "has in fact just been presence of the arresting officer arrived, but rather because the person to be arrested is suspected of having committed a crime in the future. The pertinent portion of the majority Resolution reads:

. . . Dural did not cease to be, or because less of a subversive, FOR PURPOSE OF ARREST, simply because he was, at the time of arrest, confined in the St. Agnes Hospital. . . . That Dural had shot the two (2) policemen in Caloocan City as part of his

Page 146: Warrantless Arrests, Searches and Seizures and Privacy of Communication

146

mission as a "sparrow" (NPA member) did not end there and then. Dural, given another opportunity, would have shot or would shoot other policemen anywhere as agents or representatives of organized government. It is in this sense that subversion like rebelion (or insurrection) is perceived here as a continuing offense. Unlike other so-called "common" offenses, i.e., adultery, murder, arson, etc., which generally end upon their commission, subversion and rebellion are anchored on an ideological base which compels the repetition of the same acts of lawlessness and violence until the overriding objectives of overthrowing organized government is attained. (Emphasis supplied)

9. I respectfully submit that an examination of the "continuing crimes" doctrine as actually found in our case law offers no reasonable basis for such use of the dotrine. More specifically, that doctrine, in my submission, doesnot dispence with the requirement that overt acts recognizably criminal in character must take place in the presence of the arresting officer, or must have just been committed when the arresting officer arrived, if the warrantless arrest it to be lawful. The "continuing crimes" doctrine in our case law (before rendition of Garcia-Padilla vs. Enrile 10 does not sustain warrantless arrests of person to be arrested is, as it were, merely resting in between specific lawless and commit the moment he gets an opportunity to do so.

Our case law shows that the "continuing crimes" doctrine has been used basically in relation to two (2) problems: the first problem is that of determination of whether or not a particular offense was committed within the territorial jurisdiction of the trial court; the second problem is that of determining whether a single crime or multiple crimes were committed where the defense of double jeopardy is raised.

10. In respect of the first problem, the gist of our case law is that where some of the ingredients or elements of an offense taken place within the territorial jurisdiction of one court and some other ingredients or elements of the same offense occur in the territory of another court, (e.g., estafa or malversation) either one of the two courts has jurisdiction to try the offense. Where all of the essential elements of a crime take place within the territory of one court but "by reason of he very nature of the offense committed" the violation of the law is deemed to be "continuing," then the court within whose territorial jurisdiction the offense continues to be committed, has jurisdiction to try a person charged with such offense. In the latter case, the offense is deemed to be continuing because some or all of the elements constituting the offense occurred within jurisdiction of the second court (e.g., kidnapping and illegal detention; libel; evasion of service of sentence). The criminal acts are regarded as repeated or as continuing within the province or city where the defendant was found and arrested. 11 Clearly, overt acts of the accussed constituting elements of the crime charged must be shown to have been committed within the territorial jurisdiction of the court where he is charged.

11. Turning to the second type of problem, the question is normally presented in terms of whether one crime or multiple crimes were committed by the accused. Where the series of acts actually alleged and proven to have been committed by the accused constituted only one and the same crime, the defense of double jeopardy becomes available where a second information is filed covering acts later in the series. Upon the other hand, where the acts of the accused constituted discrete, multiple offenses, each act comprising a distinct and separate offense, the double jeopardy defense is non-available. 12 The point worth stressing is that in passing upon the issue relating to the unity or multiplicity of offense committed, the overt acts of the accused constitutive either of the single offense or of the plural offenses, must be shown.

12. My final submission, is that, the doctrine of "continuing crimes," which has its own legitimate function to serve in our criminal law jurisprudence, cannot be invoked for weakening and dissolving the constitutional guarantee against warrantless arrest. Where no overt acts comprising all or some of the elements of the offense charged are shown to have been committed by the person arrested without warrant, the "continuing crime" doctrine should not be used to dress up the pretense that a crime, begun or committed elsewhere, continued to be committed by the person arrested in the presence of the arresting officer. The capacity for mischief of such a utilization of the "continuing crimes" doctrine, is infinitely increased where the crime charged does not consist of unambiguous criminal acts with a definite

Page 147: Warrantless Arrests, Searches and Seizures and Privacy of Communication

147

beginning and end in time and space (such as the killing or wounding of a person or kidnapping and illegal dentention or arson) but rather of such problematic offenses as membership in or affiliation with or becoming a member of, a subversive association or organization. For in such cases, the overt constitutive acts may be morally neutral in themselves, and the unlawfulness of the acts a function of the aims or objectives of the organization involved. Note, for instance, the following acts which constitute prima facie evidence of "membership in any subversive association:" 13

a) Allowing himself to be listed as a member in any book or any of the lists, records, correspondence, or any other document of the organization;

b) Subjecting himself to the discipline of such association or organization in any form whatsoever;

c) Giving financial contribution to such association or organization in dues, assessments, loans or in any other forms;

xxx xxx xxx

f) Conferring with officers or other members of such association or organization in furtherance of any plan or enterprise thereof;

xxx xxx xxx

h) Preparing documents, pamphlets, leaflets, books, or any other type of publication to promote the objectives and purposes of such association or organization;

xxx xxx xxx

k) Participating in any was in the activities, planning action, objectives, or purposes of such association or organization;

xxx xxx xxx

It may well be, as the majority implies, that the constitutional rule against warrantless arrests and seizures makes the law enforcement work of police agencies more difficult to carry out. It is not our Court's function, however, and the Bill of Rights was not designed, to make life easy for police forces but rather to protect the liberties of private individuals. Our police forces must simply learn to live with the requirements of the Bill of Rights, to enforce the law by modalities which themselves comply with the fundamental law. Otherwise they are very likely to destroy, whether through sheer ineptness or excess of zeal, the very freedoms which make our polity worth protecting and saving.

REGALADO, J.: Separate Opinion:

While I have heretofore concurred in the ponencia in the above-entitled cases and I reiterate such concurrence, I wish to unburden myself of some reservations on the rationale adopted in G.R. No. 86332.

It is posited in this resolution that "(a)lthough the killing of Bunye II occurred on 14 December 1988, while Nazareno's arrest without warrant was made only on 28 December 1988, or 14 days later, the arrest falls under Section 5(b) of Rule 113, since it was only on 28 December 1988 that the police authorities came to know that Nazareno was probably one of those guilty in the killing of Bunye II."

I am afraid that there has been a misapplication of Section 5(b) of Rule 113 which, while authorizing a peace officer or a private person to effect a warrantless arrest, specifically conditions that grant of

Page 148: Warrantless Arrests, Searches and Seizures and Privacy of Communication

148

authority upon the situation "(w)hen an offense has in fact just been committed, and he has personal knowledge of facts indicating that the person to be arrested has committed it."

It is significant that when the corresponding provisions of the 1964 Rules of Court were amended in the 1985 Rules of Criminal Procedure, the particular revision of paragraph (b) of the aforesaid section consisted in imposing the requirements that the person making the arrest has personal knowledge of the facts indicating that the arrestee is responsible for an offense which has just been committed.

Now, according to the resolution, "the records show that in the morning of 14 December 1988, Romulo Bunye II was killed by a group of men in Alabang, Muntinlupa, Metro Manila; that at about 5 o'clock in the morning of 28 December 1988, Ramil Regala, one of the suspects in the said killing, was arrested and he pointed to Narciso Nazareno as one of his companions during the killing of Bunye II; that at 7:20 of the same morning (28 December 1988), the police agents arrested Nazareno, without warrant, for investigation."

Since, clearly, the arresting police agents merely acted upon the information imparted by one of the suspects, Ramil Regala, the resolution has emasculated the requirement in Section 5(b) that the person making the arrest must have had personal knowledge of factual indications regarding the complicity or liability of the arrestee for the crime. Yet, that amendment requiring such personal knowledge must have been designed to obviate the practice in the past of warrantless arrests being effected on the basis of or supposed reliance upon information obtained from third persons who merely professed such knowledge or, worse, concocted such reports for variant reasons not necessarily founded on truth.

Further, and obviously as an added deterrent to the possibility that such arrest without a warrant may result from imputations based on dubious motives, it is now required that the crime must have just been committed. The recency contemplated here, in relation to the making of the warrantless arrest, is the time when the crime was in fact committed, and not the time when the crime was in fact committed, and not the time when the person making the arrest learned or was informed of such commission. Otherwise, at the risk of resorting to reductio ad absurdum, such warrantless arrests could be validly made even for a crime committed, say, more than a year ago but of which the arresting officer received information only today.

The brevity in the interval of time between the commission of the crime and the arrest, as now required by Section 5(b), must have been dictated by the consideration, among others, that by reason of such recency of the criminal occurrence, the probability of the arresting officer acquiring personal and/or reliable knowledge of such fact and the identity of the offender is necessarily enhanced, if not assured. The longer the interval, the more attenuated are the chances of his obtaining such verifiable knowledge. In the case under consideration, the obtention of information of a crime committed fourteen (14) days earlier necessarily undermines the capacity of the arresting officer to ascertain the reliability of the information he is acting upon and to acquire personal knowledge thereof after such verification.

It may be granted, as an ad hoc proposition, that the arrest of Nazareno was based on probable cause and it was not whimsical, at least, in this instance. It is correct to say that prevailing conditions affecting national security and stability must also be taken into account. However, for the reasons above elucidated, I take exception to the conclusion that the conditions in Section 5(b) of Rule 113 had been complied with in this case. It is true that the corresponding information was filed against Nazareno shortly after his arrest but that, precisely, is another cause for controversy. Definitely, if the rules on arrest are scrupulously observed, there would be no need for the usual invocation of Ilagan as a curative balm for unwarranted incursions into civil liberties.

SARMIENTO, J.: dissenting:

I reiterate my dissent. I submit that in spite of its "clarificatory" resolution, 1 the majority has not shown why the arrests in question should after all be sustained.

Page 149: Warrantless Arrests, Searches and Seizures and Privacy of Communication

149

According to the majority, Rolando Dural (G.R. No. 815667) was validly arrested without a warrant and that his arrest was sufficient compliance with the provisions of Section 5, paragraph (b), Rule 113, of the Rules of Court. According to the majority, he, Dural, was after all committing an offense (subversion being supposedly a continuing offense) and that the military did have personal knowledge that he had committed it. "Personal knowledge," according to the majority, is supposedly no more than "actual belief or reasonable grounds . . . of suspicion," and suspicion is supposedly reasonable:

. . . when, in the absence of actual belief of the arresting officers, the suspicion that the person to be arrested is probably guilty of committing the offense, is based on actual facts, i.e., supported by circumstances sufficiently strong in themselves to create the probable cause of guilty of the person to be arrested. A reasonable suspicion therefore must be founded on probable cause, coupled with good faith on the part of the peace officers making the arrest. 2

As I said, I dissent.

First, and as I held, subversion, as an offense punished by Executive Order No. 167, as amended by Executive Order No. 276, in relation to Republic Act No. 1700, 3 is made up of "overt acts." 4 In People vs. Ferrer 5 this Court defined "overt acts" as follows:

. . . Indeed, were the Anti-Subversion Act a bill of attainder, it would be totally unnecessary to charge Communists in court, as the law alone, without more would suffice to secure their punishment. But the undeniable fact is that their guilt still has to be judicially established. The Government has yet to prove at the trial that the accused joined the Party knowingly, willfully and by overt acts, and that they joined the Party, knowing its subversive character and with specific intent to further its basic objective, i.e., to overthrow the existing government by force, deceit, and other illegal means and place the country under the control and domination of a foreign power.

As Ferrer held, that above "overt acts" constitute the essence of "subversion," and as Ferrer has taken pains to explain, the law requires more than mere membership in a subversive organization to make the accused liable. I respectfully submit that for purposes of arrest without a warrant, that above "overt acts" should be visible to the eyes of the police officers (if that is possible), otherwise the accused can not be said to be committing any offense within the contemplation of the Rules of Court, to justify police action, and otherwise, we would have made "subversion" to mean mere "membership" when, as Ferrer tells us, subversion means more that mere membership.

I find strained that majority's interpretation of "personal knowledge," as the majority would interpret it, as no more than "actual belief or reasonable suspicion," that is, "suspicion . . . based on actual facts . . . [and] founded on probable cause, coupled with good faith . . . " 6 I submit that personal knowledge means exactly what it says — that the peace officer is aware that the accused has committed an offense, in this case, membership in a subversive organization with intent to further the objectives thereof. It is to be noted that prior to their amendment, the Rules (then Section 6) spoke of simple "reasonable ground" — which would have arguably encompassed "actual belief or suspicion . . . coupled with good faith" referred to by the majority. Section 5(b) as amended, however, speaks of "personal knowledge"; I respectfully submit that to give to "personal knowledge" the same meaning as "reasonable ground" is to make the amendment as useless exercise.

What, furthermore, we have here was a mere "confidential information" that a "sparrow man" had been wounded and was recuperating in the hospital, and that that person was Rolando Dural. Clearly, what we have is second-hand, indeed, hearsay, information, and needless to say, not personal knowledge.

I would like to point out that in the case of People vs. Burgos 7 this Court rejected a similar arrest because of lack of personal knowledge, and, as the Court held, "[w]hatever knowledge was possessed by

Page 150: Warrantless Arrests, Searches and Seizures and Privacy of Communication

150

the arresting officers came in its entirety from the information furnished by [another] . . ." 8 I do not see how We can act differently here.

I do not find the majority's reliance on the case of United States vs. Santos 9 to be well-taken. Santos involved a prosecution for coercion (against a peace officer for affecting an arrest without a warrant). Santos, however, did in fact affirm the illegality of the arrest but absolved the peace officer on grounds of good faith. Santos did not say that so long as he, the peace officer, was acting in good faith, as the majority here says that the military was acting in good faith, the arrest is valid. Quite to the contrary, Santos suggested that notwithstanding good faith on the part of the police, the arrest is nevertheless subject to question.

As far as the information leading to the arrest of Dural is concerned, the majority would quite evidently swallow the version of the military as if in the first place, there truly was an information, and that it was reliable, and that "it was found to be true;" 10 and as if, in the second place, the hospital authorities (the alleged informants) could have legally tipped the military under existing laws. We have, it should be noted, previously rejected such a species of information because of the lack of "compulsion for [the informant] to state truthfully his charges under pain of criminal prosecution." 11 Here, it is worse, because we do not even know who that informant was.

The majority is apparently unaware that under Executive Order No. 212, amending Presidential Decree No. 169, hospital establishments are required to report cases of acts of violence to "government health authorities" — not to the military.

I am concerned that if the military were truly armed with reliable information and if it did have personal knowledge to believe that Dural had committed an offense, there was no reason for the military to ignore the courts, to which the Constitution after all, gives the authority to issue warrants. As People vs. Burgos held:

More important, we find no compelling reason for the haste with which the arresting officers sought to arrest the accused. We fail to see why they failed to first go through the process of obtaining a warrant of arrest, if indeed they had reasonable ground to believe that the accused had truly committed a crime. There is no showing that there was a real apprehension that the accused was on the verge of flight or escape. Likewise, there is no showing that the whereabouts of the accused were unknown. 12

I do not likewise see how the petitioners Amelia Roque, Wilfredo Buenaobra, Domingo Anonuevo, Ramon Caspile, and Vicky Ocaya (G.R. Nos. 84581-82; 83162) could have been lawfully picked up under similar circumstances. As the majority points out, the military had (again) acted on a mere tip-the military had no personal knowledge (as I elaborated what personal knowledge means). Second, I do not think that the majority can say that since Amelia Roque, et al. "were NPA's anyway" (As Roque, et al. allegedly admitted), immediate arrests were "prudent" and necessary. As I said, that Roque, et al. were admitted "NPA's" is (was) the question before the trial court and precisely, the subject of controversy. I think it is imprudent for this Court to pass judgment on the guilt of the petitioners-since after all, and as the majority points out, we are talking simply of the legality of the petitioner's arrests.

More important, that Roque, et al. "were NPA's anyway" is evidently, a mere say-so of the military, and evidently, the Court is not bound by bare say-so's. Evidently, we can not approve an arrest simply because the military says it is a valid arrest (the accused being "NPA's anyway")— that would be abdication of judicial duty and when, moreover, the very basis of the claim rests on dubious "confidential information."

According to the majority, we are speaking of simple arrests; we are not talking of the guilt or innocence of the accused. I certainly hope not, after the majority referred to Rolando Dural as a "sparrow man" and having Amelia Roque, et al. admit to being NPA's."

Page 151: Warrantless Arrests, Searches and Seizures and Privacy of Communication

151

It is to gloss over at any rate, the nature of arrest as a restraining on liberty. It is to me immaterial that the guilt of the accused still has to be established, since meanwhile, the accused are in fact being deprived of liberty. Arrest to me, is something to crow about, even if in the opinion of the majority, it is nothing to crow about (a mere "administrative measure").

I can not, again, accept the validity of the arrests of Deogracia Espiritu or Narciso Nazareno (G.R. Nos. 85727; 86332). Espiritu was supposedly picked up for inciting to sedition, in uttering supposedly, on November 22, 1988, the following:

Bukas tuloy and welga natin . . . hanggang sa magkagulo na. 13

Espiritu however was arrested on November 23, 1988, a day later-and in no way is "inciting to sedition" a continuing offense. Obviously, the majority is not saying that it is either, but that:

. . . Many persons may differ as to the validity of such perception and regard the language as falling within free speech guaranteed by the Constitution. But, then, Espiritu has not lost the right to insist, during the trial on the merits, that he was just exercising his right to free speech regardless of the charged atmosphere in which it was uttered. But, the authority of the peace officers to make the arrest, without warrant, at the time the words were uttered, or soon thereafter, is still another thing. In the balancing of authority and freedom, which obviously becomes difficult at times, the Court has, in this case, titled the scale in favor of authority but only for purposes of the arrest (not conviction). Let it be noted that the Court has ordered the bail for Espiritu's release to be reduced from P60,000.00 to P10,000.00.14

And obviously, the majority is concerned about whether or not Espiritu's speech was after all, protected speech, but apparently, that is also of no moment, since: (1) that is a matter of defense; (2) we are talking of mere arrests, and as far as arrests are concerned, "the Court has, in this case, titled in favor of authority," 15 and (3) we have, anyway, given a reduced bail to the accused.

First, that the accused's statement is in the category of free speech is not only plain to my mind, it is a question I do not think the majority can rightly evade in these petitions without shirking the Court's constitutional duty. It is to my mind plain, because it does not contain enough "fighting words" recognized to be seditious. 16 Secondly, it is the very question before the Court—whether or not the statement in question constitutes an offense for purposes of a warrantless arrest. It is a perfectly legal question to my mind and I am wondering why we can not answer it.

What the majority has not answered, as I indicated, is that inciting to sedition is in no way a continuing offense, and as I said, the majority is not apparently convicted that it is, either. Of course, the majority would anyway force the issue: "But the authority of the peace officers to make the arrest, without warrant, at the time the words were uttered, or soon thereafter, is still another thing." 17 First, Espiritu was picked up the following day, and in no way is "the following day" "soon thereafter". Second, we would have stretched the authority of peace officers to make warrantless arrests for acts done days before. I do not think this is the contemplation of the Rules of Court.

As in the case of Burgos in People vs. Burgos, 18 Espiritu was neither "on the verge of flight or escape" 19 and there was no impediment for the military to go through the judicial processes, as there was none in the case of Burgos.

In the case of People vs. Aminnudin, 20 this Court held that unless there "was a crime about to be committed or had just been committed," and unless there existed an urgency as where a moving vehicle is involved, instant police action can not be justified.

Page 152: Warrantless Arrests, Searches and Seizures and Privacy of Communication

152

"In the balancing of authority and freedom," states the majority, "the Court has, in this case, titled in favor of authority but only for purposes of the arrest (not conviction)." 21 It is a strange declaration, first, because it is supported by no authority (why the Court should "tilt" on the side of Government), and second, because this Court has leaned, by tradition, on the side of liberty — as the custodian of the Bill of Rights — even if we were talking of "simple" arrests.

I do not understand why this Court should "tilt" . . . the scale in favor of authority . . . in this case," 22 as if to say that normally, this Court would have tilted the scales the other way. I do not understand why these cases are apparently, special cases, and apparently, the majority is not telling us neither. I am wondering why, apart from the fact that these cases involved, incidentally, people who think differently from the rest of us.

The majority goes on:

Although the killing of Bunye II occurred on 14 December 1988, while Nazareno's arrest without warrant was made only on 28 December 1988, or 14 days later, the arrest falls under Section 5(b) of Rule 113, since it was only on 28 December 1988 that the police authorities came to know that Nazareno was probably one of those guilty in the killing of Bunye II. 23

With all due respect, I do not think that the majority is aware of the serious implications of its pronouncement on individual rights (and statutory construction in general), and I feel I am appropriately concerned because as a member of the Court, I am co-responsible for the acts of my colleagues and I am afraid that I may, rightly or wrongly, be in time made to defend such an indefensible pronouncement.

Section 5(b) of Rule 113 is clear and categorical: the offense must have been "just committed" and the authorities must have "personal knowledge."

In no way can an offense be said to have been "just committed" fourteen days after it was in fact (allegedly) committed. In no way can the authorities be said to have "personal knowledge" two weeks thereafter; whatever "personal knowledge" they have can not possibly be "personal knowledge" of a crime that had "just been committed;" whatever "personal knowledge" they have is necessarily "personal knowledge" of a crime committed two weeks before.

In no way can Nazareno's arrest be said to be an arrest sanctioned by the exceptional provisions of the Rules.

I am not saying that the military can not act in all cases, and it is sheer ignorance to suppose that I am saying it, (or worse, that I am "coddling criminals"). I am not saying that a suspected criminal, if he can not be arrested without a warrant, can not be arrested at all — but that the military should first procure a warrant from a judge before effecting an arrest. It is not too much to ask of so-called law enforcers.

As it is, the majority has enlarged the authority of peace officers to act, when the Rules have purposely limited it by way of an exception, precisely, to the general rule, mandated by the Constitution no less, that arrests may be done only through a judicial warrant. As it is, the majority has in fact given the military the broadest discretion to act, a discretion the law denies even judges 24 — today it is fourteen days, tomorrow, one year, and sooner, a decade. I submit that a year, a decade, would not be in fact unreasonable, following the theory of the majority, since the military can claim anytime that it "found out only later," as the majority did not find it unreasonable for the Capital Command to claim that it "came to know that Nazareno was probably one of those guilty in the killing of Bunye II" 25—and none of us can possibly dispute it.

I would like to stress strongly that we are not talking of a simple "administrative measure" alone—we are talking ofarrests, of depriving people of liberty—even if we are not yet talking of whether or not people are

Page 153: Warrantless Arrests, Searches and Seizures and Privacy of Communication

153

guilty. That we are not concerned with guilt or innocence is hardly the point, I respectfully submit, and it will not minimize the significance of the petitioners' predicament.

With respect to Wilfredo Buenaobra, I submit that the majority has, as in the cases of Amelia Roque, et al., ignored the fact that Buenaobra's alleged "admission" (actually, an uncounselled confession) was precisely, the basis for Buenaobra's arrest. It is to beg the question, I respectfully submit, to approve the military's action for the reason that Buenaobra confessed, because Buenaobra confessed for the reason that the military, precisely, pounced on him. I am not to be mistaken for prejudging Buenaobra's innocence (although it is supposed to be presumed) but I can not imagine that Buenaobra would have voluntarily proclaimed to the military that he was an NPA courier so that the military could pounce on him.

I respectfully submit that the cases Garcia vs. Padilla 26 and Ilagan vs. Enrile 27 have been better days. I do not see how this court can continuously sustain them "where national security and stability are still directly challenged perhaps with greater vigor from the communist rebels." 28 First and foremost, and as the majority has conceded, we do not know if we are in fact dealing with "Communists." The case of Deogracias Espiritu, for one, hardly involves subversion. Second, "Communism" and "national security" are old hat — the dictator's own excuses to perpetuate tyranny, and I am genuinely disappointed that we would still fall for old excuses. Third, Garcia and Ilagan rested on supposed grounds that can not be possibly justified in a regime that respects the rule of law — that the Presidential Commitment Order (PCO) is a valid presidential document (Garcia) and that the filing of an information cures a defective arrest (Ilagan). Fourth and finally, it is evident that neither "Communist threat" nor "national security" are valid grounds for warrantless arrests under Section 5(b) of Rule 113.

I most respectfully submit that Garcia and Ilagan have not only been diluted by subsequent jurisprudence (e.g., People vs. Burgos, supra), they are relics of authoritarian rule that can no longer be defended, if they could have been defended, in Plaza Miranda or before our own peers in the bar.

"What is important," says the majority, "is that every arrest without warrant be tested as to its legality, via habeas corpus proceedings." 29 I supposed that goes without saying. But it is also to patronize the petitioners and simply, to offer a small consolation, when after all, this Court is validating their continued detention. 30 With all due respect, I submit that it is nothing for which the public should be elated.

A Final Word

As I began my dissent, in this Resolution and the Decision sought to be reconsidered, I reiterate one principle: The State has no right to bother citizens without infringing their right against arbitrary State action. "The right of the people," states the Constitution, "to be secure in their persons, houses, papers, and effects against unreasonable searchers and seizures of whatever nature and for any purpose shall be inviolable . . . ." 31 "The State," the Charter likewise states, "values the dignity of every human person and guarantees full respect for human rights." 32 The Constitution states the general rule — the majority would make the exception the rule, and the rule the exception. With all due respect, this is not what constitutionalism is all about.

I submit that the "actual facts and circumstances" the majority refers to are, in the first place, doubtful, the "actual facts and circumstances" being no more than "confidential information" (manufactured or genuine, we have no way of telling) and in the second place, any information with which the military (or police) were armed could no more than be hearsay, not personal, information. I submit that the "actual facts and circumstances" the majority insists on can not justify the arrests in question under Section 5(b) of Rule 113, the rule the majority insists is the applicable rule.

Apparently, Section 5(b) is not the applicable rule, as far as Deogracias Espiritu and Narciso Nazareno are concerned; certainly, it is not the Section 5(b) I know. As I indicated, Espiritu was arrested one day after the act, allegedly, inciting to sedition; Nazareno was picked up fourteen days after it (allegedly, murder). Yet, the majority would approve the police's actions nonetheless because the police supposedly

Page 154: Warrantless Arrests, Searches and Seizures and Privacy of Communication

154

"found out only later." I submit that the majority has read into Section 5(b) a provision that has not been written there.

"More than the allure of popularity of palatability to some groups," concludes the majority, "what is important is that the Court be right." 33

Nobody has suggested in the first place, that Umil was and is a question of popularity or palatability. Umil is a question, on the contrary, of whether or not the military (or police), in effecting the arrests assailed, had complied with the requirements of law on warrantless arrests. Umil is a question of whether or not this Court, in approving the military's actions, is right.

In spite of "EDSA", a climate of fear persists in the country, as incidences of disappearances, torture, hamletting, bombings, saturation drives, and various human rights violations increase in alarming rates. In its update for October, 1990, the Task Force Detainees of the Philippines found:

An average of 209 arrested for political reasons monthly since 1988, 94% of them illegally;

Four thousand four hundred eight (4,408) political detentions from January, 1989 to September, 1990, 4,419, illegally;

Of those arrested, 535 showed signs of torture; 280 were eventually salvaged, 40, of frustrated salvage, and 109 remained missing after their arrest;

Forty (40) cases of massacres, with 218 killed; 54 cases of frustrated massacre, in which 157 were wounded;

The victims belonged to neighborhood and union organizations;

Since February, 1986, 532 of those illegally arrested were women;

From January to June 1990, 361 children were detained for no apparent reason;

One million ten thousand four hundred nine (1,010,409) have been injured as a consequence of bombing, shellings, and food blockades undertaken by the military since 1988. 34

It is a bleak picture, and I am disturbed that this Court should express very little concern. I am also disappointed that it is the portrait of the Court I am soon leaving. Nonetheless, I am hopeful that despite my departure, it will not be too late.

Motions denied.

G.R. No. 95847-48. March 10, 1993.

PEOPLE OF THE PHILIPPINES, plaintiff-appellee, vs. GABRIEL GERENTE y BULLO, accused-appellant.

Page 155: Warrantless Arrests, Searches and Seizures and Privacy of Communication

155

The Solicitor General for plaintiff-appellee.

Public Attorney's Office for accused-appellant.

SYLLABUS

1. REMEDIAL LAW; CRIMINAL PROCEDURE; ARREST WITHOUT WARRANT; LAWFUL WHEN ARRESTING OFFICER HAS PERSONAL KNOWLEDGE THAT THE PERSON TO BE ARRESTED HAS COMMITTED THE CRIME; CASE AT BAR. — The policemen arrested Gerente only some three (3) hours after Gerente and his companions had killed Blace. They saw Blace dead in the hospital and when they inspected the scene of the crime, they found the instruments of death: a piece of wood and a concrete hollow block which the killers had used to bludgeon him to death. The eye-witness, Edna Edwina Reyes, reported the happening to the policemen and pinpointed her neighbor, Gerente, as one of the killers. Under those circumstances, since the policemen had personal knowledge of the violent death of Blace and of facts indicating that Gerente and two others had killed him, they could lawfully arrest Gerente without a warrant. If they had postponed his arrest until they could obtain a warrant, he would have fled the law as his two companions did.

2. ID.; ID.; SEARCH AND SEIZURE; VALID EVEN WITHOUT A WARRANT WHEN MADE AS AN INCIDENT TO LAWFUL ARREST; RATIONALE. — The search conducted on Gerente's person was likewise lawful because it was made as an incident to a valid arrest. This is in accordance with Section 12, Rule 126 of the Revised Rules of Court which provides: "Section 12. Search incident to lawful arrest. — A person lawfully arrested may be searched for dangerous weapons or anything which may be used as proof of the commission of an offense, without a search warrant." The frisk and search of appellant's person upon his arrest was a permissible precautionary measure of arresting officers to protect themselves, for the person who is about to be arrested may be armed and might attack them unless he is first disarmed. In Adams vs. Williams, 47 U.S. 143, cited in Justice Isagani A. Cruz's Constitutional Law, 1991 Edition, p. 150, it was ruled that "the individual being arrested may be frisked for concealed weapons that may be used against the arresting officer and all unlawful articles found his person, or within his immediate control may be seized."

3. CRIMINAL LAW; CONSPIRACY; LIABILITY OF CONSPIRATORS; RULE; CASE AT BAR. — There is no merit in appellant's allegation that the trial court erred in convicting him of having conspired and cooperated with Fredo and Totoy Echigoren to kill Blace despite the testimony of Dr. Valentin Bernales that the fracture on the back of the victim's skull could have been inflicted by one person only. what Dr. Bernales stated was a mere possibility that only one person dropped the concrete hollow block on the head of the victim, smashing it. That circumstance, even if true, does not absolve the other two co-conspirators in the murder of Blace for when there is a conspiracy to commit a crime, the act of one conspirator is the act of all. The conspiracy was proven by the eyewitness-testimony of Edna Edwina Reyes, that she overheard the appellant and his companions conspire to kill Blace, that acting in concert, they attacked their victim with a piece of wood and a hollow block and caused his death. "When there is no evidence indicating that the principal witness for the prosecution was moved by improper motive, the presumption is that he was not so moved and his testimony is entitled to full faith and credit" (People vs. Belibet, 199 SCRA 587, 588). Hence, the trial court did not err in giving full credit to Edna Reyes' testimony.

4. ID.; CIVIL INDEMNITY FOR DEATH; INCREASED TO P50,000.00. — The Solicitor General correctly pointed out in the appellee's brief that the award of P30,000.00 as civil indemnity for the death of Clarito Blace should be increased to P50,000.00 in accordance with our ruling in People vs. Sison, 189 SCRA 643.

D E C I S I O N

GRIÑO-AQUINO, J p:

Page 156: Warrantless Arrests, Searches and Seizures and Privacy of Communication

156

This is an appeal from the decision of the Regional Trial Court of Valenzuela, Metro Manila, Branch 172, which found the appellant guilty of Violation of Section 8 of Republic Act 6425 (Dangerous Drugs Act of 1972) and sentenced him to suffer the penalty of imprisonment for a term of twelve (12) years and one (1) day, as minimum, to twenty (20) years, as maximum; and also found him guilty of Murder for which crime he was sentenced to suffer the penalty of reclusion perpetua. The dispositive portion of the appealed decision reads:

"WHEREFORE, in view of the foregoing the Court finds the accused Gabriel Gerente in Criminal Case No. 10255-V-90 guilty beyond reasonable doubt of Violation of Section 8 of R.A. 6425 and hereby sentences him to suffer the penalty of imprisonment of twelve years and one day as minimum to twenty years as maximum, and a fine of twelve thousand, without subsidiary imprisonment in case of insolvency, and to pay the costs.

"In Criminal Case No. 10256-V-90, the Court finds the accused Gabriel Gerente guilty beyond reasonable doubt of the crime of Murder, and there by (sic) no aggravating circumstances nor mitigating circumstances, is hereby sentenced to suffer the penalty of reclusion perpetua; to indemnify the heirs of the victim in the sum of P30,000.00, and in the amount of P17,609.00 as funeral expenses, without subsidiary imprisonment in case of insolvency, and to pay the costs. The accused Gabriel Gerente shall be credited with the full term of his preventive imprisonment." (p. 25, Rollo.)

Appellant Gabriel Gerente y Bullo was charged with Violation of Section 8, Art. II of R.A. 6425, which was docketed as Criminal Case No. 10255-V-90 of the Regional Trial Court of Valenzuela, Metro Manila. The Information reads:

"That on or about the 30th day of April, 1990, in the municipality of Valenzuela, Metro Manila, Philippines, and within the jurisdiction of this Honorable Court, the above-named accused, without justification, did then and there wilfully, unlawfully and feloniously have in his possession and control dried flowering tops wrapped in foil with markings and place in a transparent plastic bag which are considered prohibited drugs." (p. 2, Rollo.)

The same accused, together with Totoy and Fredo Echigoren who are both at large, was charged with Murder in Criminal Case No. 10256-V-90 in an information of the same date and signed by the same Assistant Provincial Prosecutor, as follows:

"That on or about the 30th day of April, 1990, in the municipality of Valenzuela, Metro Manila, Philippines, and within the jurisdiction of this Honorable Court, the above-named accused together with two (2) others who are still at large and against whom the preliminary investigation has not yet been terminated by the Office of the Provincial Prosecutor of Bulacan, conspiring, confederating together and mutually helping one another, armed with a piece of wood and hallow (sic) block and with intent to kill one Clarito B. Blace, did then and there wilfully, unlawfully and feloniously, with evident premeditation and treachery, attack, assault and hit with the said piece of wood and hollow block the said Clarito B. Blace, hitting the latter on the different parts of his body, thereby inflicting serious physical injuries which directly caused the death of the said victim." (p. 3, Rollo.)

Edna Edwina Reyes testified that at about 7:00 a.m. of April 30, 1990, appellant Gabriel Gerente, together with Fredo Echigoren and Totoy Echigoren, started drinking liquor and smoking marijuana in the house of the appellant which is about six (6) meters away from the house of the prosecution witness who was in her house on that day. She overheard the three men talking about their intention to kill Clarito Blace. She testified that she heard Fredo Echigoren saying, "Gabriel, papatayin natin si Clarito Blace," and Totoy Echigoren allegedly seconded Fredo's suggestion saying: "Papatayin natin 'yan mamaya." Appellant allegedly agreed: "Sigue, papatayin natin mamaya." (pp. 3-4, tsn, August 24, 1990.)

Fredo and Totoy Echigoren and Gerente carried out their plan to kill Clarito Blace at about 2:00 p.m. of the same day. The prosecution witness, Edna Edwina Reyes, testified that she witnessed the killing.

Page 157: Warrantless Arrests, Searches and Seizures and Privacy of Communication

157

Fredo Echigoren struck the first blow against Clarito Blace, followed by Totoy Echigoren and Gabriel Gerente who hit him twice with a piece of wood in the head and when he fell, Totoy Echigoren dropped a hollow block on the victim's head. Thereafter, the three men dragged Blace to a place behind the house of Gerente.

At about 4:00 p.m. of the same day, Patrolman Jaime Urrutia of the Valenzuela Police Station received a report from the Palo Police Detachment about a mauling incident. He went to the Valenzuela District Hospital where the victim was brought. He was informed by the hospital officials that the victim died on arrival. The cause of death was massive fracture of the skull caused by a hard and heavy object. Right away, Patrolman Urrutia, together with Police Corporal Romeo Lima and Patrolman Alex Umali, proceeded to Paseo de Blas where the mauling incident took place. There they found a piece of wood with blood stains, a hollow block and two roaches of marijuana. They were informed by the prosecution witness, Edna Edwina Reyes, that she saw the killing and she pointed to Gabriel Gerente as one of the three men who killed Clarito.

The policemen proceeded to the house of the appellant who was then sleeping. They told him to come out of the house and they introduced themselves as policemen. Patrolman Urrutia frisked appellant and found a coin purse in his pocket which contained dried leaves wrapped in cigarette foil. The dried leaves were sent to the National Bureau of Investigation for examination. The Forensic Chemist found them to be marijuana.

Only the appellant, Gabriel Gerente, was apprehended by the police. The other suspects, Fredo and Totoy Echigoren, are still at large.

On May 2, 1990, two separate informations were filed by Assistant Provincial Prosecutor Benjamin Caraig against him for Violation of Section 8, Art. II, of R.A. 6425, and for Murder.

When arraigned on May 16, 1990, the appellant pleaded not guilty to both charges. A joint trial of the two cases was held. On September 24, 1990, the trial court rendered a decision convicting him of Violation of Section 8 of R.A. 6425 and of Murder.

In this appeal of the appellant, the following errors are ascribed to the trial court:

1. the court a quo gravely erred in admitting the marijuana leaves adduced in evidence by the prosecution; and

2. the court a quo gravely erred in convicting the accused-appellant of the crimes charged despite the absence of evidence required to prove his guilt beyond reasonable doubt.

The appellant contends that the trial court erred in admitting the marijuana leaves as evidence in violation of his constitutional right not to be subjected to illegal search and seizure, for the dried marijuana leaves were seized from him in the course of a warrantless arrest by the police officers. We do not agree.

The search of appellant's person and the seizure of the marijuana leaves in his possession were valid because they were incident to a lawful warrantless arrest.

Paragraphs (a) and (b), Section 5, Rule 113 of the Revised Rules of Court provide:

'SECTION 5. Arrest without warrant; when lawful. — A peace officer or a private person may, without a warrant, arrest a person:

"(a) When, in his presence, the person to be arrested has committed, is actually committing, or is attempting to commit an offense;"

Page 158: Warrantless Arrests, Searches and Seizures and Privacy of Communication

158

"(b) When an offense has in fact just been committed, and he has personal knowledge of facts indicating that the person to be arrested has committed it; . . .'

The policemen arrested Gerente only some three (3) hours after Gerente and his companions had killed Blace. They saw Blace dead in the hospital and when they inspected the scene of the crime, they found the instruments of death: a piece of wood and a concrete hollow block which the killers had used to bludgeon him to death. The eye-witness, Edna Edwina Reyes, reported the happening to the policemen and pinpointed her neighbor, Gerente, as one of the killers. Under those circumstances, since the policemen had personal knowledge of the violent death of Blace and of facts indicating that Gerente and two others had killed him, they could lawfully arrest Gerente without a warrant. If they had postponed his arrest until they could obtain a warrant, he would have fled the law as his two companions did.

In Umil vs. Ramos, 187 SCRA 311, the arrest of the accused without a warrant was effected one (1) day after he had shot to death two Capcom soldiers. The arrest was held lawful by this Court upon the rationale stated by us in People vs. Malasugui, 63 Phil. 221, 228, thus:

"To hold that no criminal can, in any case, be arrested and searched for the evidence and tokens of his crime without a warrant, would be to leave society, to a large extent, at the mercy of the shrewdest, the most expert, and the most depraved of criminals, facilitating their escape in many instances."

The search conducted on Gerente's person was likewise lawful because it was made as an incident to a valid arrest. This is in accordance with Section 12, Rule 126 of the Revised Rules of Court which provides:

"SECTION 12. Search incident to lawful arrest. — A person lawfully arrested may be searched for dangerous weapons or anything which may be used as proof of the commission of an offense, without a search warrant."

The frisk and search of appellant's person upon his arrest was a permissible precautionary measure of arresting officers to protect themselves, for the person who is about to be arrested may be armed and might attack them unless he is first disarmed. In Adams vs. Williams, 47 U.S. 143, cited in Justice Isagani A. Cruz's Constitutional Law, 1991 Edition, p. 150, it was ruled that "the individual being arrested may be frisked for concealed weapons that may be used against the arresting officer and all unlawful articles found in his person, or within his immediate control may be seized."

There is no merit in appellant's allegation that the trial court erred in convicting him of having conspired and cooperated with Fredo and Totoy Echigoren to kill Blace despite the testimony of Dr. Valentin Bernales that the fracture on the back of the victim's skull could have been inflicted by one person only.

What Dr. Bernales stated was a mere possibility that only one person dropped the concrete hollow block on the head of the victim, smashing it. That circumstance, even if true, does not absolve the other two co-conspirators in the murder of Blace for when there is a conspiracy to commit a crime, the act of one conspirator is the act of all. The conspiracy was proven by the eyewitness-testimony of Edna Edwina Reyes, that she overheard the appellant and his companions conspire to kill Blace, that acting in concert, they attacked their victim with a piece of wood and a hollow block and caused his death. "When there is no evidence indicating that the principal witness for the prosecution was moved by improper motive, the presumption is that he was not so moved and his testimony is entitled to full faith and credit" (People vs. Belibet, 199 SCRA 587, 588). Hence, the trial court did not err in giving full credit to Edna Reyes' testimony.

Appellant's failure to escape (because he was very drunk) is no indicium of his innocence.

Page 159: Warrantless Arrests, Searches and Seizures and Privacy of Communication

159

The Solicitor General correctly pointed out in the appellee's brief that the award of P30,000.00 as civil indemnity for the death of Clarito Blace should be increased to P50,000.00 in accordance with our ruling in People vs. Sison, 189 SCRA 643.

WHEREFORE, the appealed decision is hereby AFFIRMED, with modification of the civil indemnity awarded to the heirs of the victim, Clarito Blace, which is hereby increased to P50,000.00.

SO ORDERED.

Cruz, Bellosillo and Quiason, JJ ., concur.

[G.R. No. 144037.  September 26, 2003]

PEOPLE OF THE PHILIPPINES, plaintiff-appellee, vs. NOEL TUDTUD y PAYPA and DINDO BOLONG y NARET, accused-appellants.

D E C I S I O N

TINGA, J.:

….  It is desirable that criminals should be detected, and to that end that all available evidence should be used.  It also is desirable that the government should not itself foster and pay for other crimes, when they are the means by which the evidence is to be obtained.  If it pays its officers for having got evidence by crime, I do not see why it may not as well pay them for getting it in the same way, and I can attach no importance to protestations of disapproval if it knowingly accepts and pays and announces that it will pay for the fruits.  We have to choose, and for my part I think it a less evil that some criminals should escape than that the government should play an ignoble part.

So wrote Justice Oliver Wendell Holmes in Olmstead v. U.S.[1] On this occasion, this Court is made to choose between letting suspected criminals escape or letting the government play an ignoble part.

Sometime during the months of July and August 1999, the Toril Police Station, Davao City received a report from a “civilian asset” named Bobong Solier about a certain Noel Tudtud. [2] Solier related that his neighbors have been complaining about Tudtud, who was allegedly responsible for the proliferation of marijuana in their area.[3]

Reacting to the report, PO1 Ronald Desierto, PO1 Ramil Floreta and their superior, SPO1 Villalonghan,[4] all members of the Intelligence Section of the Toril Police Station, conducted surveillance in Solier’s neighborhood in Sapa, Toril, Davao City.[5] For five days, they gathered information and learned that Tudtud was involved in illegal drugs.[6] According to his neighbors, Tudtud was engaged in selling marijuana.[7]

On August 1, 1999, Solier informed the police that Tudtud had headed to Cotabato and would be back later that day with new stocks of marijuana. [8] Solier described Tudtud as big-bodied and short, and usually wore a hat.[9] At around 4:00 in the afternoon that same day, a team composed of PO1 Desierto, PO1 Floreta and SPO1 Villalonghan posted themselves at the corner of Saipon and McArthur Highway to await Tudtud’s arrival.[10] All wore civilian clothes.[11]

About 8:00 later that evening, two men disembarked from a bus and helped each other carry a carton[12] marked “King Flakes.”[13] Standing some five feet away from the men, PO1 Desierto and PO1 Floreta observed that one of the men fit Tudtud’s description.[14] The same man also toted a plastic bag.[15]

Page 160: Warrantless Arrests, Searches and Seizures and Privacy of Communication

160

PO1 Floreta and PO1 Desierto then approached the suspects and identified themselves as police officers.[16] PO1 Desierto informed them that the police had received information that stocks of illegal drugs would be arriving that night.[17] The man who resembled Tudtud’s description denied that he was carrying any drugs.[18]PO1 Desierto asked him if he could see the contents of the box.[19] Tudtud obliged, saying, “it was alright.”[20] Tudtud opened the box himself as his companion looked on.[21]

The box yielded pieces of dried fish, beneath which were two bundles, one wrapped in a striped plastic bag[22] and another in newspapers.[23] PO1 Desierto asked Tudtud to unwrap the packages.[24] They contained what seemed to the police officers as marijuana leaves.[25]

The police thus arrested Tudtud and his companion, informed them of their rights and brought them to the police station.[26] The two did not resist.[27]

The confiscated items were turned over to the Philippine National Police (PNP) Crime Laboratory for examination.[28] Forensic tests conducted by Police Chief Inspector Noemi Austero, forensic chemist of the PNP Crime Laboratory, Region XI, on specimens taken from the confiscated items confirmed the police officers’ suspicion.  The plastic bag contained 3,200 grams of marijuana leaves while the newspapers contained another 890 grams.[29] Police Chief Inspector Austero reduced her findings in her report, Physical Sciences Report No. D-220-99 dated 2 August 1999.[30]

Noel Tudtud and his companion, Dindo Bulong, were subsequently charged[31] before the Regional Trial Court (RTC) of Davao City with illegal possession of prohibited drugs. [32] Upon arraignment, both accused pleaded not guilty.[33] The defense, however, reserved their right to question the validity of their arrest and the seizure of the evidence against them.[34]

Trial ensued thereafter.

The prosecution presented five witnesses, namely, arresting officers PO1 Desierto and PO1 Floreta, their civilian informant Bobong Solier, forensic chemist Police Chief Inspector Noemi Austero, and SPO3 Nicolas Algabre, exhibit custodian of the PNP Crime Laboratory.  Said witnesses testified to the foregoing narration of facts.

The accused, denying the charges against them, cried frame-up.

Noel Tudtud recalled that on August 1, 1999 he had gone to Kabacan, North Cotabato to sell pairs of Levi’s pants, which was his “sideline.”[35] At about 5:00 in the afternoon, he returned to Davao City by bus.[36] Upon reaching Toril, Tudtud, along with less than ten passengers, got down the bus.[37]

Suddenly, a man who identified himself as a police officer approached him, pointing a .38 caliber revolver.[38] The man told him not to run.[39] Tudtud raised his arms and asked, “Sir, what is this about?”[40] The man answered that he would like to inspect the plastic bag Tudtud was carrying, and instructed Tudtud to open the bag, which revealed several pairs of Levi’s pants.[41]

The man then directed Tudtud to open a carton box some two meters away. [42] According to Tudtud, the box was already there when he disembarked the bus.[43]Tudtud told the man the box was not his, but proceeded to open it out of fear after the man again pointed his revolver at him. [44] Tudtud discovered pieces of dried fish, underneath which was something wrapped in cellophane.[45]

“What is that?” the man asked.[46] Tudtud replied that he did not know.[47] Without even unwrapping the cellophane, the man said it was marijuana and abruptly handcuffed Tudtud.[48]

Simultaneously, another man was pointing a firearm at Dindo Bolong at the other side of the street, some eight meters from Tudtud.[49]

Bolong recounted that he was on his way to a relative in Daliao after attending a cousin’s wedding in Hagonoy, Davao del Sur when he was accosted. [50] After alighting the bus, Bolong crossed the street.[51] Someone then approached him and pointed a gun at him. [52] The man ordered him not to move and handcuffed him.[53]Bolong asked why he was being arrested but the man just told him to go with them.[54]

The suspects were then taken to the police station where, they would later claim, they met each other for the first time.[55]

Page 161: Warrantless Arrests, Searches and Seizures and Privacy of Communication

161

Assailing the credibility of informant Bobong Solier, the defense offered the testimonies of Felicia Julaton,[56] Branch 3 Clerk of Court, Claudio Bohevia,[57] Branch 7 Clerk of Court, and Mercedita Abunda,[58] Branch 9 Utility Clerk, all of the Davao City Municipal Trial Circuit Court.  They testified and presented court documents showing that one “Bobo” or “Bobong” Ramirez was charged in their respective branches with various crimes, specifically, light threats, less serious physical injuries and robbery.  The defense asserted that the “Bobo” or “Bobong” Ramirez accused in these cases is the same person as the informant Bobong Solier.[59]

Swayed by the prosecution’s evidence beyond reasonable doubt, the RTC rendered judgment convicting both accused as charged and sentencing them to suffer the penalty of reclusion perpetua and to pay a fine of P500,000.00.[60]

On appeal, Noel Tudtud and Dindo Bolong assign, among other errors, the admission in evidence of the marijuana leaves, which they claim were seized in violation of their right against unreasonable searches and seizures.

The right against unreasonable searches and seizures is secured by Section 2, Article III of the Constitution, which states:

SEC. 2.  The right of the people to be secured in their persons, houses, papers, and effects against unreasonable searches and seizures of whatever nature and for any purpose shall be inviolable, and no search warrant or warrant of arrest shall issue except upon probable cause to be determined personally by the judge after examination under oath or affirmation of the complainant and the witnesses he may produce, and particularly describing the places to be searched and the persons or things to be seized.

The rule is that a search and seizure must be carried out through or with a judicial warrant; otherwise, such search and seizure becomes  “unreasonable” within the meaning of the above-quoted constitutional provision, and any evidence secured thereby, will be inadmissible in evidence “for any purpose in any proceeding.”[61]Section 3 (2), Article III of the Constitution explicitly provides:

(2) Any evidence obtained in violation of… the preceding section shall be inadmissible for any purpose in any proceeding.

The proscription in Section 2, Article III, however, covers only “unreasonable” searches and seizures.  The following instances are not deemed “unreasonable” even in the absence of a warrant:

1.       Warrantless search incidental to a lawful arrest. (Sec. 12, Rule 126 of the Rules of Court and prevailing jurisprudence);

2.       Search of evidence in “plain view.” The elements are: (a) a prior valid intrusion based on the valid warrantless arrest in which the police are legally present in the pursuit of their official duties; (b) the evidence was inadvertently discovered by the police who have the right to be where they are; (c) the evidence must be immediately apparent; (d) “plain view” justified mere seizure of evidence without further search;

3.       Search of a moving vehicle. Highly regulated by the government, the vehicle’s inherent mobility reduces expectation of privacy especially when its transit in public thoroughfares furnishes a highly reasonable suspicion amounting to probable cause that the occupant committed a criminal activity;

4.       Consented warrantless search;

5.       Customs search;

6.       Stop and Frisk; and

Page 162: Warrantless Arrests, Searches and Seizures and Privacy of Communication

162

7.       Exigent and emergency circumstances.[62]

The RTC justified the warrantless search of appellants’ belongings under the first exception, as a search incident to a lawful arrest.  It cited as authorities this Court’s rulings in People v. Claudio,[63] People v. Tangliben,[64] People v. Montilla,[65] and People v. Valdez.[66] The Office of the Solicitor General (OSG), in arguing for the affirmance of the appealed decision, invokes the cases of People v. Maspil, Jr.,[67] People v. Malmstedt,[68] and People v. Bagista.[69]

A search incidental to a lawful arrest is sanctioned by the Rules of Court.   Prior to its revision in 2000, Section 12,[70] Rule 126 of said Rules read as follows:

SEC. 12.  Search incident to lawful arrest. – A person lawfully arrested may be searched for dangerous weapons or anything which may be used as proof of the commission of an offense, without a search warrant.

Section 5 (a), Rule 113 of the Rules, in turn, allows warrantless arrests:

SEC. 5.  Arrest without warrant; when lawful. – A peace officer or a private person may, without a warrant, arrest a person:

(a) When, in his presence, the person to be arrested has committed, is actually committing, or is attempting to commit an offense;

….

It is significant to note that the search in question preceded the arrest.  Recent jurisprudence holds that the arrest must precede the search; the process cannot be reversed.[71] Nevertheless, a search substantially contemporaneous with an arrest can precede the arrest if the police have probable cause to make the arrest at the outset of the search.[72] The question, therefore, is whether the police in this case had probable cause to arrest appellants.  Probable cause has been defined as:

an actual belief or reasonable grounds of suspicion.  The grounds of suspicion are reasonable when, in the absence of actual belief of the arresting officers, the suspicion that the person to be arrested is probably guilty of committing the offense, is based on actual facts, i.e., supported by circumstances sufficiently strong in themselves to create the probable cause of guilt of the person to be arrested.  A reasonable suspicion therefore must be founded on probable cause, coupled with good faith of the peace officers making the arrest.[73]

The long-standing rule in this jurisdiction, applied with a great degree of consistency, is that “reliable information” alone is not sufficient to justify a warrantless arrest under Section 5 (a), Rule 113.  The rule requires, in addition, that the accused perform some overt act that would indicate that he “has committed, is actually committing, or is attempting to commit an offense.”

In the leading case of People v. Burgos,[74] this Court held that “the officer arresting a person who has just committed, is committing, or is about to commit an offense must have personal knowledge of that fact.  The offense must also be committed in his presence or within his view.” [75] In Burgos, the authorities obtained information that the accused had forcibly recruited one Cesar Masamlok as member of the New People’s Army, threatening the latter with a firearm. Upon finding the accused, the arresting team searched his house and discovered a gun as well as purportedly subversive documents.  This Court, in declaring then Section 6 (a), Rule 113 of the Rules of Court inapplicable, ruled that:

There is no such personal knowledge in this case.  Whatever knowledge was possessed by the arresting officers, it came in its entirety from the information furnished by Cesar Masamlok.  The location of the firearm was given by the appellant’s wife.

Page 163: Warrantless Arrests, Searches and Seizures and Privacy of Communication

163

At the time of the appellant’s arrest, he was not in actual possession of any firearm or subversive document.  Neither was he committing any act which could be described as subversive.  He was, in fact, plowing his field at the time of the arrest.

The right of a person to be secure against any unreasonable seizure of his body and any deprivation of his liberty is a most basic and fundamental one.  The statute or rule which allows exceptions to the requirement of warrants of arrest is strictly construed.  Any exception must clearly fall within the situations when securing a warrant would be absurd or is manifestly unnecessary as provided by the Rule.  We cannot liberally construe the rule on arrests without warrant or extend its application beyond the cases specifically provided by law.  To do so would infringe upon personal liberty and set back a basic right so often violated and so deserving of full protection.[76]

Consequently, the items seized were held inadmissible, having been obtained in violation of the accused’s constitutional rights against unreasonable searches and seizures.

In People v. Aminnudin,[77] this Court likewise held the warrantless arrest and subsequent search of appellant therein illegal, given the following circumstances:

… the accused-appellant was not, at the moment of his arrest, committing a crime nor was it shown that he was about to do so or that he had just done so.  What he was doing was descending the gangplank of the M/V Wilcon 9 and there was no outward indication that he called for his arrest.  To all appearances, he was like any of the other passengers innocently disembarking from the vessel.  It was only when the informer pointed to him as the carrier of the marijuana that he suddenly became suspect and so subject to apprehension.  It was the furtive finger that triggered his arrest.  The identification by the informer was the probable cause as determined by the officers (and not a judge) that authorized them to pounce upon Aminnudin and immediately arrest him.[78]

Thus, notwithstanding tips from confidential informants and regardless of the fact that the search yielded contraband, the mere act of looking from side to side while holding one’s abdomen, [79] or of standing on a corner with one’s eyes moving very fast, looking at every person who came near, [80] does not justify a warrantless arrest under said Section 5 (a).  Neither does putting something in one’s pocket,[81] handing over one’s baggage,[82] riding a motorcycle,[83] nor does holding a bag on board atrisikad[84]sanction State intrusion.  The same rule applies to crossing the street per se.[85]

Personal knowledge was also required in the case of People v. Doria.[86] Recently, in People v. Binad Sy Chua,[87] this Court declared invalid the arrest of the accused, who was walking towards a hotel clutching a sealed Zest-O juice box.  For the exception in Section 5 (a), Rule 113 to apply, this Court ruled, two elements must concur: (1) the person to be arrested must execute an overt act indicating he has just committed, is actually committing, or is attempting to commit a crime; and (2) such overt act is done in the presence or within the view of the arresting officer.  Reliable information alone is insufficient.

In the following cases, the search was held to be incidental to a lawful arrest because of “suspicious” circumstances: People v. Tangliben[88] (accused was “acting suspiciously”), People v. Malmstedt[89] (a bulge on the accused’s waist), and People v. de Guzman [90]  (likewise a bulge on the waist of the accused, who was wearing tight-fitting clothes).

There is, however, another set of jurisprudence that deems “reliable information” sufficient to justify a search incident to a warrantless arrest under Section 5 (a), Rule 113, thus deviating from Burgos.  To this class of cases belong People v. Maspil, Jr.,[91] People v. Bagista,[92] People v. Balingan,[93] People v. Lising,[94] People v. Montilla,[95] People v. Valdez,[96] and People v. Gonzales.[97] In these cases, the arresting authorities were acting on information regarding an offense but there were no overt acts or suspicious circumstances that would indicate that the accused has committed, is actually committing, or is attempting to commit the same. Significantly, these cases, except the last two, come under some other exception to the rule against warrantless searches.  Thus, Maspil, Jr. involved a checkpoint search, Balinganwas a search of a moving vehicle, Bagista was both, and Lising and Montilla were consented searches.

Page 164: Warrantless Arrests, Searches and Seizures and Privacy of Communication

164

Nevertheless, the great majority of cases conforms to the rule in Burgos, which, in turn, more faithfully adheres to the letter of Section 5(a), Rule 113.  Note the phrase “in his presence” therein, connoting personal knowledge on the part of the arresting officer.  The right of the accused to be secure against any unreasonable searches on and seizure of his own body and any deprivation of his liberty being a most basic and fundamental one, the statute or rule that allows exception to the requirement of a warrant of arrest is strictly construed.  Its application cannot be extended beyond the cases specifically provided by law.[98]

The cases invoked by the RTC and the OSG are, therefore, gravely misplaced.  In Claudio,[99] the accused, who was seated aboard a bus in front of the arresting officer, put her bag behind the latter, thus arousing the latter’s suspicion.  In Tangliben and Malmstedt, the accused had also acted suspiciously.

As noted earlier, Maspil, Jr., Bagista and Montilla were justified by other exceptions to the rule against warrantless searches.  Montilla, moreover, was not without its critics.  There, majority of the Court held:

Appellant insists that the mere fact of seeing a person carrying a traveling bag and a carton box should not elicit the slightest suspicion of the commission of any crime since that is normal. But precisely, it is in the ordinary nature of things that drugs being illegally transported are necessarily hidden in containers and concealed from view.  Thus, the officers could reasonably assume, and not merely on a hollow suspicion since the informant was by their side and had so informed them, that the drugs were in appellant’s luggage.  It would obviously have been irresponsible, if now downright absurd under the circumstances, to require the constable to adopt a “wait and see” attitude at the risk of eventually losing the quarry.

Here, there were sufficient facts antecedent to the search and seizure that, at the point prior to the search were already constitutive of probable cause, and which by themselves could properly create in the minds of the officers a well-grounded and reasonable belief that appellant was in the act of violating the law.  The search yielded affirmance both of that probable cause and the actuality that appellant was then actually committing a crime by illegally transporting prohibited drugs.  With these attendant facts, it is ineluctable that appellant was caught in flagrante delicto, hence his arrest and the search of his belongings without the requisite warrant were both justified.[100]

While concurring with the majority, Mr. Justice Vitug reserved his vote on the discussion on the warrantless search being incidental to a lawful arrest.  Mr. Justice Panganiban, joined by Messrs. Justices Melo and Puno, filed a Separate Opinion.

Although likewise concurring in the majority’s ruling that appellant consented to the inspection of his baggage, Justice Panganiban disagreed with the conclusion that the warrantless search was incidental to a lawful arrest.  He argued that jurisprudence required personal knowledge on the part of the officers making the in flagrante delicto arrest.  In Montilla, the appellant “did not exhibit any overt act or strange conduct that would reasonably arouse in their minds suspicion that he was embarking on some felonious enterprise.”

Law and jurisprudence in fact require stricter grounds for valid arrests and searches without warrant than for the issuance of warrants therefore. In the former, the arresting person must have actually witnessed the crime being committed or attempted by the person sought to be arrested; or he must have personal knowledge of facts indicating that the person to be arrested perpetrated the crime that had just occurred. In the latter case, the judge simply determines personally from testimonies of witnesses that there exists reasonable grounds to believe that a crime was committed by the accused.

….

To say that “reliable tips” constitute probable cause for a warrantless arrest or search is in my opinion, a dangerous precedent and places in great jeopardy the doctrines laid down in many decisions made by this Court, in its effort to zealously guard and protect the sacred constitutional right against unreasonable

Page 165: Warrantless Arrests, Searches and Seizures and Privacy of Communication

165

arrests, searches and seizures.  Everyone would be practically at the mercy of so-called informants, reminiscent of the makapilis during the Japanese occupation. Any one whom they point out to a police officer as a possible violator of the law could then be subject to search and possible arrest.  This is placing limitless power upon informants who will no longer be required to affirm under oath their accusations, for they can always delay their giving of tips in order to justify warrantless arrests and searches. Even law enforcers can use this as an oppressive tool to conduct searches without warrants, for they can always claim that they received raw intelligence information only on the day or afternoon before. This would clearly be a circumvention of the legal requisites for validly effecting an arrest or conducting a search and seizure.  Indeed the majority’s ruling would open loopholes that would allow unreasonable arrests, searches and seizures.[101]

Montilla would shortly find mention in Justice Panganiban’s concurring opinion in People v. Doria, supra, where this Court ruled:

Accused-Appellant Gaddao was arrested solely on the basis of the alleged identification made by her co-accused.  PO3 Manlangit, however, declared in his direct examination that appellant Doria named his co-accused in response to his (PO3 Manlangit’s) query as to where the marked money was. Appellant Doria did not point to appellant Gaddao as his associate in the drug business, but as the person with whom he left the marked bills. This identification does not necessarily lead to the conclusion that appellant Gaddao conspired with her co-accused in pushing drugs. Appellant Doria may have left the money in her house, with or without any conspiracy. Save for accused-appellant Doria’s word, the Narcom agents had no showing that the person who affected the warantless arrest had, in his own right, knowledge of facts implicating the person arrested to the perpetration of a criminal offense, the arrest is legally objectionable.[102] [Italics in the original.]

Expressing his accord with Mr. Justice Puno’s ponencia, Justice Panganiban said that Doria “rightfully brings the Court back to well-settled doctrines on warrantless arrests and searches, which have seemingly been modified through an obiter in People v. Ruben Montilla.”[103]

Montilla, therefore, has been seemingly discredited insofar as it sanctions searches incidental to lawful arrest under similar circumstances.  At any rate, Montillawas a consented search.  As will be demonstrated later, the same could not be said of this case.

That leaves the prosecution with People v. Valdez, which, however, involved an “on-the-spot information.”  The urgency of the circumstances, an element not present in this case, prevented the arresting officer therein from obtaining a warrant.

Appellants in this case were neither performing any overt act or acting in a suspicious manner that would hint that a crime has been, was being, or was about to be, committed.  If the arresting officers’ testimonies are  to be believed, appellants were merely helping each other carry a carton box.   Although appellant Tudtud did appear “afraid and perspiring,”[104] “pale”[105] and “trembling,”[106] this was only after, not before, he was asked to open the said box.

In no sense can the knowledge of the herein arresting officers that appellant Tudtud was in possession of marijuana be described as “personal,” having learned the same only from their informant Solier.  Solier, for his part, testified that he obtained his information only from his neighbors and the friends of appellant Tudtud:

Q –           What was your basis in your report to the police that Tudtud is going to Cotabato and get stocks of marijuana?

A – Because of the protest of my neighbors who were saying who will be the person whou [sic] would point to him because he had been giving trouble to the neighborhood because according to them there are [sic] proliferation of marijuana in our place.  That was the complained [sic] of our neighbors. 

Q –           Insofar as the accused Tudtud is concerned what was your basis in reporting him particularly?

Page 166: Warrantless Arrests, Searches and Seizures and Privacy of Communication

166

A – His friends were the once who told me about it.

Q –           For how long have you know [sic] this fact of alleged activity of Tudtud in proliferation of marijuana?

A – About a month.

….

Q –           Regarding the report that Tudtud went to Cotabato to get stocks of marijuana which led to his apprehension sometime in the evening of August 1 and according to the report [which] is based on your report my question is, how did you know that Tudtud will be bringing along with him marijuana stocks on August 1, 1999?

….

A – Because of the information of his neighbor.[107]

In other words, Solier’s information itself is hearsay.  He did not even elaborate on how his neighbors or Tudtud’s friends acquired their information that Tudtud was responsible for the proliferation of drugs in their neighborhood.

Indeed, it appears that PO1 Floreta himself doubted the reliablility of their informant.  He testified on cross-examination:

Q –           You mean to say that Bobot Solier, is not reliable?

A – He is trustworthy.

Q –           Why [did] you not consider his information not reliable if he is reliable?

A – (witness did not answer).

ATTY. CAÑETE:

       Never mind, do not answer anymore.  That’s all.[108]

The prosecution, on re-direct examination, did not attempt to extract any explanation from PO1 Floreta for his telling silence.

Confronted with such a dubious informant, the police perhaps felt it necessary to conduct their own “surveillance.”  This “surveillance,” it turns out, did not actually consist of staking out appellant Tudtud to catch him in the act of plying his illegal trade, but of a mere “gather[ing] of information from the assets there.”[109] The police officers who conducted such “surveillance” did not identify who these “assets” were or the basis of the latter’s information.  Clearly, such information is also hearsay, not of personal knowledge.

Neither were the arresting officers impelled by any urgency that would allow them to do away with the requisite warrant, PO1 Desierto’s assertions of lack of time [110] notwithstanding.  Records show that the police had ample opportunity to apply for a warrant, having received Solier’s information at around 9:00 in the morning; Tudtud, however, was expected to arrive at around 6:00 in the evening of the same day.[111] In People v. Encinada, supra, the Court ruled that there was sufficient time to procure a warrant where the police officers received at 4:00 in the afternoon an intelligence report that the accused, who was supposedly carrying marijuana, would arrive the next morning at 7:00 a.m.:

Even if the information was received by Bolonia about 4:00 p.m. of May 20, 1992 at his house, there was sufficient time to secure a warrant of arrest, as the M/V Sweet Pearl was not expected to dock until 7:00 a.m. the following day.  Administrative Circular No. 13 allows application for search warrants even after office hours:

“3. Raffling shall be strictly enforced, except only in case where an application for search warrant may be filed directly with any judge whose jurisdiction the place to be searched is located, after office hours, or

Page 167: Warrantless Arrests, Searches and Seizures and Privacy of Communication

167

during Saturdays, Sundays, and legal holidays, in which case the applicant is required to certify under oath the urgency of the issuance thereof after office hours, or during Saturdays, Sundays and legal holidays;” . . .. 

The same procedural dispatch finds validation and reiteration in Circular No. 19, series of 1987, entitled “Amended Guidelines and Procedures on Application for search warrants for Illegal Possession of Firearms and Other Serious Crimes Filed in Metro Manila Courts and Other Courts with Multiple Salas”:

“This Court has received reports of delay while awaiting raffle, in acting on applications for search warrants in the campaign against loose firearms and other serious crimes affecting peace and order.  There is a need for prompt action on such applications for search warrant.  Accordingly, these amended guidelines in the issuance of a search warrant are issued:

1.            All applications for search warrants relating to violation of the Anti-subversion Act, crimes against public order as defined in the Revised Penal Code, as amended, illegal possession of firearms and/or ammunition and violations of the Dangerous Drugs Act of 1972, as amended, shall no longer be raffled and shall immediately be taken cognizance of and acted upon by the Executive Judge of the Regional Trial Court, Metropolitan Trial Court, and Municipal Trial Court under whose jurisdiction the place to be searched is located.

2.            In the absence of the Executive Judge, the Vice-Executive Judge shall take cognizance of and personally act on the same.  In the absence of the Executive Judge or Vice-Executive Judge, the application may be taken cognizance of and acted upon by any judge of the Court where application is filed.

3.            Applications filed after office hours, during Saturdays, Sundays and holidays, shall likewise be taken cognizance of and acted upon by any judge of the Court having jurisdiction of the place to be searched, but in such cases the applicant shall certify and state the facts under oath, to the satisfaction of the judge, that its issuance is urgent.

4.            Any judge acting on such application shall immediately and without delay personally conduct the examination of the applicant and his witnesses to prevent the possible leakage of information.  He shall observe the procedures, safeguards, and guidelines for the issuance of search warrants provided for in this Court’s Administrative Circular No. 13, dated October 1, 1985.”[112] [Italics in the original.]

Given that the police had adequate time to obtain the warrant, PO1 Floreta’s testimony that the real reason for their omission was their belief that they lacked sufficient basis to obtain the same assumes greater significance.  This was PO1 Floreta’s familiar refrain:

Q – When Solier reported to you that fact, that Tudtud will be coming from Cotabato to get that (sic) stocks, you did not go to court to get a search warrant on the basis of the report of Bobot Solier?

A –  No.

Q – Why?

A –  Because we have no real basis to secure the search warrant.

Q – When you have no real basis to secure a search warrant, you have also no real basis to search Tudtud and Bulong at that time?

A –  Yes, sir.

….

Q – And Bobot Solier told you that Tudtud, that he would already bring marijuana?

Page 168: Warrantless Arrests, Searches and Seizures and Privacy of Communication

168

A –  Yes, sir.

Q – And this was 9:00 a.m.?

A –  Yes, sir.

Q – The arrival of Tudtud was expected at 6:00 p.m.?

A –  Yes, sir.

Q – Toril is just 16 kilometers from Davao City?

A –  Yes, sir.

Q – And the Office of the Regional Trial Court is only about 16 kilometers, is that correct?

A –  Yes, sir.

Q – And it can be negotiated by thirty minutes by a jeep ride?

A –  Yes, sir.

Q – And you can asked [sic] the assistance of any prosecutor to apply for the search warrant or the prosecutor do [sic] not assist?

A –  They help.

Q – But you did not come to Davao City, to asked [sic] for a search warrant?

A –  As I said, we do not have sufficient basis.[113]

It may be conceded that “the mere subjective conclusions of a police officer concerning the existence of probable cause is not binding on [the courts] which must independently scrutinize the objective facts to determine the existence of probable cause” and that “a court may also find probable cause in spite of an officer’s judgment that none exists.”[114] However, the fact that the arresting officers felt that they did not have sufficient basis to obtain a warrant, despite their own information-gathering efforts, raises serious questions whether such “surveillance” actually yielded any pertinent information and even whether they actually conducted any information-gathering at all, thereby eroding any claim to personal knowledge.

Finally, there is an effective waiver of rights against unreasonable searches and seizures if the following requisites are present:

1.  It must appear that the rights exist;

2.  The person involved had knowledge, actual or constructive, of the existence of such right;

3.  Said person had an actual intention to relinquish the right.[115]

Here, the prosecution failed to establish the second and third requisites. Records disclose that when the police officers introduced themselves as such and requested appellant that they see the contents of the carton box supposedly containing the marijuana, appellant Tudtud said “it was alright.” He did not resist and opened the box himself.

The fundamental law and jurisprudence require more than the presence of these circumstances to constitute a valid waiver of the constitutional right against unreasonable searches and seizures.  Courts indulge every reasonable presumption against waiver of fundamental constitutional rights; acquiescence in the loss of fundamental rights is not to be presumed.[116] The fact that a person failed to object to a search does not amount to permission thereto.

….  As the constitutional guaranty is not dependent upon any affirmative act of the citizen, the courts do not place the citizen in the position of either contesting an officer’s authority by force, or waiving his constitutional rights; but instead they hold that a peaceful submission to a search or seizure is not a

Page 169: Warrantless Arrests, Searches and Seizures and Privacy of Communication

169

consent or an invitation thereto, but is merely a demonstration of regard for the supremacy of the law.[117]  [Underscoring supplied.]

Thus, even in cases where the accused voluntarily handed her bag [118] or the chairs[119] containing marijuana to the arresting officer, this Court held there was no valid consent to the search.

On the other hand, because a warrantless search is in derogation of a constitutional right, peace officers who conduct it cannot invoke regularity in the performance of official functions and shift to the accused the burden of proving that the search was unconsented.[120]

In any case, any presumption in favor of regularity would be severely diminished by the allegation of appellants in this case that the arresting officers pointed a gun at them before asking them to open the subject box.  Appellant Tudtud testified as follows:

Q –           This person who approached you according to you pointed something at you[.]  [What] was that something?

A – A 38 cal. Revolver.

Q – How did he point it at you?

A –  Like this (Witness demonstrating as if pointing with his two arms holding something towards somebody).

Q –           This man[,]  what did he tell you when he pointed a gun at you?

A –  He said do not run.

Q – What did you do?

A –            I raised my hands and said “Sir, what is this about?”

Q – Why did you call him Sir?

A –  I was afraid because when somebody is holding a gun, I am afraid.

Q – Precisely, why did you address him as Sir?

A – Because he was holding a gun and I believed that somebody who is carrying a gun is a policeman.

Q –           When you asked him what is this?  What did he say?

A – He said “I would like to inspect what you are carrying.[”]

Q –           What did you say when you were asked to open that carton box?

A –  I told him that is not mine.

Q – What did this man say?

A –  He again pointed to me his revolver and again said to open.

Q –           What did you do?

A – So I proceeded to open for fear of being shot.[121]

Appellants’ implied acquiescence, if at all, could not have been more than mere passive conformity given under coercive or intimidating circumstances and is, thus, considered no consent at all within the purview of the constitutional guarantee.[122] Consequently, appellants’ lack of objection to the search and seizure is not tantamount to a waiver of his constitutional right or a voluntary submission to the warrantless search and seizure.[123]

Page 170: Warrantless Arrests, Searches and Seizures and Privacy of Communication

170

As the search of appellants’ box does not come under the recognized exceptions to a valid warrantless search, the marijuana leaves obtained thereby are inadmissible in evidence.  And as there is no evidence other than the hearsay testimony of the arresting officers and their informant, the conviction of appellants cannot be sustained.

The Bill of Rights is the bedrock of constitutional government.  If people are stripped naked of their rights as human beings, democracy cannot survive and government becomes meaningless.   This explains why the Bill of Rights, contained as it is in Article III of the Constitution, occupies a position of primacy in the fundamental law way above the articles on governmental power.[124]

The right against unreasonable search and seizure in turn is at the top of the hierarchy of rights,[125] next only to, if not on the same plane as, the right to life, liberty and property, which is protected by the due process clause.[126] This is as it should be for, as stressed by a couple of noted freedom advocates,[127] the right to personal security which, along with the right to privacy, is the foundation of the right against unreasonable search and seizure “includes the right to exist, and the right to enjoyment of life while existing.” Emphasizing such right, this Court declared in People v. Aruta:

Unreasonable searches and seizures are the menace against which the constitutional guarantees afford full protection.  While the power to search and seize may at times be necessary to the public welfare, still it may be exercised and the law enforced without transgressing the constitutional rights of the citizens, for the enforcement of no statute is of sufficient importance to justify indifference to the basic principles of government.

Those who are supposed to enforce the law are not justified in disregarding the rights of the individual in the name of order.  Order is too high a price to pay for the loss of liberty.  As Justice Holmes declared: “I think it is less evil that some criminals escape than that the government should play an ignoble part.” It is simply not allowed in free society to violate a law to enforce another, especially if the law violated is the Constitution itself.[128]

Thus, given a choice between letting suspected criminals escape or letting the government play an ignoble part, the answer, to this Court, is clear and ineluctable.

WHEREFORE, the Decision of the Regional Trial Court of Davao City is REVERSED.  Appellants Noel Tudtud y Paypa and Dindo Bolong y Naret are hereby ACQUITTED for insufficiency of evidence.  The Director of the Bureau of Prisons is ordered to cause the immediate release of appellants from confinement, unless they are being held for some other lawful cause, and to report to this Court compliance herewith within five (5) days from receipt hereof.

SO ORDERED.

Bellosillo, (Chairman), Austria-Martinez, and Callejo, Sr., JJ., concur.Quisumbing, J., please see dissenting opinion.

G.R. No. 93239 March 18, 1991

PEOPLE OF THE PHILIPPINES, plaintiff-appellee, vs.EDISON SUCRO, accused-appellant.

The Solicitor General for plaintiff-appellee.

Fidencio S. Raz for accused-appellant.

Page 171: Warrantless Arrests, Searches and Seizures and Privacy of Communication

171

 

GUTIERREZ, JR., J.:p

Edison Sucro was charged with and convicted of violation of Section 4, Article II of the Dangerous Drugs Act, under an Information which reads:

That on or about the 21st day of March, 1989, in the evening, in the Poblacion, Municipality of Kalibo, Province of Aklan, Republic of the Philippines, and within the jurisdiction of this Honorable Court, the above-named accused, acting as a pusher or broker in the business of selling, administering, delivery, giving away to another and/or distributing prohibited drugs, did then and there wilfully, unlawfully and feloniously and without authority of law have in his possession and control nineteen (19) pieces of marijuana cigarette sticks and four (4) tea bags of dried marijuana leaves which were confiscated from him by the police authorities of Kalibo, Aklan, shortly after having sold one tea bag of dried marijuana leaves to a customer. (Rollo, p. 9)

Upon arraignment, the accused-appellant, assisted by counsel, entered a plea of "not guilty" to the offense charged. Trial ensued and a judgment of conviction was rendered, the pertinent portion of which reads:

WHEREFORE, judgment is rendered finding the accused Edison Sucro guilty of the sale of prohibited drug under Section 4, Article II of the Dangerous Drug Act, as amended, and sentencing him to suffer the penalty of life imprisonment, and pay a fine of P20,000, and costs. He shall be entitled to full credit in the service of his sentence with the period for which he has undergone preventive imprisonment to the date of promulgation of this judgment. All the items of marijuana confiscated in this case are declared forfeited in favor of the State. (Rollo, p. 41)

From the foregoing judgment of conviction, accused-appellant interposes this appeal, assigning the following as errors allegedly committed by the court a quo, to wit:

I

THE LOWER COURT ERRED IN ADMITTING AS EVIDENCE FOR THE PROSECUTION EXHIBITS "E"-"E-4", TEA BAGS OF ALLEGED MARIJUANA, TO BE THE CORPUS DELICTI;FURTHERMORE, THAT THE SAME WERE TAKEN WITHOUT THE REQUIRED WARRANT OF SEARCH AND ARREST SINCE THE ACCUSED WAS NOT IN THE ACT OF COMMITTING ANY OFFENSE AT THE TIME OF HIS ARREST.

II

THE LOWER COURT ERRED IN FINDING THE ACCUSED EDISON SUCRO GUILTY OF THE SALE OF PROHIBITED DRUGS UNDER SECTION 4, ARTICLE II, OF THE DANGEROUS DRUGS ACT AND SENTENCING HIM TO SUFFER A PENALTY OF LIFE IMPRISONMENT AND TO PAY A FINE OF P 20,000.00. (Appellant's Brief, p. 1)

The antecedent facts of the case as summarized by the Solicitor General are as follows:

On March 21, 1989, Pat. Roy Fulgencio, a member of the INP, Kalibo, Aklan, was instructed by P/Lt. Vicente Seraspi, Jr. (Station Commander of the INP Kalibo, Aklan) to monitor the activities of appellant Edison Sucro, because of information gathered by Seraspi that Sucro was selling marijuana. (p. 6, TSN, May 2,1989).

Page 172: Warrantless Arrests, Searches and Seizures and Privacy of Communication

172

As planned, at about 5:00 P.M. on said date, Pat. Fulgencio Positioned himself under the house of a certain Arlie Regalado at C. Quimpo Street. Adjacent to the house of Regalado, about 2 meters away, was a chapel. Thereafter, Pat. Fulgencio saw appellant enter the chapel, taking something which turned out later to be marijuana from the compartment of a cart found inside the chapel, and then return to the street where he handed the same to a buyer, Aldie Borromeo. After a while appellant went back to the chapel and again came out with marijuana which he gave to a group of persons. (pp. 6-8, 15-18, Ibid). It was at this instance that Pat. Fulgencio radioed P/Lt. Seraspi and reported the activity going on. P/Lt. Seraspi instructed Pat. Fulgencio to continue monitoring developments. At about 6:30 P.M., Pat. Fulgencio again called up Seraspi to report that a third buyer later Identified as Ronnie Macabante, was transacting with appellant. (pp. 18-19, Ibid)

At that point, the team of P/Lt. Seraspi proceeded to the area and while the police officers were at the Youth Hostel at Maagma St., Pat. Fulgencio told P/Lt. Seraspi to intercept Macabante and appellant. P/Lt. Seraspi and his team caught up with Macabante at the crossing of Mabini and Maagma Sts. in front of the Aklan Medical Center. Upon seeing the police, Macabante threw something to the ground which turned out to be a tea bag of marijuana. (pp. 6-8, TSN, June 19, 1989) When confronted, Macabante readily admitted that he bought the same from appellant (Edison Sucro) in front of the chapel. (p. 6, TSN, May 24, 1989) The police team was able to overtake and arrest appellant at the corner of C. Quimpo and Veterans Sts. The police recovered 19 sticks and 4 teabags of marijuana from the cart inside the chapel and another teabag from Macabante, The teabags of marijuana were sent to the PC-INP Crime Laboratory Service, at Camp Delgado, Iloilo City for analysis. The specimens (Exhibits "G" to "G-18", Exhibits "E" to "E-4") were all found positive of marijuana. (pp. 47, TSN, Sept. 4, 1989)" (Appellee's Brief, pp. 3-6)

As can be seen from the facts, the issue hinges mainly on whether or not the arrest without warrant of the accused is lawful and consequently, whether or not the evidence resulting from such arrest is admissible.

We rule in the affirmative.

The accused-appellant contends that his arrest was illegal, being a violation of his rights granted under Section 2, Article III of the 1987 Constitution. He stresses that there was sufficient time for the police officers to apply for a search and arrest warrants considering that Fulgencio informed his Station Commander of the activities of the accused two days before March 21, 1989, the date of his arrest.

This contention is without merit.

Section 5, Rule 113 of the Rules on Criminal Procedure provides for the instances where arrest without warrant is considered lawful. The rule states:

Arrest without warrant, when lawful. — A peace officer or private person may, without warrant, arrest a person:

(a) When in his presence, the person to be arrested has committed, is actually committing, or is attempting to commit an offense;

(b) When an offense has in fact just been committed, and he has personal knowledge of facts indicating that the person to be arrested has committed it; (Emphasis supplied)

An offense is committed in the presence or within the view of an officer, within the meaning of the rule authorizing an arrest without a warrant, when the officer sees the offense, although at a distance, or hears

Page 173: Warrantless Arrests, Searches and Seizures and Privacy of Communication

173

the disturbances created thereby and proceeds at once to the scene thereof. (U.S. v. Fortaleza, 12 Phil. 472 [1909]; and U.S. v. Samonte, 16 Phil. 516 [1910])

The records show that Fulgencio went to Arlie Regalado's house at C. Quimpo Street to monitor the activities of the accused who was earlier reported to be selling marijuana at a chapel two (2) meters away from Regalado's house.

Fulgencio, within a distance of two meters saw Sucro conduct his nefarious activity. He saw Sucro talk to some persons, go inside the chapel, and return to them and exchange some things. These, Sucro did three times during the time that he was being monitored. Fulgencio would then relay the on-going transaction to P/Lt. Seraspi.

Anent the second requirement, the fact that Macabante, when intercepted by the police, was caught throwing the marijuana stick and when confronted, readily admitted that he bought the same from accused-appellant clearly indicates that Sucro had just sold the marijuana stick to Macabante, and therefore, had just committed an illegal act of which the police officers had personal knowledge, being members of the team which monitored Sucro's nefarious activity.

The court earlier indicated in the case of People v. Bati (G.R. No. 87429, August 27, 1990) that police officers have personal knowledge of the actual commission of the crime when it had earlier conducted surveillance activities of the accused. Thus, it stated:

When Luciano and Caraan reached the place where the alleged transaction would take place and while positioned at a street comer, they saw appellant Regalado Bati and Warner Marquez by the side of the street about forty to fifty meters away from them (the public officers). They saw Marquez giving something to Bati, who, thereafter handed a wrapped object to Marquez who then inserted the object inside the front of his pants in front of his abdomen while Bati, on his part, placed the thing given to him inside his pocket. (p. 2)

xxx xxx xxx

. . . Both Patrolman Luciano and Caraan actually witnessed the same and their testimonies were based on their actual and personal knowledge of the events that took place leading to appellant's arrest. They may not have been within hearing distance, specially since conversation would expectedly be carried on in hushed tones, but they were certainly near enough to observe the movements of the appellant and the buyer. Moreover, these prosecution witnesses are all law enforcers and are, therefore, presumed to have regularly performed their duties in the absence of proof to the contrary (People v. Bati, supra citing People v. Agapito, G.R. No. 73786, October 12, 1987)

The accused questions the failure of the police officers to secure a warrant considering that Fulgencio himself knew of Sucro's activities even prior to the former's joining the police force. Fulgencio reported Sucro's activities only three days before the incident.

As the records reveal, Fulgencio and Sucro had known each other since their childhood years and that after Fulgencio joined the police force, he told the accused-appellant not to sell drugs in their locality. Hence, it is possible that because of this friendship, Fulgencio hesitated to report his childhood friend and merely advised him not to engage in such activity. However, because of reliable information given by some informants that selling was going on everyday, he was constrained to report the matter to the Station Commander.

On the other hand, the failure of the police officers to secure a warrant stems from the fact that their knowledge acquired from the surveillance was insufficient to fulfill the requirements for the issuance of a

Page 174: Warrantless Arrests, Searches and Seizures and Privacy of Communication

174

search warrant. What is paramount is that probable cause existed. Thus, it has been held in the case of People v. Lo Ho Wing, et al. (G.R. No. 88017, January 21, 1991):

In the instant case, it was firmly established from the factual findings of the trial court that the authorities had reasonable ground to believe that appellant would attempt to bring in contraband and transport it within the country. The belief was based on intelligence reports gathered from surveillance activities on the suspected syndicate, of which appellant was touted to be a member. Aside from this, they were also certain as to the expected date and time of arrival of the accused from China. But such knowledge was clearly insufficient to enable them to fulfill the requirements for the issuance of a search warrant. Still and all, the important thing is that there was probable cause to conduct the warrantless search, which must still be present in such a case.

As the Solicitor General has pointed out:

There are several instances when a warrantless search and seizure can be effected without necessarily being preceded by an arrest provided the same is effected on the basis of probable cause (e.g. stop and search without warrant at checkpoints). Between warrantless searches and seizures at checkpoints and in the case at bar the latter is more reasonable considering that unlike in the former, it was effected on the basis of probable cause. Under the circumstances (monitoring of transactions) there existed probable cause for the arresting officers, to arrest appellant who was in fact selling marijuana and to seize the contraband.

That searches and seizures must be supported by a valid warrant is not an absolute rule (Manipon, Jr. v. Sandiganbayan, 143 SCRA 267 [1986]). Among the exceptions granted by law is a search incidental to a lawful arrest under Sec. 12, Rule 126 of the Rules on Criminal Procedure, which provides that a person lawfully arrested may be searched for dangerous weapons or anything which may be used as proof of the commission of an offense, without a search warrant. (People v. Castiller, G.R. No. 87783, August 6, 1990)

The accused-appellant claims that the arrest having been done without warrant, it follows that the evidence obtained therefrom is inadmissible.

As earlier discussed, there is nothing unlawful about the arrest considering its compliance with the requirements of a warrantless arrest. Ergo, the fruits obtained from such lawful arrest are admissible in evidence.

Edison Sucro assails the trial court's reliance on the statement of Macabante whose reason for testifying could be merely to escape prosecution.

We quote the trial court's finding as to the testimony of Macabante:

The non-filing of a complaint against him for possession of marijuana may have been the reason of (sic) his willingness to testify in court against the accused. But this does not necessarily taint the evidence that proceeds from his lips. As explained by Lt. Seraspi, the best sources of information against drug pushers are usually their customers, especially if as in this case, there is no other direct evidence of the selling except the testimony of the buyer. We accept this observation as a realistic appraisal of a situation in which drug users are, and should be employed by law enforcement authorities to bolster the drive against pushers who are the real felons in our society. We have observed the demeanor of the witness in court, and found him to be straightforward, unhesitating, and spontaneous in his declarations, so that we are satisfied as to his intention and disposition to tell the truth (Rollo, p. 40)

Page 175: Warrantless Arrests, Searches and Seizures and Privacy of Communication

175

Time and again it has been held that the findings of the trial court are entitled to great weight and should not be disturbed on appeal unless it is shown that the trial court had overlooked certain facts of weight and importance, it being acknowledged. that the court below, having seen and heard the witnesses during the trial, is in a better position to evaluate their testimonies (People v. Umali, et al., G.R. No. 84450, February 4, 1991 citing People v. Alvarez, 163 SCRA 745 [1988]; People v. Dorado, 30 SCRA 53 [1969]; and People v. Espejo, 36 SCRA 400 [1970]).

Furthermore, the testimony of Macabante was corroborated on material points by public officers Fulgencio and Seraspi.

There is nothing in the record to suggest that the police officers were compelled by any motive than to accomplish their mission to capture a drug pusher in the execution of the crime, the presumption being that police officers perform their duties regularly in the absence of any evidence to the contrary (Rule 131, Sec. 3(m), Revised Rules on Evidence; People v. Castiller, supra citing People v. Natipravat, 145 SCRA 483 [1986]).

The prosecution evidence was further bolstered by the findings of the Forensic Chemist that the items seized were all positive for marijuana.

In contrast to the evidence presented by the prosecution, accused-appellant's defense is alibi which is unavailing considering that he was positively identified by Macabante to be the person from whom he bought marijuana.

Sucro alleges that he could not have committed the crime since he was with his uncle and cousin distributing handbills for his Auntie's candidacy. The fact, however, remains that it does not preclude the possibility that he was present in the vicinity as established by his admission that he moved a lot and even had the occasion to meet Macabante on the street.

It is well-settled that mere denials cannot prevail against the positive identification of the appellant as the seller of the prohibited substances. (People v. Khan, 161 SCRA 406 [1988]; and People v. Paco, 170 SCRA 681 [1989])

Premises considered, this Court is convinced that appellant Edison Sucro had indeed committed the offense charged. The trial court's decision must be upheld.

WHEREFORE, the decision appealed from is hereby AFFIRMED.

SO ORDERED.

Fernan, C.J., Feliciano, Bidin and Davide, Jr., JJ., concur.

[G.R. No. 130805.  April 27, 2004]

PEOPLE OF THE PHILIPPINES, petitioner, vs. TOKOHISA KIMURA and AKIRA KIZAKI,[1] respondents.

Page 176: Warrantless Arrests, Searches and Seizures and Privacy of Communication

176

D E C I S I O N

AUSTRIA-MARTINEZ, J.:

Appellants Tomohisa Kimura and Akira Kizaki seek reversal of the decision [2] dated June 27, 1997 in Criminal Case No. 94-5606, rendered by the Regional Trial Court (Branch 66), Makati City, finding them guilty beyond reasonable doubt of violation of Section 4, Article II of Republic Act No. 6425, as amended by R.A. No. 7659, otherwise known as the Dangerous Drugs Act of 1972, and sentencing each of them to suffer the penalty of reclusion perpetua and to pay a fine of P500,000.00.

The Information dated August 8, 1994 against the accused alleges:

The undersigned State Prosecutor of the Department of Justice accuses TOMOHISA KIMURA and AKIRA KIZAKI of violation of Section 4, Article II of Republic Act 6425, as amended by R.A. 7659, otherwise known as the Dangerous Drugs Act of 1972, committed as follows:

That on or about June 27, 1994 in Makati, Metro Manila and within the jurisdiction of this Honorable Court, the above-named accused conspiring, confederating and mutually helping one another, did then and there willfully, unlawfully and feloniously transport and deliver without lawful authority approximately 40,768 grams of Indian hemp (marijuana), a prohibited drug, in violation of the aforecited law.

CONTRARY TO LAW. [3]

Upon arraignment on October 10, 1994, the two accused, through counsel, entered their separate pleas of Not Guilty to the crime charged;[4] whereupon, the trial of the case ensued.

The testimonies of the following prosecution witnesses, to wit:  SPO4 Juan Baldovino, Jr.,[5] SPO1 Rolando Cabato,[6] SPO1 Edmundo Badua, Chief Inspector Nilo Anso, PO3 Alfredo Cadoy, SPO1 Manuel Delfin and Forensic Chemist, Police Inspector Sonia Ludovico, sought to establish the following facts:

In the morning of June 27, 1994, Maj. Anso, head of Delta Group, Narcotics Command (NARCOM) I, North Metro District Command, Camp Karingal, Quezon City, received information from a confidential informant that a certain Koichi Kishi and Rey Plantilla were engaged in the selling of illegal drugs at the Cash and Carry Supermarket, Makati City.[7] Acting on said information, Maj. Anso organized a team composed of SPO4 Baldovino, Jr., SPO1 Cabato and PO3 Cadoy to conduct surveillance of the area. [8] A buy-bust operation was launched and PO3 Cadoy was designated to act as poseur-buyer and they prepared the buy-bust money consisting of one P500.00 bill and five pieces of P100.00 bill.[9]

At around 3:00 in the afternoon of the same day, the team together with the informant arrived at the Cash and Carry Supermarket and conducted surveillance of the area. [10] Later, the informant was able to contact the targets who told him that they will be arriving at 8:00 in the evening at the parking area of the Cash and Carry Supermarket.[11] At around 8:00 in the evening, Koichi and Rey arrived and were met by PO3 Cadoy and the informant.[12] PO3 Cadoy gave the marked money worthP1,000.00 to Rey and Koichi who then handed him the “shabu”.  PO3 Cadoy scratched his head as a pre-arranged signal of the consummation of the sale.[13] The operatives were about five meters from the suspects.[14] While the team was approaching, PO3 Cadoy held Koichi by the hand while Rey scampered away to the direction of the South Superhighway.[15] The team brought Koichi to a safe area within the Cash and Carry Supermarket and interrogated him.  They learned from Koichi that his friends/suppliers will arrive the same evening to fetch him.[16] Several minutes later, a white Nissan Sentra car driven by appellant Kimura with his co-appellant Kizaki seating at the passenger seat arrived at the parking area.  Koichi pointed to them as the ones who will fetch him.  Appellants remained inside the car for about ten to fifteen minutes. [17] Then, a certain Boy driving a stainless jeep, without a plate number, arrived and parked the jeep two to three parking spaces away from the Sentra car.[18] Boy approached the Sentra car and after a few minutes, appellants got out of their car.  Appellant Kizaki went to the stainless jeep and sat at the passenger seat.  Boy and appellant Kimura went to the rear of the Sentra car and opened its trunk. [19] Appellant Kimura got a package wrapped in a newspaper and gave it to Boy who walked back to his jeep. [20] While Maj. Anso and SPO4 Baldovino, Jr. were approaching to check what was inside the wrapped newspaper,

Page 177: Warrantless Arrests, Searches and Seizures and Privacy of Communication

177

appellant Kimura ran but was apprehended while Boy was able to board his jeep and together with appellant Kizaki who was seated at the passenger seat sped off towards South Superhighway. [21] The police operatives then inspected the contents of the trunk and found packages of marijuana. [22] They brought Koichi and appellant Kimura to the headquarters and turned over the seized marijuana to the investigator who made markings thereon.[23] Maj. Anso reported the escape of appellant Kizaki to their investigation section.[24]

The seized packages which were contained in 3 sacks were brought to the PNP Crime Laboratory on June 29, 1994.[25] Forensic Chemist Sonia Sahagun-Ludovico testified that the contents of the sacks weighed 40,768 grams and were positive to the test of marijuana.[26]

On June 29, 1994, appellant Kizaki while having dinner with his friends at the Nippon Ichi Restaurant located at Mabini, Malate, Manila[27] was arrested by another NARCOM group led by Maj. Jose F. Dayco.[28]

Appellants’ defense is denial and alibi.  In support thereof, both appellants were called to the witness stand.

Appellant Kimura’s testimony is as follows:  In the afternoon of June 27, 1994, Kimura was in the house of his co-appellant Kizaki at Dian Street, Makati City, together with Koichi Kishi, Luis Carlos and a certain “Sally” and “Boy”.[29] In the evening of the said date, Kimura borrowed the car of Kizaki in order to get his (Kimura’s) television from his house located in Evangelista Street, near the Cash and Carry Supermarket, and bring the same to a repair shop. [30] On their way to Kimura’s house, Koichi requested Kimura to pass by Cash and Carry Supermarket because Koichi needed to meet a certain “Rey” who was borrowing money from him.  Upon reaching Cash and Carry, Kimura parked the car about twenty meters from its entrance, then Koichi and Carlos alighted from the car and Koichi handed something to Rey.[31] Shortly thereafter, Koichi and Carlos were grabbed by two men from behind.  Then four men approached the car and one guy ordered him to sit at the back and together with Koichi and Carlos, they were all brought to Camp Karingal allegedly for violating Sec. 4 of Republic Act No. 6425.[32] Kimura was asked questions about the address and business of Kizaki.  Kimura denied that there was marijuana in the car on the night of June 27, 1994 but claims that he saw marijuana placed at the car trunk the following day at Camp Karingal.  Kizaki was not with him at Cash and Carry on the night of June 27, 1994.  There was no stainless jeep near the car on the same night.  Carlos was released and was not charged because Kimura’s girlfriend, Sally, served as Carlos’ guarantor.

On the other hand, appellant Kizaki testified that on the date that the alleged crime was committed, he was in the company of his friends, Mr. and Mrs. Takeyama, his co-appellant Kimura, and his driver Boy and maid Joan at his house in Dian Street, Makati City; [33] that appellant Kimura borrowed his car on the night of June 27, 1994 to pick up Kimura’s broken TV and bring it to the repair shop. [34]

Appellant Kizaki’s alibi was corroborated by Rosario Quintia, his former housemaid, and his friend, Akiyoshi Takeyama, who both testified that they were at Kizaki’s house on the night of June 27, 1994 from 7:00 to 10:00 in the evening and never saw Kizaki leave the house.[35]

Appellant Kizaki was arrested on June 29, 1994, two days after the Cash and Carry incident, in the Nippon Ichi Restaurant located at Mabini, Manila. He was having dinner with Lt. Col. Rodolfo Tan, Masami Y. Nishino, Anita Takeyama and Akiyoshi Takeyama.  These witnesses executed a joint affidavit[36] and testified that while they were about to leave the restaurant, a man got near Kizaki and asked for his passport whom they thought was from the Immigration.  Later, they learned that Kizaki was brought to Camp Karingal.[37]

On June 27, 1997, the trial court rendered the herein assailed judgment, the dispositive portion of which reads:

IN VIEW OF THE FOREGOING, judgment is hereby rendered finding accused Akira Kizaki and Tomohisa Kimura GUILTY beyond reasonable doubt for violation of Section 4 of Republic Act 6425, as amended by Republic Act 7659, and the Court hereby sentences them to suffer, taking into consideration the absence of mitigating or aggravating circumstances, the amount of marijuana seized from the accused which weigh 40,768 grams, the penalty of RECLUSION PERPETUA and to pay a fine of P500,000.00 each.

Page 178: Warrantless Arrests, Searches and Seizures and Privacy of Communication

178

The Bureau of Immigration and Deportation is hereby ordered to deport Akira Kizaki and Tomohisa Kimura without further proceedings after the service of their sentence.

Let the marijuana, the subject matter of this case be immediately forwarded to the Dangerous Drugs Board for proper disposition.

SO ORDERED.[38]

In convicting appellants, the trial court made the following findings:

The settled jurisprudence is that alibi is inherently a weak defense. Like the defense of alibi, denial by the accused of the offense charged against him is also inherently a weak defense. It is also the settled jurisprudence that the defense of alibi and denial cannot prosper over the positive identification of the accused by the prosecution witnesses.  For alibi to prosper, the accused must show that it was impossible for him to have been at the scene of the commission of the crime at the time of its commission.

Akira testified that on the evening of June 27, 1994, he was in his house located at Dian Street corner Ampil Street, Makati City, Metro Manila, which is a  walking distance to Cash and Carry Supermarket, the scene of the offense.  It was not therefore impossible for accused Akira Kizaki to have been present at the scene of the crime at the time of its commission.

Accused Kimura testified that on the evening of June 27, 1994, he was with his co-accused Kizaki at the Cash and Carry Supermarket but for another purpose, i.e., to meet Rey Plantilla who was borrowing money from him.  In fine accused Kimura merely denied the offense charged against him, which is weak defense.

Both accused, Kizaki and Kimura, were positively identified by prosecution witnesses SPO4 Baldomino, SPO1 Cabatu, Maj. Anso and PO3 Cadoy as the persons whom they arrested for drug trafficking in a buy-bust operation at the Cash and Carry Supermarket on June 27, 1994.

Finally, although the evidence show that there is a doubt in the illegality of the arrest of accused Kimura by Major Dayco, the jurisprudence is that “the illegality of warrantless arrest cannot deprive the state of its right to convict the guilty when all the facts on record point to their culpability.[39]

Hence, this appeal before us.  Appellants assert the following:

I

THE COURT A QUO GRAVELY ERRED IN DISREGARDING ACCUSED-APPELLANTS’ DEFENSE.

II

THE COURT A QUO GRAVELY ERRED IN FINDING THAT THE GUILT OF THE ACCUSED-APPELLANTS HAD BEEN PROVEN BEYOND REASONABLE DOUBT.

Appellants claim that although the defense of alibi and denial are weak, it is still the duty of the prosecution to prove the guilt of the accused beyond reasonable doubt to support a judgment of conviction; that the trial court mainly relied on the weakness of the defense rather than on the strength of the evidence for the prosecution.  They argue that appellant Kizaki’s claim that he was not at the Cash and Carry Supermarket on the night of June 27, 1994 was corroborated by three independent witnesses including appellant Kimura who testified that he was not with appellant Kizaki at Cash and Carry Supermarket on the said night.

Page 179: Warrantless Arrests, Searches and Seizures and Privacy of Communication

179

Appellants further question how the trial court could have been certain that the marijuana presented in court are the same articles confiscated from the appellants when the arresting officers did not place identifying marks on the confiscated items.

Appellant Kizaki further contends that he was arrested two days after the alleged buy-bust operation without a valid warrant of arrest.  He points out that although the trial court expressed doubts as to the legality of his arrest, it nevertheless convicted him of the crime charged, which is in violation of the Constitution.  Kizaki argues that he could not have been caught in flagrante delicto to justify the warrantless arrest when he was arrested two days after the alleged Cash and Carry incident while he was only having dinner with his friends at a restaurant.

In the appellee’s brief, the Solicitor General prays that the decision of the trial court finding appellants guilty as charged be affirmed.  He argues that appellants were positively identified by four prosecution witnesses, all police officers, as among the three persons engaged in the transportation and delivery of about 40,768 grams of marijuana on June 27, 1994 at the Cash and Carry Supermarket; that the police operatives were able to seize the marijuana from the Sentra car they were using to transport the marijuana; that the marijuana introduced and offered at the trial were positively identified by the arresting officers as those seized from the car of the appellants; that the contention of appellant Kizaki that his warrantless arrest two days after the alleged incident, was unlawful, is legally inconsequential in this case considering that his conviction was not based on his arrest on June 29, 1994 but on his having participated in the transport and delivery of marijuana on June 27, 1994; that appellant Kizaki  never questioned the validity of the warrantless arrest of his co-appellant Kimura on June 27, 1994, either before the trial court or before this Court; thus, any challenge against the search and  seizure of the marijuana based on constitutional ground is deemed waived insofar as appellant Kizaki is concerned.

We will first resolve the issue on the alleged warrantless arrest of appellant Kizaki.

Appellant Kizaki assails the legality of his warrantless arrest.  Indeed, SPO1 Delfin, one of those who arrested appellant Kizaki at the Nippon Ichi restaurant, admitted that they did not have a warrant of arrest when his group arrested Kizaki on the night of June 29, 1994. Rule 113, Section 5 of the Revised Rules of Criminal Procedure provides that a peace officer or a private person may, without a warrant, arrest a person only under the following circumstances:

(a)     When, in his presence, the person to be arrested has committed, is actually committing, or is attempting to commit an offense;

(b)     When an offense has just been committed and he has probable cause to believe based on personal knowledge of facts or circumstances that the person to be arrested has committed it; and

(c)     When the person to be arrested is a prisoner who has escaped from a penal establishment or place where he is serving final judgment or is temporarily confined while his case is pending, or has escaped while being transferred from one confinement to another.

The alleged crime happened on June 27, 1994 and appellant Kizaki was arrested on June 29, 1994 or two days after the subject incident.  At the time appellant Kizaki was arrested, he was at a restaurant having dinner with a group of friends, thus, he was not committing or attempting to commit a crime.  Neither was he an escaped prisoner whose arrest could be effected even without a warrant.  It bears stressing that none of the arresting officers of appellant Kizaki was present on the night of June 27 where appellant Kizaki allegedly sold and transported marijuana and escaped, thus the arresting officers had no personal knowledge of facts or circumstances that appellant Kizaki committed the crime. None of the exceptions enumerated above was present to justify appellant Kizaki’s warrantless arrest.

However, notwithstanding the unjustified warrantless arrest of appellant Kizaki, the records show that he did not raise such question before he pleaded to the offense charged. [40] Neither did he move to quash the information on that ground before the trial court. [41] He thus waived objection to the illegality of his

Page 180: Warrantless Arrests, Searches and Seizures and Privacy of Communication

180

arrest.[42]Moreover, appellant Kizaki’s application for bail[43] which was denied by the trial court likewise constitutes a waiver of his right to question whatever irregularities and defects which attended his arrest.[44]

Nevertheless, we find the other claims of appellants meritorious.

In all prosecutions for violation of the Dangerous Drugs Act, the existence of all dangerous drugs is a sine qua non for conviction.  The dangerous drug is the verycorpus delicti of the crime of violation of the Dangerous Drugs Act.[45]

In People vs. Casimiro,[46] we acquitted appellant for failure of the prosecution to establish the identity of the prohibited drug which constitutes the corpus delictiand held:

In People vs. Mapa, the accused-appellant was granted an acquittal after the prosecution failed to clarify whether the specimen submitted to the NBI for laboratory examination was the same one allegedly taken from the accused.  In People vs. Dismuke, this Court ruled that the failure to prove that the specimen of marijuana examined by the forensic chemist was that seized from the accused was fatal to the prosecution’s case.  In People vs. Laxa, the policemen composing the buy-bust team failed to mark the confiscated marijuana immediately after the alleged apprehension of the accused-appellant.  One policeman admitted that he marked the seized items only after seeing them for the first time in the police headquarters. It was held:

This deviation from the standard procedure in the anti-narcotics operations produces doubts as to the origins of the marijuana.  Were the allegedly confiscated from the scene of the crime the same ones which the investigator    marked in the police headquarters?  This question gives rise to surmises and speculations, and cannot prove beyond reasonable doubt the guilt of accused-appellant.

In this case, the prosecution failed to prove the crucial first link in the chain of custody.  The prosecution witnesses PO2 Supa, SPO2 Madlon and PO3 Piggangay admitted they did not write their initials on the brick of marijuana immediately after allegedly seizing from accused-appellant outside the grocery store but only did so in their headquarters. The narcotics field test, which initially identified the seized item as marijuana, was likewise not conducted at the scene of the crime, but only at the narcotics office.  There is thus reasonable doubt as to whether the item allegedly seized from accused-appellant is the same brick of marijuana marked by the policemen in their headquarters and given by them to the crime laboratory for examination.

After examining the evidence for the prosecution, and tested in the light of the Casimiro case, we find that the prosecution failed to establish the identity of the marijuana allegedly seized from appellants Kimura and Kizaki.  Extant in the records were the admissions made by the police operatives of their failure to place any markings on the seized marijuana immediately after they had allegedly apprehended appellants, thus failing to prove that the marijuana presented in court was the very same marijuana seized from appellants.  Maj. Anso, head of the police operatives, testified on cross-examination as follows:

ATTY. BALICUD:

       With respect to the packages which you identified yesterday, before you showed that to your investigation section, did you make any markings thereat?

WITNESS:

       None, sir.

ATTY. BALICUD:

       Did any of your men place any markings at least to identify that that is the drugs confiscated by you at the Cash and Carry?

WITNESS:

Page 181: Warrantless Arrests, Searches and Seizures and Privacy of Communication

181

       What I know your honor, is that the investigation section is the one who will mark the evidence.

. . .

COURT:

       You mean to say when you have already surrender(sic) the shabu(sic) to the investigation section that was the time when the investigator mark them?

WITNESS:

       It is already their duty to mark them, your honor.

. . .

ATTY. BALICUD:

       And did you see if any of those men in the investigation section did the corresponding markings?

WITNESS:

       I did not already see sir.[47]

The testimony of Maj. Anso was confirmed by SPO4 Baldovino, Jr. when the latter testified on cross-examination as follows:

ATTY. SENSON:

Q.   When the packages contained in Exhibits “B”, “C” and “D” were recovered at the car, did you not make any markings on them, is that correct?

WITNESS:

A.    That is true, sir.[48] 

SPO4 Baldovino, Jr. further clarified on his re-direct examination why no markings were made, thus:

FISCAL MANABAT:

Q.   Why is it that no markings were made on these marijuana packages?

. . .

WITNESS:

We did not put markings there because after we confiscated those packages, there was a press conference conducted and after that we submitted it to PCCL or Philippine Crime Laboratory, sir.[49]

The failure to establish the chain of custody of the evidence is further shown by the testimony of SPO1 Badua, the person assigned to bring the alleged seized marijuana to the PNP Crime Laboratory. His testimony is as follows:

PROS. MANABAT:

       Do you recall your activities on that day, June 29, 1994?

WITNESS:

       I was ordered to bring the marijuana to the Crime Laboratory.

PROS. MANABAT:

       Who ordered you to bring the marijuana to the Crime Laboratory?

Page 182: Warrantless Arrests, Searches and Seizures and Privacy of Communication

182

WITNESS:

       Superintendent Eduardo Cariño, sir.

PROS. MANABAT:

       Where did this marijuana come from, if you know?

WITNESS:

       In our office confiscated from Japanese nationals.

PROS. MANABAT:

       Do you know the name of the Japanese nationals you are referring to?

WITNESS;

       I do not know, sir.

PROS. MANABAT:

       Can you describe this marijuana which you said you were required to bring to the PNP Crime Laboratory?

WITNESS:

       They are contained in sacks, sir.

COURT:

How many sacks?

WITNESS:

       Three (3), sir.

PROS. MANABAT:

       What kind of sacks were these, can you recall?

WITNESS:

       Rice sacks.

. . .

PROS. MANABAT:

       Now, if you see this marijuana you said you were required to bring to the PNP Crime Laboratory which you described as being contained in three (3) sacks, will you be able to identify these three (3) sacks of marijuana.

WITNESS:

       Yes, sir.

PROS. MANABAT:

       Now, I am showing to you SPO1 Badua, there are three (3) sacks (sic) here already deposited in Court, please examine these three (3) sacks carefully and tell us the relation of these three sacks to that marijuana contained in sacks which you said you were required to bring to PNP Crime Laboratory.

WITNESS:

       These are the three sacks I brought.

Page 183: Warrantless Arrests, Searches and Seizures and Privacy of Communication

183

PROS. MANABAT:

. . .

       Now, you said that this marijuana was contained in three sacks, three rice sacks,  will you please examine the sacks and tell us if these are the same sacks which you brought to the PNP Crime Laboratory?

WITNESS:

       Yes, sir, these are the same sacks I brought.

COURT:

       What made you so sure that these are the same sacks that you brought from your office to the Crime Laboratory?

WITNESS:

       Because of the markings A, B, C.

COURT:

       Who affixed those markings?

WITNESS:

       The investigator, sir.

COURT:

       Did you see the investigator affixed those markings?

WITNESS:

       Yes, sir.

COURT:

Who was the investigator?

WITNESSS:

SPO1 Delfin, sir.

PROS. MANABAT:

       Now, what proof do you have that you actually brought these three sacks of marijuana which you identified to the PNP Crime Laboratory?

WITNESS:

       There is a request for laboratory examination.

PROS. MANABAT:

       Who prepared this request for laboratory examination of the marijuana?

WITNESS:

       SPO1 Delfin.[50]

And on cross-examination as follows:

ATTY. BALICUD:

       Now, were you present when this request for laboratory examination was prepared?

Page 184: Warrantless Arrests, Searches and Seizures and Privacy of Communication

184

WITNESS:

Yes, sir.

ATTY. BALICUD:

       Who specifically typed the request?

WITNESS:

       SPO1 Delfin, sir.

ATTY. BALICUD:

       And this was made on June 28, 1994?

WITNESS:

       Yes, sir.

. . .

ATTY. BALICUD:

       And then about what time on June 28 was it prepared?

WITNESS:

       Morning, sir.

ATTY. BALICUD:

       But then this request was received already by the Crime Laboratory on June 29, 1994, where were the three sacks deposited from June 28 up to the time you picked it up on June 29 to be brought to the Crime Laboratory?

WITNESS:

       It was deposited inside our supply room.

ATTY. BALICUD:

       Why did you not deposit or deliver it immediately to the Crime Laboratory?

WITNESS:

       We were still preparing the necessary papers.

ATTY. BALICUD:

       What papers were still being prepared?

WITNESS:

       Request for laboratory, medical, drug dependency.

. . .

ATTY. BALICUD:

Now, when the request for laboratory examination was made, did you already see the contents inside the sack?

WITNESS:

       Yes, sir.

ATTY. BALICUD:

Page 185: Warrantless Arrests, Searches and Seizures and Privacy of Communication

185

Why did you open the sacks?

WITNESS:

       Yes, sir.

ATTY. BALICUD:

       Who opened the sacks?

WITNESS:

       The investigator.

ATTY. BALICUD:

       So that they were placed in three sacks?

WITNESS:

       Yes, sir.

ATTY. BALICUD:

       And you opened each and every sack?

WITNESS:

       Yes, sir.

ATTY. BALICUD:

       You brought the same to the Crime Laboratory?

WITNESS:

       Yes, sir.

ATTY. BALICUD:

Were there markings in the 3 sacks when the same were brought to the PNP Crime Laboratory?

WITNESS:

       Yes, sir. A, B, C.

ATTY. BALICUD:

So that one sack is marked A, the other sack is B and the other is marked C.

WITNESS:

       Yes, sir.

ATTY. BALICUD:

       How about the contents of these three sacks, were they also marked when you brought the same to the PNP Crime Laboratory?

WITNESS:

       Yes, sir, but it was marked at the Crime Laboratory already.

ATTY. BALICUD:

       So, it is clear that when the alleged marijuana was brought to the PNP Crime Laboratory, there was no marking yet?

Page 186: Warrantless Arrests, Searches and Seizures and Privacy of Communication

186

WITNESS:

Yes, sir. [51]

While SPO1 Badua’s testimony showed that it was investigator SPO1 Delfin who made the markings A, B, C on the three sacks containing the marijuana which he brought to the laboratory, nowhere in his testimony did he say that such markings were made on the night the appellants were arrested, i.e., on June 27, 1994. Investigator Delfin did not initial said markings nor did he testify affirming his markings.

Moreover, although the three sacks of alleged marijuana were marked as A, B, C, the contents of these three sacks however had no markings when they were kept inside the supply room on June 28 since as Badua intimated, the contents of these three sacks were only marked when he brought the same to the PNP Crime Laboratory on June 29, 1994.

The records of the case do not show that the police operatives complied with the procedure in the custody of seized prohibited and regulated drugs as embodied in the Dangerous Drugs Board Regulation No. 3 Series of 1979 amending  Board Regulation No. 7 Series of 1974,[52] i.e., any apprehending team having initial custody and control of said drugs and/or paraphernalia, should immediately after seizure or confiscation, have the same physically inventoried and  photographed in the presence of the accused, if there be any, and/or his representative, who shall be required to sign the copies of the inventory and be given a copy thereof.  In this case, there was no inventory made in the crime scene despite the fact that Maj. Anso testified that he saw eighteen packages neatly wrapped in a newspaper but the inventory was made already in the headquarters.  SPO1 Badua testified that the marijuana confiscated from appellant Kimura was contained in three sacks.

Consequently, the failure of the NARCOM operatives to place markings on the alleged seized marijuana coupled with their failure to observe the procedure in the seizure and taking custody of said drug seriously bring to question the existence of the seized prohibited drug.  It is not positively and convincingly clear that what was submitted for laboratory examination and presented in court was actually recovered from the appellants.

Evidently, the prosecution has not proven the indispensable element of corpus delicti of the crime which failure produces a grevious doubt as to the guilt of the appellants.  In criminal cases, proof beyond reasonable doubt is required to establish the guilt of the accused. Similarly, in establishing the corpus delicti, thatunwavering exactitude is necessary.  Every fact necessary to constitute the crime must be established by proof beyond reasonable doubt.[53]

Although the defense raised by appellants Kimura and Kizaki were denial and alibi, respectively, which are inherently weak, we have repeatedly declared that the conviction of the accused must rest not on the weakness of the defense but on the strength of the prosecution. [54] The denial of appellant Kimura that he was caught in the Cash and Carry Supermarket delivering marijuana on the night of  June 27, 1994 may be weak but the evidence for the prosecution is clearly even weaker.  InPeople vs. Laxa,[55] we acquitted the appellant for failure of the prosecution to establish the identity of the prohibited drug which constitutes the corpus delicti, an essential requirement in a drug related case. In the present case, the prosecution also failed to indubitably show the identity of the marijuana which mere allegedly seized from appellants.

The alibi of appellant Kizaki that he was in his house on the same night assumes weight and significance considering that the scenario depicted by the prosecution on the alleged escape of appellant Kizaki at the Cash and Carry left much to speculations and surmises.  The prosecution tried to show that appellant Kizaki who was on board the stainless jeep was able to escape even if the police operatives were only about five meters away from the jeep[56] which was heading to the entrance of the Cash and Carry along South Superhighway.  It is quite difficult for us to accept its veracity considering that despite the short distance of the operatives from the jeep when it started to speed off, the operatives who were all armed with service revolvers[57] chased on foot the stainless  jeep and did  not even fire any warning shot to stop the driver and appellant Kizaki nor did they fire a shot at the tire of the jeep to immobilize it.   The alibi of Kizaki found corroboration from his friend Akiyoshi Takeyama and appellant Kizaki’s former housemaid Rosaria Quintia that he was in his house and never left it on the night of the alleged delivery or transport of marijuana in Cash and Carry Supermarket.  In fact, co-appellant Kimura testified that

Page 187: Warrantless Arrests, Searches and Seizures and Privacy of Communication

187

appellant Kizaki was not one of his companions in going to Cash and carry Supermarket on June 27, 1994.[58] Moreover, in the request for laboratory examination dated June 28, 1994, signed by P/CI Jose F. Dayco, Chief, Investigation Section, NMDU, NARCOM, the suspects named therein were only Koichi Kishi and Tomohisa Kimura.[59] Hence, the constitutional presumption of innocence has not been overcome by the prosecution.

In fine, for failure of the prosecution to establish the guilt of both appellants beyond reasonable doubt, they must perforce be exonerated from criminal liability.

WHEREFORE, the decision of the trial court in Criminal Case No. 94-5606 is hereby REVERSED and appellants Tomohisa Kimura and Akira Kizaki, are herebyACQUITTED on ground of reasonable doubt.  They are ordered immediately released from prison, unless they are being detained for some other lawful cause.  The Director of Prisons is DIRECTED to inform this Court of the action taken hereon within five (5) days from receipt hereof.

Let the PNP Director be furnished a copy of herein decision for the proper information and guidance of his police operatives. The marijuana is hereby ordered confiscated in favor of the government for its proper disposition under the law.

Costs de oficio.

SO ORDERED.

Puno, (Chairman), Quisumbing, Callejo, Sr., and Tinga, JJ., concur.

[G.R. No. 121917.  March 12, 1997]

ROBIN CARIÑO PADILLA @ ROBINHOOD PADILLA, petitioner, vs. COURT OF APPEALS and PEOPLE of the PHILIPPINES,respondents.

D E C I S I O N

FRANCISCO, J.:

On October 26, 1992, high-powered firearms with live ammunitions were found in the possession of petitioner Robin Padilla @ Robinhood Padilla, i.e.:

"(1)    One .357 Caliber revolver, Smith and Wesson, SN-32919 with six (6) live ammunitions;

"(2)    One M-16 Baby Armalite rifle, SN-RP 131120 with four (4) long and one (1) short magazine with ammunitions;

"(3)    One .380 Pietro Beretta, SN-A 35723 Y with clip and eight (8) ammunitions; and

"(4)    Six additional live double action ammunitions of .38 caliber revolver."[1]

Petitioner was correspondingly charged on December 3, 1992, before the Regional Trial Court (RTC) of Angeles City with illegal possession of firearms and ammunitions under P.D. 1866 [2] thru the following Information:[3]

"That on or about the 26th day of October, 1992, in the City of Angeles, Philippines, and within the jurisdiction of this Honorable Court, the above-named accused, did then and there willfully, unlawfully and feloniously have in his possession and under his custody and control one (1) M-16 Baby Armalite rifle, SN-RP 131120 with four (4) long and one (1) short magazines with

Page 188: Warrantless Arrests, Searches and Seizures and Privacy of Communication

188

ammunitions, one (1) .357 caliber revolver Smith and Wesson, SN-32919 with six (6) live ammunitions and one (1) .380 Pietro Beretta, SN-A35723Y with clip and eight (8) ammunitions, without having the necessary authority and permit to carry and possess the same.

ALL CONTRARY TO LAW."[4]

The lower court then ordered the arrest of petitioner, [5] but granted his application for bail.[6] During the arraignment on January 20, 1993, a plea of not guilty was entered for petitioner after he refused, [7] upon advice of counsel,[8] to make any plea.[9] Petitioner waived in writing his right to be present in any and all stages of the case.[10]

After trial, Angeles City RTC Judge David Rosete rendered judgment dated April 25, 1994 convicting petitioner of the crime charged and sentenced him to an "indeterminate penalty from 17 years, 4 months and 1 day of reclusion temporal as minimum, to 21 years of reclusion perpetua, as maximum".[11] Petitioner filed his notice of appeal on April 28, 1994. [12] Pending the appeal in the respondent Court of Appeals,[13] the Solicitor-General, convinced that the conviction shows strong evidence of guilt, filed on December 2, 1994 a motion to cancel petitioner's bail bond. The resolution of this motion was incorporated in the now assailed respondent court's decision sustaining petitioner's conviction, [14] the dispositive portion of which reads:

"WHEREFORE, the foregoing circumstances considered, the appealed decision is hereby AFFIRMED, and furthermore, the P200,000.00 bailbond posted by accused-appellant for his provisional liberty, FGU Insurance Corporation Bond No. JCR (2) 6523, is hereby cancelled.  The Regional Trial Court, Branch 61, Angeles City, is directed to issue the Order of Arrest of accused-appellant and thereafter his transmittal to the National Bureau of Prisons thru the Philippine National Police where the said accused-appellant shall remain under confinement pending resolution of his appeal, should he appeal to the Supreme Court.  This shall be immediately executory.  The Regional Trial Court is further directed to submit a report of compliance herewith.

SO ORDERED."[15]

Petitioner received a copy of this decision on July 26, 1995. [16] On August 9, 1995 he filed a "motion for reconsideration (and to recall the warrant of arrest)"[17] but the same was denied by respondent court in its September 20, 1995 Resolution,[18] copy of which was received by petitioner on September 27, 1995.  The next day, September 28, petitioner filed the instant petition for review on certiorari with application for bail[19] followed by two "supplemental petitions" filed by different counsels, [20] a "second supplemental petition"[21] and an urgent motion for the separate resolution of his  application for bail.  Again, the Solicitor-General[22] sought the denial of the application for bail, to which the Court agreed in a Resolution promulgated on July 31, 1996.[23] The Court also granted the Solicitor-General's motion to file a consolidated comment on the petitions and thereafter required the petitioner to file his reply. [24] However, after his vigorous resistance and success on the intramural of bail (both in the respondent court and this Court) and thorough exposition of petitioner's guilt in his 55-page Brief in the respondent court, the Solicitor-General now makes a complete turnabout by filing a "Manifestation In Lieu Of Comment" praying for petitioner's acquittal.[25]

The People's detailed narration of facts, well-supported by evidence on record and given credence by respondent court, is as follows:[26]

"At about 8:00 o'clock in the evening of October 26, 1992, Enrique Manarang and his compadre Danny Perez were inside the Manukan sa Highway Restaurant in Sto. Kristo, Angeles City where they took shelter from the heavy downpour (pp. 5-6, TSN, February 15, 1993) that had interrupted their ride on motorcycles (pp. 5-6, ibid.) along McArthur Highway (ibid). While inside the restaurant, Manarang noticed a vehicle, a Mitsubishi Pajero, running fast down the highway prompting him to remark that the vehicle might get into an accident considering the inclement weather.  (p. 7, Ibid)  In the local vernacular, he said thus: 'Ka bilis na, mumuran pa naman pota makaaksidente ya.' (p. 7, ibid).  True enough, immediately after the vehicle had passed the restaurant, Manarang and Perez heard a screeching sound produced by the sudden and hard

Page 189: Warrantless Arrests, Searches and Seizures and Privacy of Communication

189

braking of a vehicle running very fast (pp. 7-8, ibid) followed by a sickening sound of the vehicle hitting something (p. 8, ibid).  Danny Cruz, quite sure of what had happened, remarked 'oy ta na' signifying that Manarang had been right in his observation (pp. 8-9, ibid).

"Manarang and Cruz went out to investigate and immediately saw the vehicle occupying the edge or shoulder of the highway giving it a slight tilt to its side (pp. 9-10, ibid). Manarang, being a member of both the Spectrum, a civic group and the Barangay Disaster Coordinating Council, decided to report the incident to the Philippine National Police of Angeles City (p. 10, ibid).  He took out his radio and called the Viper, the radio controller of the Philippine National Police of Angeles City (p. 10, ibid).  By the time Manarang completed the call, the vehicle had started to leave the place of the accident taking the general direction to the north (p. 11, ibid).

"Manarang went to the location of the accident and found out that the vehicle had hit somebody (p. 11, ibid).

"He asked Cruz to look after the victim while he went back to the restaurant, rode on his motorcycle and chased the vehicle (p. 11 ibid).  During the chase he was able to make out the plate number of the vehicle as PMA 777 (p. 33, TSN, February 15, 1993).  He called the  Viper through the radio once again (p. 34, ibid) reporting that a vehicle heading north with plate number PMA 777 was involved in a hit and run accident (p. 20, TSN, June 8, 1993).  The Viper, in the person of SPO2 Ruby Buan, upon receipt of the second radio call flashed the message to all units of PNP Angeles City with the order to apprehend the vehicle (p. 20, ibid).  One of the units of the PNP Angeles City reached by the alarm was its Patrol Division at Jake Gonzales Street near the Traffic Division (pp. 5-7, TSN, February 23, 1993).  SPO2 Juan C. Borja III and SPO2 Emerlito Miranda immediately borded a mobile patrol vehicle (Mobile No. 3) and positioned themselves near the south approach of Abacan bridge since it was the only passable way going to the north (pp. 8-9, ibid). It took them about ten (10) seconds to cover the distance between their office and the Abacan bridge (p. 9, ibid).

"Another PNP mobile patrol vehicle that responded to the flash message from SPO2 Buan was Mobile No. 7 of the Pulongmaragal Detachment which was then conducting patrol along Don Juico Avenue (pp. 8-9, TSN, March 8, 1993).  On board were SPO Ruben Mercado and SPO3 Tan and SPO2 Odejar (p. 8, ibid). SPO Ruben Mercado immediately told SPO3 Tan to proceed to the MacArthur Highway to intercept the vehicle with plate number PMA 777 (p. 10, ibid).

"In the meantime, Manarang continued to chase the vehicle which figured in the hit and run incident, even passing through a flooded portion of the MacArthur Highway two (2) feet deep in front of the Iglesia ni Kristo church but he could not catch up with the same vehicle (pp. 11-12, February 15, 1993).  When he saw that the car he was chasing went towards Magalang, he proceeded to Abacan bridge because he knew Pulongmaragal was not passable (pp. 12-14, ibid).  When he reached the Abacan bridge, he found Mobile No. 3 and SPO2 Borja and SPO2 Miranda watching all vehicles coming their way (p. 10, TSN, February 23, 1993).  He approached them and informed them that there was a hit and run incident (p. 10, ibid).  Upon learning that the two police officers already knew about the incident, Manarang went back to where he came from (pp. 10-11;ibid).  When Manarang was in front of Tina's Restaurant, he saw the vehicle that had figured in the hit and run incident emerging from the corner adjoining Tina's Restaurant (p. 15, TSN, February 15, 1993).  He saw that the license plate hanging in front of the vehicle bore the identifying number PMA 777 and he followed it (p. 15, ibid) towards the Abacan bridge.

"Soon the vehicle was within sight of SPO2 Borja and SPO2 Miranda of Mobile No. 3 (p. 10, TSN, February 23, 1993).  When the vehicle was about twelve (12) meters away from their position, the two police officers boarded their Mobile car, switched on the engine, operated the siren and strobe light and drove out to intercept the vehicle (p. 11,ibid).  They cut into the path of the vehicle forcing it to stop (p. 11, ibid).

"SPO2 Borja and SPO2 Miranda alighted from Mobile No. 3 (P. 12, TSN, February 23, 1993).  SPO2 Miranda went to the vehicle with plate number PMA 777 and instructed its driver

Page 190: Warrantless Arrests, Searches and Seizures and Privacy of Communication

190

to alight (p. 12, ibid).  The driver rolled down the window and put his head out while raising both his hands.  They recognized the driver as Robin C. Padilla, appellant in this case (p. 13, ibid).  There was no one else with him inside the vehicle (p. 24).  At that moment, Borja noticed that Manarang arrived and stopped his motorcycle behind the vehicle of appellant (p. 14, ibid).  SPO2 Miranda told appellant to alight to which appellant complied.  Appellant was wearing a short leather jacket (p. 16, TSN, March 8, 1993) such that when he alighted with both his hands raised, a gun (Exhibit 'C') tucked on the left side of his waist was revealed (p. 15, TSN, February 23, 1993), its butt protruding (p. 15, ibid).  SPO2 Borja made the move to confiscate the gun but appellant held the former's hand alleging that the gun was covered by legal papers (p. 16, ibid).  SPO2 Borja, however, insisted that if the gun really was covered by legal papers, it would have to be shown in the office (p. 16, ibid).  After disarming appellant, SPO2 Borja told him about the hit and run incident which was angrily denied by appellant (p. 17, ibid).  By that time, a crowd had formed at the place (p. 19, ibid).  SPO2 Borja checked the cylinder of the gun and find six (6) live bullets inside (p. 20, ibid).

"While SPO2 Borja and appellant were arguing, Mobile No. 7 with SPO Ruben Mercado, SPO3 Tan and SPO2 Odejar on board arrived (pp. 11-12, TSN, March 8, 1993).  As the most senior police officer in the group, SPO Mercado took over the matter and informed appellant that he was being arrested for the hit and run incident (p. 13, ibid).  He pointed out to appellant the fact that the plate number of his vehicle was dangling and the railing and the hood were dented (p. 12, ibid).  Appellant, however, arrogantlydenied his misdeed and, instead, played with the crowd by holding their hands with one hand and pointing to SPO3 Borja with his right hand saying 'iyan, kinuha ang baril ko'(pp. 13-15, ibid).  Because appellant's jacket was short, his gesture exposed a long magazine of an armalite rifle tucked in appellant's back right pocket (p. 16, ibid).  SPOMercado saw this and so when appellant turned around as he was talking and proceeding to his vehicle, Mercado confiscated the magazine from appellant (pp. 16-17,ibid).  Suspecting that appellant could also be carrying a rifle inside the vehicle since he had a magazine, SPO2 Mercado prevented appellant from going back to his vehicle by opening himself the door of appellant's vehicle (16-17, ibid).  He saw a baby armalite rifle (Exhibit D) lying horizontally at the front by the driver's seat.  It had a long magazine filled with live bullets in a semi-automatic mode (pp. 17-21, ibid). He asked appellant for the papers covering the rifle and appellant answered angrily that they were at his home (pp. 26-27, ibid).  SPO Mercado modified the arrest of appellant by including as its ground illegal possession of firearms (p. 28, ibid).  SPO Mercado then read to appellant his constitutional rights (pp. 28-29, ibid).

"The police officers brought appellant to the Traffic Division at Jake Gonzales Boulevard (pp. 31-32, ibid) where appellant voluntarily surrendered a third firearm, a pietro berreta pistol (Exhibit 'L') with a single round in its chamber and a magazine (pp. 33-35, ibid) loaded with seven (7) other live bullets.  Appellant also voluntarily surrendered a black bag containing two additional long magazines and one short magazine (Exhibits M, N, and O, pp. 36-37, ibid).  After appellant had been interrogated by the Chief of the Traffic Division, he was transferred to the Police Investigation Division at Sto. Rosario Street beside the City Hall Building where he and the firearms and ammunitions were turned over to SPO2 Rene Jesus Gregorio (pp. 5-10, TSN, July 13, 1993).  During the investigation, appellant admitted possession of the firearms stating that he used them for shooting (p. 14, ibid).  He was not able to produce any permit to carry or memorandum receipt to cover the three firearms (pp. 16-18, TSN, January 25, 1994).

"On November 28, 1992, a certification (Exhibit 'F') was issued by Captain, Senior Inspector Mario Espino, PNP, Chief, Record Branch of the Firearms and Explosives Office (pp. 7-8, TSN, March 4, 1993).  The Certification stated that the three firearms confiscated from appellant, an M-16 Baby armalite rifle SN-RP 131280, a .357 caliber revolver Smith and Wesson SN 32919 and a .380 Pietro Beretta SN-A35720, were not registered in the name of Robin C. Padilla (p. 6, ibid).  A second Certification dated December 11, 1992 issued by Captain Espino stated that the three firearms were not also registered in the name of Robinhood C. Padilla (p. 10, ibid)."

Page 191: Warrantless Arrests, Searches and Seizures and Privacy of Communication

191

Petitioner's defenses are as follows: (1) that his arrest was illegal and consequently, the firearms and ammunitions taken in the course thereof are inadmissible in evidence under the exclusionary rule; (2) that he is a confidential agent authorized, under a Mission Order and Memorandum Receipt, to carry the subject firearms; and (3) that the penalty for simple illegal possession constitutes excessive and cruel punishment proscribed by the 1987 Constitution.

After a careful review of the records[27]of this case, the Court is convinced that petitioner's guilt of the crime charged stands on terra firma, notwithstanding the Solicitor-General's change of heart.

Anent the first defense, petitioner questions the legality of his  arrest.  There is no dispute that no warrant was issued for the arrest of petitioner, but that per sedid not make his apprehension at the Abacan bridge illegal.

Warrantless arrests are sanctioned in the following instances:[28]

"Sec. 5.  Arrest without warrant; when lawful. - A peace officer or a private person may, without a warrant, arrest a person:

(a)  When, in his presence, the person to be arrested has committed, is actually committing, or is attempting to commit an offense;

(b)  When an offense has in fact just been committed, and he has personal knowledge of facts indicating that the person to be arrested has committed it.

(c)  When the person to be arrested is a prisoner who has escaped from a penal establishment or place where he is serving final judgment or temporarily confined while his case is pending, or has escaped while being transferred from one confinement to another.

Paragraph (a) requires that the person be arrested (i) after he has committed or while he is actually committing or is at least attempting to commit an offense, (ii) in the presence of the arresting officer or private person.[29] Both elements concurred here, as it has been established that petitioner's vehicle figured in a hit and run - an offense committed in the "presence" of Manarang, a private person, who then sought to arrest petitioner.  It must be stressed at this point that "presence" does not only require that the arresting person sees the offense, but also when he "hears the disturbance created thereby AND proceeds at once to the scene."[30] As testified to by Manarang, he heard the screeching of tires followed by a thud, saw the sideswiped victim (balut vendor), reported the incident to the police and thereafter gave chase to the erring Pajero vehicle using his motorcycle in order to apprehend its driver.  After having sent a radio report to the PNP for assistance, Manarang proceeded to the Abacan bridge where he found responding policemen SPO2 Borja and SPO2 Miranda already positioned near the bridge who effected the actual arrest of petitioner.[31]

Petitioner would nonetheless insist on the illegality of his arrest by arguing that the policemen who actually arrested him were not at the scene of the hit and run. [32]We beg to disagree.  That Manarang decided to seek the aid of the policemen (who admittedly were nowhere in the vicinity of the hit and run) in effecting petitioner's arrest, did not in any way affect the propriety of the apprehension. It was in fact the most prudent action Manarang could have taken rather than collaring petitioner by himself, inasmuch as policemen are unquestionably better trained and well-equipped in effecting an arrest of a suspect (like herein petitioner) who , in all probability, could have put up a degree of resistance which an untrained civilian may not be able to contain without endangering his own life.  Moreover, it is a reality that curbing lawlessness gains more success when law enforcers function in collaboration with private citizens.   It is precisely through this cooperation, that the offense herein involved fortunately did not become an additional entry to the long list of unreported and unsolved crimes.

It is appropriate to state at this juncture that a suspect, like petitioner herein, cannot defeat the arrest which has been set in motion in a public place for want of a warrant as the police was confronted by an urgent need to render aid or take action.[33] The exigent circumstances of - hot pursuit,[34] a fleeing suspect, a moving vehicle, the public place and the raining nighttime - all created a situation in which speed is essential and delay improvident.[35] The Court acknowledges police authority to make the forcible stop since they had more than mere "reasonable and articulable" suspicion that the occupant of the

Page 192: Warrantless Arrests, Searches and Seizures and Privacy of Communication

192

vehicle has been engaged in criminal activity. [36] Moreover, when caught in flagrante delicto with possession of an unlicensed firearm (Smith & Wesson) and ammunition (M-16 magazine), petitioner's warrantless arrest was proper as he was again actually committing another offense (illegal possession of firearm and ammunitions) and this time in the presence of a peace officer.[37]

Besides, the policemen's warrantless arrest of petitioner could likewise be justified under paragraph (b) as he had in fact just committed an offense.  There was no supervening event or a considerable lapse of time between the hit and run and the actual apprehension.  Moreover, after having stationed themselves at the Abacan bridge in response to Manarang's report, the policemen saw for themselves the fast approaching Pajero of petitioner,[38] its dangling plate number (PMA 777 as reported by Manarang), and the dented hood and railings thereof. [39] These formed part of the arresting police officer's personal knowledge of the facts indicating that petitioner's Pajero was indeed the vehicle involved in the hit and run incident.  Verily then, the arresting police officers acted upon verified personal knowledge and not on unreliable hearsay information.[40]

Furthermore, in accordance with settled jurisprudence, any objection, defect or irregularity attending an arrest must be made before the accused enters his plea. [41]Petitioner's belated challenge thereto aside from his failure to quash the information, his participation in the trial and by presenting his evidence, placed him in estoppelto assail the legality of his arrest.[42] Likewise, by applying for bail, petitioner patently waived such irregularities and defects.[43]

We now go to the firearms and ammunitions seized from petitioner without a search warrant, the admissibility in evidence of which, we uphold.

The five (5) well-settled instances when a warrantless search and seizure of property is valid, [44] are as follows:

1.      warrantless search incidental to a lawful arrest recognized under Section 12, Rule 126 of the Rules of Court[45] and by prevailing jurisprudence[46],

2.      Seizure of evidence in "plain view", the elements of  which are:[47]

(a).        a prior valid intrusion based on the valid warrantless arrest in which the police are legally present in the pursuit of their official duties;

(b).        the evidence was inadvertently discovered by the police who had the right to be where they are;

(c).        the evidence must be immediately apparent, and

(d).        "plain view" justified mere seizure of evidence without further search.[48]

3.      search of a moving vehicle.[49] Highly regulated by the government, the vehicle's inherent mobility reduces expectation of privacy especially when its transit in public thoroughfares furnishes a highly reasonable suspicion amounting to probable cause that the occupant committed a criminal activity.[50]

4.      consented warrantless search, and

5.      customs search.

In conformity with respondent court's observation, it indeed appears that the authorities stumbled upon petitioner's firearms and ammunitions without even undertaking any active search which, as it is commonly understood, is a prying into hidden places for that which is concealed. [51] The seizure of the Smith & Wesson revolver and an M-16 rifle magazine was justified for they came within "plain view" of the policemen who inadvertently discovered the revolver and magazine tucked in petitioner's waist and back pocket respectively, when he raised his hands after alighting from his Pajero.  The same justification applies to the confiscation of the M-16 armalite rifle which was immediately apparent to the policemen as

Page 193: Warrantless Arrests, Searches and Seizures and Privacy of Communication

193

they took a casual glance at the Pajero and saw said rifle lying horizontally near the driver's seat. [52]Thus it has been held that:

"(W)hen in pursuing an illegal action or in the commission of a criminal offense, the . . . police officers should happen to discover a criminal offense being committed by any person, they are not precluded from performing their duties as police officers for the apprehension of the guilty person and the taking of the corpus delicti."[53]

"Objects whose possession are prohibited by law inadvertently found in plain view are subject to seizure even without a warrant."[54]

With respect to the Berreta pistol and a black bag containing assorted magazines, petitioner voluntarily surrendered them to the police.[55] This latter gesture of petitioner indicated a waiver of his right against the alleged search and seizure[56], and that his failure to quash the information estopped him from assailing any purported defect.[57]

Even assuming  that the firearms and ammunitions were products of an active search done by the authorities on the person and vehicle of petitioner, their seizure without a search warrant nonetheless can still be justified under a search  incidental to a lawful arrest (first instance).  Once the lawful arrest was effected, the police may undertake a protective search [58] of the passenger compartment and containers in the vehicle[59] which are within petitioner's grabbing distance regardless of the nature of the offense.[60] This satisfied the two-tiered test of an incidental search: (i) the item to be searched (vehicle) was within the arrestee's custody or area of immediate control [61] and (ii) the search was contemporaneous with the arrest.[62] The products of that search are admissible evidence not excluded by the exclusionary rule.  Another justification is a search of a moving vehicle (third instance).  In connection therewith, a warrantless search is constitutionally permissible when, as in this case, the officers conducting the search have reasonable or probable cause to believe, before the search, that either the motorist is a law-offender (like herein petitioner with respect to the hit and run) or the contents or cargo of the vehicle are or have been instruments or the subject matter or the proceeds of some criminal offense.[63]

Anent his second defense, petitioner contends that he could not be convicted of violating P.D. 1866 because he is an appointed civilian agent authorized to possess and carry the subject firearms and ammunition as evidenced by a Mission Order[64] and Memorandum Receipt duly issued by PNP Supt. Rodialo Gumtang, the deputy commander of Task Force Aguila, Lianga, Surigao del Sur.  The contention lacks merit.

In crimes involving illegal possession of firearm, two requisites must be established, viz.:  (1) the existence of the subject firearm and, (2) the fact that the accused who owned or possessed the firearm does not have the corresponding license or permit to possess. [65] The first element is beyond dispute as the subject firearms and ammunitions[66] were seized from petitioner's possession via a valid warrantless search, identified and offered in evidence during trial.  As to the second element, the same was convincingly proven by the prosecution.  Indeed, petitioner's purported Mission Order and Memorandum Receipt are inferior in the face of the more formidable evidence for the prosecution as our meticulous review of the records reveals that the Mission Order and Memorandum Receipt were mere afterthoughts contrived and issued under suspicious circumstances.  On this score, we lift from respondent court's incisive observation.  Thus:

"Appellant's contention is predicated on the assumption that the Memorandum Receipts and Mission Order were issued before the subject firearms were seized and confiscated from him by the police officers in Angeles City.  That is not so.  The evidence adduced indicate that the Memorandum Receipts and Mission Order were prepared and executed long after appellant had been apprehended on October 26, 1992.

"Appellant, when apprehended, could not show any document as proof of his authority to possess and carry the subject firearms.  During the preliminary investigation of the charge against him for illegal possession of firearms and ammunitions he could not, despite the ample time given him, present any proper document showing his authority.  If he had, in actuality, the Memorandum Receipts and Missions Order, he could have produced those documents easily, if not at the time of apprehension, at least during the preliminary investigation.  But neither

Page 194: Warrantless Arrests, Searches and Seizures and Privacy of Communication

194

appellant nor his counsel inform the prosecutor that appellant is authorized to possess and carry the subject firearms under Memorandum Receipt and Mission Order.  At the initial presentation of his evidence in court, appellant could have produced these documents to belie the charged against him.  Appellant did not.  He did not even take the witness stand to explain his possession of the subject firearms.

"Even in appellant's Demurrer to Evidence filed after the prosecution rested contain no allegation of a Memorandum Receipts and Mission Order authorizing appellant to possess and carry the subject firearms.

"At the initial presentation of appellant's evidence, the witness cited was one James Neneng to whom a subpoena was issued.  Superintendent Gumtang was not even mentioned.  James Neneng appeared in court but was not presented by the defense.  Subsequent hearings were reset until the defense found Superintendent Gumtang who appeared in court without subpoena on January 13, 1994."[67]

The Court is baffled why petitioner failed to produce and present the Mission Order and Memorandum Receipt if they were really issued and existing before his apprehension.  Petitioner's alternative excuses that the subject firearms were intended for theatrical purposes, or that they were owned by the Presidential Security Group, or that his Mission Order and Memorandum Receipt were left at home, further compound their irregularity.  As to be reasonably expected, an accused claiming innocence, like herein petitioner, would grab the earliest opportunity to present the Mission Order and Memorandum Receipt in question and save himself from the long and agonizing public trial and spare him from proffering inconsistent excuses.  In fact, the Mission Order itself, as well as the Letter-Directive of the AFP Chief of Staff, is explicit in providing that:

"VIII. c.  When a Mission Order is requested for verification by enforcement units/personnels such as PNP, Military Brigade and other Military Police Units of AFP, the Mission Order should be shown without resentment to avoid embarrassment and/or misunderstanding.

"IX.  d.  Implicit to this Mission Order is the injunction that the confidential instruction will be carried out through all legal means and do not cover an actuation in violation of laws.  In the latter event, this Mission Order is rendered inoperative in respect to such violation."[68]

which directive petitioner failed to heed without cogent explanation.

The authenticity and validity of the Mission Order and Memorandum Receipt, moreover, were ably controverted.  Witness for the prosecution Police Supt. Durendes denied under oath his signature on the dorsal side of the Mission Order and declared further that he did not authorize anyone to sign in his behalf.[69] His surname thereon, we note, was glaringly misspelled as "Durembes."[70] In addition, only Unit Commanders and Chief of Offices have the authority to issue Mission Orders and Memorandum Receipts under the Guidelines on the Issuance of MOs, MRs, & PCFORs.[71] PNP Supt. Rodialo Gumtang who issued petitioner's Mission Order and Memorandum Receipt is neither a Unit Commander nor the Chief of Office, but a mere deputy commander.  Having emanated from an unauthorized source, petitioner's Mission Order and Memorandum Receipt are infirm and lacking in force and effect.  Besides, the Mission Order covers "Recom 1-12-Baguio City,"[72] areas outside Supt. Gumtang's area of responsibility thereby needing prior approval "by next higher Headquarters" [73] which is absent in this case.  The Memorandum Receipt is also unsupported by a certification as required by the March 5, 1988 Memorandum of the Secretary of Defense which pertinently provides that:

"No memorandum receipt shall be issued for a CCS firearms without corresponding certification from the corresponding Responsible Supply Officer of the appropriate AFP unit that such firearm has been officially taken up in that units property book, and that report of such action has been reported to higher AFP authority."

Had petitioner's Memorandum Receipt been authentic, we see no reason why he cannot present the corresponding certification as well.

Page 195: Warrantless Arrests, Searches and Seizures and Privacy of Communication

195

What is even more peculiar is that petitioner's name, as certified to by the Director for Personnel of the PNP, does not even appear in the Plantilla of Non-Uniform Personnel or in the list of Civilian Agents or Employees of the PNP which could justify the issuance of a Mission Order, a fact admitted by petitioner's counsel.[74] The implementing rules of P.D. 1866 issued by the then PC-INP Chief and Director-General Lt. Gen. Fidel V. Ramos are clear and unambiguous, thus:

"No Mission Order shall be issued to any civilian agent authorizing the same to carry firearms outside residence unless he/she is included in the regular plantilla of the government agency  involved in law enforcement and is receiving regular compensation for the services he/she is rendering in the agency.  Further, the civilian agent must be included in a specific law enforcement/police/intelligence project proposal or special project which specifically required the use of firearms(s) to insure its accomplishment and that the project is duly approved at the PC Regional Command level or its equivalent level in other major services of the AFP, INP and NBI, or at higher levels of command."[75]

Circular No. 1, dated January 6, 1986, of the then Ministry of Justice likewise provides as follows:

"If mission orders are issued to civilians (not members of the uniformed service), they must be civilian agents included in the regular plantilla of the government agency involved in law enforcement and are receiving regular compensation for the service they are rendering."

That petitioner's Mission Order and Memorandum Receipt were fabricated pieces of evidence is accentuated all the more by the testimony and certification of the Chief of the Records Branch of the firearms and Explosives Office of the PNP declaring that petitioner's confiscated firearms are not licensed or registered in the name of the petitioner.[76] Thus:

"Q.  In all these files that you have just mentioned Mr. Witness, what did you find, if any?

"A.   I found that a certain Robin C. Padilla is a licensed registered  owner of one 9 mm pistol, Smith and  Wesson with Serial No. TCT 8214 and the following firearms being asked whether it is registered or not, I did not find any records, the M-16 and the caliber .357 and the caliber .380 but there is a firearm with the same serial number which is the same as that licensed and/or registered in the name of one Albert Villanueva Fallorina.

"Q.  So in short, the only licensed firearms in the name of accused Robin C. Padilla is a pistol, Smith and Wesson, caliber 9 mm with Serial No. TCT 8214?

"A.   Yes, sir.

"Q.  And the firearms that were the subject of this case are not listed in the names of the accused in this case?

"A.   Yes, sir.[77]

xxx                                                                        xxx                                                                               xxx

And the certification which provides as follows:

                                                          Republic of the Philippines                                           Department of the Interior and Local Government                          GENERAL HEADQUARTERS PHILIPPINE NATIONAL POLICE                                         FIREARMS AND EXPLOSIVES OFFICE

                                                   Camp Crame, Quezon City

"PNPFEO5                                                                                             28 November 1992

                                                    "C E R T I F I C A T I O N

Page 196: Warrantless Arrests, Searches and Seizures and Privacy of Communication

196

"TO WHOM IT MAY CONCERN:

"THIS IS TO CERTIFY  that Robin C. Padilla of 59 Labo St., Quezon City is a licensed/registered holder of Pistol Smith and Wesson Cal 9mm with serial number TCT8214 covered by License No. RL M76C4476687.

"Further certify that the following firearms are not registered with this Office per verification from available records on file this Office as of this date:

M16 Baby Armalite SN-RP131120Revolver Cal 357 SN-3219

Pistol Cal 380 Pietro Beretta SN-35723

"However,  we have on file one Pistol Cal 380, Beretta with serial number 35723Y, licensed/registered to one Albert Villanueva Fallorina of 29 San Juan St., Capitol Pasig, MM under Re-Registered License.

"This certification is issued pursuant to Subpoena from City of Angeles.

"FOR THE CHIEF,  FEO:

(Sgd.)

JOSE MARIO M. ESPINOSr. Inspector, PNP

Chief, Records Branch" [78]

In several occasions, the Court has ruled that either the testimony of a representative of, or a certification from, the PNP Firearms and Explosives Office (FEO) attesting that a person is not a licensee of any firearm would suffice to prove beyond reasonable doubt the second element of illegal possession of firearm.[79] In People vs. Tobias,[80] we reiterated that such certification is sufficient to show that a person has in fact no license.  From the foregoing discussion, the fact that petitioner does not have the license or permit to possess was overwhelmingly proven by the prosecution.  The certification may even be dispensed with in the light of the evidence [81]that an M-16 rifle and any short firearm higher than a .38 caliber pistol, akin to the confiscated firearms, cannot be licensed to a civilian, [82] as in the case of petitioner. The Court, therefore, entertains no doubt in affirming petitioner's conviction especially as we find no plausible reason, and none was presented, to depart from the factual findings of both the trial court and respondent court which, as a rule, are accorded by the Court with respect and finality. [83]

Anent his third defense, petitioner faults respondent court "in applying P.D. 1866 in a democratic ambience (sic) and a non-subversive context"  and adds that respondent court should have applied instead the previous laws on illegal possession of firearms since the reason for the penalty imposed under P.D. 1866 no longer exists.[84] He stresses that the penalty  of 17 years and 4 months to 21 years for simple illegal possession of firearm is cruel and excessive in contravention of the Constitution. [85]

The contentions do not merit serious consideration. The trial court and the respondent court are bound to apply the governing law at the time of appellant's commission of the offense for it is a rule that laws are repealed only by subsequent ones.[86] Indeed, it is the duty of judicial officers to respect and apply the law as it stands.[87] And until its repeal, respondent court can not be faulted for applying P.D. 1866 which abrogated the previous statutes adverted to by petitioner. 

Equally lacking in merit is appellant's allegation that the penalty for simple illegal possession is unconstitutional. The penalty for simple possession of firearm, it should be stressed, ranges from reclusion temporal maximum to reclusion perpetua contrary to appellant's erroneous averment.  The severity of a penalty does notipso facto make the same cruel and excessive.

Page 197: Warrantless Arrests, Searches and Seizures and Privacy of Communication

197

"It takes more than merely being harsh, excessive, out of proportion, or severe for a penalty to be obnoxious to the Constitution. 'The fact that the punishment authorized by the statute is severe does not make it cruel and unusual.'  (24 C.J.S., 1187-1188).  Expressed in other terms, it has been held that to come under the ban, the punishment must be  'flagrantly and plainly oppressive',  'wholly disproportionate  to the nature of the offense as to shock the moral sense of the community' "[88]

It is well-settled that as far as the constitutional prohibition goes, it is not so much the extent as the nature of the punishment that determines whether it is, or is not, cruel and unusual and that sentences of imprisonment, though perceived to be harsh, are not cruel or unusual if  within statutory limits.[89]

Moreover, every law has in its favor the presumption of constitutionality.  The burden of proving the invalidity of the statute in question lies with the appellant which burden, we note, was not convincingly discharged. To justify nullification of the law, there must be a clear and unequivocal breach of the Constitution, not a doubtful and argumentative implication, [90] as in this case.  In fact, the constitutionality of P.D. 1866 has been upheld twice by this Court. [91] Just recently, the Court declared that "the pertinent laws on illegal  possession of firearms [are not] contrary to any provision  of the Constitution. . ."[92] Appellant's grievance on the wisdom of the prescribed penalty should not be addressed to us.  Courts are not concerned with the wisdom, efficacy or morality of laws.  That question falls exclusively within the province of Congress  which enacts them and the Chief Executive who approves or vetoes them.  The only function of the courts, we reiterate,  is to interpret and apply the laws.

With respect  to the penalty imposed by the trial court as affirmed by respondent court (17 years 4 months and 1 day  of  reclusion temporal, as minimum, to 21 years of reclusion perpetua, as maximum), we reduce  the same in line with the fairly recent case of People v. Lian[93] where the Court en banc provided that the indeterminate penalty imposable for simple illegal possession of firearm, without any mitigating or aggravating circumstance, should be within the range of ten (10) years and one (1) day to twelve years (12) of prision mayor, as minimum, to eighteen (18) years, eight (8) months and one (1) day to twenty (20) of reclusion temporal, as maximum.  This is discernible from the following explanation by the Court:

"In the case at bar, no mitigating or aggravating circumstances have been alleged or proved, In accordance with the doctrine regarding special laws explained in People v. Simon,[94]although Presidential Decree No. 1866 is a special law, the penalties therein were taken from the Revised Penal Code, hence the rules in said Code for graduating by degrees or determining the proper period should be applied. Consequently, the penalty for the offense of simple illegal possession of firearm is the medium period of the complex penalty in said Section 1, that is, 18 years, 8 months and 1 day to 20 years.

"This penalty, being that which is to be actually imposed in accordance with the rules therefor and not merely imposable as a general prescription under the law, shall be the maximum of the range of the indeterminate sentence. The minimum thereof shall be taken, as aforesaid, from any period of the penalty next lower in degree, which is, prision mayor in its maximum period to reclusion temporal in its medium period.[95]

WHEREFORE, premises considered, the decision of the Court of Appeals sustaining petitioner's conviction by the lower court of the crime of simple illegal possession of firearms and ammunitions is AFFIRMED EXCEPT that petitioner's indeterminate penalty is MODIFIED to ten (10) years and one (1) day, as minimum, to eighteen (18) years, eight (8) months and one (1) day, as maximum.

SO ORDERED

Narvasa, C.J. (Chairman), Davide, Jr., Melo, and Panganiban, concur.

Page 198: Warrantless Arrests, Searches and Seizures and Privacy of Communication

198

[G.R. No. 114791.  May 29, 1997]

NANCY GO AND ALEX GO, petitioners, vs. THE HONORABLE COURT OF APPEALS, HERMOGENES ONG and JANE C. ONG,respondents.

D E C I S I O N

ROMERO, J.:

No less than the Constitution commands us to protect marriage as an inviolable social institution and the foundation of the family.[1] In our society, the importance of a wedding ceremony cannot be underestimated as it is the matrix of the family and, therefore, an occasion worth reliving in the succeeding years.

It is in this light that we narrate the following undisputed facts:

Private respondents spouses Hermogenes and Jane Ong were married on June 7, 1981, in Dumaguete City.  The video coverage of the wedding was provided by petitioners at a contract price of P1,650.00. Three times thereafter, the newlyweds tried to claim the video tape of their wedding, which they planned to show to their relatives in the United States where they were to spend their honeymoon,

Page 199: Warrantless Arrests, Searches and Seizures and Privacy of Communication

199

and thrice they failed because the tape was apparently not yet processed. The parties then agreed that the tape would be ready upon private respondents’ return.

When private respondents came home from their honeymoon, however, they found out that the tape had been erased by petitioners and therefore, could no longer be delivered.

Furious at the loss of the tape which was supposed to be the only record of their wedding, private respondents filed on September 23, 1981 a complaint for specific performance and damages against petitioners before the Regional Trial Court, 7th Judicial District, Branch 33, Dumaguete City.  After a protracted trial, the court a quorendered a decision, to wit:

“WHEREFORE, judgment is hereby granted:

1.       Ordering the rescission of the agreement entered into between plaintiff Hermogenes Ong and defendant Nancy Go;

2.       Declaring defendants Alex Go and Nancy Go jointly and severally liable to plaintiffs Hermogenes Ong and Jane C. Ong for the following sums:

a)           P450.00, the down payment made at contract time;

b)           P75,000.00, as moral damages;

c)           P20,000.00, as exemplary damages;

d)           P5,000.00, as attorney’s fees; and

e)           P2,000.00, as litigation expenses;

Defendants are also ordered to pay the costs.

SO ORDERED.”

Dissatisfied with the decision, petitioners elevated the case to the Court of Appeals which, on September 14, 1993, dismissed the appeal and affirmed the trial court’s decision.

Hence, this petition.

Petitioners contend that the Court of Appeals erred in not appreciating the evidence they presented to prove that they acted only as agents of a certain Pablo Lim and, as such, should not have been held liable.  In addition, they aver that there is no evidence to show that the erasure of the tape was done in bad faith so as to justify the award of damages.[2]

The petition is not meritorious.

Petitioners claim that for the video coverage, the cameraman was employed by Pablo Lim who also owned the video equipment used. They further assert that they merely get a commission for all customers solicited for their principal.[3]

This contention is primarily premised on Article 1883 of the Civil Code which states thus:

“ART. 1883.  If an agent acts in his own name, the principal has no right of action against the persons with whom the agent has contracted; neither have such persons against the principal.

In such case the agent is the one directly bound in favor of the person with whom he has contracted, as if the transaction were his own, except when the contract involves things belonging to the principal.

Page 200: Warrantless Arrests, Searches and Seizures and Privacy of Communication

200

xxx                                                                        xxx                                                                               xxx”

Petitioners’ argument that since the video equipment used belonged to Lim and thus the contract was actually entered into between private respondents and Lim is not deserving of any serious consideration.  In the instant case, the contract entered into is one of service, that is, for the video coverage of the wedding. Consequently, it can hardly be said that the object of the contract was the video equipment used.  The use by petitioners of the video equipment of another person is of no consequence.

It must also be noted that in the course of the protracted trial below, petitioners did not even present Lim to corroborate their contention that they were mere agents of the latter.  It would not be unwarranted to assume that their failure to present such a vital witness would have had an adverse result on the case.[4]

As regards the award of damages, petitioners would impress upon this Court their lack of malice or fraudulent intent in the erasure of the tape.  They insist that since private respondents did not claim the tape after the lapse of thirty days, as agreed upon in their contract, the erasure was done in consonance with consistent business practice to minimize losses.[5]

We are not persuaded.

As correctly observed by the Court of Appeals, it is contrary to human nature for any newlywed couple to neglect to claim the video coverage of their wedding; the fact that private respondents filed a case against petitioners belies such assertion.  Clearly, petitioners are guilty of actionable delay for having failed to process the video tape.  Considering that private respondents were about to leave for the United States, they took care to inform petitioners that they would just claim the tape upon their return two months later.  Thus, the erasure of the tape after the lapse of thirty days was unjustified.

In this regard, Article 1170 of the Civil Code provides that “those who in the performance of their obligations are guilty of fraud, negligence or delay, and those who is any manner contravene the tenor thereof, are liable for damages.”

In the instant case, petitioners and private respondents entered into a contract whereby, for a fee, the former undertook to cover the latter’s wedding and deliver to them a video copy of said event.   For whatever reason, petitioners failed to provide private respondents with their tape. Clearly, petitioners are guilty of contravening their obligation to said private respondents and are thus liable for damages.

The grant of actual or compensatory damages in the amount of P450.00 is justified, as reimbursement of the downpayment paid by private respondents to petitioners. [6]

Generally, moral damages cannot be recovered in an action for breach of contract because this case is not among those enumerated in Article 2219 of the Civil Code.  However, it is also accepted in this jurisdiction that liability for a quasi-delict may still exist despite the presence of contractual relations, that is, the act which violates the contract may also constitute a quasi-delict.[7] Consequently, moral damages are recoverable for the breach of contract which was palpably wanton, reckless, malicious or in bad faith, oppresive or abusive.[8]

Petitioners’ act or omission in recklessly erasing the video coverage of private respondents’ wedding was precisely the cause of the suffering private respondents had to undergo.

As the appellate court aptly observed:

“Considering the sentimental value of the tapes and the fact that the event therein recorded — a wedding which in our culture is a significant milestone to be cherished and remembered — could no longer be reenacted and was lost forever, the trial court was correct in awarding the appellees moral damages albeit in the amount of P75,000.00, which was a great reduction from plaintiffs’ demand in the complaint, in compensation for the mental anguish, tortured feelings, sleepless nights and humiliation that the appellees suffered and which under the circumstances could be awarded as allowed under Articles 2217 and 2218 of the Civil Code.”[9]

Page 201: Warrantless Arrests, Searches and Seizures and Privacy of Communication

201

Considering the attendant wanton negligence committed by petitioners in the case at bar, the award of exemplary damages by the trial court is justified [10] to serve as a warning to all entities engaged in the same business to observe due diligence in the conduct of their affairs.

The award of attorney’s fees and litigation expenses are likewise proper, consistent with Article 2208[11] of the Civil Code.

Finally, petitioner Alex Go questions the finding of the trial and appellate courts holding him jointly and severally liable with his wife Nancy regarding the pecuniary liabilities imposed.  He argues that when his wife entered into the contract with private respondent, she was acting alone for her sole interest. [12]

We find merit in this contention.  Under Article 117 of the Civil Code (now Article 73 of the Family Code), the wife may exercise any profession, occupation or engage in business without the consent of the husband.  In the instant case, we are convinced that it was only petitioner Nancy Go who entered into the contract with private respondent. Consequently, we rule that she is solely liable to private respondents for the damages awarded below, pursuant to the principle that contracts produce effect only as between the parties who execute them.[13]

WHEREFORE, the assailed decision dated September 14, 1993 is hereby AFFIRMED with the MODIFICATION that petitioner Alex Go is absolved from any liability to private respondents and that petitioner Nancy Go is solely liable to said private respondents for the judgment award.  Costs against petitioners.

SO ORDERED.

Regalado, (Chairman), Puno, Mendoza, and Torres, Jr., JJ., concur.

G.R. No. L-63630 April 6, 1990

PEOPLE OF THE PHILIPPINES, plaintiff-appellee, vs.MEDEL TANGLIBEN Y BERNARDINO, defendant-appellant.

The Office of the Solicitor General for plaintiff-appellee.

Katz N. Tierra for defendant-appellant.

 

GUTIERREZ, JR., J.:

This is an appeal from the decision of the Regional Trial Court, Branch 41, Third Judicial Region at San Fernando, Pampanga, Branch 41, finding appellant Medel Tangliben y Bernardino guilty beyond reasonable doubt of violating Section 4, Article II of Republic Act 6425 (Dangerous Drugs Act of 1972 as amended) and sentencing him to life imprisonment, to pay a fine of P20,000 and to pay the costs.

The information filed against the appellant alleged:

Page 202: Warrantless Arrests, Searches and Seizures and Privacy of Communication

202

That on or about the 2nd day of March, 1982, in the municipality of San Fernando, Province of Pampanga, Philippines, and within the jurisdiction of this Honorable Court, the above-named accused MEDEL TANGLIBEN y BERNARDINO, knowing fully well that Marijuana is a prohibited drug, did then and there willfully, unlawfully and feloniously have his possession, control and custody one (1) bag of dried marijuana leaves with an approximate weight of one (1) kilo and to transport (sic) the same to Olongapo City, without authority of law to do so. (At p. 6, Rollo)

The prosecution's evidence upon which the finding of guilt beyond reasonable doubt was based is narrated by the trial court as follows:

It appears from the evidence presented by the prosecution that in the late evening of March 2, 1982, Patrolmen Silverio Quevedo and Romeo L. Punzalan of the San Fernando Police Station, together with Barangay Tanod Macario Sacdalan, were conducting surveillance mission at the Victory Liner Terminal compound located at Barangay San Nicolas, San Fernando, Pampanga; that the surveillance was aimed not only against persons who may commit misdemeanors at the said place but also on persons who may be engaging in the traffic of dangerous drugs based on informations supplied by informers; that it was around 9:30 in the evening that said Patrolmen noticed a person caring a traveling bag (Exhibit G) who was acting suspiciously and they confronted him; that the person was requested by Patrolmen Quevedo and Punzalan to open the red traveling bag but the person refused, only to accede later on when the patrolmen identified themselves; that found inside the bag were marijuana leaves (Exhibit B) wrapped in a plastic wrapper and weighing one kilo, more or less; that the person was asked of his name and the reason why he was at the said place and he gave his name as Medel Tangliben and explained that he was waiting for a ride to Olongapo City to deliver the marijuana leaves; that the accused was taken to the police headquarters at San Fernando, Pampanga, for further investigation; and that Pat. Silverio Quevedo submitted to his Station Commander his Investigator's Report (Exhibit F).

It appears also from the prosecution's evidence that in the following morning or on March 3, 1982, Pat. Silverio Quevedo asked his co-policeman Pat. Roberto Quevedo, who happens to be his brother and who has had special training on narcotics, to conduct a field test on a little portion of the marijuana leaves and to have the remaining portion examined by the PCCL at Camp Olivas, San Fernando, Pampanga; that Pat. Roberto Quevedo conducted a field test (Exhibit H) on the marijuana leaves and found positive result for marijuana (Exhibit E); that the remaining bigger quantity of the marijuana leaves were taken to the PCCL at Camp Olivas by Pat. Roberto Quevedo that same day of March 3, 1982 (Exhibit A and A-1) and when examined, the same were also found to be marijuana (Exhibit C and C-1). (At pp. 9-10, Rollo)

Only the accused testified in his defense. His testimony is narrated by the trial court as follows:

The accused declared that he got married on October 25, 1981 and his wife begot a child on June 10, 1982; that he was formerly employed in the poultry farm of his uncle Alejandro Caluma in Antipolo, Rizal; that he is engaged in the business of selling poultry medicine and feeds, including chicks, and used to conduct his business at Taytay, Rizal; that he goes to Subic at times in connection with his business and whenever he is in Subic, he used to buy C-rations from one Nena Ballon and dispose the same in Manila; that he never left his residence at Antipolo, Rizal, on March 2, 1982; that on March 3, 1982, he went to Subic to collect a balance of P100.00 from a customer thereat and to buy C-rations; that he was able to meet Nena Ballon at 6:00 o'clock in the evening and he stayed in Nena's house up to 8:00 o'clock because he had a drinking spree with Nena's son; that he tried to catch the 8:00 o'clock trip to Manila from Olongapo City but he failed and was able to take the bus only by 9:00 o'clock that evening that it was a Victory Liner

Page 203: Warrantless Arrests, Searches and Seizures and Privacy of Communication

203

Bus that he rode and because he was tipsy, he did not notice that the bus was only bound for San Fernando, Pampanga; that upon alighting at the Victory Liner Compound at San Fernando, Pampanga he crossed the street to wait for a bus going to Manila; that while thus waiting for a bus, a man whom he came to know later as Pat. Punzalan, approached him and asked him if he has any residence certificate; that when he took out his wallet, Pat. Punzalan got the wallet and took all the money inside the wallet amounting to P545.00; that Pat. Punzalan told him that he'll be taken to the municipal building for verification as he may be an NPA member; that at the municipal building, he saw a policeman, identified by him later as Pat. Silverio Quevedo, sleeping but was awakened when he arrived that Pat. Quevedo took him upstairs and told him to take out everything from his pocket saying that the prisoners inside the jail may get the same from him; that inside his pocket was a fifty-peso bill and Pat. Quevedo took the same, telling him that it shall be returned to him but that it was never returned to him; that he was thereafter placed under detention and somebody told him that he is being charged with possession of marijuana and if he would like to be bailed out, somebody is willing to help him; and, that when he was visited by his wife, he told his wife that Patrolman Silverio Quevedo took away all his money but he told his wife not to complain anymore as it would be useless. (Rollo, pp. 10-11)

Appellant, through counsel de oficio Atty. Enrique Chan, raised the lone assignment of error in his appeal:

THE COURT A QUO ERRED IN CONVICTING THE ACCUSED-APPELLANT AND FINDING HIM GUILTY OF THE CRIME CHARGED ON INSUFFICIENT AND DOUBTFUL EVIDENCE. (At p. 48,Rollo)

The Solicitor-General likewise filed his brief, basically reiterating ating the lower court's findings.

However, before this Court had the chance to act on appeal, counsel de oficio Atty. Enrique Chan died. Thereafter, this court appointed a new counsel de oficio, Atty. Katz Tierra and pursuant thereto, the Deputy Clerk of Court, in behalf of the Clerk of Court, required the new counsel to file her appellant's brief. The latter complied and, in her brief, raised the following assignment of errors:

I

THE LOWER COURT ERRED IN ADMITTING AS EVIDENCE THE PACKAGE OF MARIJUANA ALLEGEDLY SEIZED FROM DEFENDANT-APPELLANT AS IT WAS A PRODUCT OF AN UNLAWFUL SEARCH WITHOUT A WARRANT.

II

THE LOWER COURT ERRED IN ADMITTING AS EVIDENCE THE ALLEGED PACKAGE OF MARIJUANA LEAVES AS THE LEAVES SUPPOSEDLY SEIZED FROM ACCUSED WHEN IT WAS NEVER AUTHENTICATED.

III

THE LOWER COURT ERRED IN NOT RULING THAT THE PROSECUTION FAILED TO PROVE THE GUILT OF DEFENDANT-APPELLANT. (At pp. 92-93, Rollo)

It is contended that the marijuana allegedly seized from the accused was a product of an unlawful search without a warrant and is therefore inadmissible in evidence.

This contention is devoid of merit.

Page 204: Warrantless Arrests, Searches and Seizures and Privacy of Communication

204

One of the exceptions to the general rule requiring a search warrant is a search incident to a lawful arrest. Thus, Section 12 of Rule 126 of the 1985 Rules on Criminal Procedure provides:

Section 12. Search incident to a lawful arrest. A person lawfully arrested may be searched for dangerous weapons or anything which may be used as proof of the commission of an offense, without a search warrant.

Meanwhile, Rule 113, Sec. 5(a) provides:

. . . A peace officer or a private person may, without a warrant, arrest a person:

(a) When, in his presence, the person to be arrested has committed, is actually committing, or is attempting to commit an offense.

Accused was caught in flagrante, since he was carrying marijuana at the time of his arrest. This case therefore falls squarely within the exception. The warrantless search was incident to a lawful arrest and is consequently valid.

In the case of People v. Claudia, 160 SCRA 646, [1988] this Court, confronted with the same issue, held that:

Appellant Claudio was caught transporting prohibited drugs. Pat. Daniel did not need a warrant to arrest Claudio as the latter was caught in flagrante delicto. The warrantless search being an incident to a lawful arrest is in itself lawful. (Nolasco V. Paño, 147 SCRA 509). Therefore, there was no infirmity in the seizure of the 1.1 kilos of marijuana.

We are not unmindful of the decision of this Court in People v. Amininudin, 163 SCRA 402 [1988]. In that case the PC officers had earlier received a tip from an informer that accused-appellant. was on board a vessel bound for Iloilo City and was carrying marijuana. Acting on this tip, they waited for him one evening, approached him as he descended from the gangplank, detained him and inspected the bag he was carrying. Said bag contained marijuana leaves. The Court held that the marijuana could not be admitted in evidence since it was seized illegally. The records show, however, that there were certain facts, not sing in the case before us, which led the Court to declare the seizure as invalid. As stated therein:

The present case presented no such urgency From the conflicting declarations of the PC witnesses, it is clear that they had at react two days within which they could have obtained a warrant of arrest and search Aminnudin who was coming to Iloilo on the M/V Wilcon 9. His name was known. The vehicle was identified. The date of its arrival was certain. And from the information they had received, they could have persuaded a judge that there was probable cause, indeed, to justify the issuance of a warrant. Yet they did nothing. No effort was made to comply with the law. The Bill of Rights was ignored altogether because the PC lieutenant who was the head of the arresting team, had determined on his own authority that a "search warrant was not necessary."

In contrast, the case before us presented urgency. Although the trial court's decision did not mention it, the transcript of stenographic notes reveals that there was an informer who pointed to the accused-appellant as carrying marijuana. (TSN, pp. 52-53) Faced with such on-the-spot information, the police officers had to act quickly. There was not enough time to secure a search warrant. We cannot therefore apply the ruling inAminnudin to the case at bar. To require search warrants during on-the-spot apprehensions of drug pushers, illegal possessors of firearms, jueteng collectors, smugglers of contraband goods, robbers, etc. would make it extremely difficult, if not impossible to contain the crimes with which these persons are associated.

Page 205: Warrantless Arrests, Searches and Seizures and Privacy of Communication

205

Accused-appellant likewise asserts that the package of marijuana leaves supposedly seized from him was never authenticated and therefore should not have been admitted as evidence. He capitalizes on the fact that the marijuana package brought by patrolman Roberto Quevedo to the PC Crime Laboratory for examination did not contain a tag bearing the name of the accused. We rule, however, that since Patrolman Quevedo testified that he gave the marijuana package together with a letter-request for examination, and the forensic chemist Marilene Salangad likewise testified that she received the marijuana together with the letter-request and said letter-request bore the name of the accused, then the requirements of proper authentication of evidence were sufficiently complied with. The marijuana package examined by the forensic checklist was satisfactorily identified as the one seized from accused.

Even assuming arguendo that the marijuana sent to the PC Crime Laboratory was not properly authenticated, still, we cannot discount the separate field test conducted by witness Roberto Quevedo which yielded positive results for marijuana.

Lastly, the appellant claims that the evidence upon which he was convicted was insufficient and doubtful and that the prosecution failed to prove his guilt.

In attacking the sufficiency of evidence, the appellant avers that the informer should have been presented before the lower court. We discard this argument as a futile attempt to revive an already settled issue. This Court has ruled in several cases that non-presentation of the informer, where his testimony would be merely corroborative or cumulative, is not fatal to the prosecution's case. (People v. Asio, G.R. No. 84960, September 1, 1989; (People v. Viola, G.R. No. 64262, March 16, 1989; People v. Capulong, 160 SCRA 533 [1988]; People v. Cerelegia, 147 SCRA 538).

As to doubtfulness of evidence, well-settled is the rule that findings of the trial court on the issue of credibility of witnesses and their testimonies are entitled to great respect and accorded the highest consideration by the appellate court. Since credibility is a matter that is peculiarly within the province of the trial judge, who had first hand opportunity to watch and observe the demeanor and behavior of witnesses both for the prosecution and the defense at the time of their testimony (People v. Tejada, G.R. No. 81520, February 21, 1989; People v. Turla, 167 SCRA 278), we find no reason to disturb the following findings:

The testimony of prosecution witnesses Patrolmen Silverio Quevedo and Romeo Punzalan are positive and sufficiently clean to show the commission by the accused of the offense herein chatted. These prosecution witnesses have no motive to fabricate the facts and to foist a very serious offense against the accused. The knowledge on what these witnesses testified to were (sic) acquired by them in the official performance of their duties and then, (sic) being no showing that they are prejudiced against the accused, their testimonies deserve full credit.

The testimonies of the afore-mentioned petitioner that what they found in the possession of the accused were marijuana leaves were corroborated by the examination findings conducted by Pat. October to Salangad of the PCCL, with station at camp Olivas, San Fernando, Pampanga (Exhibits C and C-1). (Rollo, p. 11)

Moreover, if there is truth in the testimony of the accused to the effect that Pat. Punzalan got all the money from his wallet when he was accosted at the Victory Liner Terminal and was told just to keep quiet otherwise he will be "salvaged" why will Pat. Punzalan still bring the accused to the municipal Building for interrogation and/or verification? Would not Pat. Punzalan be exposing his identity to the accused? This is unnatural. And this is also true on the testimony to the accused that Pat. Silverio Quevedo got his fifty-peso bill arid never returned the same to him. If the policemen really got any money from the accused and that the marijuana leaves do not belong to the accused, why will the two policemen still produce in Court as evidence that expensive-looking traveling red bag

Page 206: Warrantless Arrests, Searches and Seizures and Privacy of Communication

206

(Exhibit G) taken from the accused and which contained the marijuana in question if the instant case is a mere fabrication?

As already stated, all the evidence, oral and documentary, presented by the prosecution in this case were all based on personal knowledge acquired by the prosecution witnesses in the regular performance of their official duties and there is nothing in their testimonies to show that they are bias (sic) or that they have any prejudice against the herein accused. Between the testimonies of these prosecution witnesses and that of the uncorroborated and self-serving testimony of the accused, the former should prevail. (Rollo, p. 13)

Likewise, the appellant chose to limit his defense to his own testimony. He could have availed himself through compulsory court processes of several witnesses to buttress his defense. Since not one other witness was presented nor was any justification for the non-appearance given, the inadequacy of his lone and uncorroborated testimony remains. It cannot prevail vis-a-vis the positive testimonies given by the prosecution witnesses.

Moreover, the appellant's having jumped bail is akin to flight which, as correctly observed by the lower court, is an added circumstance tending to establish his guilt.

We take exception, however, to the trial court's finding that:

The dried marijuana leaves found in the possession of the accused weighs one (1) kilo, more or less. The intent to transport the same is clear from the testimony of Pat. Silverio Quevedo who declared, among other things, that when he confronted the accused that night, the latter told him that he (accused) is bringing the marijuana leaves to Olongapo City. Moreover, considering the quantity of the marijuana leaves found in the possession of the accused and the place he was arrested which is at San Fernando, Pampanga, a place where the accused is not residing, it can be said that the intent to transport the marijuana leaves has been clearly established. (Rollo, pp. 13-14)

The alleged extrajudicial confession of the accused which, on the other hand, he categorically denied in court, that he is transporting the marijuana leaves to Olongapo City cannot be relied upon. Even assuming it to be true, the extrajudicial confession cannot be admitted because it does not appear in the records that the accused, during custodial investigation, was apprised of his rights to remain silent and to counsel and to be informed of such rights. In People v. Duero 104 SCRA 379 [1981], the Court pronounced that "inasmuch as the prosecution failed to prove that before Duero made his alleged oral confession he was informed of his rights to remain silent and to have counsel and because there is no proof that he knowingly and intelligently waived those rights, his confession is inadmissible in evidence. This ruling was reiterated in People v. Tolentino, 145 SCRA 597 [1986], where the Court added that:

In effect, the Court not only abrogated the rule on presumption of regularity of official acts relative to admissibility of statements taken during in-custody interrogation but likewise dispelled any doubt as to the full adoption of the Miranda doctrine in this jurisdiction. It is now incumbent upon the prosecution to prove during a trial that prior to questioning, the confessant was warned of his constitutionally protected rights.

The trial judge likewise found the marijuana to weigh one kilo, more or less, and from this finding extracted a clear intent to transport the marijuana leaves. It may be pointed out, however, that although the information stated the weight to be approximately one kilo, the forensic chemist who examined the marijuana leaves testified that the marijuana weighed only 600 grams Such amount is not a considerable quantity as to conclusively confer upon the accused an intent to transport the marijuana leaves.

Page 207: Warrantless Arrests, Searches and Seizures and Privacy of Communication

207

Nor can it be said that the intent to transport is clearly established from the fact that the accused was arrested at San Fernando, Pampanga, a place which is not his residence. Conviction of a crime with an extremely severe penalty must be based on evidence which is clearer and more convincing than the inferences in this case.

What was therefore proved beyond reasonable doubt is not his intent to transport the marijuana leaves but his actual session.

The offense committed by the appellant is possession of marijuana under Section 8 of Republic Act No. 6425 (Dangerous Drugs Act of 1972 as amended).

WHEREFORE, the judgment of conviction by the trial court is hereby AFFIRMED but MODIFIED. The appellant is sentenced to suffer the penalty of imprisonment ranging from six (6) years and one (1) day to twelve (12) years and fine of Six Thousand (P6,000.00) Pesos.

SO ORDERED.

Fernan, C.J., Feliciano, Bidin and Cortes, JJ., concur.

G.R. No. L-69803 October 8, 1985

CYNTHIA D. NOLASCO, MILA AGUILAR-ROQUE and WILLIE C. TOLENTINO, petitioners, vs.HON. ERNANI CRUZ PAÑO, Executive Judge, Regional Trial Court of Quezon City; HON. ANTONIO P. SANTOS, Presiding Judge, Branch XLII, Metropolitan Trial Court of Quezon City: HON. SERGIO F. APOSTOL, City Fiscal, Quezon City; HON. JUAN PONCE ENRILE, LT. GEN. FIDEL RAMOS and COL. JESUS ALTUNA, respondents.

Jose W .Diokno, Joker P. Arroyo, Rene A. V. Sarmiento, Dan Malabonga and Cesar Maravilla for petitioners.

 

MELENCIO-HERRERA, J.:

The facts before the Court in these Certiorari, Prohibition, and mandamus proceedings will be briefly stated. The three petitioners will be referred to through their surnames of NOLASCO, AGUILAR-ROQUE and TOLENTINO.

1. Prior to August 6, 1984 (hereinafter to be referred to without the year), AGUILAR-ROQUE was one of the accused of Rebellion in Criminal Case No. MC-25-113 of Military Commission No. 25, both cases being entitled "People of the Philippines vs. Jose Ma. Sison, et al." She was then still at large.

2. At 11:30 A.M. on August 6th, AGUILAR-ROQUE and NOLASCO were arrested by a Constabulary Security Group (CSG) at the intersection of Mayon Street and P. Margall Street, Quezon City. The stated time is an allegation of petitioners, not denied by respondents. The record does not disclose that a warrant of arrest had previously beeen issued against NOLASCO.

3. At 12:00 N. on August 6th, elements of the CSG searched the premises at 239-B Mayon Street, Quezon City. The stated time is an allegation of petitioners, not specifically denied by respondents. In their COMMENT, however, respondents have alleged that the search was conducted "late on the same day"; that is late on august 6th.

Page 208: Warrantless Arrests, Searches and Seizures and Privacy of Communication

208

4. On August 6th, at around 9:00 A.M., Lt. Col. Virgilio G. Saldajeno of the CSG, applied for a Search Warrant from respondent Hon. Ernani Cruz Paño, Executive Judge of the Regional Trial Court in Quezon City, to be served at No. 239-B Mayon Street, Quezon City, determined tyo be the leased residence of AGUILAR-ROQUE, after almost a month of "round the clock surveillance" of the premises as a "suspected underground house of the CPP/NPA." AGUILAR-ROQUE has been long wanted by the military for being a high ranking officer of the Communist Party of the Philippines, particularly connected with the MV Karagatan/Doña Andrea cases.

In connection with the Search Warrant issued, the following may be stated:

(a) The Search Warrant was issued in proceedings entitled "People of the Philippines vs. Mila Aguilar-Roque, Accused, Search Warrant No. 80- 84 for rebellion" (the SEARCH WARRANT CASE). Judge Panos Court was Branch 88.

(b) It does not appear from the records before us that an application in writing was submitted by Lt. Col. Saldajeno to Judge Paño.

(c) According to the record, Lt. Col. Saldajeno and his witness S/A Dionicio A. Lapus, were examined under oath by Judge Paño but only the deposition of S/A Lapus has been submitted to us. The latter deposed that to his personal knowledge, there were kept in the premises to be searched records, documents and other papers of the CPP/NPA and the National Democratic Front, including support money from foreign and local sources intended to be used for rebellion. 1

5. In connection with the search made at 12:00 N. of August 6th the following may be stated:

(a) TOLENTINO was a person then in charge of the premises. He was arrested by the searching party presumably without a warrant of arrest.

(b) The searching party seized 428 documents and written materials, 2 and additionally a portable typewriter, and 2 wooden boxes, making 431 items in all. 3

(c) According to the Return, submitted in the SEARCH WARRANT CASE on August 10th, 4 the search was made in the presence of Dra. Marciana Galang, owner of the premises, and of two (2) Barangay Tanods. No mention was made that TOLENTINO was present. The list of the 428 articles and documents attached to the Return was signed by the two Barangay Tanods, but not by Dra. Galang.

6. (a) On August 10th, the three petitioners, AGUILAR-ROQUE, NOLASCO and TOLENTINO, were charged before the Quezon City Fiscal's Office (the CITY FISCAL, for short) upon complaint filed by the CSG against petitioners for "Subversion/Rebellion and/or Conspiracy to Commit Rebellion/Subversion."

(b) On August 13th, the CITY FISCAL filed an Information for Violation of Presidential Decree No. 33 (Illegal Possession of Subversive Documents) against petitioners before Branch 42 of the Metropolitan Trial Court of Quezon City (the SUBVERSIVE DOCUMENTS CASE), respondent Judge Antonio P. Santos, presiding.

(c) On August 16th, CSG filed a Motion for Reconsideration with the CITY FISCAL, praying that AGUILAR-ROQUE and NOLASCO be charged with Subversion. The Motion was denied on November 16th.

7. (a) On September 10th, the CSG submitted an Amended Return in the SEARCH WARRANT CASE praying, inter alia, that the CSG be allowed to retain the seized 431 documents and articles, in connection with cases that are presently pending against Mila Aguilar Roque before the Quezon City Fiscal's Office and the court. 5

Page 209: Warrantless Arrests, Searches and Seizures and Privacy of Communication

209

(b) On September 28th, petitioners were required by Judge Pano to comment on the Amended Return, which AGUILAR-ROQUE did on October 18th, raising the issue of the inadmissibility of any evidence obtained pursuant to the Search Warrant.

(c) On December 13, 1984, Judge Paño admitted the Amended Return and ruled that the seized documents "shall be subject to disposition of the tribunal trying the case against respondent."

8. (a) On December 12th, petitioners filed a Motion to Suppress in the SUBVERSIVE DOCUMENTS CASE, praying that such of the 431 items belonging to them be returned to them. It was claimed that the proceedings under the Search Warrant were unlawful. Judge Santos denied the Motion on January 7, 1985 on the ground that the validity of the Search Warrant has to be litigated in the SEARCH WARRANT CASE. He was apparently not aware of the Order of Judge Paño of December 13th issued in the SEARCH WARRANT CASE.

Hence, this Petition for Certiorari, Prohibition and mandamus to annul and set aside the (1) Search Warrant issued by respondent RTC Judge Paño; (2) his Order admitting the Amended Return and granting the Motion to Retain Seized Items; and (3) Order of respondent MTC Judge Santos denying petitioners' Motion to Suppress.

This Court, on February 12, 1985, issued a Temporary Restraining Order enjoining the respondents or their duly authorized representatives from introducing evidence obtained under the Search Warrant.

The PETITIONERS principally assert that the Search Warrant is void because it is a general warrant since it does not sufficiently describe with particularity the things subject of the search and seizure, and that probable cause has not been properly established for lack of searching questions propounded to the applicant's witness. The respondents, represented by the Solicitor General, contend otherwise, adding that the questions raised cannot be entertained in this present petition without petitioners first moving for the quashal of the disputed Search Warrant with the issuing Judge.

We find merit in the Petition.

Section 3, Article IV of the Constitution, guarantees the right of the people to be secure in their persons, houses, papers and effects against unreasonable searches and seizures of whatever nature and for any purpose. It also specifically provides that no Search Warrant shall issue except upon probable cause to be determined by the Judge or such other responsible officer as may be authorized by law, after examination under oath or affirmation of the complainant and the witnesses he may produce, and particularly describing the place to be searched and the things to be seized.

The disputed Search Warrant (No. 80-84) describes the personalities to be seized as follows:

Documents, papers and other records of the Communist Party of the Phihppines/New Peoples Army and/or the National Democratic Front, such as Minutes of the Party Meetings, Plans of these groups, Programs, List of possible supporters, subversive books and instructions, manuals not otherwise available to the public, and support money from foreign or local sources.

It is at once evident that the foregoing Search Warrant authorizes the seizure of personal properties vaguely described and not particularized. It is an all- embracing description which includes everything conceivable regarding the Communist Party of the Philippines and the National Democratic Front. It does not specify what the subversive books and instructions are; what the manuals not otherwise available to the public contain to make them subversive or to enable them to be used for the crime of rebellion. There is absent a definite guideline to the searching team as to what items might be lawfully seized thus giving the officers of the law discretion regarding what articles they should seize as, in fact, taken also were a portable typewriter and 2 wooden boxes. It is thus in the nature of a general warrant and infringes on the

Page 210: Warrantless Arrests, Searches and Seizures and Privacy of Communication

210

constitutional mandate requiring particular description of the things to be seized. In the recent rulings of this Court, search warrants of similar description were considered null and void for being too general. Thus:

Subversive documents, pamphlets, leaflets, books, and other publications to promote the objectives and purposes of the subversive organizations known as Movement for Free Philippines. Light-a-Fire Movement and April 6 Movement. 6

The things to be seized under the warrant issued by respondent judge were described as 'subversive documents, propaganda materials, FAs, printing paraphernalia and all other subversive materials Such description hardly provided a definite guideline to the search team as to what articles might be lawfully seized thereunder. Said description is no different from if not worse than, the description found in the search warrants in "Burgos, et al. v. the Chief of Staff"which this Court declared null and void for being too general. 7

In the case at bar, the search warrant issued by respondent judge allowed the seizure of printed copies of the Philippine Times, manuscripts/drafts of articles for publication, newspaper dummies subversive documents, articles, etc., and even typewriters, duplicating machines, mimeographing and tape recording machines. Thus, the language used is so all embracing as to include all conceivable records and equipment of petitioner regardless of whether they are legal or illegal. The search warrant under consideration was in the nature of a general warrant which is constitutionally objectionable. 8

The lack of particularization is also evident in the examination of the witness presented by the applicant for Search Warrant.

Q Mr. Dionicio Lapus, there is an application for search warrant filed by Lt. Col. Virgilio Saldajeno and the Court would like to know if you affirm the truth of your answer in this deposition?

(The deposition instead)—

A Yes, sir,

Q How long did it take you for the surveillance?

A Almost a month, sir.

Q Are you a lawyer, Mr. Lapus?

A No, Your Honor, but I was a student of law.

Q So, you are more or less familiar with the requisites of the application for search warrant?

A Yes, Your Honor.

Q How did you come to know of the person of Mila Aguilar-Roque?

A Because of our day and night surveillance, Your Honor, there were so many suspicious persons with documents.

Page 211: Warrantless Arrests, Searches and Seizures and Privacy of Communication

211

Q What kind of documents do you refer to?

A Documents related to the Communist Party of Philippines and New People's Army.

Q What else?

A Conferences of the top ranking officials from the National Democratic Front, Organization of the Communist Party of the Philippines ...

Q And may include what else?

A Other papers and documents like Minutes of the Party Meetings, Plans of these groups, Programs, List of possible supporters, subversive books and instructions, manuals not otherwise available to the public and support money from foreign and local sources. 9

The foregoing questions propounded by respondent Executive Judge to the applicant's witness are not sufficiently searching to establish probable cause. The "probable cause" required to justify the issuance of a search warrant comprehends such facts and circumstances as will induce a cautious man to rely upon them and act in pursuant thereof. 10 Of the 8 questions asked, the 1st, 2nd and 4th pertain to Identity. The 3rd and 5th are leading not searching questions. The 6th, 7th and 8th refer to the description of the personalities to be seized, which is Identical to that in the Search Warrant and suffers from the same lack of particularity. The examination conducted was general in nature and merely repetitious of the deposition of said witness. Mere generalization will not suffice and does not satisfy the requirements of probable cause upon which a warrant may issue. 11

Respondents claim, however, that the proper forum for questioning the illegality of a Search Warrant is with the Court that issued it instead of this original, independent action to quash. The records show, however, that petitioners did raise that issue in the SEARCH WARRANT CASE in their Comment, dated October 18, 1984. In fact, they already questioned the admissibility of the evidence obtained under the Search Warrant, even during the inquest investigation on August 10, 1984. And in the SUBVERSIVE DOCUMENTS CASE, they filed a Motion to Suppress on December 12, 1984 claiming that the proceedings under the Search Warrant were unlawful. Substantially, therefore, while not denominated as a motion to quash, petitioners had questioned the legality of the Search Warrant.

Parenthetically, it strikes the Court that the pendency of the SEARCH WARRANT CASE and of the SUBVERSIVE DOCUMENTS CASE before two different Courts is not conducive to an orderly administration of justice. It should be advisable that, whenever a Search Warrant has been issued by one Court, or Branch, and a criminal prosecution is initiated in another Court, or Branch, as a result of the service of the Search Warrant, the SEARCH WARRANT CASE should be consolidated with the criminal case for orderly procedure. The later criminal case is more substantial than the Search Warrant proceeding, and the Presiding Judge in the criminal case should have the right to act on petitions to exclude evidence unlawfully obtained.

Notwithstanding the irregular issuance of the Search Warrant and although, ordinarily, the articles seized under an invalid search warrant should be returned, they cannot be ordered returned in the case at bar to AGUILAR-ROQUE. Some searches may be made without a warrant. Thus, Section 12, Rule 126, Rules of Court, explicitly provides:

Section 12. Search without warrant of person arrested.—A person charged with an offense may be searched for dangerous weapons or anything which may be used as proof of the commission of the offense.

Page 212: Warrantless Arrests, Searches and Seizures and Privacy of Communication

212

The provision is declaratory in the sense that it is confined to the search, without a search warrant, of a person who had been arrested. It is also a general rule that, as an incident of an arrest, the place or premises where the arrest was made can also be search without a search warrant. In this latter case, "the extent and reasonableness of the search must be decided on its own facts and circumstances, and it has been stated that, in the application of general rules, there is some confusion in the decisions as to what constitutes the extent of the place or premises which may be searched. 12 "What must be considered is the balancing of the individual's right to privacy and the public's interest in the prevention of crime and the apprehension of criminals." 13

Considering that AGUILAR-ROQUE has been charged with Rebellion, which is a crime against public order; that the warrant for her arrest has not been served for a considerable period of time; that she was arrested within the general vicinity of her dwelling; and that the search of her dwelling was made within a half hour of her arrest, we are of the opinion that in her respect, the search at No. 239-B Mayon Street, Quezon City, did not need a search warrant; this, for possible effective results in the interest of public order.

Such being the case, the personalities seized may be retained. by CSG, for possible introduction as evidence in the Rebellion Case, leaving it to AGUILAR-ROQUE to object to their relevance and to ask Special Military Commission No.1 to return to her any and all irrelevant documents and articles.

WHEREFORE, while Search Warrant No. 80-84 issued on August 6, 1984 by respondent Executive Judge Ernani Cruz Paño is hereby annulled and set aside, and the Temporary Restraining Order enjoining respondent from introducing evidence obtained pursuant to the Search Warrant in the Subversive Documents case hereby made permanent, the, personalities seized may be retained by the Constabulary Security Group for possible introduction as evidence in Criminal Case No. SMC-1-1, pending before Special Military commission No. 1, without prejudice to petitioner Mila Aguilar-Roque objecting to their relevance and asking said Commission to return to her any and all irrelevant documents and articles.

SO ORDERED.

Plana, Escolin Relova, Gutierrez, Jr., De la Fuente, Alampay and Patajo concur.

Makasiar, C.J., concurs in the result.

Aquino, J.; took no part.

Concepcion Jr., J., reserves his vote.

 

Separate Opinions

TEEHANKEE, J., concurring and dissenting:

I concur with the concurring and dissenting opinion of Mr. Justice Vicente Abad Santos. The questioned search warrant has correctly been declared null and void in the Court's decision as a general warrant issued in gross violation of the constitutional mandate that "the right of the people to be secure in their persons, houses, papers and effects against unreasonable searches and seizures of whatever nature and for any purpose shall not be violated" (Bill of Rights, sec. 3). The Bill of Rights orders the absolute exclusion of all illegally obtained evidence: "Any evidence obtained in violation of this . . . section shall be inadmissible for any purpose in any proceeding" (Sec. 4[2]). This constitutional mandate expressly adopting the exclusionary rule has proved by historical experience to be the only practical means of

Page 213: Warrantless Arrests, Searches and Seizures and Privacy of Communication

213

enforcing the constitutional injunction against unreasonable searches and seizures by outlawing all evidence illegally seized and thereby removing the incentive on the part of state and police officers to disregard such basic rights. What the plain language of the Constitution mandates is beyond the power of the courts to change or modify.

All the articles thus seized fall under the exclusionary rule totally and unqualifiedly and cannot be used against any of the three petitioners, as held by the majority in the recent case of Galman vs. Pamaran (G.R. Nos. 71208-09, August 30, 1985). The Court has held that "in issuing a search warrant the judge must strictly comply with the requirements of the Constitution and the statutory provisions. A liberal construction should be given in favor of the individual to prevent stealthy encroachment upon, or gradual depreciation of the rights secured by the Constitution. No presumptions of regularity are to be invoked in aid of the process when an officer undertakes to justify it." (Mata vs. Bayona, 128 SCRA 388, 393-394)

The majority pronouncement that "as an incident to (petitioner Mila Aguilar- Roque's) arrest, her dwelling at No. 239-B Mayon Street, Quezon City could have been searched, even without a warrant, for evidence of rebellion" is patently against the constitutional proscription and settled law and jurisprudence. Mr. Justice Cuevas amply discusses this in the dissenting portion of his separate opinion. Suffice it to add and stress that the arresting CSG Group themselves knew that they needed a search warrant and obtained the void warrant in question. The exception of Rule 126, sec. 12 which allows a warrantless search of a person who is lawfully arrested is absolutely limited to his person, at the time of and incident to his arrest and to dangerous weapons or anything which may be used as proof of the commission of the offense." Such warrantless search obviously cannot be made in a place other than the place of arrest. In this case, petitioner Aguilar-Roque was arrested at 11:30 a.m. on board a public vehicle on the road (at Mayon and P. Margall Streets). To hold that her dwelling could "later on the same day" be searched without warrant is to sanction an untenable violation, if not nullification, of the cited basic constitutional rights against unreasonable searches and seizures.

I vote to grant the petition in toto.

ABAD SANTOS, J., concurring and dissenting:

I concur in the judgment insofar as it annuls and sets aside Search Warrant No. 80-84 issued by Executive Judge Ernani Cruz Paño for the reasons adduced by Justice Melencio Herrera. In addition I wish to state the judge either did not fully know the legal and constitutional requirements for the issuance of a search warrant or he allowed himself to be used by the military. In either case his action can only be described as deplorable.

I do not agree with the ponencia when it says that personalities seized may be retained by the Constabulary Security Group for possible introduction as evidence in Criminal Case No. SMC-1-1 pending before Special Military Commission No. 1. I agree with Justice Cuevas. for the reasons stated by him, that their retention cannot be justified by the provisions of Sec. 12, Rule 126 of the Rules of Court. But then again I cannot agree with Justice Cuevas, statement that not all the things seized can be ordered returned to their owners. He refers to "the subversive materials seized by the government agents." What are subversive materials? Whether a material is subversive or not is a conclusion of law, not of fact. Who will make the determination? Certainly not the military for it is not competent to do so aside from the fact that it has its own peculiar views on the matter. thus copies of Playboy magazines were seized from a labor leader now deceased and medicines were also seized from a physician who was suspected of being a subversive. I say return everything to the petitioners.

CUEVAS, J., concurring and dissenting

I fully agree with the pronouncement in the majority opinion nullifying Search warrant No. 80-84 issued by the Hon. Ernani Cruz Paño Executive Judge of the Regional Trial Court of Quezon City which was served

Page 214: Warrantless Arrests, Searches and Seizures and Privacy of Communication

214

at 239B Mayon St., Quezon City It does not specify with requisite particularity the things, objects or properties that may be seized hereunder. Being in the nature of a general warrant, it violates the constitutional mandate that the place to be searched and the persons or things to be seized, 'must be particularly described. (Art. IV, Sec. 3, 1973 Constitution)

I, however, regret being unable to concur with the dictum justifying the said search on the basis of Sec. 12, Rule 126 of the Rules of Court which provides:

SEC. 12. Search without warrant of person arrested.—A person charged with an offense may be searched for dangerous weapons or anything which may be used as proof of the commission of the offense.

The lawful arrest being the sole justification for the validity of the warrantless search under the aforequoted provision (Sec. 12, Rule 126) the same must be limited to and circumscribed by, the subject, time, and place of said arrest. As to subject, the warrantless search is sanctioned only with respect to the person of the suspect, and things that may be seized from him are limited to "dangerous weapons" or "anything which may be used as proof of the commission of the offense." Hence—

An officer making an arrest may take from the person arrested any money or property found upon his person which was used in the commission of the crime or might furnish the prisoner with the means of committing violence or escaping or which may be used as evidence in the trial of the cause ... (In Re Moreno vs. Ago Chi, 12 Phil. 439: People vs. Veloso, 48 Phil. 169)

With respect to the time and place of the warrantless search allowed by law, it must be contemporaneous with the lawful arrest. Stated otherwise, to be valid, the search must have been conducted at about the time of the arrest or immediately thereafter and only at the place where the suspect was arrested,

The right without a search warrant contemporaneously to search a person lawfully arrested while committing a crime and to search the place where the arrest is made in order to find and seize things connected with the crime as its fruits or as the means by which it was committed, as well as weapons or other things to effect an escape from custody is not to be doubted. CAROLL vs. US 267 US 122. 158. ... But the right does not extend to other places. Frank Agnello's house was several blocks distant from Alba's house where the arrest was made. When it was entered and searched, the conspiracy was ended and the defendants were under arrest and in custody elsewhere. That search cannot be sustained as an incident of the arrests. MARSON vs. US, 275 US 192, 199. (Emphasis supplied) (Agnello vs. U.S., 269 U.S. 20,30)

The second element which must exist in order to bring the case within the exception to the general rule is that, in addition to a lawful arrest, the search must be incident to the arrest.

The search must be made at the place of the arrest, otherwise, it is not incident to the arrest.AGNELLO vs. U.S. supra. In this latter case, 269 U.S. 20 at 30, it is said that the officers have a right to make a search contemporaneously with the arrest. And if the purpose of the officers in making their entry is not to make an arrest, but to make a search to obtain evidence for some future arrest, then search is not incidental to arrest. BYARS vs. U.S. 273 U.S., 28 ET AL. (Papani vs, U. S. 84 F 2d 160, 163)

In the instant case, petitioners were arrested at the intersection of Mayon St. and P. Margall St. at 11:30 A.M. of August 6. 1976. The search, on the other hand, was conducted after the arrest, that was at around 12:00 noon of the same day or "late that same day (as respondents claim in their "COMMENT") at the residence of petitioner AGUILAR-ROQUE in 239B Mayn St., Quezon City. How far or how many �kilometers is that place from the place where petitioner was arrested do not appear shown by the record.

Page 215: Warrantless Arrests, Searches and Seizures and Privacy of Communication

215

But what appears undisputed is that the search was made in a place other than the place of arrest and, not on the occasion of nor immediately after the arrest. It cannot be said, therefore, that such a search was incidental to the arrest of the petitioners. Not being an incident of a lawful arrest, the search of the premises at 239B Mayon St., Quezon City WITHOUT A VALID SEARCH WARRANT is ILLEGAL and violative of the constitutional rights of the respondent. The things and properties seized on the occasion of said illegal search are therefore INADMISSIBLE in evidence under the exclusionary rule. However, not all the things so seized can be ordered returned to their owners. Objects and properties the possession of which is prohibited by law, cannot be returned to their owners notwithstanding the illegality of their seizure. (Mata vs. Bayona, 128 SCRA 388, 1984 citing Castro vs. Pabalan, 70 SCRA 478). Thus, the subversive materials seized by the government agents which cannot be legally possessed by anyone under the law can and must be retained by the government.

[G.R. No. 128222.  June 17, 1999]

PEOPLE OF THE PHILIPPINES, plaintiff-appellee, vs. CHUA HO SAN @ TSAY HO SAN, accused-appellant.

D E C I S I O N

DAVIDE, JR., C.J.:

Chua Ho San @ Tsay Ho San (hereafter CHUA) prays for his acquittal and the reversal of the judgment of 10 February 1997 of the Regional Trial Court (RTC) of San Fernando, La Union, Branch 66, finding him guilty of transporting, without appropriate legal authority, the regulated substance methamphetamine hydrochloride, in violation of Section 15, [1]Article III of Republic Act No. 6425, otherwise known as the Dangerous Drugs Act of 1972 as further amended by R.A. No. 7659, [2] and sentencing him to "die by lethal injection." In view thereof, the judgment was brought to this Court for automatic review pursuant to Article 47 of the Revised Penal Code, as amended by Section 11 of R.A. No. 7659.

In response to reports of rampant smuggling of firearms and other contraband, Jim Lagasca Cid (hereafter CID), as Chief of Police of the Bacnotan Police Station, of La Union began patrolling the Bacnotan coastline with his officers.  While monitoring the coastal area of Barangay Bulala on 29 March 1995, he intercepted a radio call at around 12:45 p.m. from Barangay Captain Juan Almoite (hereafter ALMOITE) of Barangay Tammocalao requesting police assistance regarding an unfamiliar speedboat the latter had spotted.  According to ALMOITE, the vessel looked different from the boats ordinarily used by fisherfolk of the area and was poised to dock at Tammocalao shores.  CID and six of his men led by his Chief Investigator, SPO1 Reynoso Badua (hereafter BADUA), proceeded forthwith to Tammocalao beach and there conferred with ALMOITE.  CID then observed that the speedboat ferried a lone male passenger.  As it was routine for CID to deploy his men in strategic places when dealing with similar situations, he ordered his men to take up positions thirty meters from the coastline.   When the speedboat landed, the male passenger alighted, and using both hands, carried what appeared a multicolored strawbag.  He then walked towards the road. By this time, ALMOITE, CID and BADUA, the latter two conspicuous in their uniform and issued side-arms, became suspicious of the man as he suddenly changed direction and broke into a run upon seeing the approaching officers.  BADUA, however, prevented the man from fleeing by holding on to his right arm.  Although CID introduced themselves as police officers, the man appeared impassive.  Speaking in English, CID then requested the man to open his bag, but he seemed not to understand.  CID thus tried speaking Tagalog, then Ilocano, but still to no avail.  CID then resorted to what he termed “sign language;” he motioned with his hands for the man to open the bag.  This time, the man apparently understood and acceded to the request.  A search of the

Page 216: Warrantless Arrests, Searches and Seizures and Privacy of Communication

216

bag yielded several transparent plastic packets containing yellowish crystalline substances.  CID then gestured to the man to close the bag, which he did.  As CID wished to proceed to the police station, he signaled the man to follow, but the latter did not to comprehend.  Hence, CID placed his arm around the shoulders of the man and escorted the latter to the police headquarters.

At the police station, CID surmised, after having observed the facial features of the man, that he was probably Taiwanese.  CID then "recited and informed the man of his constitutional rights" to remain silent, to have the assistance of a counsel, etc.  Eliciting no response from the man, CID ordered his men to find a resident of the area who spoke Chinese to act as an interpreter.  In the meantime, BADUA opened the bag and counted twenty-nine (29) plastic packets containing yellowish crystalline substances which he and CID suspected was shabu.  The interpreter, Mr. Go Ping Guan, finally arrived, through whom the man was "apprised of his constitutional rights." The police authorities were satisfied that the man and the interpreter perfectly understood each other despite their uncertainty as to what language was spoken.  But when the policemen asked the man several questions, he retreated to his obstinate reticence and merely showed his I.D. with the name Chua Ho San printed thereon.  CHUA's bag and its contents were sent to the PNP Crime Laboratory at Camp Diego Silang, Carlatan, San Fernando, La Union for laboratory examination.  In the meantime, CHUA was detained at the Bacnotan Police Station.

Later that same day, Police Chief Inspector and Forensic Chemist Theresa Ann Bugayong Cid of the Philippine National Police, Region I, received a letter request[3] from CID – incidentally her husband – to conduct a laboratory examination of twenty-nine (29) plastic packets placed inside a multicolored strawbag.  In her Chemistry Report No. D-025-95,[4] she stated that her qualitative examination established the contents of the plastic packets, weighing 28.7 kilos, to be positive of methamphetamine hydrochloride or shabu, a regulated drug.

CHUA was initially charged with illegal possession of methamphetamine hydrochloride before the RTC which docketed the case as Criminal Case No. 4037.  However, pursuant to the recommendation of the Office of the Provincial Prosecutor of San Fernando, La Union, that the facts of the case could support an indictment for illegal transport of a regulated drug, the information was subsequently amended to allege that CHUA "willfully, unlawfully and feloniously transpor(ted) 28.7 kilos of [m]ethamphetamine [h]ydrochloride (shabu) without the necessary permit or authority to transport the same" in violation of Section 15, Article III of R.A. 6425 as amended by R.A. 7659.

At his arraignment on 31 July 1995, CHUA entered a plea of not guilty.   The RTC was satisfied that CHUA understood the amended information read to him in Fukien by the Fukien-speaking interpreter, Thelma Sales Go.

Thereafter, the RTC exerted all efforts to obtain the services of a Taiwanese Interpreter through the auspices of the Department of Foreign Affairs.  However, it was only after directing the request to the Taipei Economic and Cultural Office in the Philippines that  interpreters were assigned to CHUA.

Trial finally ensued.  The State presented evidence tending to establish the above narration of facts which were culled chiefly from the testimony of CID, its first witness, and whose testimony, in turn, was substantially corroborated by witnesses BADUA and ALMOITE.

Expert witness Theresa Ann Cid, confirmed the entries of her chemistry report in that the contents of the 29 plastic packets weighing 28.7 kilos sent to her for chemical analysis were pure, unadulterated methamphetamine hydrochloride or shabu.  She also explained that they were unwashed, hence they appeared yellowish.

For the defense, CHUA testified in his own behalf through interpreter Steven Yu.  He disclosed that he hails from Taiwan and was employed in a shipbuilding and repairing company.  On 21 March 1995, he was instructed by his employer Cho Chu Rong (hereafter RONG) to board the latter’s 35-tonner ship which would embark for Nan Au Port, Mainland China where they would buy fish.  Upon arrival at their destination, RONG left the ship, came back without the fish, but with two bags, the contents of which he never divulged to CHUA.  RONG then showed to CHUA a document purportedly granting them authority to fish on Philippine waters.  So they sailed towards the Philippines and reached Dagupan, Pangasinan on 29 March 1995.  At around 10:30 a.m., they disembarked on a small speedboat with the two bags RONG brought with him from China.  While sailing, RONG made several phone calls using his mobile

Page 217: Warrantless Arrests, Searches and Seizures and Privacy of Communication

217

phone.  CHUA heard RONG asked the person on the other side of the line if he could see the speedboat they were riding.  Apparently, the person on shore could not see them so they cruised over the waters for about five hours more when finally, low on fuel and telephone battery, they decided to dock.  CHUA anchored the boat while RONG carried the bags to shore.  The tasks completed, RONG left to look for a telephone while CHUA rested and sat one and half (1 1/2) meters away from one bag. A child thereafter pointed out to him that one bag was missing much to RONG’s dismay when he learned of it.  When a crowd started to mill around them, the police arrived.  CHUA then realized that RONG was nowhere to be found.  The police immediately approached CHUA, and with nary any spoken word, only gestures and hand movements, they escorted him to the precinct where he was handcuffed and tied to a chair.   Later, the police, led by an officer who CHUA guessed as the Chief of Police arrived with the motor engine of the speedboat and a bag.  They presented the bag to him, opened it, inspected and weighed the contents, then proclaimed them as methamphetamine hydrochloride.

CHUA denounced the prosecution’s story as a distortion of the truth.  He denied he was ever favored with an interpreter or informed of his "constitutional rights," particularly of his right to counsel.  Consequently, his arrest was tainted with illegality and the methamphetamine hydrochloride found in the bag should have been regarded inadmissible as evidence.  He also maintained that CID never graced the occasion of his setting foot for the first time at Tammocalao beach.   BADUA certainly never prevented him from running away, as such thought failed to make an impression in his mind.   Most significantly, he denied ownership and knowledge of the contents of the bag, emphasizing that RONG alone exercised dominion over the same.

Elmer Parong, (hereafter PARONG) a Sangguniang Bayan member, recalled that on the date in question, he arrived at the beach with the police.  He saw CHUA standing with a bag beside him.  He also remembered hearing from the people congregating at the beach that CHUA arrived with a companion and a certain policeman Anneb had chased the latter’s car.  He additionally claimed that when the crowd became unruly, the police decided to bring CHUA to police headquarters.  There, the mayor took charge of the situation -- he opened CHUA's bag with the assistance of the police, he called for a forensic chemist surnamed CID to take a sample of the contents of the bag, and he ordered his officials to find an interpreter.  Throughout the proceedings, photographers were busy taking pictures to document the event.

Last to testify was Arsenio CRAIG, a farmer and resident of Tammocalao who narrated that he was standing with CHUA on the beach when two men and a lady arrived.  They were about to get a bag situated near CHUA when they detected the arrival of the local police.  They quickly disappeared.  CRAIG then noticed ALMOITE and PARONG at the beach but not CID.

In a decision promulgated on 10 February 1997, the RTC found that the prosecution successfully discharged its burden of proving that CHUA transported 28.7 kilos of methamphetamine hydrochloride without legal authority to do so.  Invoking People v. Tagliben[5] as authority, the RTC characterized the search as incidental to a valid in flagrante delicto arrest, hence it allowed the admission of the methamphetamine hydrochloride as corpus delicti.  The RTC also noted the futility of informing CHUA of his constitutional rights to remain silent, and to have competent and independent counsel preferably of his own choice, considering the language barrier and the observation that such irregularity was “rectified when accused was duly arraigned and … (afterwards) participated in the trial of this case.” The RTC then disregarded the inconsistencies and contradictions in the testimonies of the prosecution witnesses as these referred to minor details which did not impair the credibility of the witnesses or tarnish the credence conferred on the testimonies thus delivered.

The RTC also believed that CHUA conspired not only with his alleged employer RONG and the Captain of the 35-tonner vessel in the illegal trade of prohibited drugs on Philippine shores, but with several other members of an organized syndicate bent on perpetrating said illicit traffic.   Such predilection was plainly evident in the dispositive portion, to wit:

WHEREFORE, and in view of all the foregoing, as proven and established by convincing and satisfactory evidence that the accused had conspired and acted in concert with one Cho Chu Rong, not to mention Chen Ho Fa, the Skipper of the 35-tonner ship they used in coming to the Country from China and Taiwan, this Court finds the accused Chua Ho San @ Tsay Ho San guilty beyond reasonable doubt of the

Page 218: Warrantless Arrests, Searches and Seizures and Privacy of Communication

218

offense of Violation of Sec. 15, Art. III of R.A. No. 6425, as amended by R.A. No. 7659 as charged in the Information, and considering the provisions of Sec. 20 of R.A. No. 7659 that the maximum penalty shall be imposed if the quantity sold/possessed/transported is ‘200 grams or more’ in the case of Shabu, and considering, further that the quantity involved in this case is 28.7 kilograms which is far beyond the weight ceiling specified in said Act, coupled with the findings of conspiracy or that accused is a member of an organized syndicated crime group, this Court, having no other recourse but to impose the maximum penalty to accused, this Court hereby sentences the said accused Chua Ho San @ Tsay Ho San to die by lethal injection; to pay a fine of Ten Million Pesos (P10,000,000.00); and to pay the costs.

The Court hereby orders Director Ricareido [sic] Sarmiento of the Philippine National Police to immediately form an investigating Committee to be composed by [sic] men of unimpeachable integrity, who will conduct an exhaustive investigation regarding this case to determine whether there was negligence or conspiracy in the escape of Cho Chu Rong and the two (2) or three (3) persons who approached the accused in the seashore of Tammocalao, Bacnotan, La Union, and attempted to take the remaining bag from accused, as well as the whereabouts of the other bag; and to furnish this Court a copy of the report/result of the said investigation in order to show compliance herewith sixty (60) days from receipt hereof.

The confiscated 28.7 kilograms of Methamphetamine Hydrochloride or Shabu is ordered turned over immediately to the Dangerous Drugs Board for destruction in accordance with the law.

The fiberglass boat with its motor engine is hereby ordered confiscated in favor of the government and to be turned over to the Philippine National Police, La Union Command, for use in their Bantay-Dagat operations against all illegal seaborne activities.

SO ORDERED.[6]

Before this Court, CHUA posits that the RTC erred in (1) admitting as competent evidence the 29 plastic packets of methamphetamine hydrochloride since they were indubitably "forbidden fruits;" (2) granting weight and credence to the testimonies of prosecution witnesses despite glaring inconsistencies on material points; and in (3) appreciating conspiracy between him and an organized syndicate in the illicit commerce of prohibited drugs since this was not alleged in the information.

The Solicitor General traverses CHUA's contentions by asserting that:  (1) the search was licitly conducted despite the absence of search and seizure warrants as circumstances immediately preceding to and contemporaneous with the search necessitated and validated the police action; and (2) that there was an effective and valid waiver of CHUA's right against unreasonable searches and seizures since he consented to the search.

We reverse the RTC.

Enshrined in the Constitution is the inviolable right to privacy of home and person.   It explicitly ordains that people have the right to be secure in their persons, houses, papers and effects against unreasonable searches and seizures of whatever nature and for any purpose. [7] Inseparable, and not merely corollary or incidental to said right and equally hallowed in and by the Constitution, is the exclusionary principle which decrees that any evidence obtained in violation of said right is inadmissible for any purpose in any proceeding.[8]

The Constitutional proscription against unreasonable searches and seizures does not, of course, forestall reasonable searches and seizure.  What constitutes a reasonable or even an unreasonable search in any particular case is purely a judicial question, determinable from a consideration of the circumstances involved.[9] Verily, the rule is, the Constitution bars State intrusions to a person's body, personal effects or residence except if conducted by virtue of a valid search warrant issued in compliance with the procedure outlined in the Constitution and reiterated in the Rules of Court; “otherwise such search and seizure become ‘unreasonable’ within the meaning of the aforementioned constitutional provision.”[10] This interdiction against warrantless searches and seizures, however, is not absolute and

Page 219: Warrantless Arrests, Searches and Seizures and Privacy of Communication

219

such warrantless searches and seizures have long been deemed permissible by jurisprudence [11] in instances of (1) search of moving vehicles, (2) seizure in plain view, (3) customs searches, (4) waiver or consent searches, (5) stop and frisk situations (Terry search),[12] and (6) search incidental to a lawful arrest.  The last includes a valid warrantless search and seizure pursuant to an equally valid warrantless arrest, for, while as a rule, an arrest is considered legitimate if effected with a valid warrant of arrest, the Rules of Court recognize permissible warrantless arrests, to wit:  (1) arrests in flagrante delicto, (2) arrests effected in hot pursuit, and (3) arrests of escaped prisoners.[13]

This Court is therefore tasked to determine whether the warrantless arrest, search and seizure conducted under the facts of the case at bar constitute a valid exemption from the warrant requirement.  Expectedly and quite understandably, the prosecution and the defense painted extremely divergent versions of the incident.  But this Court is certain that CHUA was arrested and his bag searched without the benefit of a warrant.

In cases of in flagrante delicto arrests, a peace officer or a private person may without a warrant, arrest a person, when, in his presence, the person to be arrested has committed, is actually committing, or is attempting to commit an offense.  The arresting officer, therefore, must have personal knowledge of such fact[14] or as recent case law[15] adverts to, personal knowledge of facts or circumstances convincingly indicative or constitutive of probable cause.  The term probable cause had been understood to mean a reasonable ground of suspicion supported by circumstances sufficiently strong in themselves to warrant a cautious man’s belief that the person accused is guilty of the offense with which he is charged.[16]Specifically with respect to arrests, it is such facts and circumstances which would lead a reasonably discreet and prudent man to believe that an offense has been committed by the person sought to be arrested.[17] In People v. Montilla,[18]  the Court acknowledged that “the evidentiary measure for the propriety of filing criminal charges, and correlatively, for effecting warrantless arrest, has been reduced and liberalized.” Noting that the previous statutory and jurisprudential evidentiary standard was "prima facie evidence" and that it had been dubiously equated with probable cause, the Court explained:

[F]elicitously, those problems and confusing concepts (referring to prima facie evidence and probable cause) were clarified and set aright, at least on the issue under discussion, by the 1985 amendment of the Rules of Court which provides in Rule 112 thereof that the quantum of evidence required in preliminary investigation is such evidence as suffices to ‘engender as well founded belief’ as to the fact of the commission of the crime and the respondent’s probable guilt thereof.  It has the same meaning as the related phraseology used in other parts of the same Rule, that is, that the investigating fiscal ‘finds cause to hold the respondent for trial,’ or where ‘a probable cause exists.’ It should, therefore, be in that sense, wherein the right to effect a warrantless arrest should be considered as legally authorized.” (emphasis supplied)[19]

Guided by these principles, this Court finds that there are no facts on record reasonably suggestive or demonstrative of CHUA’s participation in an ongoing criminal enterprise that could have spurred police officers from conducting the obtrusive search.  The RTC never took the pains of pointing to such facts, but predicated mainly its decision on the finding that "accused was caught red-handed carrying the bagful of [s]habu when apprehended.” In short, there is no probable cause.  At least in People v. Tangliben, the Court agreed with the lower court's finding that compelling reasons (e.g., accused was acting suspiciously, on the spot identification by an informant that accused was transporting prohibitive drug, and the urgency of the situation) constitutive of probable cause impelled police officers from effecting an in flagrante delicto arrest.  In the case at bar, the Solicitor General proposes that the following details are suggestive of probable cause -- persistent reports of rampant smuggling of firearm and other contraband articles, CHUA's watercraft differing in appearance from the usual fishing boats that commonly cruise over the Bacnotan seas, CHUA’s illegal entry into the Philippines (he lacked the necessary travel documents or visa), CHUA’s suspicious behavior, i.e. he attempted to flee when he saw the police authorities, and the apparent ease by which CHUA can return to and navigate his speedboat with immediate dispatch towards the high seas, beyond the reach of Philippine laws.

This Court, however, finds that these do not constitute “probable cause.” None of the telltale clues, e.g., bag or package emanating the pungent odor of marijuana or other prohibited drug,[20] confidential report and/or positive identification by informers of courier(s) of prohibited drug and/or the

Page 220: Warrantless Arrests, Searches and Seizures and Privacy of Communication

220

time and place where they will transport/deliver the same,[21]suspicious demeanor or behavior[22] and suspicious bulge in the waist[23]-- accepted by this Court as sufficient to justify a warrantless arrest exists in this case.  There was no classified information that a foreigner would disembark at Tammocalao beach bearing prohibited drug on the date in question.  CHUA was not identified as a drug courier by a police informer or agent.  The fact that the vessel that ferried him to shore bore no resemblance to the fishing boats of the area did not automatically mark him as in the process of perpetrating an offense. And despite claims by CID and BADUA that CHUA attempted to flee, ALMOITE testified that the latter was merely walking and oblivious to any attempt at conversation when the officers approached him.  This cast serious doubt on the truthfulness of the claim, thus:

Q     How far were you when the accused put the bag on his shoulder?

A     We were then very near him about three meters away from the male person carrying the bag.

Q     To what direction was he facing when he put the bag on his shoulder?

A     To the east direction.

Q     In relation to you, where were you.

A     With the company of Sgt. Reynoso and Maj. Cid we approached the accused and when Maj. Cid went near him, he spoke in Tagalog, English and Ilocano which accused did not understand because he did not respond.

Q     When Maj. Cid was talking, what was the accused doing at that time?

A     He was walking.

Q     To what direction he was walking?

A     He was walking to the east direction.  (sic)

Q     He was walking away from you or going near you?

A     He was going away from us.  That is why Sgt. Reynoso held the right arm of the accused.

Q     Was Sgt. Badua able to hold the right arm of the accused?

A     Yes sir and he stopped.[24]

True, CHUA entered Philippine territory without a visa.  This was not obvious to the police.  But gossamer to the officers’ sense perception and view were CHUA disembarking from a speedboat, CHUA walking casually towards the road, and CHUA carrying a multicolored strawbag.  These acts did not convey any impression that he illegally entered Philippine shores.  Neither were these overt manifestations of an ongoing felonious activity nor of CHUA’s criminal behavior as clearly established in CID’s testimony, thus:

Q     Was the accused committing a crime when you introduced yourselves:

A     No, sir.

Q     No, so there was no reason for you to approach the accused because he was not doing anything wrong?

A     No, sir, that is our objective, to approach the person and if ever or whatever assistance that we can give we will give.[25]

The search cannot therefore be denominated as incidental to an arrest.  While a contemporaneous search of a person arrested may be effected to deliver dangerous weapons or proofs or implements used in the commission of the crime and which search may extend to the area within his immediate control where he might gain possession of a weapon or evidence he can destroy, [26] a valid arrest must precede the search.  The process cannot be reversed.

Page 221: Warrantless Arrests, Searches and Seizures and Privacy of Communication

221

In a search incidental to a lawful arrest, as the precedent arrest determines the validity of the incidental search, the legality of the arrest is questioned in a large majority of these cases, e.g., whether an arrest was merely used as a pretext for conducting a search.  In this instance, the law requires that there be first a lawful arrest before a search can be made - the process cannot be reversed.[27]

To reiterate, the search was not incidental to an arrest.  There was no warrant of arrest and the warrantless arrest did not fall under the exemptions allowed by the Rules of Court [28] as already shown.  From all indications, the search was nothing but a fishing expedition.  It is worth mentioning here that after introducing themselves, the police officers immediately inquired about the contents of the bag.  What else could have impelled the officers from displaying such inordinate interest in the bag but to ferret out evidence and discover if a felony had indeed been committed by CHUA -- in effect to "retroactively establish probable cause and validate an illegal search and seizure."

The State then attempted to persuade this Court that there was a consented search, a legitimate waiver of the constitutional guarantee against obtrusive searches.  It is fundamental, however, that to constitute a waiver, it must first appear that the right exists; secondly, that the person involved had knowledge, actual or constructive, of the existence of such a right; and lastly, that said person had an actual intention to relinquish the right.[29] CHUA never exhibited that he knew, actually or constructively of his right against unreasonable searches or that he intentionally conceded the same.  This can be inferred from the manner by which the search was performed, thus:

Q     Together with your Chief Investigator, what was the first thing that you did when you approached him (CHUA)?

A     We introduced ourselves as police officers, sir.

Q     Okey, in the first place why did you introduce yourselves?

A     That is normal practice in our part, sir.

* * *

Q     If it is possible .  Okey (sic) now, after introducing yourselves what did you do?

A     He did not answer me and he did not utter any word,

Q     When he did not utter any word.  What else did he do?

A     I asked again a question that if he can open his bag sir.

Q     And did he understand your question when you requested him to open his bag?

A     No, sir, there is no answer.

Q     No answer?

A     Yes, sir, no answer.

Q     And when there was no answer what did you do next?

A     I used sign language sir.

Q     Will you demonstrate to this Honorable Court how you demonstrated that sign language of opening the bag mr. (sic) witness?

A     I pointed to the zipper of the bag and then made an action like this sir.

* * *

SHERIFF:

Page 222: Warrantless Arrests, Searches and Seizures and Privacy of Communication

222

The witness demonstrating (sic) by pointing to the straw bag and then manifesting a sign to open the zipper of the straw bag moving his right hand from left to right or from the opening to the end of the zipper.

COURT:  From the start of the zipper where you open it up to the end of the zipper.

Witness:  Yes, sir, and then I made a motion like this.

(The witness repeating the motion described on record.)

COURT:  Did you open that personally?

WITNESS:

A     No, your honor.

Q     Now, mr. (sic) witness, why did you request the accused to open the bag?

A     Because it is our duty also to inspect his belongings sir.

Q     Why, why was it - no, I reform my question your honor.  Is it normal procedure for you to examine anybody or to request anybody to open his bag?

A     The fact that he was a foreigner, sir, it is also our duty to inspect the baggage, it is our routine duty of a police (sic), sir.

Q     Is that the normal duty of a police officer to request a person to open his bag?

A     yes, sir.

Q     Okey, (sic) you did not ask the accused, mr. (sic) witness, to open his bag?

A     No, sir.

Q     But you simply requested him to open the nag?

A     Yes, sir.[30]

CHUA obviously failed to understand the events that overran and overwhelmed him.  The police officers already introduced themselves to CHUA in three languages, but he remained completely deadpan.  The police hence concluded that CHUA failed to comprehend the three languages.  When CHUA failed to respond again to the police’s request to open the bag, they resorted to what they called “sign language.” They claimed that CHUA finally understood their hand motions and gestures.  This Court disagrees.  If CHUA could not understand what was orally articulated to him, how could he understand the police’s “sign language.” More importantly, it cannot logically be inferred from his alleged cognizance of the “sign language” that he deliberately, intelligently, and consciously waived his right against such an intrusive search.  This Court is not unmindful of cases upholding the validity of consented warrantless searches and seizure.  But in these cases, the police officers' request to search personnel effects was orally articulated to the accused and in such language that left no room for doubt that the latter fully understood what was requested.  In some instances, the accused even verbally replied to the request demonstrating that he also understood the nature and consequences of such request.[31]

It was eventually discovered that the bag contained the regulated substance.  But this is a trifling matter.  If evidence obtained during an illegal search even if tending to confirm or actually confirming initial information or suspicion of felonious activity is absolutely considered inadmissible for any purpose in any proceeding, the same being the fruit of a poisonous tree [32] how much more of "forbidden fruits" which did not confirm any initial suspicion of criminal enterprise as in this case - because the police admitted that they never harbored any initial suspicion.  Casting aside the regulated substance as evidence, the remaining evidence on record are insufficient, feeble and ineffectual to sustain CHUA’s conviction.

Indeed, the likelihood of CHUA having actually transported methamphetamine hydrochloride cannot be quickly dispelled.  But the constitutional guarantee against unreasonable searches and seizures

Page 223: Warrantless Arrests, Searches and Seizures and Privacy of Communication

223

cannot be so carelessly disregarded as overzealous police officers are sometimes wont to do.  Fealty to the Constitution and the rights it guarantees should be paramount in their minds, otherwise their good intentions will remain as such simply because they have blundered.  "There are those who say that… 'the criminal is to go free because the constable has blundered.'…  In some cases this will undoubtedly be the result.  But… 'there is another consideration -- the imperative of judicial integrity.'…  The criminal goes free, if he must, but it is the law that sets him free.   Nothing can destroy a government more quickly than its failure to observe its own laws, or worse, its disregard of the charter of its own existence." [33]

As to the averred glaring inconsistencies in the testimonies of the prosecution witnesses, this Court considers them trivial as they refer to insignificant details which will not affect the outcome of the case.  On a passing note, this Court calls the attention of the trial court regarding its erroneous appreciation of conspiracy.  This aggravating circumstance is without question unsupported by the records.  Conspiracy was not included in the indictment nor raised in the pleadings or proceedings of the trial court.  It is also fundamental that conspiracy must be proven just like any other criminal accusation, that is, independently and beyond reasonable doubt.[34]

WHEREFORE, for all the foregoing, the decision of the Regional Trial Court, Branch 66, San Fernando, La Union in Criminal Case No. 4037 is hereby REVERSED and SET ASIDE and accused-appellant CHUA HO SAN @ TSAY HO SAN is hereby ACQUITTED of the crime charged, the evidence not being sufficient to establish his guilt beyond reasonable doubt.

Costs de oficio.

SO ORDERED.

Romero, Bellosillo, Melo, Vitug, Kapunan, Mendoza, Quisumbing, Purisima, Pardo, Buena, Gonzaga-Reyes, and Ynares-Santiago, JJ., concur.

Puno, J., no part. On official leave.Panganiban, J., on leave.

Page 224: Warrantless Arrests, Searches and Seizures and Privacy of Communication

224

[G.R. No. 133917.  February 19, 2001]

PEOPLE OF THE PHILIPPINES, plaintiff-appellee, vs. NASARIO MOLINA y MANAMAT @ “BOBONG” and GREGORIO MULA y MALAGURA @ “BOBOY”, accused-appellants.

D E C I S I O N

YNARES-SANTIAGO, J.:

To sanction disrespect and disregard for the Constitution in the name of protecting the society from lawbreakers is to make the government itself lawless and to subvert those values upon which our ultimate freedom and liberty depend.[1]

For automatic review is the Decision[2] of the Regional Trial Court of Davao City, Branch 17, in Criminal Case No. 37,264-96, finding accused-appellants Nasario Molina y Manamat alias “Bobong” and Gregorio Mula y Malagura alias “Boboy,” guilty beyond reasonable doubt of violation of Section 8, [3] of the Dangerous Drugs Act of 1972 (Republic Act No. 6425), as amended by Republic Act No. 7659, [4] and sentencing them to suffer the supreme penalty of death.

The information against accused-appellants reads:

That on or about August 8, 1996, in the City of Davao, Philippines, and within the jurisdiction of this Honorable Court, the above-named accused, in conspiracy with each other, did then and there willfully, unlawfully and feloniously was found in their possession 946.9 grams of dried marijuana which are prohibited.

CONTRARY TO LAW.[5]

Upon arraignment on September 4, 1996, accused-appellants pleaded not guilty to the accusation against them.[6] Trial ensued, wherein the prosecution presented Police Superintendent Eriel Mallorca, SPO1 Leonardo Y. Pamplona, Jr., and SPO1 Marino S. Paguidopon, Jr. as witnesses.

The antecedent facts are as follows:

Sometime in June 1996, SPO1 Marino Paguidopon, then a member of the Philippine National Police detailed at Precinct No. 3, Matina, Davao City, received an information regarding the presence of an alleged marijuana pusher in Davao City.[7] The first time he came to see the said marijuana pusher in person was during the first week of July 1996.  SPO1 Paguidopon was then with his informer when a motorcycle passed by. His informer pointed to the motorcycle driver, accused-appellant Mula, as the pusher.  As to accused-appellant Molina, SPO1 Paguidopon had no occasion to see him before the arrest.  Moreover, the names and addresses of the accused-appellants came to the knowledge of SPO1 Paguidopon only after they were arrested.[8]

At about 7:30 in the morning of August 8, 1996, SPO1 Paguidopon received an information that the alleged pusher will be passing at NHA, Ma-a, Davao City any time that morning. [9] Consequently, at around 8:00 A.M. of the same day, he called for assistance at the PNP, Precinct No. 3, Matina, Davao City, which immediately dispatched the team of SPO4 Dionisio Cloribel (team leader), SPO2 Paguidopon (brother of SPO1 Marino Paguidopon), and SPO1 Pamplona, to proceed to the house of SPO1 Marino Paguidopon where they would wait for the alleged pusher to pass by.[10]

At around 9:30 in the morning of August 8, 1996, while the team were positioned in the house of SPO1 Paguidopon, a “trisikad” carrying the accused-appellants passed by.  At that instance, SPO1 Paguidopon pointed to the accused-appellants as the pushers.  Thereupon, the team boarded their vehicle and overtook the “trisikad.”[11] SPO1 Paguidopon was left in his house, thirty meters from where the accused-appellants were accosted.[12]

The police officers then ordered the “trisikad” to stop.  At that point, accused-appellant Mula who was holding a black bag handed the same to accused-appellant Molina. Subsequently, SPO1 Pamplona

Page 225: Warrantless Arrests, Searches and Seizures and Privacy of Communication

225

introduced himself as a police officer and asked accused-appellant Molina to open the bag. [13] Molina replied, “Boss, if possible we will settle this.”[14]SPO1 Pamplona insisted on opening the bag, which revealed dried marijuana leaves inside.  Thereafter, accused-appellants Mula and Molina were handcuffed by the police officers.[15]

On December 6, 1996, accused-appellants, through counsel, jointly filed a Demurrer to Evidence, contending that the marijuana allegedly seized from them is inadmissible as evidence for having been obtained in violation of their constitutional right against unreasonable searches and seizures. [16] The demurrer was denied by the trial court.[17] A motion for reconsideration was filed by accused-appellants, but this was likewise denied.  Accused-appellants waived presentation of evidence and opted to file a joint memorandum.

On April 25, 1997, the trial court rendered the assailed decision,[18] the decretal portion of which reads:

WHEREFORE, finding the evidence of the prosecution alone without any evidence from both accused who waived presentation of their own evidence through their counsels, more than sufficient to prove the guilt of both accused of the offense charged beyond reasonable doubt, pursuant to Sec. 20, sub. par. 5 of Republic Act 7659, accused NASARIO MOLINA and GREGORIO MULA, are sentenced to suffer a SUPREME PENALTY OF DEATH through lethal injection under Republic Act 8176, to be effected and implemented as therein provided for by law, in relation to Sec. 24 of Rep. Act 7659.

The Branch Clerk of Court of this court, is ordered to immediately elevate the entire records of this case with the Clerk of Court of the Supreme Court, Manila, for the automatic review of their case by the Supreme Court and its appropriate action as the case may be.

SO ORDERED.[19]

Pursuant to Article 47 of the Revised Penal Code and Rule 122, Section 10 of the Rules of Court, the case was elevated to this Court on automatic review.  Accused-appellants contend:

I.

THAT THE MARIJUANA IS INADMISSIBLE IN EVIDENCE FOR HAVING BEEN SEIZED IN VIOLATION OF APPELLANTS’ CONSTITUTIONAL RIGHTS AGAINST UNREASONABLE SEARCHES AND SEIZURES;

II.

THAT ASSUMING IT IS ADMISSIBLE IN EVIDENCE, THE GOVERNMENT HAS NOT OTHERWISE PROVED THEIR GUILT BEYOND REASONABLE DOUBT; AND

III.

THAT, FINALLY, ASSUMING THEIR GUILT HAS BEEN PROVED BEYOND REASONABLE DOUBT, THE IMPOSABLE PENALTY FOR VIOLATION OF SEC. 8 OF RA No. 7659 (sic), IN THE ABSENCE OF ANY AGGRAVATING CIRCUMSTANCE, IS LIFE IMPRISONMENT, NOT DEATH. [20]

The Solicitor General filed a Manifestation and Motion (In Lieu of Brief), wherein he prayed for the acquittal of both accused-appellants.

The fundamental law of the land mandates that searches and seizures be carried out in a reasonable fashion, that is, by virtue or on the strength of a search warrant predicated upon the existence of a probable cause.  The pertinent provision of the Constitution provides:

Page 226: Warrantless Arrests, Searches and Seizures and Privacy of Communication

226

SEC. 2. The right of the people to be secure in their persons, houses, papers, and effects against unreasonable searches and seizures of whatever nature and for any purpose shall be inviolable, and no search warrant or warrant of arrest shall issue except upon probable cause to be determined personally by the judge after examination under oath or affirmation of the complainant and the witnesses he may produce, and particularly describing the place to be searched and the persons or things to be seized. [21]

Complementary to the foregoing provision is the exclusionary rule enshrined under Article III, Section 3, paragraph 2, which bolsters and solidifies the protection against unreasonable searches and seizures.[22] Thus:

Any evidence obtained in violation of this or the preceding section shall be inadmissible for any purpose in any proceeding.

Without this rule, the right to privacy would be a form of words, valueless and undeserving of mention in a perpetual charter of inestimable human liberties; so too, without this rule, the freedom from state invasions of privacy would be so ephemeral and so neatly severed from its conceptual nexus with the freedom from all brutish means of coercing evidence as not to merit this Court’s high regard as a freedom implicit in the concept of ordered liberty.[23]

The foregoing constitutional proscription, however, is not without exceptions.  Search and seizure may be made without a warrant and the evidence obtained therefrom may be admissible in the following instances: (1) search incident to a lawful arrest; (2) search of a moving motor vehicle; (3) search in violation of customs laws; (4) seizure of evidence in plain view; (5) when the accused himself waives his right against unreasonable searches and seizures;[24] and (6) stop and frisk situations (Terry search).[25]

The first exception (search incidental to a lawful arrest) includes a valid warrantless search and seizure pursuant to an equally valid warrantless arrest which must precede the search.  In this instance, the law requires that there be first a lawful arrest before a search can be made --- the process cannot be reversed.[26] As a rule, an arrest is considered legitimate if effected with a valid warrant of arrest.  The Rules of Court, however, recognizes permissible warrantless arrests.  Thus, a peace officer or a private person may, without warrant, arrest a person: (a) when, in his presence, the person to be arrested has committed, is actually committing, or is attempting to commit an offense (arrest in flagrante delicto); (b) when an offense has just been committed and he has probable cause to believe based on personal knowledge of facts or circumstances that the person to be arrested has committed it (arrest effected in hot pursuit); and (c) when the person to be arrested is a prisoner who has escaped from a penal establishment or a place where he is serving final judgment or is temporarily confined while his case is pending, or has escaped while being transferred from one confinement to another (arrest of escaped prisoners).[27]

In the case at bar, the court a quo anchored its judgment of conviction on a finding that the warrantless arrest of accused-appellants, and the subsequent search conducted by the peace officers, are valid because accused-appellants were caught in flagrante delicto in possession of prohibited drugs.[28] This brings us to the issue of whether or not the warrantless arrest, search and seizure in the present case fall within the recognized exceptions to the warrant requirement.

In People v. Chua Ho San,[29] the Court held that in cases of in flagrante delicto arrests, a peace officer or a private person may, without a warrant, arrest a person when, in his presence, the person to be arrested has committed, is actually committing, or is attempting to commit an offense.  The arresting officer, therefore, must have personal knowledge of such fact or, as recent case law adverts to,  personal knowledge of facts or circumstances convincingly indicative or constitutive of probable cause.    As discussed in People v. Doria,[30] probable cause means an actual belief or reasonable grounds of suspicion.  The grounds of suspicion are reasonable when, in the absence of actual belief of the arresting officers, the suspicion that the person to be arrested is probably guilty of committing the offense, is based on actual facts, i.e., supported by circumstances sufficiently strong in themselves to create the probable cause of guilt of the person to be arrested.  A reasonable suspicion therefore must be founded on probable cause, coupled with good faith on the part of the peace officers making the arrest.

Page 227: Warrantless Arrests, Searches and Seizures and Privacy of Communication

227

As applied to in flagrante delicto arrests, it is settled that “reliable information” alone, absent any overt act indicative of a felonious enterprise in the presence and within the view of the arresting officers, are not sufficient to constitute probable cause that would justify an in flagrante delicto arrest.  Thus, in People v. Aminnudin,[31] it was held that “the accused-appellant was not, at the moment of his arrest, committing a crime nor was it shown that he was about to do so or that he had just done so.  What he was doing was descending the gangplank of the M/V Wilcon 9 and there was no outward indication that called for his arrest.  To all appearances, he was like any of the other passengers innocently disembarking from the vessel.  It was only when the informer pointed to him as the carrier of the marijuana that he suddenly became suspect and so subject to apprehension.”

Likewise, in People v. Mengote,[32] the Court did not consider “eyes... darting from side to side ... [while] holding ... [one’s] abdomen”, in a crowded street at 11:30 in the morning, as overt acts and circumstances sufficient to arouse suspicion and indicative of probable cause.  According to the Court, “[b]y no stretch of the imagination could it have been inferred from these acts that an offense had just been committed, or was actually being committed, or was at least being attempted in [the arresting officers’] presence.”  So also, inPeople v. Encinada,[33] the Court ruled that no probable cause is gleanable from the act of riding a motorela while holding two plastic baby chairs.

Then, too, in Malacat v. Court of Appeals,[34] the trial court concluded that petitioner was attempting to commit a crime as he was “‘standing at the corner of Plaza Miranda and Quezon Boulevard’ with his eyes ‘moving very fast’ and ‘looking at every person that come (sic) nearer (sic) to them.’” [35] In declaring the warrantless arrest therein illegal, the Court said:

Here, there could have been no valid in flagrante delicto ... arrest preceding the search in light of the lack of personal knowledge on the part of Yu, the arresting officer, or an overt physical act, on the part of petitioner, indicating that a crime had just been committed, was being committed or was going to be committed.[36]

It went on to state that -

Second, there was nothing in petitioner’s behavior or conduct which could have reasonably elicited even mere suspicion other than that his eyes were “moving very fast” - an observation which leaves us incredulous since Yu and his teammates were nowhere near petitioner and it was already 6:30 p.m., thus presumably dusk.  Petitioner and his companions were merely standing at the corner and were not creating any commotion or trouble...

Third, there was at all no ground, probable or otherwise, to believe that petitioner was armed with a deadly weapon.  None was visible to Yu, for as he admitted, the alleged grenade was “discovered” “inside the front waistline” of petitioner, and from all indications as to the distance between Yu and petitioner, any telltale bulge, assuming that petitioner was indeed hiding a grenade, could not have been visible to Yu. [37]

Clearly, to constitute a valid in flagrante delicto arrest, two requisites must concur: (1) the person to be arrested must execute an overt act indicating that he has just committed, is actually committing, or is attempting to commit a crime; and (2) such overt act is done in the presence or within the view of the arresting officer.[38]

In the case at bar, accused-appellants manifested no outward indication that would justify their arrest.  In holding a bag on board a trisikad, accused-appellants could not be said to be committing, attempting to commit or have committed a crime. It matters not that accused-appellant Molina responded “Boss, if possible we will settle this” to the request of SPO1 Pamplona to open the bag.  Such response which allegedly reinforced the “suspicion” of the arresting officers that accused-appellants were committing a crime, is an equivocal statement which standing alone will not constitute probable cause to effect an inflagrante delicto arrest.  Note that were it not for SPO1 Marino Paguidopon (who did not participate in the arrest but merely pointed accused-appellants to the arresting officers), accused-appellants could not be the subject of any suspicion, reasonable or otherwise.

Page 228: Warrantless Arrests, Searches and Seizures and Privacy of Communication

228

While SPO1 Paguidopon claimed that he and his informer conducted a surveillance of accused-appellant Mula, SPO1 Paguidopon, however,  admitted that he only learned Mula’s name and address after the arrest.  What is more, it is doubtful if SPO1 Paguidopon indeed recognized accused-appellant Mula.  It is worthy to note that, before the arrest, he was able to see Mula in person only once, pinpointed to him by his informer while they were on the side of the road.  These circumstances could not have afforded SPO1 Paguidopon a closer look at accused-appellant Mula, considering that the latter was then driving a motorcycle when SPO1 Paguidopon caught a glimpse of him.  With respect to accused-appellant Molina, SPO1 Paguidopon admitted that he had never seen him before the arrest.

This belies the claim of SPO1 Pamplona that he knew the name of  accused-appellants even before the arrest, to wit -

“Q-  When you said that certain Mula handed a black bag to another person and how did you know that it was Mula who handed the black bag to another person?

A-            Because I have already information from Paguidopon, regarding Mula and Molina, when they pass by through the street near the residence of Paguidopon. He told that the one who is big one that is Gregorio Mula and the thin one is Nazario Molina”[39]

The aforecited testimony of SPO1 Pamplona, therefore, is entirely baseless.  SPO1 Pamplona could not have learned the name of accused-appellants from SPO1 Paguipodon because Paguipodon himself, who allegedly conducted the surveillance, was not even aware of accused-appellants’ name and address prior to the arrest.

Evidently, SPO1 Paguidopon, who acted as informer of the arresting officers, more so the arresting officers themselves, could not have been certain of accused-appellants’ identity, and were, from all indications, merely fishing for evidence at the time of the arrest.

Compared to People v. Encinada, the arresting officer in the said case knew appellant Encinada even before the arrest because of the latter’s illegal gambling activities, thus, lending at least a semblance of validity on the arrest effected by the peace officers.  Nevertheless, the Court declared in said case that the warrantless arrest and the consequent search were illegal, holding that “[t]he prosecution’s evidence did not show any suspicious behavior when the appellant disembarked from the ship or while he rode the motorela.  No act or fact demonstrating a felonious enterprise could be ascribed to appellant under such bare circumstances.”[40]

Moreover, it could not be said that accused-appellants waived their right against unreasonable searches and seizure.  Implied acquiescence to the search, if there was any, could not have been more than mere passive conformity given under intimidating or coercive circumstances and is thus considered no consent at all within the purview of the constitutional guarantee.[41]

Withal, the Court holds that the arrest of accused-appellants does not fall under the exceptions allowed by the rules.  Hence, the search conducted on their person was likewise illegal.  Consequently, the marijuana seized by the peace officers could not be admitted as evidence against accused-appellants, and the Court is thus, left with no choice but to find in favor of accused-appellants.

While the Court strongly supports the campaign of the government against drug addiction and commends the efforts of our law-enforcement officers towards this drive, all efforts for the achievement of a drug-free society must not encroach on the fundamental rights and liberties of individuals as guaranteed in the Bill of Rights, which protection extends even to the basest of criminals.

WHEREFORE, the Decision of the Regional Trial Court of Davao City, Branch 17, in Criminal Case No. 37, 264-96, is REVERSED and SET ASIDE.  For lack of evidence to establish their guilt beyond reasonable doubt, accused-appellants Nasario Molina y Manamat alias “Bobong” and Gregorio Mula y Malagura alias “Boboy”, are ACQUITTED and ordered RELEASED from confinement unless they are validly detained for other offenses.  No costs.

SO ORDERED.

Page 229: Warrantless Arrests, Searches and Seizures and Privacy of Communication

229

Davide, Jr., C.J., Bellosillo, Melo, Puno, Vitug, Kapunan, Mendoza, Panganiban, Quisumbing, Pardo, Buena, Gonzaga-Reyes, De Leon, Jr., and Sandoval-Gutierrez, JJ.,concur.

[G.R. Nos. 138539-40.  January 21, 2003]

PEOPLE OF THE PHILIPPINES, appellee, vs. ANTONIO C. ESTELLA, appellant.

D E C I S I O N

PANGANIBAN, J.:

The Constitution bars the admission of evidence gathered in violation of the right against unreasonable search and seizure.  In the present case, the illegal drug was searched for and found in a hut that has not been proven to be owned, controlled, or used by appellant for residential or any other purpose.  Hence, he cannot be held guilty of illegal possession of the illegal drug found therein.

The Case

Antonio C. Estella appeals the August 25, 1998 Decision [1] of the Regional Trial Court (RTC) of Iba, Zambales (Branch 69) in Criminal Case No. RTC 2143-I.  The trial court found him guilty of violating Section 8, Article II of RA 6425, as amended by RA 7659, and sentenced him to reclusion perpetua as follows:

“WHEREFORE, foregoing considered, in Criminal Case No. RTC 2143-I, accused Antonio C. Estella is found GUILTY beyond reasonable doubt for Violation of Section 8, Article II of R.A. 6425 as amended by R.A. 7659 and is sentenced to suffer the penalty of reclusion perpetua.

“The 8.320 kilograms of dried marijuana is ordered confiscated in favor of the government.  The Sheriff is directed to deliver the subject marijuana to the Dangerous Drugs Board for its proper disposition.

“In Criminal Case No. RTC 2144-I, accused Antonio C. Estella is ACQUITTED and the Information dated 07 January 1997 filed against him for violation of P.D. 1866 is dismissed with costs de oficio.

“The .38 caliber revolver without serial number and four (4) live ammunitions, subject of the offense, are ordered delivered to any authorized representative of the Philippine National Police, Firearms and Explosives Division, Camp Crame, Quezon City.”[2]

The Information dated January 7, 1997, charged appellant thus:

Page 230: Warrantless Arrests, Searches and Seizures and Privacy of Communication

230

“That on or about the 20th day of November, 1996 at about 11:15 o’clock in the morning, at Purok Yakal, Barangay Baloganon, in the Municipality of Masinloc, Province of Zambales, Philippines, and within the jurisdiction of this Honorable Court, said accused, did then and there, wil[l]fully, unlawfully and feloniously have in his possession, custody and control, [o]ne (1) tin can labeled ‘CLASSIC’ containing twenty (20) small bricks of dried marijuana fruiting tops having a total weight of 589.270 grams each wrapped with a piece of reading material; [o]ne (1) tin can labeled ‘CLASSIC’ containing dried marijuana fruiting tops weighing 41.126 grams; [t]wo (2) white sando plastic bag each containing one (1) [brick] of dried marijuana fruiting tops having a total weight of 1.710 kilograms each wrapped with a piece of newspaper; [o]ne (1) white sando plastic bag containing two (2) bricks of dried marijuana fruiting tops having a total weight of 1.820 kilograms each wrapped with a piece of newspaper, all in the total of 8.320 kilograms of dried marijuana, without any authority to possess the same.”[3]

After the Information had been read to him in Filipino, a language he fully understood, [4] appellant, assisted by his counsel de parte,[5] pleaded not guilty when arraigned on March 11, 1997.  After due trial, the RTC convicted appellant of illegal possession of dangerous drugs (marijuana), but acquitted him of illegal possession of firearms.  On November 4, 1998, his counsel filed a Notice of Appeal.[6]

The Facts

Version of the Prosecution

In its Brief,[7] the Office of the Solicitor General (OSG) presents the prosecution’s version of the facts as follows:

“Prior to November 20, 1996, Executive Judge Romulo Estrada of the Regional Trial Court of Zambales issued a warrant for the conduct of a search and seizure in the residence of appellant at Purok Yakal, Barangay Baloganon, Masinloc, Zambales.

“In the morning of November 20, 1996, Senior Police Officer 1 (SPO1) Antonio Bulor[o]n, then Intelligence and Investigation Officer, together with SPO1 Jose Arca and several other members of the Provincial Special Operation Group based in Burgos, San Marcelino, Zambales proceeded to Masinloc.  They coordinated with the members of the Philippine National Police (PNP) in Masinloc and sought the assistance of Barangay Captain Rey Barnachea of Baloganon, Masinloc for the enforcement of the search warrant.  Barangay Captain Barnaceha accompanied the police officers to Purok Yakal, Barangay Baloganon, Masinloc, the place mentioned in the search warrant.

“On their way to Purok Yakal, SPO1 Buloron saw appellant sitting on a rocking chair located about two (2) meters away from a hut owned by Narding Estella, brother of appellant, and being rented by appellant’s live-in partner, named Eva.  They approached appellant and introduced themselves as police officers.  They showed appellant the search warrant and explained the contents to him.  SPO1 Buloron asked appellant if indeed he had in his possession prohibited drug and if so, to surrender the same so he would deserve a lesser penalty.

“While inside the hut, appellant surrendered to the team two cans containing dried marijuana fruiting tops.  One can contained twenty (20) bricks of fruiting tops.  The team searched the hut in the presence of appellant and his live-in partner.  They found a plastic container under the kitchen table, which contained four (4) big bricks of dried marijuana leaves and a .38 caliber revolver with four live ammunitions.  The team seized the prohibited drug, the revolver and ammunitions.  The team seized and signed a receipt for the seized items. Barangay Captain Barnachea and SPO1 Edgar Bermudez of the Masinloc Police Station also signed the receipt as witnesses.  SPO1 Buloron and his companions arrested appellant and brought him to San Marcelino, Zambales.

Page 231: Warrantless Arrests, Searches and Seizures and Privacy of Communication

231

“At their office in San Marcelino, Zambales, SPO1 Buloron and SPO1 Arca placed their markings on the seized items for purposes of identification.  SPO1 Arca kept the seized items under his custody.  The next day, SPO1 Buloron and SPO1 Arca brought the seized items to San Antonio, Zambales, where Police Senior Inspector Florencio Sahagun examined the suspected marijuana dried leaves.  Inspector Sahagun prepared a certification of field test.

“On November 29, 1996, the suspected marijuana dried leaves were delivered to the PNP Crime Laboratory at Camp Olivas for further examination.  Senior Inspector Daisy Babor, a forensic chemist, examined the suspected marijuana dried leaves and issued Chemistry Report No. D-768-96 stating that the specimens are positive for marijuana, a prohibited drug. Specimen A weighed 1.710 kilograms, while Specimen D weighed 1.820 kilograms.”[8] (Citations omitted)

Version of the Defense

For his version of the facts, appellant merely reproduced the narration in the assailed RTC Decision as follows:

“Accused Antonio C. Estella [I]s married to Gloria Atrero Estella.  They have three (3) children, namely: Carmen Estella (8 years old), Antonio Estella, Jr. (5 years old) and Roen Estella (3 years old).  Since 1982, Antonio Estella has been [a] resident of Barangay Baloganon, Masinloc, Zambales.

“On 20 November 1996 between 10:30 o’clock and 11:00 o’clock in the morning, while accused was talking with his friends Rael Tapado and Victor de Leon at a vacant lot just outside the house of Camillo Torres and about 70 meters away from his house, a group of men approached them.  The group introduced themselves as policemen and told them that they were looking for Antonio Estella because they have a search warrant issued against him.  Accused identified himself to them.  The policemen inquired from the accused as to where his house is located and accused told them that his house is located across the road.  The police did not believe him and insisted that accused’s house (according to their asset) is that house located about 5-8 meters away from them.  Accused told the policemen to inquire from the Barangay Captain Barnachea as to where his house is and heard the latter telling the policemen that his house is located near the Abokabar junk shop.  After about half an hour, the policemen went inside the house nearby and when they came out, they had with them a bulk of plastic and had it shown to the accused.  They photographed the accused and brought him to their office at San Marcelino, Zambales.  Accused Antonio Estella was investigated a[t] San Marcelino, Zambales where he informed the police officers of the fact that the house they searched was occupied by Spouses Vicente and Fely Bakdangan.

“Accused denied having surrendered to policeman Buloron tin cans containing marijuana and likewise having any firearm.

“Miguel Buccat, who personally knew the accused for about ten (10) years, identified the house depicted on a photograph as that house belonging to the accused.”[9] (Citations omitted)

Ruling of the Trial Court

In finding appellant guilty of violating the Dangerous Drugs Act, the court a quo relied heavily on the testimony of the prosecution’s principal witness, Intelligence and Investigation Officer SPO1 Antonio Buloron.  He was among the members of the police team that searched appellant’s alleged house.  Since the defense failed to present proof of any intent on the part of SPO1 Buloron to falsely impute to appellant such a serious crime, the trial court accorded full faith and credence to the police officer’s testimony.

Page 232: Warrantless Arrests, Searches and Seizures and Privacy of Communication

232

Moreover, the RTC held that no less than the barangay captain of the place named in the search warrant led the police to the house.  Thus, appellant could not deny that he owned it.

As to the charge of illegal possession of firearms, the lower court ruled that the search warrant did not cover the seized firearm, making it inadmissible against appellant.  He was thus acquitted of the charge.

Hence, this recourse.[10]

The Issues

In his appeal, appellant assigns the following alleged errors for our consideration:

“A.     The trial court erred in convicting the accused based on the conjectural and conflicting testimonies of the prosecution witnesses;

“B.     The trial court gravely failed to consider the serious contradictions in the facts and evidences adduced by the prosecution;

“C.     The trial court gravely erred in finding that the guilt of the accused-appellant for the crime charged has been prove[n] beyond reasonable doubt, instead of judgment of acquittal demanded by the constitutional presumption of innocence[.]”[11]

Though not clearly articulated by appellant, the pivotal issue here is the legality of the police search undertaken in the hut where the subject marijuana was seized.

The Court’s Ruling

The appeal is meritorious.

Main Issue:

Legality of the Search Undertaken

Once again, this Court is confronted with a situation that involves a well-enshrined dogma in our Constitution: the inviolable right of the people to be secure in their persons and properties against unreasonable searches and seizures.[12]  The exclusionary rule prescribed by Section 3(2), Article III of the Constitution, bars the admission of evidence obtained in violation of this right.[13]

The conviction or the acquittal of appellant hinges primarily on the validity of the police officers’ search and seizure, as well as the admissibility of the evidence obtained by virtue thereof.  Without that evidence, the prosecution would not be able to prove his guilt beyond reasonable doubt.

Ownership of the Subject House

Page 233: Warrantless Arrests, Searches and Seizures and Privacy of Communication

233

Appellant claims that the hut,[14] which was searched by the police and where the subject marijuana was recovered, does not belong to him.  He points to another house[15] as his real residence.  To support his claim, he presents a document[16] that shows that the subject hut was sold to his brother Leonardo C. Estella by one Odilon Eclarinal.  The OSG, on the other hand, argues that just because “appellant has another house in a place away from the hut that was searched does not necessarily mean that the hut is not occupied by him or under his full control.” [17] The prosecution cites the testimony of Rey Barnachea, the barangay captain of that place, to show that the hut in question belongs to appellant.

The only link that can be made between appellant and the subject hut is that it was bought by his brother Leonardo a.k.a. “Narding” Estella.[18] We cannot sustain the OSG’s supposition that since it was being rented by the alleged live-in partner of appellant, it follows that he was also occupying it or was in full control of it.  In the first place, other than SPO1 Buloron’s uncorroborated testimony, no other evidence was presented by the prosecution to prove that the person renting the hut was indeed the live-in partner of appellant -- if he indeed had any.  Moreover, the testimony of Barnachea serves to undermine, not advance, the position of the prosecution.  We quote from his testimony:

“Q   Do you know who is the owner of that house?

A     What I know is that Narding Estella bought that house, sir.

Q    Who is that Narding Estella?

A     The brother of Tony Estella, sir.

Q    And you know that that has been rent[ed] to people?

A     Yes, sir.

Q    Now, so far how many people [rented]  that place or that house?

A     I do not have any information about that[,] sir.

Q    Why did you know that that place was rented?

A     Because when I asked Eva she replied that they [were] only renting that house, sir.

Q    How long has Eva been renting that house?

A     I do not have any information about that[,] sir.

Q    Do you know who was living with Eva?

A     No, sir.

Q    So, what you know is that Eva lives alone in that house?

A     Yes, sir.

Q    And you do not know anybody who is renting that house?

A     I have no information, sir.

Q    And you do not know if the accused was renting [it] or not?

A     I don’t have any information, sir.”[19]

At most, the testimony shows that the subject hut was bought by Narding Estella and rented by someone named Eva.  The attempt to make it appear that appellant occupied it, or that it was under his full control, is merely conjectural and speculative.  We have often ruled that courts do not rely on evidence that arouses mere suspicion or conjecture.[20] To lead to conviction, evidence must do more than raise the mere possibility or even probability of guilt.[21] It must engender moral certainty.

Page 234: Warrantless Arrests, Searches and Seizures and Privacy of Communication

234

Neither do we find merit in the OSG’s argument that appellant cannot deny ownership or control of the hut, since he was found in front of it, sitting on a rocking chair and drinking coffee. [22] Indeed, to uphold this proposition would be to stretch our imagination to the extreme.

The OSG maintains that when appellant was “shown the search warrant and asked about the existence of prohibited drug in his possession, appellant went inside the hut, took his stock of marijuana and turned it [over] to the police officers.” [23] This, according to the prosecution, clearly showed that he was not only occupying the hut, but was in fact using it to store the prohibited drug.[24]

It is well-settled that this Court is not precluded from assessing the probative value of witnesses’ testimonies on the basis of the transcript of stenographic notes (TSNs).[25]

In the case at bar, we believe that the trial court erred in adopting the prosecution’s dubious story.  It failed to see patent inconsistencies in the prosecution witnesses’ testimonies about the search undertaken.

A review of the TSNs shows that SPO1 Buloron, the prosecution's principal witness, testified that appellant had allegedly gone inside the hut; and that the latter had done so to get his stock of illegal drugs, which he turned over to the police.  Ironically, Captain Barnachea, who was purposely presented by the prosecution to corroborate SPO1 Buloron's story, belied it when he testified thus:

“PROS. QUINTILLAN:

Q    When the police officer showed that search warrant what did Antonio Estella said, if any, if you hear[d]?

A     What I saw is that Tony Estella is sitting in the rocking chair outside the house drinking coffee, sir.

Q    And you saw him and then the search warrant was presented, isn’t it?

A     Yes, sir.

Q    And when it was presented what did Tony Estella do?

A     What they did they show to Tony the search warrant and I also read the contents of the search warrant, sir.

Q    And when Tony was shown that search warrant what did he do immediately after being shown that search warrant?

A     He just [sat] and then he stood up, sir.

Q    And when he stood up what else did he do?

A     Nothing, sir.  The NARCOM g[o]t inside the house, sir.

Q    And where did Antonio Estella go when the police entered the house?

A     He was just outside the house, sir.

Q    And how far is that house from Antonio Estella?

INTERPRETER:

Witness estimating the distance of about five (5) meters.

COURT:

Do the prosecution and defense agree to 5 meters?

BOTH COUNSEL:

Yes, Your Honor.

PROS. QUINTILLAN:

Page 235: Warrantless Arrests, Searches and Seizures and Privacy of Communication

235

Q    And when the police entered the house did not Tony go with them?

A     I did not notice, sir.”[26]

It is undisputed that even before arriving at the hut, the police officers were already being assisted by Barangay Captain Barnachea.  Thus, it was highly improbable for him not to see personally appellant’s alleged voluntary surrender of the prohibited drug to the authorities.  And yet, his testimony completely contradicted the policemen’s version of the events.  He testified that appellant, after being served the search warrant, remained outside the hut and did nothing.  In fact, the former categorically stated that when the police officers had gone inside the hut to conduct the search, appellant remained seated on a rocking chair outside.[27] Barnachea’s statements sow doubts as to the veracity of SPO1 Buloron’s claim that, after being apprised of the contents of the search warrant, appellant voluntarily surrendered the prohibited drug to the police.[28]

Apart from the testimony of Barnachea -- which contradicted rather than validated the story of SPO1 Buloron -- no other evidence was presented to corroborate the latter’s narration of the events.  Without any independent or corroborative proof, it has little or no probative value at all.

In a criminal prosecution, the court is always guided by evidence that is tangible, verifiable, and in harmony with the usual course of human experience -- not by mere conjecture or speculation. [29] While the guilty should not escape, the innocent should not suffer.[30]

Search Incident to Lawful Arrest

The OSG argues that “[e]ven assuming that appellant was not the occupant of the hut, the fact remains that he voluntarily surrendered the marijuana to the police officers.   After appellant had surrendered the prohibited stuff, the police had a right to arrest him even without a warrant and to conduct a search of the immediate vicinity of the arrestee for weapons and other unlawful objects as an incident to the lawful arrest.”[31]

The above argument assumes that the prosecution was able to prove that appellant had voluntarily surrendered the marijuana to the police officers.  As earlier adverted to, there is no convincing proof that he indeed surrendered the prohibited drug, whether voluntarily or otherwise.  In fact, the testimony of Prosecution Witness Barnachea clouds rather than clarifies the prosecution’s story.

Given this backdrop, the police authorities cannot claim that the search was incident to a lawful arrest.  Such a search presupposes a lawful or valid arrest and can only be invoked through Section 5, Rule 113 of the Revised Rules on Criminal Procedure, which we quote:

“SEC. 5. Arrest without warrant; when lawful - A peace officer or a private person may, without a warrant, arrest a person:

“(a) When, in his presence, the person to be arrested has committed, is actually committing, or is attempting to commit an offense;

“(b)    When an offense has just been committed and he has probable cause to believe based on personal knowledge of facts or circumstances that the person to be arrested has committed it; and

“(c)    When the person to be arrested is a prisoner who has escaped from a penal establishment or place where he is serving final judgment or is temporarily confined while his case is pending, or has escaped while being transferred from one confinement to another.

“In cases falling under paragraphs (a) and (b) above, the person arrested without a warrant shall be forthwith delivered to the nearest police station or jail and shall be proceeded against in accordance with Section 7 Rule 112.”

Page 236: Warrantless Arrests, Searches and Seizures and Privacy of Communication

236

Never was it proven that appellant, who was the person to be arrested, was in possession of the subject prohibited drug during the search.  It follows, therefore, that there was no way of knowing if he had committed or was actually committing an offense in the presence of the arresting officers.   Without that knowledge, there could have been no search incident to a lawful arrest.

Assuming arguendo that appellant was indeed committing an offense in the presence of the arresting officers, and that the arrest without a warrant was lawful, it still cannot be said that the search conducted was within the confines of the law.  Searches and seizures incident to lawful arrests are governed by Section 12, Rule 126 of the Revised Rules of Criminal Procedure, which reads:

“Section 12.   Search incident to lawful arrest. – A person lawfully arrested may be searched for dangerous weapons or anything which may have been used or constitute proof in the commission of an offense without a search warrant.”

However, the scope of the search should be limited to the area within which the person to be arrested can reach for a weapon or for evidence that he or she can destroy. [32]  The prevailing rule is that the arresting officer may take from the arrested individual any money or property found upon the latter’s person -- that which was used in the commission of the crime or was the fruit of the crime, or which may provide the prisoner with the means of committing violence or escaping, or which may be used in evidence in the trial of the case.[33]

In the leading case Chimel v. California,[34] the Supreme Court of the United States of America laid down this rule:

“When an arrest is made, it is reasonable for the arresting officer to search the person arrested in order to remove any weapons that the latter might seek to use in order to resist arrest or effect his escape.  Otherwise, the officer’s safety might well be endangered, and the arrest itself frustrated.  In addition, it is entirely reasonable for the arresting officer to search for and seize any evidence on the arrestee’s person in order to prevent its concealment or destruction.  And the area into which an arrestee might reach in order to grab a weapon or evidentiary items must, of course, be governed by a like rule.  A gun on a table or in a drawer in front of one who is arrested can be as dangerous to the arresting officer as one concealed in the clothing of the person arrested. There is ample justification, therefore, for a search of the arrestee’s person and the area ‘within his immediate control’ – construing that phrase to mean the area from within which he might gain possession of a weapon or destructible evidence.

“There is no comparable justification, however, for routinely searching any room other than that in which an arrest occurs – or, for that matter, for searching through all the desk drawers or other closed or concealed areas in that room itself.”[35]

The purpose of the exception is to protect the arresting officer from being harmed by the person being arrested, who might be armed with a concealed weapon, and to prevent the latter from destroying evidence within reach.  The exception, therefore, should not be strained beyond what is needed to serve its purpose.[36]

In the case before us, searched was the entire hut, which cannot be said to have been within appellant’s immediate control.  Thus, the search exceeded the bounds of that which may be considered to be incident to a lawful arrest.

The Presence of the Accused or the

Witnesses During the Search

Page 237: Warrantless Arrests, Searches and Seizures and Privacy of Communication

237

Having ruled that the prosecution failed to prove appellant’s ownership, control of or residence in the subject hut, we hold that the presence of appellant or of witnesses during the search now becomes moot and academic.

Obviously, appellant need not have been present during the search if he was neither the owner nor the lawful occupant of the premises in question.  Besides, as we have noted, the testimonies of the prosecution witnesses regarding these crucial circumstances were contradictory.  They erode SPO1 Buloron’s credibility as a prosecution witness and raise serious doubts concerning the prosecution’s evidence.  This Court is thus constrained to view his testimony with caution and care.

With the failure of the prosecution to establish the propriety of the search undertaken -- during which the incriminating evidence was allegedly recovered -- we hold that the search was illegal.  Without the badge of legality, any evidence obtained therein becomes ipso facto inadmissible.

Objections to the

Legality of the Search

Finally, the OSG argues that appellant is deemed to have waived his right to object to the legality of the search and the admissibility of the evidence seized through that search because, during the trial, he did not raise these issues.

On the contrary, during the trial, appellant constantly questioned the legality of the search.  In fact, when SPO1 Buloron was presented as a prosecution witness, the former’s counsel objected to the offer of the latter’s testimony on items allegedly confiscated during the search.  Appellant’s counsel argued that these items, which consisted of the marijuana and the firearm, had been seized illegally and were therefore inadmissible.[37]

Further, in his Comments and Objections to Formal Offer of Exhibits,[38] appellant once again questioned the legality of the search conducted by the police, a search that had yielded the evidence being used against him.

Finally, on October 21, 1997, he filed a Demurrer to Evidence [39] reiterating his objection to the search and to the eventual use against him of the evidence procured therefrom.

All told, without sufficient admissible evidence against appellant, the prosecution failed to establish his guilt with moral certainty.[40]  Not only did its evidence fall short of the quantum of proof required for a conviction, it has also failed to present any evidence at all.  Under our Bill of Rights, among the fundamental rights of the accused is to be presumed innocent until the contrary is proved. [41]  To overcome such presumption, the prosecution must establish guilt beyond reasonable doubt. Our criminal justice system dictates that if the prosecution fails to do so, it becomes not only the right of the accused to be set free, but also the constitutional duty of the court to set them free. [42]  This principle leaves this Court no option but to acquit Appellant Antonio C. Estella for insufficiency of evidence.

WHEREFORE, the appealed Decision is SET ASIDE.  Antonio C. Estella is ACQUITTED and ordered immediately RELEASED from custody, unless he is being held for some other lawful cause.

The director of the Bureau of Corrections is ORDERED to implement this Decision forthwith and to INFORM this Court, within five (5) days from receipt hereof, of the date appellant was actually released from confinement.  Costs de oficio.

SO ORDERED.

Puno, (chairman), Sandoval-Gutierrez, Corona and Carpio-Morales, JJ., concur.

Page 238: Warrantless Arrests, Searches and Seizures and Privacy of Communication

238

[G.R. No. 123872.  January 30, 1998]

PEOPLE OF THE PHILIPPINES, plaintiff-appellee, vs. RUBEN MONTILLA y GATDULA, accused-appellant.

D E C I S I O N

REGALADO, J.:

Accused-Appellant Ruben Montilla y Gatdula, alias "Joy," was charged on August 22, 1994 for violating Section 4, Article II of the Dangerous Drugs Act of 1972, Republic Act No. 6425, as amended by Republic Act No. 7659, before the Regional Trial Court, Branch 90, of Dasmariñas, Cavite in an information which alleges:

That on or about the 20th day of June 1994, at Barangay Salitran, Municipality of Dasmariñas, Province of Cavite, Philippines and within the jurisdiction of this Honorable Court, the above-named accused, not being authorized by law, did then and there, wilfully, unlawfully and feloniously, administer, transport, and deliver twenty-eight (28) kilos of dried marijuana leaves, which are considered prohibited drugs, in violation of the provisions of R.A. 6425 thereby causing damage and prejudice to the public interest. [1]

The consequent arraignment conducted on September 14, 1994 elicited a plea of not guilty from appellant who was assisted therein by his counsel de parte.[2] Trial was held on scheduled dates thereafter, which culminated in a verdict of guilty in a decision of the trial court dated June 8, 1995 and which imposed the extreme penalty of death on appellant.  He was further ordered to pay a fine in the amount of P500,000.00 and to pay the costs of the proceedings.[3]

It appears from the evidence of the prosecution that appellant was apprehended at around 4:00 A.M. of June 20, 1994 near a waiting shed located at Barangay Salitran, Dasmariñas, Cavite by SPO1 Concordio Talingting and SPO1 Armando Clarin, both members of the Cavite Philippine National Police Command based in Dasmariñas.  Appellant, according to the two officers, was caught transporting 28 marijuana bricks contained in a traveling bag and a carton box, which marijuana bricks had a total weight of 28 kilos.

These two officers later asserted in court that they were aided by an informer in the arrest of appellant.  That informer, according to Talingting and Clarin, had informed them the day before, or on June 19, 1994 at about 2:00 P.M., that a drug courier, whom said informer could recognize, would be arriving somewhere in Barangay Salitran, Dasmariñas from Baguio City with an undetermined amount of marijuana.  It was the same informer who pinpointed to the arresting officers the appellant when the latter alighted from a passenger jeepney on the aforestated day, hour, and place. [4]

Upon the other hand, appellant disavowed ownership of the prohibited drugs.  He claimed during the trial that while he indeed came all the way from Baguio City, he traveled to Dasmariñas, Cavite with only some pocket money and without any luggage.  His sole purpose in going there was to look up his cousin who had earlier offered a prospective job at a garment factory in said locality, after which he would return to Baguio City.  He never got around to doing so as he was accosted by SPO1 Talingting and SPO1 Clarin at Barangay Salitran.

He further averred that when he was interrogated at a house in Dasmariñas, Cavite, he was never informed of his constitutional rights and was in fact even robbed of the P500.00 which he had with

Page 239: Warrantless Arrests, Searches and Seizures and Privacy of Communication

239

him.  Melita Adaci, the cousin, corroborated appellant's testimony about the job offer in the garment factory where she reportedly worked as a supervisor, [5] although, as the trial court observed, she never presented any document to prove her alleged employment.

In the present appellate review, appellant disputes the trial court's finding that he was legally caught in flagrante transporting the prohibited drugs.  This Court, after an objective and exhaustive review of the evidence on record, discerns no reversible error in the factual findings of the trial court.   It finds unassailable the reliance of the lower court on the positive testimonies of the police officers to whom no ill motives can be attributed, and its rejection of appellant's fragile defense of denial which is evidently self-serving in nature.

1. Firstly, appellant asserts that the court a quo grossly erred in convicting him on the basis of insufficient evidence as no proof was proffered showing that he wilfully, unlawfully, and feloniously administered, transported, and delivered 28 kilos of dried marijuana leaves, since the police officers "testified only on the alleged transporting of Marijuana from Baguio City to Cavite."

Further, the failure of the prosecution to present in court the civilian informant is supposedly corrosive of the People's cause since, aside from impinging upon appellant's fundamental right to confront the witnesses against him, that informant was a vital personality in the operation who would have contradicted the hearsay and conflicting testimonies of the arresting officers on how appellant was collared by them.

The pertinent provision of the penal law here involved, in Section 4 of Article II thereof, as amended, is as follows:

SEC. 4. Sale, Administration, Delivery, Distribution and Transportation of Prohibited Drugs. - The penalty of reclusion perpetua to death and a fine ranging from five hundred thousand pesos to ten million pesos shall be imposed upon any person who, unless authorized by law, shall sell, administer, deliver, give away to another, distribute, dispatch in transit or transport any prohibited drug, or shall act as a broker in any of such transactions.

Notwithstanding the provision of Section 20 of this Act to the contrary, if the victim of the offense is a minor, or should a prohibited drug involved in any offense under this Section be the proximate cause of the death of a victim thereof, the maximum penalty herein provided shall be imposed.

Now, the offense ascribed to appellant is a violation of the Dangerous Drugs Act, some of the various modes of commission[6] being the sale, administration, delivery, distribution, and transportation of prohibited drugs as set forth in the epigraph of Section 4, Article II of said law.  The text of Section 4 expands and extends its punitive scope to other acts besides those mentioned in its headnote by including these who shall sell, administer, deliver, give away to another, distribute, dispatch in transit or transport any prohibited drug, or shall act as a broker in any of such transactions."  Section 4 could thus be violated by the commission of any of the acts specified therein, or a combination thereof, such as selling, administering, delivering, giving away, distributing, dispatching in transit or transporting, and the like.

As already stated, appellant was charged with a violation of Section 4, the transgressive acts alleged therein and attributed to appellant being that he administered, delivered, and transported marijuana.  The governing rule with respect to an offense which may be committed in any of the different modes provided by law is that an indictment would suffice if the offense is alleged to have been committed in one, two or more modes specified therein.  This is so as allegations in the information of the various ways of committing the offense should be considered as a description of only one offense and the information cannot be dismissed on the ground of multifariousness.[7] In appellant's case, the prosecution adduced evidence clearly establishing that he transported marijuana from Baguio City to Cavite.  By that act alone of transporting the illicit drugs, appellant had already run afoul of that particular section of the statute, hence, appellant's asseverations must fail.

Page 240: Warrantless Arrests, Searches and Seizures and Privacy of Communication

240

The Court also disagrees with the contention of appellant that the civilian informer should have been produced in court considering that his testimony was "vital" and his presence in court was essential in order to give effect to or recognition of appellant's constitutional right to confront the witnesses arrayed by the State against him.  These assertions are, however, much too strained.  Far from compromising the primacy of appellant's right to confrontation, the non-presentation of the informer in this instance was justified and cannot be faulted as error.

For one, the testimony of said informer would have been, at best, merely corroborative of the declarations of SPO1 Talingting and SPO1 Clarin before the trial court, which testimonies are not hearsay as both testified upon matters in which they had personally taken part.  As such, the testimony of the informer could be dispensed with by the prosecution,[8] more so where what he would have corroborated are the narrations of law enforcers on whose performance of duties regularity is the prevailing legal presumption.  Besides, informants are generally not presented in court because of the need to hide their identities and preserve their invaluable services to the police.[9] Moreover, it is up to the prosecution whom to present in court as its witnesses, and not for the defense to dictate that course. [10] Finally, appellant could very well have resorted to the coercive process of subpoena to compel that eyewitness to appear before the court below,[11] but which remedy was not availed of by him.

2. Appellant contends that the marijuana bricks were confiscated in the course of an unlawful warrantless search and seizure.  He calls the attention of the Court to the fact that as early as 2:00 P.M. of the preceding day, June 19, 1994, the police authorities had already been apprised by their so-called informer of appellant's impending arrival from Baguio City, hence those law enforcers had the opportunity to procure the requisite warrant.  Their misfeasance should therefore invalidate the search for and seizure of the marijuana, as well as the arrest of appellant on the following dawn.  Once again, the Court is not persuaded.

Section 2, Article III of the Constitution lays down the general rule that a search and seizure must be carried out through or on the strength of a judicial warrant, absent which such search and seizure becomes "unreasonable" within the meaning of said constitutional provision. [12] Evidence secured on the occasion of such an unreasonable search and seizure is tainted and should be excluded for being the proverbial fruit of a poisonous tree.  In the language of the fundamental law, it shall be inadmissible in evidence for any purpose in any proceeding.  This exclusionary  rule  is  not,  however,  an  absolute  and  rigid  proscription.  Thus, (1) customs searches;[13] (2) searches of moving vehicles,[14] (3) seizure of evidence in plain view;[15] (4) consented searches;[16] (5) searches incidental to a lawful arrest;[17] and (6) "stop and frisk" measures[18] have been invariably recognized as the traditional exceptions.

In appellant's case, it should be noted that the information relayed by the civilian informant to the law enforcers was that there would be delivery of marijuana at Barangay Salitran by a courier coming from Baguio City in the "early morning" of June 20, 1994.  Even assuming that the policemen were not pressed for time, this would be beside the point for, under these circumstances, the information relayed was too sketchy and not detailed enough for the obtention of the corresponding arrest or search warrant.   While there is an indication that the informant knew the courier, the records do not reveal that he knew him by name.

While it is not required that the authorities should know the exact name of the subject of the warrant applied for, there is the additional problem that the informant did not know to whom the drugs would be delivered and at which particular part of the barangay there would be such delivery.  Neither did this asset know the precise time of the suspect's arrival, or his means of transportation, the container or contrivance wherein the drugs were concealed and whether the same were arriving together with, or were being brought by someone separately from, the courier.

On such bare information, the police authorities could not have properly applied for a warrant, assuming that they could readily have access to a judge or a court that was still open by the time they could make preparations for applying therefor, and on which there is no evidence presented by the defense.  In determining the opportunity for obtaining warrants, not only the intervening time is controlling but all the coincident and ambient circumstances should be considered, especially in rural areas.  In fact, the police had to form a surveillance team and to lay down a dragnet at the possible entry points to

Page 241: Warrantless Arrests, Searches and Seizures and Privacy of Communication

241

Barangay Salitran at midnight of that day notwithstanding the tip regarding the "early morning" arrival of the courier.  Their leader, SPO2 Cali, had to reconnoiter inside and around the barangay as backup, unsure as they were of the time when and the place in Barangay Salitran, where their suspect would show up, and how he would do so.

On the other hand, that they nonetheless believed the informant is not surprising for, as both SPO1 Clarin and SPO1 Talingting recalled, he had proved to be a reliable source in past operations. Moreover, experience shows that although information gathered and passed on by these assets to law enforcers are vague and piecemeal, and not as neatly and completely packaged as one would expect from a professional spymaster, such tip-offs are sometimes successful as it proved to be in the apprehension of appellant.  If the courts of justice are to be of understanding assistance to our law enforcement agencies, it is necessary to adopt a realistic appreciation of the physical and tactical problems of the latter, instead of critically viewing them from the placid and clinical environment of judicial chambers.

3. On the defense argument that the warrantless search conducted on appellant invalidates the evidence obtained from him, still the search on his belongings and the consequent confiscation of the illegal drugs as a result thereof was justified as a search incidental to a lawful arrest under Section 5(a), Rule 113 of the Rules of Court.  Under that provision, a peace officer or a private person may, without a warrant, arrest a person when, in his presence, the person to be arrested has committed, is actually committing, or is attempting to commit an offense.

A legitimate warrantless arrest, as above contemplated, necessarily cloaks the arresting police officer with authority to validly search and seize from the offender (1) dangerous weapons, and (2) those that may be used as proof of the commission of an offense. [19] On the other hand, the apprehending officer must have been spurred by probable cause in effecting an arrest which could be  classified  as  one  in cadence with  the instances of permissible arrests set out in Section 5(a).[20] These instances have been applied to arrests carried out on persons caught in flagrante delicto.  The conventional view is that probable cause, while largely a relative term the determination of which must be resolved according to the facts of each case, is understood as having reference to such facts and circumstances which could lead a reasonable, discreet, and prudent man to believe and conclude as to the commission of an offense, and that the objects sought in connection with the offense are in the place sought to be searched.[21]

Parenthetically, if we may digress, it is time to observe that the evidentiary measure for the propriety of filing criminal charges and, correlatively, for effecting a warrantless arrest, has been reduced and liberalized.  In the past, our statutory rules and jurisprudence required prima facie evidence, which was of a higher degree or quantum,[22] and was even used with dubiety as equivalent to "probable cause." Yet, even in the American jurisdiction from which we derived the term and its concept, probable cause is understood to merely mean a reasonable ground for belief in the existence of facts warranting the proceedings complained of,[23] or an apparent state of facts found to exist upon reasonable inquiry which would induce a reasonably intelligent and prudent man to believe that the accused person had committed the crime.[24]

Felicitously, those problems and confusing concepts were clarified and set aright, at least on the issue under discussion, by the 1985 amendment of the Rules of Court which provides in Rule 112 thereof that the quantum of evidence required in preliminary investigation is such evidence as suffices to "engender a well founded belief" as to the fact of the commission of a crime and the respondent's probable guilt thereof.[25] It has the same meaning as the related phraseology used in other parts of the same Rule, that is, that the investigating fiscal "finds cause to hold the respondent for trial," or where "a probable cause exists."[26] It should, therefore, be in that sense, wherein the right to effect a warrantless arrest should be considered as legally authorized.

In the case at bar, as soon as appellant had alighted from the passenger jeepney the informer at once indicated to the officers that their suspect was at hand by pointing to him from the waiting shed. SPO1 Clarin recounted that the informer told them that the marijuana was likely hidden inside the traveling bag and carton box which appellant was carrying at the time.  The officers thus realized that he was their man even if he was simply carrying a seemingly innocent looking pair of luggage for personal effects.  Accordingly, they approached appellant, introduced themselves as policemen, and requested

Page 242: Warrantless Arrests, Searches and Seizures and Privacy of Communication

242

him to open and show them the contents of the traveling bag, which appellant voluntarily and readily did.  Upon cursory inspection by SPO1 Clarin, the bag yielded the prohibited drugs, so, without bothering to further search the box, they brought appellant and his luggage to their headquarters for questioning.

Appellant insists that the mere fact of seeing a person carrying a traveling bag and a carton box should not elicit the slightest suspicion of the commission of any crime since that is normal.  But, precisely, it is in the ordinary nature of things that drugs being illegally transported are necessarily hidden in containers and concealed from view.  Thus, the officers could reasonably assume, and not merely on a hollow suspicion since the informant was by their side and had so informed them, that the drugs were in appellant's luggage.  It would obviously have been irresponsible, if not downright absurd under the circumstances, to require the constable to adopt a "wait and see" attitude at the risk of eventually losing the quarry.

Here, there were sufficient facts antecedent to the search and seizure that, at the point prior to the search, were already constitutive of probable cause, and which by themselves could properly create in the minds of the officers a well-grounded and reasonable belief that appellant was in the act of violating the law.  The search yielded affirmance both of that probable cause and the actuality that appellant was then actually committing a crime by illegally transporting prohibited drugs.  With these attendant facts, it is ineluctable that appellant was caught in flagrante delicto, hence his arrest and the search of his belongings without the requisite warrant were both justified.

Furthermore, that appellant also consented to the search is borne out by the evidence. To repeat, when the officers approached appellant and introduced themselves as policemen, they asked him about the contents of his luggage, and after he replied that they contained personal effects, the officers asked him to open the traveling bag.  Appellant readily acceded, presumably or in all likelihood resigned to the fact that the law had caught up with his criminal activities.  When an individual voluntarily submits to a search or consents to have the same conducted upon his person or premises, he is precluded from later complaining thereof.

After all, the right to be secure from unreasonable search may, like other rights, be waived either expressly or impliedly.[27] Thus, while it has been held that the silence of the accused during a warrantless search should not be taken to mean consent to the search but as a demonstration of that person's regard for the supremacy of the law,[28] the case of herein appellant is evidently different for, here, he spontaneously performed affirmative acts of volition by himself opening the bag without being forced or intimidated to do so, which acts should properly be construed as a clear waiver of his right.[29]

4. Appellant likewise harps on the alleged failure of the prosecution to "legally, properly and adequately establish that the 28 bricks of marijuana allegedly confiscated from (him) were the same marijuana examined by the forensic chemist and presented in court."  Indeed, the arresting officers did not identify in court the marijuana bricks seized from appellant since, in fact they did not have to do so.   It should be noted that the prosecution presented in the court below and formally offered in evidence those 28 bricks of marijuana together with the traveling bag and the carton box in which the same were contained.  The articles were properly marked as confiscated evidence and proper safeguards were taken to ensure that the marijuana turned over to the chemist for examination, and which subsequently proved positive as such, were the same drugs taken from appellant.  The trial court, therefore, correctly admitted them in evidence, satisfied that the articles were indubitably no other than those taken from appellant.

Complementarily, the corpus delicti was firmly established by SPO1 Clarin and SPO1 Talingting who categorically related that when they had ascertained that the contents of the traveling bag of appellant appeared to be marijuana, they forthwith asked him where he had come from, and the latter readily answered "Baguio City," thus confirming the veracity of the report of the informer.  No other conclusion can therefore be derived than that appellant had transported the illicit drugs all the way to Cavite from Baguio City.  Coupled with the presentation in court of the subject matter of the crime, the marijuana bricks which had tested positive as being indian hemp, the guilt of appellant for transporting the prohibited drugs in violation of the law is beyond doubt.

Appellant questions the interrogation conducted by the police authorities, claiming that he was not allowed to communicate with anybody, and that he was not duly informed of his right to remain silent and

Page 243: Warrantless Arrests, Searches and Seizures and Privacy of Communication

243

to have competent and independent counsel preferably of his own choice.  Indeed, appellant has a point.  The police authorities here could possibly have violated the provision of Republic Act No. 7438[30] which defines certain rights of persons arrested, detained, or under custodial investigation, as well as the duties of the arresting, detaining, and investigating officers, and providing corresponding penalties for violations thereof.

Assuming the existence of such irregularities, however, the proceedings in the lower court will not necessarily be struck down.  Firstly, appellant never admitted or confessed anything during his custodial investigation.  Thus, no incriminatory evidence in the nature of a compelled or involuntary confession or admission was elicited from him which would otherwise have been inadmissible in evidence.  Secondly and more importantly, the guilt of appellant was clearly established by other evidence adduced by the prosecution, particularly the testimonies of the arresting officers together with the documentary and object evidence which were formally offered and admitted in evidence in the court below.

5. The reversible error of the trial court lies in its imposition of the penalty of death on appellant.   As amended by Republic Act No. 7659, Section 20, Article IV of the Dangerous Drugs Act now provides inter alia that the penalty in Section 4 of Article II shall be applied if the dangerous drugs involved is, in the case of indian hemp or marijuana, 750 grams or more.  In said Section 4, the transporting of prohibited drugs carries with it the penalty of reclusion perpetua to death and a fine ranging from five hundred thousand pesos to ten million pesos.  Thus, the law prescribes a penalty composed of two indivisible penalties, reclusion perpetua and death.  In the present case, Article 63 of the Revised Penal Code consequently provides the rules to be observed in the application of said penalties.

As found by the trial court, there were neither mitigating nor aggravating circumstances attending appellant's violation of the law, hence the second paragraph of Article 63 must necessarily apply, in which case the lesser penalty of reclusion perpetua is the proper imposable penalty.  Contrary to the pronouncement of the court a quo, it was never intended by the legislature that where the quantity of the dangerous drugs involved exceeds those stated in Section 20, the maximum penalty of death shall be imposed.  Nowhere in the amendatory law is there a provision from which such a conclusion may be gleaned or deduced.  On the contrary, this Court has already concluded that Republic Act No. 7659 did not amend Article 63 of the Revised Penal Code,[31] the rules wherein were observed although the cocaine subject of that case was also in excess of the quantity provided in Section 20.

It is worth mentioning at this juncture that the law itself provides a specific penalty where the violation thereof is in its aggravated form as laid down in the second paragraph of Section 4 whereby, regardless of Section 20 of Article IV, if the victim is a minor, or should a prohibited drug involved in any offense in said section be the proximate cause of the death of a victim thereof, the maximum penalty shall be imposed.[32] While the minority or the death of the victim will increase the liability of the offender, these two facts do not constitute generic aggravating circumstances, as the law simply provides for the imposition of the single indivisible penalty of death if the offense is attended by either of such factual features.   In that situation, obviously the rules on the graduation of penalties in Article 63 cannot apply.  In herein appellant's case, there was neither a minor victim nor a consequent death of any victim.  Hence, the basic rules in Article 63 of the Code govern.

WHEREFORE, the judgment of the Regional Trial Court, Branch 90, of Dasmariñas, Cavite in Criminal Case No. 3401-94 is hereby MODIFIED in the sense that accused-appellant Ruben Montilla y Gatdula shall suffer the penalty of reclusion perpetua.  In all other respects, the judgment of the trial court is hereby AFFIRMED, with costs against accused-appellant.

SO ORDERED.

Narvasa, C.J., Davide, Jr., Romero, Bellosillo, Kapunan, Mendoza, Francisco, and  Martinez, JJ, concur.

Melo, and Puno, JJ., join Panganiban J., separate opinion.Vitug, J., concur but reserve his vote on the discussion on the warrantless search of appellant as his

incidental to a lawful arrest.

Page 244: Warrantless Arrests, Searches and Seizures and Privacy of Communication

244

[G.R. No. 143944.  July 11, 2002]

THE PEOPLE OF THE PHILIPPINES, plaintiff-appellee, vs. BASHER BONGCARAWAN y MACARAMBON, accused-appellant.

D E C I S I O N

PUNO, J.:

This is an appeal from the Decision[1] dated December 27, 1999 of the Regional Trial Court of Iligan City, Branch 06, in Criminal Case No. 06-7542, finding accused Basher Bongcarawan y Macarambon guilty beyond reasonable doubt of violation of Section 16, Article III of Republic Act No. 6425 [2] as amended, and sentencing him to suffer the penalty of reclusion perpetua, and to pay a fine of Five Hundred Thousand Pesos (P500,000.00) without subsidiary imprisonment in case of insolvency.

Accused Basher Bongcarawan y Macarambon was charged in an Information which reads, thus:

“That on or about March 13, 1999, in the City of Iligan, Philippines, and within the jurisdiction of this Honorable Court, the said accused, without authority of law, did then and there wilfully, unlawfully and feloniously have in his possession, custody and control eight (8) packs of Methamphetamine Hydrochloride, a regulated drug commonly known as Shabu, weighing approximately 400 grams, without the corresponding license or prescription.

Contrary to and in violation of Section 16, Article III of RA 6425, otherwise known as the Dangerous Drugs Act of 1972, as amended by RA 7659.”[3]

During the arraignment, the accused pleaded not guilty.  Trial ensued.

Evidence for the prosecution shows that on March 11, 1999, an interisland passenger ship, M/V Super Ferry 5, sailed from Manila to Iligan City.  At about 3:00 a.m. on March 13, 1999, the vessel was about to dock at the port of Iligan City when its security officer, Mark Diesmo, received a complaint from passenger Lorena Canoy about her missing jewelry. Canoy suspected one of her co-passengers at cabin no. 106 as the culprit.  Diesmo and four (4) other members of the vessel security force accompanied Canoy to search for the suspect whom they later found at the economy section. [4] The suspect was identified as the accused, Basher Bongcarawan. The accused was informed of the complaint and was invited to go back to cabin no. 106.  With his consent, he was bodily searched, but no jewelry was found.  He was then escorted by two (2) security agents back to the economy section to get his baggage.  The accused took a Samsonite suitcase and brought this back to the cabin. When requested by the security, the accused opened the suitcase, revealing a brown bag and small plastic packs containing white crystalline substance.  Suspecting the substance to be “shabu,” the security personnel immediately reported the matter to the ship captain and took pictures of the accused beside the suitcase and its contents.  They also called the Philippine Coast Guard for assistance.[5] At about 6:00 a.m., Lt. Robert Patrimonio, YN Aurelio Estoque, CD2 Phoudinie Lantao and RM3 Merchardo De Guzman of the Philippine Coast Guard arrived and took custody of the accused and the seized items--the Samsonite

Page 245: Warrantless Arrests, Searches and Seizures and Privacy of Communication

245

suitcase, a brown bag[6] and eight (8) small plastic packs of white crystalline substance.[7] When asked about the contraband articles, the accused explained that he was just requested by a certain Alican “Alex” Macapudi to bring the suitcase to the latter’s brother in Iligan City. [8] The accused and the seized items were later turned over by the coast guard to the Presidential Anti-Organized Crime Task Force (PAOCTF).  Chief Inspector Graciano Mijares and his men brought the accused to the PAOCTF Headquarters,[9] while the packs of white crystalline substance were sent to the NBI Regional Office in Cagayan de Oro City for laboratory examination.  NBI Forensic Chemist Nicanor Cruz later confirmed the substance to be methamphetamine hydrochloride, commonly known as “shabu,” weighing 399.3266 grams.[10]

The accused testified and proffered  his own version.  On March 11, 1999, at about 10:00 p.m., he was in Quiapo, Manila where he met Alican “Alex” Macapudi, a neighbor who has a store in Marawi City.  He was requested by Macapudi to bring a Samsonite suitcase containing sunglasses and watches to Iligan City, and to give it to Macapudi’s brother at the Iligan port. He boarded the M/V Super Ferry 5 on the same night, carrying a big luggage full of clothes, a small luggage or “maleta” containing the sunglasses and brushes he bought from Manila, and the Samsonite suitcase of Macapudi. [11] He stayed at cabin no. 106.  At about 4:00 a.m of March 13, 1999, as the vessel was about to dock at the Iligan port, he took his baggage and positioned himself at the economy section to be able to disembark ahead of the other passengers.  There, he met a friend, Ansari Ambor.  While they were conversing, five (5) members of the vessel security force and a woman whom he recognized as his co-passenger at cabin no. 106 came and told him that he was suspected of stealing jewelry.  He voluntarily went with the group back to cabin no. 106 where he was frisked.  Subsequently, he was asked to get his baggage, so he went back to the economy section and took the big luggage and Macapudi’s Samsonite suitcase.  He left the small “maleta” containing sunglasses and brushes for fear that they would be confiscated by the security personnel.  When requested, he voluntarily opened the big luggage, but refused to do the same to the Samsonite suitcase which he claimed was not his and had a secret combination lock.  The security personnel forcibly opened the suitcase and found packs of white crystalline substance inside which they suspected to be “shabu.” They took pictures of him with the merchandise, and asked him to sign a turn over receipt which was later given to the Philippine Coast Guard, then to the PAOCTF.[12]

On December 27, 1999, the trial court rendered judgment, the dispositive portion of which reads:

“WHEREFORE, the court finds the accused Basher Bongcarawan y Macarambon GUILTY beyond reasonable doubt as principal of the offense of violation of Section 16, Art. III, R.A. No. 6425 as amended by R.A. No. 7659 and hereby imposes upon him the penalty of RECLUSION PERPETUA and a fine of FIVE HUNDRED THOUSAND (P500,000.00) PESOS, without subsidiary imprisonment in case of insolvency.

Having been under preventive imprisonment since March 13, 1999 until the present, the period of such preventive detention shall be credited in full in favor of the accused in the service of his sentence.

The 399.3266 grams of methamphetamine hydrochloride or shabu is hereby ordered delivered to the National Bureau of Investigation for proper disposition.

SO ORDERED.”[13]

Hence, this appeal where the accused raises the following assignment of errors:

“I.

THE COURT A QUO ERRED IN SO HOLDING THAT THE DRUG CONFISCATED IS ADMISSIBLE IN EVIDENCE AGAINST THE ACCUSED/APPELLANT.

II.

Page 246: Warrantless Arrests, Searches and Seizures and Privacy of Communication

246

THE COURT A QUO ERRED IN SO HOLDING THAT THE APPELLANT OWNED THE CONFISCATED EVIDENCE AND THEREFORE ADMISSIBLE IN EVIDENCE AGAINST HIM.“[14]

On the first assignment of error, the accused-appellant contends that the Samsonite suitcase containing the methamphetamine hydrochloride or “shabu” was forcibly opened and searched without his consent, and hence, in violation of his constitutional right against unreasonable search and seizure. Any evidence acquired pursuant to such unlawful search and seizure, he claims, is inadmissible in evidence against him.  He also contends that People v. Marti[15] is not applicable in this case because a vessel security personnel is deemed to perform the duties of a policeman.

The contentions are devoid of merit.

The right against unreasonable search and seizure is a fundamental right protected by the Constitution.[16] Evidence acquired in violation of this right shall be inadmissible for any purpose in any proceeding.[17] Whenever this right is challenged, an individual may choose between invoking the constitutional protection or waiving his right by giving consent to the search and seizure.   It should be stressed, however, that protection is against transgression committed by the government or its agent.  As held by this Court in the case of People v. Marti,[18] “[i]n the absence of governmental interference, liberties guaranteed by the Constitution cannot be invoked against the State.” [19] The constitutional proscription against unlawful searches and seizures applies as a restraint directed only against the government and its agencies tasked with the enforcement of the law.  Thus, it could only be invoked against the State to whom the restraint against arbitrary and unreasonable exercise of power is imposed.[20]

In the case before us, the baggage of the accused-appellant  was searched by the vessel security personnel. It was only after they found “shabu” inside the suitcase that they called the Philippine Coast Guard for assistance.  The search and seizure of the suitcase and the contraband items was therefore carried out without government intervention, and hence, the constitutional protection against unreasonable search and seizure does not apply.

There is no merit in the contention of the accused-appellant that the search and seizure performed by the vessel security personnel should be considered as one conducted by the police authorities for like the latter, the former are armed and tasked to maintain peace and order.  The vessel security officer in the case at bar is a private employee and does not discharge any governmental function.   In contrast, police officers are agents of the state tasked with the sovereign function of enforcement of the law.  Historically and until now, it is against them and other agents of the state that the protection against unreasonable searches and seizures may be invoked.

On the second assignment of error, the accused-appellant contends that he is not the owner of the Samsonite suitcase and he had no knowledge that the same contained “shabu.”  He submits that without knowledge or intent to possess the dangerous drug, he cannot be convicted of the crime charged. [21]

We are not persuaded.

In a prosecution for illegal possession of dangerous drugs, the following facts must be proven beyond reasonable doubt, viz:  (1) that the accused is in possession of the object identified as a prohibited or a regulated drug;  (2) that such possession is not authorized by law; and (3) that the accused freely and consciously possessed the said drug. [22] The first two elements were sufficiently proven in this case, and were in fact undisputed.  We are left with the third.

As early as 1910 in the case of United States v. Tan Misa,[23] this Court has ruled that to warrant conviction, the possession of dangerous drugs must be with knowledge of the accused, or that animus possidendi existed together with the possession or control of such articles. [24] It has been ruled, however, that possession of dangerous drugs constitutes prima facie evidence of knowledge or animus possidendi sufficient to convict an accused in the absence of a satisfactory explanation of such possession.[25] Hence, the burden of evidence is shifted to the accused to explain the absence of knowledge or animus possidendi.[26]

Page 247: Warrantless Arrests, Searches and Seizures and Privacy of Communication

247

In this respect, the accused-appellant has utterly failed.  His testimony, uncorroborated, self-serving and incredulous, was not given credence by the trial court.  We find no reason to disagree.  Well-settled is the rule that in the absence of palpable error or grave abuse of discretion on the part of the trial judge, the trial court’s evaluation of the credibility of witnesses will not be disturbed on appeal. [27] Moreover, evidence must be credible in itself to deserve credence and weight in law.  In this case, the accused-appellant admits that when he was asked to get his baggage, he knew it would be inspected. [28] Why he got the Samsonite suitcase allegedly not owned by him and which had a combination lock known only to the owner remains unclear.  He also claims that he did not present his small “maleta” for inspection for fear that its contents consisting of expensive sunglasses and brushes would be confiscated, [29] but he brought the Samsonite suitcase which is not his and also contained expensive sunglasses, and even watches.[30]

The things in possession of a person are presumed by law to be owned by him. [31] To overcome this presumption, it is necessary to present clear and convincing evidence to the contrary.  In this case, the accused points to a certain Alican “Alex” Macapudi as the owner of the contraband, but presented no evidence to support his claim. As aptly observed by the trial judge:

“First, who is Alex Macap[u]di aka Ali[c]an Macap[u]di?  Does he really exist or simply a figment of the imagination?  He says that Alex Macap[u]di is a friend and a fellow businessman who has a stall selling sunglasses in Marawi City.  But no witnesses were presented to prove that there is such a living, breathing, flesh and blood person named Alex Macap[u]di who entrusted the Samsonite to the accused.  Surely, if he does exist, he has friends, fellow businessmen and acquaintances who could testify and support the claim of the accused.”[32]

Mere denial of ownership will not suffice especially if, as in the case at bar, it is the keystone of the defense of the accused-appellant.  Stories can easily be fabricated. It will take more than bare-bone allegations to convince this Court that a courier of dangerous drugs is not its owner and has no knowledge or intent to possess the same.

WHEREFORE, the decision of the Regional Trial Court of Iligan City, Branch 06, in Criminal Case No. 06-7542, convicting accused-appellant Basher Bongcarawan of violation of Section 16, Article III of Republic Act No. 6425, as amended, and sentencing him to suffer the penalty of Reclusion Perpetua and to pay a fine of Five Hundred Thousand Pesos (P500,000.00) without subsidiary imprisonment in case of insolvency, is AFFIRMED.

Costs against the accused-appellant.

SO ORDERED.

Panganiban, Sandoval-Gutierrez, and Carpio, JJ., concur.

G.R. No. L-45950             June 20, 1938

LEONA PASION VIUDA DE GARCIA, petitioner, vs.DIEGO LOCSIN, Judge of First Instance of Tarlac, FELIX IMPERIAL, Provincial Fiscal of Tarlac, and the ANTI-USURY BOARD, respondents.

Benigo S. Aquino and Marcial P. Lichauco for petitionerAdolfo N. Feliciano for the respondent Anti-Usury Board.Office of the Solicitor-General Tuason for other respondents.

LAUREL, J.:

Page 248: Warrantless Arrests, Searches and Seizures and Privacy of Communication

248

This is a petition for mandamus presented to secure the annulment of a search warrant and two orders of the respondent judge, and the restoration of certain documents alleged to have been illegally seized by an agent of the Anti-Usuary Board.

It appears that on November 10, 1934, Mariano G. Almeda, an agent of the Anti-Usuary Board, obtained from the justice of the peace of Tarlac, Tarlac, a search warrant(Exhibit B) commanding any officer of the law to search the person, house or store of the petitioner at Victoria, Tarlac, for "certain books, lists, chits, receipts, documents and other papers relating to her activities as usurer." The search warrant was issued upon an affidavit given by the said Almeda "that he has and there (is) just and probable cause to believe and he does believe that Leona Pasion de Garcia keeps and conceals in her house and store at Victoria, Tarlac, certain books, lists, chits, receipts, documents, and other papers relating to her activities as usurer, all of which is contrary to the statute in such cases made and provided." On the same date, the said Mariano G. Almeda, accompanied by a captain of the Philippine Constabulary, went to the office of the petitioner in Victoria, Tarlac and, after showing the search warrant to the petitioner's bookkeeper, Alfredo Salas, and, without the presence of the petitioner who was ill and confined at the time, proceeded with the execution thereof. Two packages of records and a locked filing cabinet containing several Papers and documents were seized by Almeda and a receipt therefor issued by him to Salas. The papers and documents seized were kept for a considerable length of time by the Anti-Usury Board and thereafter were turned over by it to the respondent fiscal who subsequently filed, in the Court of First Instance of Tarlac, six separate criminal cases against the herein petitioner for violation of the Anti-Usury Law. On several occasions, after seizure, the petitioner, through counsel, demanded from the respondent Anti-Usury Board the return of the documents seized. On January 7. and, by motion, on June 4, 1937, the legality of the search warrant was challenged by counsel for the petitioner in the six criminal cases and the devolution of the documents demanded. By resolution of October 5, 1937, the respondent Judge of First Instance denied the petitioner's motion of June 4 for the reason that though the search warrant was illegal, there was a waiver on the part of the petitioner. "En el caso presente," declared the respondent judge, "teniendo en cuenta que la acusada Por si o por medio de su representante, no presento protests alguna contra el registro de autos, at verificarse el mismo, o despues de un tiempo rezonable, el juzgado declare que la citada con su silencio y conducta, ha renunciado implicitanmente a su derecho a no ser sometido a un registro irrazonable, por lo que no le es pemitido quejarse despues, puesto que cualquier defecto queha adolecido lo expedicion de la orden de registro y su ejecucion, ha quidado implilcitamente subsanado." A motion for reconsideration was presented but was denied by order of January 3, 1938. Petitioner registered her exception. The resolution of October 5, 1937 and the order of January 3, 1938 are sought, together with the search warrant, Exhibit B, to be nullified in these proceedings.

Paragraph 3, section 1 of the bill of right of our Constitution provides as follows:

The right of the people to be secure in their persons, houses, papers, and effects against unreasonable searches and seizures shall not be violated, and no warrants shall issue but upon probable cause, to be judge after examination under oath or affirmation of the complainant and the witnesses he may produce, and particularly describing the place to be searched, and the persons or things to be seized.

Freedom from unreasonable searches and seizures is declared a popular right and for a search warrant to be valid, (1) it must be issued upon probable cause; (2) the probable cause must be determined by the judge himself and not by the applicant or any other person; (3) in the determination of probable cause, the judge must examine, under oath or affirmation, the complainant and such witnesses as the latter may produce; and (4) the warrant issued must particularly describe the place to be searched and persons or things to be seized. These requirements are complemented by the Code of Criminal Procedure (G. O. No. 58), particularly with reference to the duration of the validity of the search warrant and the obligation of the officer seizing the property to deliver the same to the corresponding court (secs. 102-104). On more than one occasion, since the approval of the Constitution, we had emphasized the necessity of adherence to the constitutional requirements on this subject (Alvarez vs. Court of First Instance of Tayabas and Anti-Usury Board [1937], 35 Off. Gaz., 1183; People vs. Sy Juco [1937], G.R. No. 41957; Rodriguez vs.

Page 249: Warrantless Arrests, Searches and Seizures and Privacy of Communication

249

Villamiel [1937], G.R. No. 44328; and Molo vs. Yatco [1936], 35 Off. Gaz., 1935) and we do not deem it necessary to reiterate what has been said or observed in these cases.

In the instant case the existence of probable cause was determined not by the judge himself but by the applicant. All that the judge did was to accept as true the affidavit made by agent Almeda. He did not decide for himself. It does not appear that he examined the applicant and his witnesses, if any. Even accepting the description of the properties to be seized to be sufficient and on the assumption that the receipt issued is sufficiently detailed within the meaning of the law, the properties seized were not delivered to the court which issued the warrant, as required by law. (See, secs. 95 and 104, G. O. No. 58.) instead, they were turned over to the respondent provincial fiscal and used by him in building up cases against the petitioner. Considering that at the time the warrant was issued there was no case pending against the petitioner, the averment that the warrant was issued primarily for exploration purposes is not without basis. The lower court is, therefore, correct in reaching the conclusion that the search warrant (Exhibit B) was illegally issued by the justice of the peace of Tarlac, Tarlac.

The important question presented is whether upon the facts and under the circumstances of the present case, there has been a waiver by the petitioner of her constitutional immunity against unreasonable searches and seizures. While the Solicitor-General admits that, in the light of decisions of this court, the search warrant was illegally issued, he maintains "(1) that the petitioner had waived her constitutional right by her acquiescence after the search and seizure, and (2) that the application for the return of the documents illegally seized was made after an unreasonable length of time after the date of seizure." Doubtless, the constitutional immunity against unreasonable searches and seizures is a personal right which may be waived. (People vs. Kagui Malasugui, 34 Off. Gaz., pp. 2163, 2164; 56 C.J., pp. 1178, 1179; Cf. Rodriguez vs. Villamiel, supra.) The waiver may be either express or implied (67 C.J., p. 304). No express waiver has been made in the case before us. It is urged, however, that there has been a waiver by implication. It is well-settled that to constitute a waiver of a constitutional right, it must appear, first, that the right exists; secondly, that the persons involved had knowledge, either actual or constructive, of the existence of such right; and, lastly, that said person had an actual intention to relinquish the right. (67 C. J., 299.) It is true that the petitioner did not object to the legality of the search when it was made. She could not have objected because she was sick and was not present when the warrant was served upon Alfredo Salas. Certainly, the constitutional immunity from unreasonable searches and seizures, being a personal one, cannot be waived by anyone except the person whose rights are invaded or one who is expressly authorized to do so in his or her behalf. (56 C. J., p. 1183.) Of course, the petitioner came to know later of the seizure of some of her papers and documents. But this was precisely the reason why she sent her attorneys to the office of the Anti-Usuary Board to demand the return of the documents seized. In any event, the failure on the part of the petitioner and her bookkeeper to resist or object to the execution of the warrant does not constitute an implied waiver of constitutional right. It is, as Judge Cooley observes, but a submission to the authority of the law. (Const. Lim., 8th ed., Vol., I, p. 630.) As the constitutional guaranty is not dependent upon any affirmative act of the citizen, the courts do not place the citizen in the position of either contesting an officer's authority by force, or waiving his constitutional rights; but instead they hold that a peaceful submission to a search or seizure is not a consent or an invitation thereto, but is merely a demonstration of regard for the supremacy of the law. (56 C.J., pp. 1180, 1181.)

As a general proposition, it may be admitted that waiver may be the result of a failure to object within a reasonable time to a search and seizure illegally made. It must be observed, however, that the petitioner, on several occasions, and prior to the filing of criminal actions against her, had demanded verbally, through counsel, the return by the Anti-Usuary Board of the properties seized. This is admitted by Adolfo N. Feliciano, acting chief of the board, who said that the demand was refused simply because no habiamos terminado con nuestra investigacion. (T.s.n., pp. 24-25.) On July 7, 1936, counsel for the petitioner wrote a letter to the Anti-Usuary Board demanding again the return of the documents withheld. And in connection with the criminal cases pending against the petitioner, similar demands were made on January 7, 1937 and on June 4, 1937. In the light of these circumstances, we find that the petitioner did not waive her constitutional right. The delay in making demand for the return of the documents seized is not such as to result in waiver by implication.

Page 250: Warrantless Arrests, Searches and Seizures and Privacy of Communication

250

In view of the foregoing, the writ prayed for is granted. The search warrant, Exhibit B, is hereby declared void and of no effect; the orders of October 5, 1937 and January 3, 1938 of the respondent judge are set aside; and the respondents Anti-Usuary Board and the provincial fiscal of Tarlac or those acting in their behalf, are hereby ordered to return and restore to the petitioner all the properties, documents, papers and effects illegally seized from her, within forty-eight (48) hours from the time this decision becomes final. Without costs. So ordered.

Avanceña, C.J., Villa-Real, Abad Santos, Imperial, Diaz and Concepcion, JJ., concur.

G.R. No. L-95630 June 18, 1992

SPOUSES LEOPOLDO and MA. LUISA VEROY, petitioners, vs.THE HON. WILLIAM L. LAYAGUE, Presiding Judge, Branch XIV, Regional Trial Court at Davao City; and BRIG. GEN. PANTALEON DUMLAO, Commanding General, PC-Criminal Investigation Service,respondents.

 

PARAS, J.:

This was originally a petition for certiorari, mandamus and prohibition under Rule 65 of the Rules of Court:certiorari, to review the Order of the respondent Judge dated October 2, 1990 denying herein petitioner's Motion for Hospital Confinement; mandamus, to compel respondent Judge to resolve petitioners' long pending motion for bail; and prohibition, to enjoin further proceedings on the ground that the legal basis therefore is unconstitutional for being violative of the due process and equal protection clauses of the Constitution.

The facts of this case are as follows:

Petitioners are husband and wife who owned and formerly resided at No. 13 Isidro St., Skyline Village. Catalunan Grande, Davao City. When petitioner Leopoldo Veroy was promoted to the position of Assistant Administrator of the Social Security System sometime in June, 1988, he and his family transferred to 130 K-8th St., East Kamias, Quezon City, where they are presently residing. The care and upkeep of their residence in Davao City was left to two (2) houseboys, Jimmy Favia and Eric Burgos, who had their assigned quarters at a portion of the premises. The Veroys would occasionally send money to Edna Soguilon for the salary of the said houseboys and other expenses for the upkeep of their house. While the Veroys had the keys to the interior of the house, only the key to the kitchen, where the circuit breakers were located, was entrusted to Edna Soguilon to give her access in case of an emergency. Hence, since 1988, the key to the master's bedroom as well as the keys to the children's rooms were retained by herein Petitioners so that neither Edna Soguilon nor the caretakers could enter the house.

On April 12, 1990, Capt. Reynaldo Obrero of the Talomo Patrol Station, PC/INP, acting upon a directive issued by Metrodiscom Commander Col. Franco Calida, raided the house of herein petitioners in Davao City on information that the said residence was being used as a safehouse of rebel soldiers. They were able to enter the yard with the help of the caretakers but did not enter the house since the owner was not present and they did not have a search warrant. Petitioner Ma. Luisa was contacted by telephone in her Quezon City residence by Capt. Obrero to ask permission to search the house in Davao City as it was reportedly being used as a hideout and recruitment center of rebel soldiers. Petitioner Ma. Luisa Veroy responded that she is flying to Davao City to witness the search but relented if the search would not be conducted in the presence of Major Ernesto Macasaet, an officer of the PC/INP, Davao City and a long time family friend of the Veroys. The authority given by Ma. Luisa Veroy was relayed by Capt. Obrero to Major Macasaet who answered that Ma. Luisa Veroy has called him twice by telephone on the matter and that the permission was given on the condition that the search be conducted in his presence.

Page 251: Warrantless Arrests, Searches and Seizures and Privacy of Communication

251

The following day, Capt. Obrero and Major Macasaet met at the house of herein petitioners in Skyline Village to conduct the search pursuant to the authority granted by petitioner Ma. Luisa Veroy. The caretakers facilitated their entry into the yard, and using the key entrusted to Edna Soguilon, they were able to gain entrance into the kitchen. However, a locksmith by the name of George Badiang had to be employed to open the padlock of the door leading to the children's room. Capt. Obrero and Major Macasaet then entered the children's room and conducted the search. Capt. Obrero recovered a .45 cal. handgun with a magazine containing seven (7) live bullets in a black clutch bag inside an unlocked drawer. Three (3) half-full jute sacks containing printed materials of RAM-SFP (samples of which were attached as Annexes "H" and "H-1" of the petition) (Rollo, pp. 49-55) were also found in the children's room. A search of the children's recreation and study area revealed a big travelling bag containing assorted polo shirts, men's brief, two (2) pieces polo barong and short sleeve striped gray polo. sweat shirt, two (2) pairs men's socks, a towel made in U.S.A., one blanket, a small black bag, Gandhi brand, containing a book entitled "Islamic Revolution Future Path of the Nation", a road map of the Philippines, a telescope, a plastic bag containing assorted medicines and religious pamphlets was found in the master's bedroom. Sgt. Leo Justalero was instructed by Capt. Obrero to make an inventory and receipt of the articles seized, in the house (Annex "F" of the Petition, Rollo, p. 48). Said receipt was signed by Eric Burgos, one of the caretakers, and George Badiang, the locksmith, as witnesses. Sgt. Justalero turned over the articles to Sgt. Rodolfo Urbano at the police station.

The case was referred for preliminary investigation to Quezon City Assistant Prosecutor Rodolfo Ponferrada who was designated Acting Provincial Prosecutor for Davao City by the Department of Justice through Department Order No. 88 dated May 16, 1990. In a resolution dated August 6, 1990, Fiscal Ponferrada recommended the filing of an information against herein petitioners for Violation of Presidential Decree No. 1866 (Illegal Possession of Firearms and Ammunitions in Furtherance of Rebellion) (Annex "L" of the Petition, Rollo, p. 71). Hence, on August 8, 1990. an Information for the said offense was filed by the Office of the City Prosecutor of Davao City before the Regional Trial Court, 11th Judicial Region, Davao City, docketed as Criminal Case No. 20595-90 and entitled "People of the Philippines v. Atty. Leopoldo Veroy and Mrs. Maria Luisa Veroy" (Annex "K" of the Petition,Rollo, p. 70). No bail was recommended by the prosecution.

The aforementioned resolution dated August 6, 1990 of Fiscal Ponferrada was received by the petitioners on August 13, 1990. On the same day, the latter filed a Motion for Bail before herein respondent Judge Layague which was denied on August 17, 1990 for being premature since at that time, petitioners had not yet been arrested. Despite the fact that the warrants for their arrest have not yet been served on them, herein petitioners voluntarily surrendered themselves to Brig. Gen. Pantaleon Dumlao, PC-CIS Chief, since it was the CIS that initiated the complaint. However, the latter refused to receive them on the ground that his office has not yet received copies of their warrants of arrest.

In the meantime, on August 15, 1990, herein petitioners were admitted to the St. Luke's Hospital for various ailments brought about or aggravated by the stress and anxiety caused by the filing of the criminal complaint. On August 17, 1990, Brig. Gen. Dumlao granted their request that they be allowed to be confined at the hospital and placed under guard thereat.

In an Indorsement dated August 20, 1990, the CIS through Capt. Benjamin de los Santos, made its return to the trial court informing the latter of the voluntary surrender of herein petitioners and the fact that they were under hospital confinement. Herein Petitioner reiterated their Motion for Bail. In an Order dated August 24, 1990 (Annex "M" of the Petition, Rollo, p. 74), the hearing for the Motion for Ball was set for August 31, 1990 to enable the prosecution to present evidence it opposition to said motion. The prosecution filed its written opposition (Annex "N" of the Petition, Rollo, p. 75) on August 28, 1990, arguing that the evidence of petitioners' guilt was strong and thereafter presented its evidence.

On September 21, 1990, respondent Judge required the CIS to produce the bodies of herein petitioners on October 1, 1990 for arraignment (Annex "O" of the Petition, Rollo, p. 76). Upon their arraignment, herein Petitioners entered a plea of not guilty and filed an "Urgent Motion for Hospital Confinement" (Annex "OO" of the Petition Rollo, p. 77) which was denied by the court in its Order dated October 2,

Page 252: Warrantless Arrests, Searches and Seizures and Privacy of Communication

252

1990 (Annex "P" of the Petition,Rollo, p. 80). It likewise ordered their commitment at the Davao City Rehabilitation Center, Ma-a, Davao City pending trial on the merits. Herein petitioners argued orally a motion for reconsideration which was opposed by the prosecution. At the conclusion thereof, the court a quo issued a second order annex "Q" of the Petition, Rollo, p. 83) denying then motion for reconsideration and as to the alternative prayer to reopen the motion for hospital confinement, set the continuance thereof to October 17, 1990. It was further ordered that the petitioners shall remain under the custody of the PC-CIS pending resolution of the case.

Meanwhile, petitioners were returned to the St. Luke's Hospital where their physical condition remained erratic. On or about October 18, 1990, herein petitioners were informed that Brig. Gen. Dumlao had issued a directive for their transfer from the St. Luke's Hospital to Camp Crame on the basis of the October 2, 1990 Order (Annex "Q" of the Petition, Rollo, p. 83). Petitioners made representations that the tenor of the court order warranted maintenance of the status quo, i.e., they were to continue their hospital confinement. However, Brig, Gen. Dumlao informed them that unless otherwise restrained by the court, they would proceed with their transfer pursuant to the order of the trial court.

Hence, this petition on October 25, 1990 this Court issued a Temporary Restraining Order, effective immediately and continuing until further orders from this Court, ordering: (a) respondent Hon. William L. Layague to refrain from further proceeding with petitioners' "Motion for Hospital Confinement" in Criminal Case No. 20595-90 entitled "People of the Philippines v. Leopoldo Veroy and Ma. Luisa Veroy"; and (b) respondent Brig. Gen. Pantaleon Dumlao to refrain from transferring petitioners from the St. Luke's Hospital (Rollo, pp. 84-A to 84-C).

On November 2, 1990, respondent Judge issued an order denying petitioners' Motion for Bail (Annex "A" of the Second Supplemental Petition, Rollo, p. 133). Petitioners filed a Supplemental Petition on November 7, 1990 (Rollo, P. 105) and a Second Supplemental Petition on November 16, 1990 (Rollo, p. 120) which sought to review the order of the trial court dated November 2, 1990 denying their petition for bail.

Acting on the Supplemental Petition filed by Petitioners and taking into consideration several factors such as: a) that the possibility that they will flee or evade the processes of the court is fairly remote; b) their poor medical condition; and c) the matters in their Second Supplemental Petition especially since the prosecution's evidence refers to constructive possession of the disputed firearms in Davao City through the two (2) caretakers while petitioners lived in Manila since 1988, this Court, on November 20, 1990, granted petitioners' provisional liberty and set the bail bond at P20,000.00 each (Rollo, p. 141). Petitioners posted a cash bond in the said amount on November 23, 1990 (Rollo, pp. 143-145).

The petition was given due course on July 16, 1991 (Rollo, p. 211). Respondents adopted their Comment dated December 28, 1990 (Rollo, pp. 182-191) as their Memorandum while, petitioners filed their Memorandum on September 9, 1991 (Rollo, pp. 218-269).

As submitted by the respondents, and accepted by petitioners, the petition for mandamus to compel respondent Judge to resolve petitioners' Motion for Bail, and the petition for certiorari to review the order of respondent judge initially denying their Motion for Hospital Confinement, were rendered moot and academic by the resolutions of this Court dated November 20, 1990 and October 25, 1990, respectively. What remains to be resolved is the petition for prohibition where petitioners raised the following issues:

1. Presidential Decree No. 1866, or at least the third paragraph of Section 1 thereof, is unconstitutional for being violative of the due process and equal protection clauses of the Constitution;

2. Presidential Decree No. 1866 has been repealed by Republic Act No. 6968;

Page 253: Warrantless Arrests, Searches and Seizures and Privacy of Communication

253

3. Assuming the validity of Presidential Decree No. 1866 the respondent judge gravely abused his discretion in admitting in evidence certain articles which were clearly inadmissible for being violative of the prohibition against unreasonable searches and seizures.

The issue of constitutionality of Presidential Decree No. 1866 has been laid to rest in the case of Misolas v. Panga, G.R. No. 83341, January 30, 1990 (181 SCRA 648), where this Court held that the declaration of unconstitutionality of the third paragraph of Section 1 of Presidential Decree No. 1866 is wanting in legal basis since it is neither a bill of attainder nor does it provide a possibility of a double jeopardy.

Likewise, petitioners' contention that Republic Act 6968 has repealed Presidential Decree No. 1866 is bereft of merit. It is a cardinal rule of statutory construction that where the words and phrases of a statute are not obscure or ambiguous. its meaning and the intention of the legislature must be determined from the language employed, and where there is no ambiguity in the words, there is no room for construction (Provincial Board of Cebu v. Presiding Judge of Cebu, CFI, Br. IV, G.R. No. 34695, March 7, 1989 [171 SCRA 1]). A perusal of the aforementioned laws would reveal that the legislature provided for two (2) distinct offenses: (1) illegal possession of firearms under Presidential Decree No. 1866; and (2) rebellion, coup d' etat, sedition and disloyalty under Republic Act 6968; evidently involving different subjects which were not clearly shown to have eliminated the others.

But petitioners contend that Section 1 of Presidential Decree No. 1866 is couched in general or vague terms. The terms "deal in", "acquire", "dispose" or "possess" are capable of various interpretations such that there is no definiteness as to whether or not the definition includes "constructive possession" or how the concept of constructive possession should be applied. Petitioners were not found in actual possession of the firearm and ammunitions. They were in Quezon City while the prohibited articles were found in Davao City. Yet they were being charged under Presidential Decree No. 1866 upon the sole circumstance that the house wherein the items were found belongs to them (Memorandum for Petitioners, Rollo, pp. 242-244).

Otherwise stated, other than their ownership of the house in Skyline Village, there was no other evidence whatsoever that herein petitioners possessed or had in their control the items seized (Ibid., pp. 248-250). Neither was it shown that they had the intention to possess the Firearms or to further rebellion (Ibid., P. 252).

In a similar case, the revolver in question was found in appellant's store and the question arouse whether he had possession or custody of it within the meaning of the law.

This Court held that:

The animus possidendi must be proved in opium cases where the prohibited drug was found on the premises of the accused and the same rule is applicable to the possession of firearms. The appellant denied all knowledge of the existence of the revolver, and the Government's principal witness stated that there were a number of employees in the store. The only testimony which tends to show that the appellant had the possession or custody of this revolver is the inference drawn from the fact that it was found in his store, but we think that this inference is overcome by the positive testimony of the appellant, when considered with the fact that there were a number of employees in the store, who, of course, could have placed the revolver in the secret place where it was found without the knowledge of the appellant. At least there is a very serious doubt whether he knew of the existence of this revolver. In such case the doubt must be resolved in favor of the appellant. (U.S. v. Jose and Tan Bo., 34 Phil. 724 [1916])

But more importantly, petitioners question the admissibility in evidence of the articles seized in violation of their constitutional right against unreasonable search and seizure.

Page 254: Warrantless Arrests, Searches and Seizures and Privacy of Communication

254

Petitioners aver that while they concede that Capt. Obrero had permission from Ma. Luisa Veroy to break open the door of their residence, it was merely for the purpose of ascertaining thereat the presence of the alleged "rebel" soldiers. The permission did not include any authority to conduct a room to room search once inside the house. The items taken were, therefore, products of an illegal search, violative of their constitutional rights As such, they are inadmissible in evidence against them.

The Constitution guarantees the right of the people to be secure in their persons, houses, papers and effects against unreasonable searches and seizures (Article III, Section 2 of the 1987 Constitution). However, the rule that searches and seizures must be supported by a valid warrant is not an absolute one. Among the recognized exceptions thereto are: (1) a search incidental to an arrest; (2) a search of a moving vehicle; and (3) seizure of evidence in plain view (People v. Lo Ho Wing, G.R. No. 88017, January 21, 1991 [193 SCRA 122]).

None of these exceptions pertains to the case at bar. The reason for searching the house of herein petitioners is that it was reportedly being used as a hideout and recruitment center for rebel soldiers. While Capt. Obrero was able to enter the compound, he did not enter the house because he did not have a search warrant and the owners were not present. This shows that he himself recognized the need for a search warrant, hence, he did not persist in entering the house but rather contacted the Veroys to seek permission to enter the same. Permission was indeed granted by Ma. Luisa Veroy to enter the house but only to ascertain the presence of rebel soldiers. Under the circumstances it is undeniable that the police officers had ample time to procure a search warrant but did not.

In a number of cases decided by this Court, (Guazon v. De Villa, supra.; People v. Aminnudin, G.R. No. L-74869, July 6, 1988 [163 SCRA 402]; Alih v. Castro, G.R. No. L-69401, June 23, 1987 [151 SCRA 279]), warrantless searches were declared illegal because the officials conducting the search had every opportunity to secure a search Warrant. The objects seized, being products of illegal searches, were inadmissible in evidence in the criminal actions subsequently instituted against the accused-appellants (People v. Cendana, G.R. No. 84715, October 17, 1990 [190 SCRA 538]).

Undeniably, the offense of illegal possession of firearms is malum prohibitum but it does not follow that the subject thereof is necessarily illegal per se. Motive is immaterial in mala prohibita but the subjects of this kind of offense may not be summarily seized simply because they are prohibited. A search warrant is still necessary. Hence, the rule having been violated and no exception being applicable, the articles seized were confiscated illegally and are therefore protected by the exclusionary principle. They cannot be used as evidence against the petitioners in the criminal action against them for illegal possession of firearms. (Roan v. Gonzales, 145 SCRA 689-690 [1986]). Besides, assuming that there was indeed a search warrant, still in mala prohibita, while there is no need of criminal intent, there must be knowledge that the same existed. Without the knowledge or voluntariness there is no crime.

PREMISES CONSIDERED, the petition as granted and the criminal case against the petitioners for illegal possession of firearms is DISMISSED.

SO ORDERED.

Narvasa, C.J., Gutierrez, Jr., Cruz, Feliciano, Padilla, Bidin, Griño-Aquino, Medialdea, Regalado, Davide, Jr., Romero and Bellosillo, JJ., concur.

Nocon, J., is on leave.

Page 255: Warrantless Arrests, Searches and Seizures and Privacy of Communication

255

G.R. No. 90640 March 29, 1994

PEOPLE OF THE PHILIPPINES, plaintiff-appellee, vs.BONIFACIO BARROS, accused-appellant.

The Solicitor General for plaintiff-appellee.

Bartolome F. Macliing for accused-appellant.

 

FELICIANO, J.:

Bonifacio Barros was charged with violating Section 4 of R.A. No. 6425, as amended (known as the Dangerous Drugs Act of 1972), in an information which read as follows:

That on or about September 6, 1987, from Chackchakan, Bontoc, Mountain Province, to Nacagang, Sabangan, Mountain Province, and within the jurisdiction of this Honorable Court, the above-named accused while being a passenger in a Dangwa Bus with Plate No. ABZ 242, destined for Baguio City, without lawful authority did then and there willfully, unlawfully and feloniously carry with him as part of his baggage and transport about four (4) kilos of dried marijuana which the accused intended for distribution and sale at Baguio City, knowing fully well that said marijuana is a prohibited drug or [a] source of [a] prohibited drug.

Contrary to law. 1

After trial, the trial court convicted Bonifacio Barros of violation of Section 4 of R.A. No. 6425 as amended and sentenced him to suffer the penalty of reclusion perpetua 2 and to pay a fine of P20,000.00.

Barros now appeals from the judgment of conviction and essentially asks this Court to determine —

Whether the [trial] court deprived [the] accused of his right to due process by:

(1) ignoring manifest absence of the mandatory warrant in the arrest and search of the accused;

Page 256: Warrantless Arrests, Searches and Seizures and Privacy of Communication

256

(2) admitting confessions extracted from the accused after two hours of interrogation conducted by four (4) soldiers one after the other under intimidating circumstances; and

(3) misappreciation of facts. 3

The relevant facts as found by the trial court and as set forth in the court's decision are as follows:

That on September 6, 1987, M/Sgt. Francis Yag-as and S/Sgt. James Ayan, both members of the P.C. Mountain Province Command, rode the Dangwa Bus bearing Plate No. ABZ-242 bound for Sabangan, Mountain Province. Upon reaching Chackchakan, Bontoc, Mountain Province, the bus stopped and both M/Sgt. Yag-as and S/Sgt. Ayan, who were seated at the back, saw accused carrying a carton, board the bus and seated himself on seat No. 18 after putting the carton under his seat. Thereafter, the bus continued and upon reaching Sabangan, M/Sgt. Yag-as and S/Sgt. Ayan before they alighted, it being their station, called C2C [Fernando] Bongyao to inspect the carton under seat No. 18. After C2C Bongyao inspected the carton, he found out that it contained marijuana and he asked the passengers [who] the owner of the carton [was] but nobody answered. Thereafter, C2C Bongyao alighted with the carton and S/Sgt. Ayan and C2C Bongyao invited the herein accused to the detachment for questioning as accused was the suspected owner of the carton containing marijuana. As both P.C. officers Yag-as and Ayan saw accused, Bonifacio Barros carrying that same carton when he boarded the bus at Chackchakan. That upon entering the detachment the carton was opened in the presence of accused and accused Bonifacio Barros was asked if he owned the carton of marijuana and accused denied [this]. That when accused denied ownership of the carton of marijuana, the P.C. officers called for the bus conductor who pinpointed to Bonifacio Barros as the owner of the carton of marijuana. That during the oral investigation of accused, he finally admitted ownership of the carton (Exhibit "B") containing [four] 4 paper-wrapped packages of dried marijuana. (Exhibits "B-1", "B-2", "B-3" and "B-4").

. . . [A]fter he was orally investigated, [the accused] was brought to the Abatan General Hospital, Bauko, Mountain Province, for physical examination and a Medico Legal Certificate was issued (Exhibits "F" and "F-1"), indicating that accused suffered no physical injuries and that accused was probably under the influence of marijuana. That Dra. Danna Aleta inquired from accused Bonifacio Barros if he smoked marijuana and accused admitted having smoked marijuana. That after accused was medically examined, he was escorted by three members of the P.C. to the P.C. detachment at Tadian, Mountain Province, where the carton of marijuana (Exhibit "B") was also brought. That at Tadian, a seizure receipt was made together with a certification (Exhibit "C") pointing out to the fact that approximately 4 kilos of dried marijuana leaves were from accused Bonifacio Barros and which certification was signed by the accused (Exhibit "C-1") and subscribed before Judge Romualdo P. Awisan (Exhibit "C-2"). That in connection with the confiscation of the marijuana subject of the instant case and the apprehension of accused Bonifacio Barros, the P.C. officers who figured in this case namely M/Sgt. Yag-as and S/Sgt. Ayan and C2C Bongyao have correspondingly executed their sworn statements (Exhibits "A", "A-1", "A-2", "D", "D-1", "D-2").

. . . [S]amples of the marijuana were taken from each of the four packages marked Exhibits "B-1", "B-2", "B-3", and "B-4" and placed in four separate envelopes, following an order of the court to that effect and were hand-carried by Police Officer Jack Masilian to Camp Dangwa, La Trinidad, Benguet for laboratory test. That Capt. Carlos Figueroa, the Forensic Expert conducted two kinds of test on the four samples sent by the court and found them to be positive of marijuana as per his report No. D-011-88. (Exhibits "I" and "I-1"). 4

Page 257: Warrantless Arrests, Searches and Seizures and Privacy of Communication

257

The defense of the accused on the facts consisted of a simple denial of the ownership or possession of the carton box containing the four (4) kilos of marijuana. The trial court summarized the story of the accused in the following manner:

That accused Bonifacio Barros since 1984 was employed at the Honeymoon Disco Pad, Baguio City. That on September 5, 1987, accused was sent by his Manager, Engineer Arsenio Cuanguey to Bontoc, Mountain Province, to get their records from one Billy Cuanguey at Chackchakan, Bontoc, Mountain Province. That upon arriving at Chackchakan, Bontoc, Mountain Province, accused looked for the residence of Billy Cuanguey and he was pointed to a house where someone was tending a store. That accused asked the man if Billy Cuanguey was there and the man answered that he did not know where Billy went. So accused asked the man if Billy left [in] his room the tapes and records and the man said he did not know. Thereafter, accused asked the man to stay over night in that house where Billy was staying as it was the instruction of his manager. That the following day, September 6, 1987, after taking breakfast, accused, was going back to Baguio. On that morning of September 6, 1987, accused Bonifacio Barros boarded the Dangwa Bus at Chackchakan, Bontoc, Mountain Province bound for Baguio City. That when the Dangwa Bus reached the P.C. Checkpoint, soldiers went inside the bus and checked the baggages. That a soldier fished out a carton under the seat of [the] accused and shouted who owns the carton but nobody answered. Thereafter, the soldier went down with the carton and moments later returned to the bus and called accused Bonifacio Barros to alight from the bus. That Mr. Barros was surprised why he was ordered to alight and accused took his baggage which consisted of a pasiking and went down the bus. That accused was led by the soldiers to a house where his pasiking was taken and his clothes removed and his wallet taken. Accused was made to accept ownership of the carton of marijuana but he refused.

. . . [A]t 11:00 o'clock that same day, September 6, 1987, three soldiers escorted accused to the hospital and from the hospital, they proceeded to the Municipality of Tadian, Mountain Province. That upon reaching Tadian, accused was brought to the P.C. Camp and there he saw someone typing. Later, the soldiers allegedly presented to accused some papers which he was asked to sign but accused refused. That accused was threatened and if he refused to sign the papers that something will happen to him. That moments later, accused was threatened [by] a soldier [who] pointed a gun to him and told him to sign the paper and because of fear, he had to sign the document marked Exhibit "C." Thereafter, the soldiers allegedly threatened again accused and asked him to sign his name on the inside part of the cover of the carton of marijuana. Exhibit "X" for the court and Exhibit "B-5" for the prosecution. That after staying at Tadian for one night, accused was brought back to Sabangan and later transferred to the Bontoc Provincial Jail. 5

Turning to the legal defenses of the accused, we consider first his allegation that the police authorities had impermissibly extracted confessions from him after two (2) hours of interrogation, "under intimidating circumstances," by four (4) soldiers one after the other. The accused complains that he was not informed of his rights to remain silent and to counsel, that he had not waived his rights as an accused person, and that he had signed a confession involuntarily and without the assistance of counsel. He essentially contends that the confession is inadmissible as evidence against him.

We find, however, that it is not necessary to pass upon the above contention of appellant Barros. For the trial court in reaching its judgment of conviction had not taken into consideration the statements which had been obtained from the appellant during the interrogation conducted by the police officers. The trial court, so far as can be determined from its decision, totally disregarded Exhibits "C", "E" and "B-5," the alleged uncounselled confessions. The trial court made very clear the bases of its conclusion that the accused was guilty beyond reasonable doubt of the offense charged; those bases did not include the alleged confessions:

Page 258: Warrantless Arrests, Searches and Seizures and Privacy of Communication

258

First — M/Sgt. Francis Yag-as and S/Sgt. James Ayan testified that they saw the accused carrying the carton (Exhibit "B") when he boarded the bus at Chackchakan, Bontoc, Mountain Province. That the bus conductor pointed to accused at the checkpoint of Sabangan, Mountain Province. That accused is the owner of the carton (Exhibit "B"). That the carton (Exhibit "B") which contained four packages of dried marijuana leaves (Exhibits "B-1", "B-2", "B-3" and "B-4") was fished out from under the seat of the accused which fact was admitted by the accused himself.

Second — That per testimony of Dra. Danna Aleta, she examined accused Bonifacio Barros and that he suffered no physical injuries that would show that the accused was in anyway maltreated by the police authorities, and this fact was also admitted by accused to the effect that he was never harmed by the police nor the soldiers. Dra. Aleta also found that the accused was under the influence of drug[s] and that the accused admitted [to] her that he, accused, smoked marijuana. This is clear evidence that accused is not only a pusher of marijuana but also a user of said prohibited drugs. (See Exhibits "F" and "F-1" and TSN — Page 24 — Orpecio).

Third — The samples taken from Exhibits "B-1", "B-2", "B-3" and "B-4" sent by the court for laboratory test at Camp Dangwa, La Trinidad, Benguet were all positive of marijuana per Report No. D-011-88 (Exhibits "I" and "I-1") of Captain Carlos Figueroa, forensical expert.

Lastly, accused's testimony in his own behalf does not impress the court at it lacks the ring of truth. Besides, it is devoid of any corroboration. Our Supreme Court in this respect said:

The weak and uncorroborated denial of the accused cannot prevail over the clear, positive and straightforward testimony of prosecution witnesses [sic]." (People vs. Acelajao, 148 SCRA 142)." 6

We turn, therefore, to the second legal defense asserted by appellant Barros — i.e., that his constitutional right against unreasonable searches and seizures had been violated by the police authorities. The relevant constitutional provisions are found in Sections 2 and 3 [2], Article III of the 1987 Constitution which read as follows:

Sec. 2. The right of the people to be secure in their persons, houses, papers and effects against unreasonable searches and seizures of whatever nature and for any purpose shall be inviolable, and no search warrant or warrant of arrest shall issue except upon probable cause to be determined personally by the judge after examination under oath or affirmation of the complainant and the witness as he may produce, and particularly describing the place to be searched and the persons or things to be seized.

Sec. 3. . . .

(2) Any evidence obtained in violation of this or the preceding section shall be inadmissible for any purpose in any proceeding.

The general rule is that a search and seizure must be carried out through or with a judicial warrant; otherwise such search and seizure becomes "unreasonable" within the meaning of the above quoted constitutionalprovision. 7 The evidence secured thereby — i.e., the "fruits" of the search and seizure — will be inadmissible in evidence "for any purpose in anyproceeding. 8

Page 259: Warrantless Arrests, Searches and Seizures and Privacy of Communication

259

The requirement that a judicial warrant must be obtained prior to the carrying out of a search and seizure is, however, not absolute. There are certain exceptions recognized in our law, one of which relates to the search of moving vehicles. 9 Peace officers may lawfully conduct searches of moving vehicles — automobiles, trucks, etc. — without need of a warrant, it not being practicable to secure a judicial warrant before searching a vehicle, since such vehicle can be quickly moved out of the locality or jurisdiction in which the warrant may be sought. 10In carrying out warrantless searches of moving vehicles, however, peace officers are limited to routine checks, that is, the vehicles are neither really searched nor their occupants subjected to physical or body searches, the examination of the vehicles being limited to visual inspection. In Valmonte vs. De Villa, 11 the Court stated:

[N]ot all searches and seizures are prohibited. Those which are reasonable are not forbidden. A reasonable search is not to be determined by any fixed formula but is to be resolved according to the facts of each case.

Where, for example, the officer merely draws aside the curtain of a vacant vehicle which is parked on the public fair grounds, or simply looks into a vehicle, or flashes a light therein, these do not constitute unreasonable search. (Citations omitted)

When, however, a vehicle is stopped and subjected to an extensive search, such a warrantless search would be constitutionally permissible only if the officers conducting the search have reasonable or probable cause to believe, before the search, that either the motorist is a law-offender or the contents or cargo of the vehicle are or have been instruments or the subject matter or the proceeds of some criminal offense. 12

This Court has in the past found probable cause to conduct without a judicial warrant an extensive search of moving vehicles in situations where (1) there had emanated from a package the distinctive smell of marijuana; 13(2) agents of the Narcotics Command ("Narcom") of the Philippine National Police ("PNP") had received a confidential report from informers that a sizeable volume of marijuana would be transported along the route where the search was conducted; 14 (3) Narcom agents were informed or "tipped off" by an undercover "deep penetration" agent that prohibited drugs be brought into the country on a particular airline flight on a given date;15 (4) Narcom agents had received information that a Caucasian coming from Sagada, Mountain Province, had in his possession prohibited drugs and when the Narcom agents confronted the accused Caucasian, because of a conspicuous bulge in his waistline, he failed to present his passport and other identification papers when requested to doso; 16 and (5) Narcom agents had received confidential information that a woman having the same physical appearance as that of the accused would be transporting marijuana. 17

In the case at bar, however, we have been unable to find in the record of this case any circumstance which constituted or could have reasonably constituted probable cause for the peace officers to search the carton box allegedly owned by appellant Barros. The carrying of such a box by appellant onto a passenger bus could not, by itself, have convinced M/Sgt. Francis Yag-as and S/Sgt. James Ayan either that the appellant was a law violator or the contents of the box were instruments or the subject matter or proceeds of some criminal offense. The carrying of carton boxes is a common practice among our people, especially those coming from the rural areas since such boxes constitute the most economical kind of luggage possible. The peace officers here involved had not received any information or "tip-off" from an informer; no such a "tip-off" was alleged by the police officers before or during the trial. The police officers also did not contend that they had detected the odor of dried marijuana, or appellant Barros had acted suspiciously in the course of boarding the bus and taking a seat during the trip to Sabangan, nor in the course of being asked whether he owned the carton box later ascertained to contain four (4) kilos of marijuana. The testimony of the law enforcement officers who had apprehended the accused (M/Sgt. Francis Yag-as and S/Sgt. James Ayan), and who had searched the box in his possession, (C2C Fernando Bongyao), simply did not suggest or indicate the presence of any such probable cause.

M/Sgt. Francis Yag-as testified as follows:

Page 260: Warrantless Arrests, Searches and Seizures and Privacy of Communication

260

Direct Examination by Fiscal Moises Ayochok:

xxx xxx xxx

Q: On September 6, 1987, do you recall if you reported for duty?

A: Yes, sir.

Q: And where did you go on the morning of September 6, 1987?

A: I went to Sabangan, sir.

Q: What transportation did you use?

A: Dangwa Bus with Plate No. ABZ-242.

Q: Where did you board the Dangwa Bus?

A: At the Dangwa Terminal at Bontoc.

Q: When you said you boarded the bus with Plate No. ABZ-242 which started for Baguio City from Bontoc, Mountain Province, and while it stopped at Chackchakan, Bontoc, Mountain Province, was there anything that happened?

xxx xxx xxx

A: When the bus stopped at Sitio Chackchakan, we saw a person carrying a baggage or carton and boarded the bus then took his seat, seat No. 18.

Q: What was he carrying that time Mr. witness?

A: A carton.

Q: And where did he place that carton which he was carrying?

A: In front of seat No. 18 where he sat.

Q: You mean inside the bus?

A: Yes.

Q: And after this person boarded the bus at sitio Chackchakan and holding a carton and placed it in front of seat No. 18, what happened to the bus afterwards?

A: It proceeded to Sabangan.

Q: And at Sabangan, Mountain Province, what happened, if any?

Page 261: Warrantless Arrests, Searches and Seizures and Privacy of Communication

261

A: The bus stopped for the routinary checkpoint and inspection.

Q: When they [were at] the routinary checkpoint, what happened?

Atty. Sokoken:

He did not say routinary checkpoint. He said routinary inspection.

Fiscal Ayochok:

We substitute the words inspection with checkpoint to satisfy the objection of counsel.

Q: What happened when you stopped for the routinary inspection?

A: We called C2C Bongyao a member of the detachment to inspect the baggage of the suspect and when C2C .

Atty. Sokoken:

We request that [the] witness answers the question that he testifies [to] not in the narrative way.

Fiscal Ayochok:

He is answering the question.

Court:

Let the witness finish.

A: When Bongyao inspected the baggage of the suspect and he found out that it contained MJ.

Q: What do you mean MJ?

A: Marijuana.

xxx xxx xxx 18

For his part, S/Sgt. James Ayan testified as follows:

Direct Examination:

xxx xxx xxx

Q: And in the morning of September 6, 1987, do you recall where you were particularly in the afternoon?

A: In the morning of September 6, 1987, we rode on a Dangwa bus [with Plate] No. ABZ-242 going to Sabangan.

Page 262: Warrantless Arrests, Searches and Seizures and Privacy of Communication

262

Q: You said we. Who was your companion that time?

A: Master Sgt. Yag-as, sir.

Q: And when this bus reached Chackchakan, Bontoc, Mountain Province, what did you see?

A: We saw a civilian board the bus we were riding carrying a carton.

Q: And where did this civilian who boarded the bus which you were riding on place that carton?

A: He placed the carton under the seat of No. 18.

Q: Inside the bus, Mr. witness?

A: Inside the bus, sir.

Q: And what about the passenger who boarded the bus carrying the carton baggage, where did he go?

A: He sat facing the seat No. 18.

Q: Between seat No. 18 and the seat seated by the civilian who brought the carton, where was the carton exactly located?

A: As far as I know, sir, it was located just beneath seat No. 18.

Q: When this bus which you rode on which the passenger carrying the carton luggage you saw reached Sabangan what happened there?

A: When the bus reached Sabangan that we were riding, it was stopped for routinary inspection.

Q: What happened next?

A: We called C2C Bongyao to inspect the baggage that we have just seen at Chackchakan.

Q: Did he inspect the baggage?

A: Yes, sir.

Q: And what was the contents of that baggage if there was any?

A: It turned out that the contents of the baggage was MJ sir.

Q: You mean marijuana?

A: Yes, sir.

Page 263: Warrantless Arrests, Searches and Seizures and Privacy of Communication

263

xxx xxx xxx

Cross Examination:

xxx xxx xxx

Q: You stated that on September 6, 1987, a Dangwa bus stopped at Sabangan, Mt. Province for purposes of military check-up, is that correct?

A: Routinary inspection, sir.

Q: But it was not you who entered the Dangwa bus for routinary check-up?

A: We were there riding in the bus, sir, and we called C2C Bongyao to come.

Q: So your purpose in riding inside the Dangwa bus was actually to see that person carrying this carton which is marked Exhibit "B"?

A: No, sir, because I am a detachment commander at Sabangan and that is why I called one of my men, sir.

Q: So that you have full knowledge that from Chackchakan, Bontoc, going to Sabangan, there is already marijuana being carried inside that bus?

A: That is only our suspect [should be suspicion], sir.

Q: Would you please tell this Honorable Court why you have not inspected it when you arrived at Alab? Why have you waited to reach Sabangan to inspect it?

A: Because it is the checkpoint, sir, at Nacagang, Sabangan.

Q: Are you now admitting that you do not have authority to inspect the baggage here in Bontoc?

A: We just wanted it checked in Sabangan, sir.

Q: Could you give us a very special reason why you have to wait in Sabangan?

A: Because we are stationed in Sabangan and that is the checkpoint.

Fiscal Ayochok:

Why argue with the witness? It is up for them to check it at the proper checkpoint.

Court:

Page 264: Warrantless Arrests, Searches and Seizures and Privacy of Communication

264

Sustained.

xxx xxx xxx 19

The testimony of C2C Fernando Bongyao is much briefer, but equally uninformative:

Direct Examination:

Q: On September 6, 1987, at around 9:30 a.m., do you recall having reported for duty at Nacagang, Sabangan, Mountain Province?

A: Yes, sir.

Q: And while you were on duty at Nacagang, Sabangan, was there anything unusual that happened that time?

A: Yes, sir.

Q: What was that Mr. witness?

A: When we were on the checkpoint, the bus stopped bearing Plate No. ABZ-242.

Q: When the bus stopped, what did you do?

A: While on my way to check the bus, Master Sergeant Yag-as and Ayan called for me, sir, and they told me that a carton was placed under seat No. 18, sir.

Q: And when you were told to inspect that carton under seat No. 18, did you inspect that carton?

A: I inspected it, sir.

Q: You said you inspected that carton, what did you do in inspecting that carton?

A: I inserted my hand inside and when I removed my hand, it was a stuff of marijuana, sir.

xxx xxx xxx 20

So far as the record itself is concerned, therefore, it would appear that there existed no circumstance which might reasonably have excited the suspicion of the two (2) police officers riding in the same bus as appellant Barros. They asked the police officers at the checkpoint at Sabangan to inspect the box allegedly carried by appellant Barros apparently on a mere guess that appellant Barros might be carrying something in the nature of contraband goods. There was, in other words, nothing to show that appellant Barros was then in the process of "actually committing" or "attempting to commit" a crime. 21 There was, moreover, nothing on the record that could have reasonably led the two (2) police officers to believe that "an offense [had] in fact just been committed" when appellant Barros boarded the bus at Chackchakan or when he was asked whether he owned the box here involved at the checkpoint in Sabangan. The two (2) police officers, according to the record, had no "personable knowledge of facts indicating that the person

Page 265: Warrantless Arrests, Searches and Seizures and Privacy of Communication

265

to be arrested (appellant Barros) had committed it." There was, in brief, no basis for a valid warrantless arrest. Accordingly, the search and seizure of the carton box was equally non-permissible and invalid. 22 The "fruits" of the invalid search and seizure — i.e., the four (4) kilos of marijuana — should therefore not have been admitted in evidence against appellant Barros.

The Solicitor General, however, contends that appellant Barros had waived any irregularities which may have attended his arrest. Presumably, the Solicitor General also argues that appellant Barros has waived the non-admissibility of the carton (Exhibit "B") which contained four (4) packages of dried marijuana leaves (Exhibits "B-1", "B-2", "B-3" and "B-4"). The Solicitor General said:

. . . [E]ven assuming in gratia argumenti that irregularities attended the arrest of appellant, still the same cannot be questioned at this late stage. Well-settled is the doctrine laid down in the case of Callanta vs. Villanueva (77 SCRA 377), and later reiterated in the more recent case of Bagcal vs. Villaraza (120 SCRA 525), that "posting of [a] bail bond constitutes waiver of any irregularity attending the arrest of a person and estops him from questioning its validity." Here, appellant had in fact posted the required bail to obtain his provisional liberty, albeit his application was subsequently denied (see TSN, Feb. 10, 1988, p. 65). Consistent with jurisprudence, therefore, he should be deemed to have waived any irregularity attending his arrest, if any there be, and cannot now be heard to assail the same. 23

It might be supposed that the non-admissibility of evidence secured through an invalid warrantless arrest or a warrantless search and seizure may be waived by an accused person. The a priori argument is that the invalidity of an unjustified warrantless arrest, or an arrest effected with a defective warrant of arrest may be waived by applying for and posting of bail for provisional liberty, so as to estop as accused from questioning the legality or constitutionality of his detention or the failure to accord him a preliminary investigation. We do not believe, however, that waiver of the latter (by, e.g., applying for and posting of bail) necessarily constitutes, or carries with it, waiver of the former — an argument that the Solicitor General appears to be making impliedly. Waiver of the non-admissibility of the "fruits" of an invalid warrantless arrest and of a warrantless search and seizure is not casually to be presumed, if the constitutional right against unlawful searches and seizures is to retain its vitality for the protection of our people. In the case at bar, defense counsel had expressly objected on constitutional grounds to the admission of the carton box and the four (4) kilos of marijuana when these were formally offered in evidence by the prosecution. 24 We consider that appellant's objection to the admission of such evidence was made clearly and seasonably and that, under the circumstances, no intent to waive his rights under the premises can be reasonably inferred from his conduct before or during during the trial.

In the dissenting opinion, my learned brother Melo, J. takes the view that appellant Barros had waived his rights by his "stoic deportment" consisting of failure to object to the search by the police authorities immediately after the opening of the carton box:

. . . In point of fact, when the police authorities inspected the carton of marijuana and asked accused-appellant who owned the box, accused-appellant denied ownership of the box or carton and failed to even mutter the least bit of protest (p. 3, Decision). His demeanor should therefore be construed as implicit acquiescence to the search inasmuch as the objection thereto is vulnerable to express or implied waiver (People vs. Kagui Malasugui (63 Phil. 221 [1936]); 1 Bernas, Constitution of the Republic of the Philippines, First ed., 1987, p. 108). . . . . 25

It is submitted, with respect, that Kagui Malasugui is not applicable to the case at bar; rather it is People vs. Burgos, 26 promulgated fifty (50) years after Kaqui Malasuqui, that is applicable. In Burgos, this Court ruled that the accused is not to be presumed to have waived the unlawful search conducted on the occasion of his warrantless arrest "simply because he failed to object" —

Page 266: Warrantless Arrests, Searches and Seizures and Privacy of Communication

266

. . . To constitute a waiver, it must appear first that the right exists; secondly, that the person involved had knowledge, actual or constructive, of the existence of such a right; and lastly, that said person had an actual intention to relinquish the right (Pasion Vda. de Garcia vs. Locsin, 65 Phil. 689). The fact that the accused failed to object to the entry into his house does not amount to a permission to make a search therein (Magoncia vs. Palacio, 80 Phil. 770). As pointed out by Justice Laurel in the case of Pasion Vda. de Garcia vs. Locsin (supra):

xxx xxx xxx

. . . As the constitutional guaranty is not dependent upon any affirmative act of the citizen, the courts do not place the citizen in the position of either contesting an officer's authority by force, or waiving his constitutional rights; but instead they hold that a peaceful submission to a search or seizure is not a consent or an invitation thereto, but is merely a demonstration of regard for the supremacy of the law. (Citation omitted).

We apply the rule that: "courts indulge every reasonable presumption against waiver of fundamental constitutional rights and that we do not presume acquiescence in the loss of fundamental rights." (Johnson vs. Zerbts, 304 U.S. 458). 27 (Emphasis supplied) .

Kagui Malasugui is not applicable to the instant case, because there the Court explicitly found that there was probable cause for the warrantless arrest of the accused and therefore, the warrantless search effected immediately thereafter was equally lawful. In Kagui Malasugui, a Chinese merchant was found lying on the ground with several nasty wounds in the head; one resulted in skull fracture and proved fatal. He died in the hospital to which he had been immediately brought by a policeman. Mr. Malasuqui became a suspect because when the victim was found, still alive, and upon being asked who had attacked him, laconically answered, "Kagui." On the same day, the accused Kagui Malasugui was arrested and a search of his person was conducted without objection from the accused. Before the body search of the accused was carried out, the accused voluntarily surrendered to the police authorities a couple of bracelets belonging to the deceased victim and when asked if he had anything else to surrender, he, in a trembling voice, answered in the negative. The police thereupon conducted a body search of the accused, without any objection from him; the search resulted in the production of additional personal effects belonging to the deceased victim. Under these circumstances, the Court ruled that:

When one voluntarily submits to a search or consents to have it made of his person or premises, he is precluded from complaining later thereof. (Cooley, Constitutional Limitations, 8th ed., [V]ol. I, p. 631.) The right to be secure from unreasonable search may, like every right, be waived and such waiver may be made either expressly or impliedly.

A propos my distinguished brother Melo, J.'s suggestion that the right against an unlawful warrantless search or arrest is personal and may not be invoked by the accused's counsel during trial, it is relevant to note that the law (the Rules of Court) specifies the proper time when objections to admission of evidence must be raised and that in the case at bar, a timely objection was made by appellant Barros. Finally, the accused's silence during the warrantless search should not be lightly taken as consent to that search, but rather construed as explained by the Court in Burgos, 28 and as pointed out by Mr. Justice Laurel, a "demonstration of regard for the supremacy of the law."

It is, of course, possible that appellant Barros may in fact have been guilty of transporting the four (4) kilos of marijuana. His guilt must, however, be established by constitutional means. The non-admissibility of evidence secured through a disregard of the constitutional right of the accused against unreasonable searches and seizures is the sanction imposed by the Constitution for disregard of such right; the sanction is a powerful one, for it renders inutile the work done by the police officers, by the prosecutor and by the trial court. It is a sanction which this Court has no choice but to apply in the instant case.

Page 267: Warrantless Arrests, Searches and Seizures and Privacy of Communication

267

WHEREFORE, for all the foregoing, the decision of the Regional Trial Court, Branch 35, Bontoc, Mountain Province, in Criminal Case No. 687 is hereby REVERSED and SET ASIDE and appellant is hereby ACQUITTED of the crime charged, the evidence lawfully before the trial court not being sufficient to establish his guilt thereof beyond reasonable doubt. No costs.

SO ORDERED.

Bidin, Romero and Vitug, JJ., concur.

Melo, J., dissents.

[G.R. Nos. 129756-58. January 28, 2000]

PEOPLE OF THE PHILIPPINES, plaintiff-appellee, vs. JULIAN DEEN ESCAñO, VIRGILIO TOME USANA and JERRY CASABAAN LOPEZ, accused.

VIRGILIO TOME USANA and JERRY CASABAAN LOPEZ, accused-appellants.

D E C I S I O N

DAVIDE, JR., C.J.:

Accused-appellants Virgilio T. Usana and Jerry C. Lopez, together with Julian D. Escaño, were charged before the Regional Trial Court of Makati City, Branch 64, in Criminal Case No. 95-936 with violation of Section 4, Article II of Republic Act No. 6425,[1] as amended. Escaño and Usana were also charged in Criminal Case No. 95-937 and No. 95-938 with illegal possession of firearms and ammunition in violation of Presidential Decree No. 1866.

The accusatory portion of the Information in Criminal Case No. 95-936 reads as follows:

That on or about the 5th day of April, 1995, in the City of Makati, Metro Manila, Philippines and within the jurisdiction of this Honorable Court, the above-named accused, conspiring and confederating together and all of them mutually helping and aiding one another, without being authorized by law, did then and there willfully, unlawfully and

Page 268: Warrantless Arrests, Searches and Seizures and Privacy of Communication

268

feloniously sell, distribute and transport 3.3143 kilograms of "HASHISH", a prohibited drug, in violation of the above-cited law.[2]

The charge against accused Julian D. Escaño in Criminal Case No. 95-937 reads as follows:

That on or about the 5th day of April, 1995, in the City of Makati, Metro Manila, Philippines and within the jurisdiction of this Honorable Court, the above-named accused did then and there willfully, unlawfully and feloniously have in his possession, direct custody and control one (1) pc. of cal. .45 pistol, government model with Serial No. 990255, with magazine containing 7 live ammos and two (2) more magazines for cal. .45 pistol containing 7 live ammos each, without first securing the necessary license or permit from the proper government authorities and which firearm and ammunitions he carried outside of his residence.[3]

The accusatory portion of the information against Virgilio Usana in Criminal Case No. 95-938 reads:

That on or about the 5th day of April, 1995, in the City of Makati, Metro Manila, Philippines and within the jurisdiction of this Honorable Court, the above-named accused, did then and there willfully, unlawfully and feloniously have in his possession, direct custody and control One (1) pc. of rifle carbine with Serial No. 7176644 with a banana type magazine loaded with 28 live ammunitions without first securing the necessary license or permit from the proper government authorities and which firearms and ammunitions he carried outside of his residence.[4] Es-mso

The cases were consolidated and jointly tried.

In its Decision of 30 May 1997,[5] which was promulgated on 17 June 1997, the trial court convicted Escaño and herein appellants in Criminal Case No. 95-936, Escaño in Criminal Case No. 95-937, and appellant Usana in Criminal Case No. 95-938.

Escaño filed on 19 June 1997 a Notice of Appeal, but on 16 July 1997, he filed a Manifestation and Withdrawal of Appeal,[6] which was granted by the trial court in its Order of 17 July 1997.[7]

Usana and Lopez filed a Notice of Appeal on 30 June 1997,[8] manifesting therein that they were appealing to this Court and to the Court of Appeals. Considering the penalties imposed, the decision in Criminal Case No. 95-936 was appealed to this Court, while the Court of Appeals took cognizance of the appeal from Criminal Case No. 95-938. In its Order of 30 June 1997,[9] the trial court gave due course to the appeal and ordered the transmittal of the record in Criminal Case No. 95-936 to this Court and the record of Criminal Case No. 95-938 to the Court of Appeals.

Accordingly, it is only the appeal from the judgment in Criminal Case No. 95-936 that is now before this Court.

Due to the differing versions of the parties, there is a need to narrate each of the testimonies of the key players in this case.

The prosecution has this version of the events: On the 5th of April 1995 and during a COMELEC gun ban, some law enforcers of the Makati Police, namely, PO3 Eduardo P. Suba, PO3 Bernabe Nonato, SPO4 Juan de los Santos, and Inspector Ernesto Guico,[10] were manning a checkpoint at the corner of Senator Gil Puyat Ave. and the South Luzon Expressway (SLEX).[11] They were checking the cars going to Pasay City, stopping those they found suspicious, and imposing merely a running stop on the others. At about past midnight, they stopped a Kia Pride car with Plate No. TBH 493.[12] PO3 Suba saw a long firearm on the lap of the person seated at the passenger seat, who was later identified as Virgilio Usana. They asked

Page 269: Warrantless Arrests, Searches and Seizures and Privacy of Communication

269

the driver, identified as Escaño, to open the door. PO3 Suba seized the long firearm, an M-1 US Carbine, from Usana. When Escaño, upon order of the police, parked along Sen. Gil Puyat Ave., the other passengers were search for more weapons. Their search yielded a .45 caliber firearm which they seized from Escaño.[13]

The three passengers were thereafter brought to the police station Block 5 in the Kia Pride driven by PO3 Nonato.[14] Upon reaching the precinct, Nonato turned over the key to the desk officer. Since SPO4 de los Santos was suspicious of the vehicle, he requested Escaño to open the trunk.[15] Escaño readily agreed and opened the trunk himself using his key.[16]They noticed a blue bag inside it,[17] which they asked Escaño to open. The bag contained a parcel wrapped in tape,[18] which, upon examination by National Bureau of Investigation Forensic Chemist Emilia A. Rosaldos, was found positive for hashish weighing 3.3143 kilograms.[19] lex

A certification was issued by the Firearms and Explosive Office of the National Police Commission (NAPOLCOM) to the effect that Escaño was not a licensed/registered firearms holder of any kind and caliber. Usana, however, according to the same certification is a licensed/registered holder of a pistol Colt .45 caliber with license issued on 14 October 1994 and to expire on April 1996. Usana also has an application for a pistol Uzi Cal. 9mm. Neither of the two guns seized were licensed/registered with the NAPOLCOM.[20]

For his part, Escaño (or Jovy) testified that on the 4th of April 1995, between 11:00 and 11:30 in the morning, he was at the lobby of Legend Hotel, at Pioneer St., Mandaluyong City, to meet with his business partners, including Usana and Lopez. He saw his friend and erstwhile co-employee at Philippine Airlines, Ramon Cabrera, who had borrowed his wife’s car. Since it was his wife’s birthday the following day, he asked Cabrera if he could get back the car. Cabrera readily gave him the keys of the car. [21]

He left the hotel at around 11:45 in the evening with Usana and Lopez. Using his wife’s car, they cruised southward along Epifanio de los Santos Avenue (EDSA) and turned right at Sen. Gil Puyat Avenue. They stopped before crossing SLEX because the traffic light turned red. From the other side of SLEX, he could see a group of policemen. Upon crossing SLEX, they were flagged down by one of the policemen, so he slowed down and stopped. PO3 Nonato asked him to roll down the window and demanded to see his license. He asked if he had committed any violation, but PO3 Nonato accused him of being drunk, which he denied. The policemen persisted in asking for his license, but he did not budge and instead reiterated that there was no reason for him to surrender his license for he had not committed any violation. A verbal tussle ensued resulting in the drawing of firearms by the policemen which prompted Usana to suggest that they go to the police station because the policemen were carrying guns and they have not done anything wrong.[22] Jjjä uris

He stated further that he was the one who drove to the police station along Dian St. with his companions. He parked the car then they were brought to the office of the Deputy Station Commander, Lieutenant Eco.[23] The policemen asked if they could search his car. He then inquired if he was not entitled to a lawyer and why they needed to conduct a search when they had not even told him what he had violated. Apparently, he thought they were there only for verification purposes. Lt. Eco explained that that was the reason why they were going to search his car, to see if he had done anything illegal. Although the police were insistent in asking for the keys to his car, he continuously refused. Lt. Eco asked his men to usher the trio into the detention cell.[24]

After two hours, he was brought back to Lt. Eco’s office. Lt. Eco pointed to a bag, a rifle, a pistol and a package wrapped in masking tape or packing tape on his desk, and said these items constituted evidence of illegal possession of firearms and transporting of drugs. He was surprised that they found those items from his car because his key had been with him all the time. He was handcuffed, brought to his car, and again was surprised to see its trunk open.[25]

Page 270: Warrantless Arrests, Searches and Seizures and Privacy of Communication

270

On the other hand, Lopez had a different story. He claimed he was the mechanic of Usana and they lived in the same subdivision.[26] On 4 April 1995, he was working on Usana’s pick-up truck at the latter’s house when Escaño dropped by at around 4:30 in the afternoon looking for Usana who was then working in Forbes Park.[27] At around 5:30 p.m., they left Usana’s house in Escaño’s metallic gray Kia Pride. Inside the car, he saw a .45 caliber pistol and two spare magazines tucked in the right side and left side of Escaño’s waist. He also saw a carbine under the right passenger seat. When he inquired about the guns, Escaño replied that such did not pose any problem since they were licensed. Before going to Usana, they went to Pasay City to see a certain Jerry.[28] They met Usana at the Sen. Gil Puyat Station of the LRT at around 9:00 p.m. He gave his seat to Usana but was unaware if the latter noticed the rifle beneath the seat.[29] LexjÓ uris

They went home via Sen. Gil Puyat Avenue but were stopped at a checkpoint after crossing SLEX. The policemen directed their flashlights at them and one opened the front passenger door.[30] The latter saw the rifle under Usana’s seat. Usana and Escaño were ordered to get out of the car. PO3 Nonato immediately saw the gun tucked in Escaño’s waist and asked if he was a policeman. Escaño replied that everything would be explained at the police station. He was also asked to step out. No firearm was, however, found in his possession.[31]

When confronted about the guns, Escaño tried to intercede for his two companions and said that "... these two don’t know anything about it, I just took them for a drive." They subsequently went to police station Block 5. A certain Toto, a policeman, drove the Kia Pride to Block 5.[32]

Upon reaching the police station, Escaño was immediately brought to the office of Lt. Eco while he and Usana were asked to sit on the bench. After a few minutes, PO3 de los Santos came out of the office of Lt. Eco to talk to him. He told him that all he knew about Escaño is that he was a wealthy flight attendant with military connections. After returning to Lt. Eco’s office, PO3 de los Santos went out of the police station with Lt. Eco and Escaño. The three came back with a blue bag which he had never seen before. The bag was opened before the three suspects. Escaño reiterated that his two companions had nothing to do with the bag.[33]

He and Usana stayed overnight in their cell and only saw Escaño in the morning of April 5. At around 4:00 p.m., they were transferred to the CID and stayed in the office of a certain Inspector Sipin. Escaño admitted he owned the bag/case.[34]

For his part, Usana testified that he was a duly licensed architect who was gainfully employed by Rolando de Asis and Taytay Management Corporation.[35] He admitted owning a licensed .45 caliber pistol.[36] In March 1995, he hired as mechanic Lopez, who lives in Bernabe Subdivision Phase II where he also lives. Escaño on the other hand, was introduced to him by a certain Roberto Samparado, a neighbor of Lopez. Escaño, an international flight attendant of Philippine Airlines and a businessman who owns Verge Enterprises, also supplied materials to the Philippine Army and planned to engage in a construction business.[37]

On 4 April 1995, at around 7:30 p.m., he paged Escaño to talk about the materials for the five prototype gunship helicopters they were supposed to supply. They talked on the phone, agreeing to meet between 8:30 and 9:00 p.m. at the Sen. Gil Puyat Ave. Station of the Light Rail Transit, [38] and met at around a quarter past nine. Escaño was on board a metallic Kia Pride with Lopez on the passenger seat. Lopez vacated the seat for him. They went to Magallanes Village to meet a certain Norman Garcia and talk about the documents[39] relating to the helicopter gunship of the Air Force. They arrived there at 11:30 p.m. While they were talking with Garcia, he noticed a gun and magazines tucked in Escaño’s waist. Upon inquiry, Escaño said it was not a problem and only for his protection.[40] On their way to Roxas Boulevard, they were stopped at a checkpoint along Sen. Gil Puyat Ave. Policemen knocked on the car windows so he and Escaño rolled down their windows. A person in civilian clothes suddenly opened the right door, took something from the side of his seat and shouted, "There’s a gun." He was surprised because he did not carry anything when he boarded the car; neither did he see anything inside the car because it was dark and he was not wearing his eyeglasses.[41] The person who took the gun asked if he

Page 271: Warrantless Arrests, Searches and Seizures and Privacy of Communication

271

was a policeman, and he said he was an architect. He was then asked to alight from the car, then frisked. Escaño was also asked to alight from a car. They saw a gun tucked in his waist, so they asked if he was a policeman, and Escaño answered in the negative. Lopez was then ordered to get out of the car by the person in civilian clothes and was also searched. They rode the Anfra service vehicle of the police. One of the policemen asked Lopez to handcuff him and Escaño. The policeman who asked Escaño to get out of the car drove the Anfra van to Block 5 where they arrived at 1:30 in the morning of 5 April. [42] Jä lexj

He and Lopez waited outside the office of Lt. Eco while Escaño was inside with the arresting officers. Lt. Eco came out of his office and urged Lopez to tell the truth. He heard Lopez say that they were both just with Escaño and that they knew nothing about the guns; neither do they own any. SPO4 de los Santos entered the office of Lt. Eco and came out five minutes later with Escaño, Lt. Eco, and the other arresting officers, Nonato, Suba and Erwin Eco, the person in civilian clothes. All six went out to the parking area and returned after about five minutes. Lt. Eco was carrying a bag which he placed on top his desk. Lopez and Escaño were asked about the contents of the bag. The two replied it was the first time they saw that bag. Lt. Eco opened the bag before them. They all saw something in brown paper. He and Lopez simultaneously exclaimed that they knew nothing about the contents of the bag, and they implored Escaño to tell the police that they had nothing to do with it.[43]

The trial court found the prosecution’s version more credible than that of any one of the accused, and ruled that the evidence presented by the prosecution was sufficient to convict the accused as charged. It decreed:

WHEREFORE, in view of the foregoing judgment is hereby rendered as follows:

1. In Criminal Case No. 95-936, accused JULIAN ESCAÑO y DEEN, VIRGILIO USANA y TOME and JERRY LOPEZ y CASABAAN are GUILTY as charged and are sentenced to suffer imprisonment of RECLUSION PERPETUA, and to pay a fine of P500,000.00.

The Branch Clerk of Court is directed to turn over to the Dangerous Drugs Board the 3.314 kilograms of Hashish (marijuana) for its appropriate disposition in accordance with law; and

2. In criminal Cases Nos. 95-937 and 95-938, accused JULIAN ESCAÑO y DEEN and VIRGILIO USANA y TOME are GUILTY as charged in the two separate informations respectively filed against them and are sentenced to suffer the indeterminate prison term from TEN (10) YEARS of PRISION MAYOR maximum, as minimum to SEVENTEEN (17) YEARS, FOUR (4) MONTHS and ONE (1) DAY of RECLUSION TEMPORAL maximum as maximum.[44] Courtä

The firearms and ammunitions subject matter of these cases which are still with the City Prosecutor’s Office are forfeited in favor of the Government are directed to be turned over to the Firearms and Explosive Unit, PNP, Camp Crame, Quezon City for its appropriate disposition.

SO ORDERED. [45]

Accused-appellants Usana and Lopez anchor their appeal on the following arguments:

1. The trial court erred in admitting in evidence the hashish seized without search warrant when the police officers already had the opportunity to secure a search warrant before searching the bag found at the baggage compartment at the back of the car;

Page 272: Warrantless Arrests, Searches and Seizures and Privacy of Communication

272

2. Assuming that the hashish is admissible in evidence, the trial court erred in finding appellants to have conspired with Escaño in transporting the hashish when the evidence clearly shows that the hashish was owned and possessed solely by Escaño;

3. The trial court erred in convicting appellants of illegal possession of hashish despite the fact that they were neither in actual nor constructive possession of the illegal drug; and

4. The trial court erred in not considering the exculpatory testimony of Julian Escaño in favor of appellants.

Before going any further, some words are in order regarding the establishment of checkpoints.

Accused-appellants assail the manner by which the checkpoint in question was conducted. They contend that the checkpoint manned by elements of the Makati Police should have been announced. They also complain of its having been conducted in an arbitrary and discriminatory manner.

We take judicial notice of the existence of the COMELEC resolution[46] imposing a gun ban during the election period issued pursuant to Section 52(c) in relation to Section 26(q) of the Omnibus Election Code (Batas Pambansa Blg. 881). The national and local elections in 1995 were held on 8 May, the second Monday of the month. The incident, which happened on 5 April 1995, was well within the election period. Supremeä

This Court has ruled that not all checkpoints are illegal. Those which are warranted by the exigencies of public order and are conducted in a way least intrusive to motorists are allowed.[47] For, admittedly, routine checkpoints do intrude, to a certain extent, on motorists’ right to "free passage without interruption," but it cannot be denied that, as a rule, it involves only a brief detention of travelers during which the vehicle’s occupants are required to answer a brief question or two. For as long as the vehicle is neither searched nor its occupants subjected to a body search, and the inspection of the vehicle is limited to a visual search, said routine checks cannot be regarded as violative of an individual’s right against unreasonable search. In fact, these routine checks, when conducted in a fixed area, are even less intrusive. [48]

The checkpoint herein conducted was in pursuance of the gun ban enforced by the COMELEC. The COMELEC would be hard put to implement the ban if its deputized agents were limited to a visual search of pedestrians. It would also defeat the purpose for which such ban was instituted. Those who intend to bring a gun during said period would know that they only need a car to be able to easily perpetrate their malicious designs.

The facts adduced do not constitute a ground for a violation of the constitutional rights of the accused against illegal search and seizure. PO3 Suba admitted that they were merely stopping cars they deemed suspicious, such as those whose windows are heavily tinted just to see if the passengers thereof were carrying guns. At best they would merely direct their flashlights inside the cars they would stop, without opening the car’s doors or subjecting its passengers to a body search. There is nothing discriminatory in this as this is what the situation demands.

We see no need for checkpoints to be announced, as the accused have invoked. Not only would it be impractical, it would also forewarn those who intend to violate the ban. Even so, badges of legitimacy of checkpoints may still be inferred from their fixed location and the regularized manner in which they are operated.[49]

Usana and Lopez also question the validity of the search. The trial court, in convicting the three accused for violation of R.A. No. 6425, accepted as aboveboard the search done by the Makati Police of the trunk of the car. Jurisprudence recognizes six generally accepted exceptions to the warrant requirement: (1)

Page 273: Warrantless Arrests, Searches and Seizures and Privacy of Communication

273

search incidental to an arrest; (2) search of moving vehicles; (3) evidence in plain view; (4) customs searches; (5) consented warrantless search;[50] and (6) stop-and-frisk situations.[51]

Even though there was ample opportunity to obtain a search warrant, we cannot invalidate the search of the vehicle, for there are indications that the search done on the car of Escaño was consented to by him. Both Lopez and Usana testified that Escaño was with the police officers when they searched the car.[52] There was no apparent objection made by Escaño as he seemed to have freely accompanied the police officers to the car. PO3 Suba, on the other hand, testified that "Escaño readily agreed to open the trunk," upon request of SPO4 de los Santos.[53] But according to Escaño, he refused the request of the police officers to search his car.[54] We must give credence to the testimony of PO3 Suba. Not only is it buttressed by the testimony of Usana and Lopez that Escaño freely accompanied the police officers to the car, it is also deemed admitted by Escaño in failing to appeal the decision. The findings of fact of the trial court are thus deemed final as against him. Esmsc

Despite the validity of the search, we cannot affirm the conviction of Usana and Lopez for violation of R.A. No. 6425, as amended. The following facts militate against a finding of conviction: (1) the car belonged to Escaño; (2) the trunk of the car was not opened soon after it was stopped and after the accused were searched for firearms; (3) the car was driven by a policeman from the place where it was stopped until the police station; (4) the car’s trunk was opened, with the permission of Escaño, without the presence of Usana and Lopez; and (5) after arrival at the police station and until the opening of the car’s trunk, the car was in the possession and control of the police authorities. No fact was adduced to link Usana and Lopez to the hashish found in the trunk of the car. Their having been with Escaño in the latter’s car before the "finding" of the hashish sometime after the lapse of an appreciable time and without their presence left much to be desired to implicate them to the offense of selling, distributing, or transporting the prohibited drug. In fact, there was no showing that Usana and Lopez knew of the presence of hashish in the trunk of the car or that they saw the same before it was seized.

IN VIEW WHEREOF, that portion of the challenged decision of 30 May 1997 of the Regional Trial Court, Makati, Branch 64, insofar as Criminal Case No. 95-936 is concerned with regard to accused-appellants VIRGILIO T. USANA and JERRY C. LOPEZ, holding them guilty of violation of Section 4, Article II of R.A. No 6425, as amended, is hereby REVERSED and SET ASIDE and another is hereby rendered ACQUITTING them therein on ground of reasonable doubt and ORDERING their immediate release from confinement at the New Bilibid Prison, unless their further detention is justified for any lawful ground. The Director of the Bureau of Corrections is hereby directed to report to the Court the release of said accused-appellants within five (5) days from notice of this decision.

SO ORDERED.

Puno, Kapunan, Pardo, and Ynares-Santiago, JJ., concur.

G.R. No. 81561 January 18, 1991

PEOPLE OF THE PHILIPPINES, plaintiff-appelleevs.ANDRE MARTI, accused-appellant.

The Solicitor General for plaintiff-appellee.

Reynaldo B. Tatoy and Abelardo E. Rogacion for accused-appellant.

Page 274: Warrantless Arrests, Searches and Seizures and Privacy of Communication

274

 

BIDIN, J.:p

This is an appeal from a decision * rendered by the Special Criminal Court of Manila (Regional Trial Court, Branch XLIX) convicting accused-appellant of violation of Section 21 (b), Article IV in relation to Section 4, Article 11 and Section 2 (e) (i), Article 1 of Republic Act 6425, as amended, otherwise known as the Dangerous Drugs Act.

The facts as summarized in the brief of the prosecution are as follows:

On August 14, 1987, between 10:00 and 11:00 a.m., the appellant and his common-law wife, Shirley Reyes, went to the booth of the "Manila Packing and Export Forwarders" in the Pistang Pilipino Complex, Ermita, Manila, carrying with them four (4) gift wrapped packages. Anita Reyes (the proprietress and no relation to Shirley Reyes) attended to them. The appellant informed Anita Reyes that he was sending the packages to a friend in Zurich, Switzerland. Appellant filled up the contract necessary for the transaction, writing therein his name, passport number, the date of shipment and the name and address of the consignee, namely, "WALTER FIERZ, Mattacketr II, 8052 Zurich, Switzerland" (Decision, p. 6)

Anita Reyes then asked the appellant if she could examine and inspect the packages. Appellant, however, refused, assuring her that the packages simply contained books, cigars, and gloves and were gifts to his friend in Zurich. In view of appellant's representation, Anita Reyes no longer insisted on inspecting the packages. The four (4) packages were then placed inside a brown corrugated box one by two feet in size (1' x 2'). Styro-foam was placed at the bottom and on top of the packages before the box was sealed with masking tape, thus making the box ready for shipment (Decision, p. 8).

Before delivery of appellant's box to the Bureau of Customs and/or Bureau of Posts, Mr. Job Reyes (proprietor) and husband of Anita (Reyes), following standard operating procedure, opened the boxes for final inspection. When he opened appellant's box, a peculiar odor emitted therefrom. His curiousity aroused, he squeezed one of the bundles allegedly containing gloves and felt dried leaves inside. Opening one of the bundles, he pulled out a cellophane wrapper protruding from the opening of one of the gloves. He made an opening on one of the cellophane wrappers and took several grams of the contents thereof (tsn, pp. 29-30, October 6, 1987; Emphasis supplied).

Job Reyes forthwith prepared a letter reporting the shipment to the NBI and requesting a laboratory examination of the samples he extracted from the cellophane wrapper (tsn, pp. 5-6, October 6, 1987).

He brought the letter and a sample of appellant's shipment to the Narcotics Section of the National Bureau of Investigation (NBI), at about 1:30 o'clock in the afternoon of that date, i.e., August 14, 1987. He was interviewed by the Chief of Narcotics Section. Job Reyes informed the NBI that the rest of the shipment was still in his office. Therefore, Job Reyes and three (3) NBI agents, and a photographer, went to the Reyes' office at Ermita, Manila (tsn, p. 30, October 6, 1987).

Job Reyes brought out the box in which appellant's packages were placed and, in the presence of the NBI agents, opened the top flaps, removed the styro-foam and took out the cellophane wrappers from inside the gloves. Dried marijuana leaves were found to have been contained inside the cellophane wrappers (tsn, p. 38, October 6, 1987; Emphasis supplied).

Page 275: Warrantless Arrests, Searches and Seizures and Privacy of Communication

275

The package which allegedly contained books was likewise opened by Job Reyes. He discovered that the package contained bricks or cake-like dried marijuana leaves. The package which allegedly contained tabacalera cigars was also opened. It turned out that dried marijuana leaves were neatly stocked underneath the cigars (tsn, p. 39, October 6, 1987).

The NBI agents made an inventory and took charge of the box and of the contents thereof, after signing a "Receipt" acknowledging custody of the said effects (tsn, pp. 2-3, October 7, 1987).

Thereupon, the NBI agents tried to locate appellant but to no avail. Appellant's stated address in his passport being the Manila Central Post Office, the agents requested assistance from the latter's Chief Security. On August 27, 1987, appellant, while claiming his mail at the Central Post Office, was invited by the NBI to shed light on the attempted shipment of the seized dried leaves. On the same day the Narcotics Section of the NBI submitted the dried leaves to the Forensic Chemistry Section for laboratory examination. It turned out that the dried leaves were marijuana flowering tops as certified by the forensic chemist. (Appellee's Brief, pp. 9-11, Rollo, pp. 132-134).

Thereafter, an Information was filed against appellant for violation of RA 6425, otherwise known as the Dangerous Drugs Act.

After trial, the court a quo rendered the assailed decision.

In this appeal, accused/appellant assigns the following errors, to wit:

THE LOWER COURT ERRED IN ADMITTING IN EVIDENCE THE ILLEGALLY SEARCHED AND SEIZED OBJECTS CONTAINED IN THE FOUR PARCELS.

THE LOWER COURT ERRED IN CONVICTING APPELLANT DESPITE THE UNDISPUTED FACT THAT HIS RIGHTS UNDER THE CONSTITUTION WHILE UNDER CUSTODIAL PROCEEDINGS WERE NOT OBSERVED.

THE LOWER COURT ERRED IN NOT GIVING CREDENCE TO THE EXPLANATION OF THE APPELLANT ON HOW THE FOUR PARCELS CAME INTO HIS POSSESSION (Appellant's Brief, p. 1; Rollo, p. 55)

1. Appellant contends that the evidence subject of the imputed offense had been obtained in violation of his constitutional rights against unreasonable search and seizure and privacy of communication (Sec. 2 and 3, Art. III, Constitution) and therefore argues that the same should be held inadmissible in evidence (Sec. 3 (2), Art. III).

Sections 2 and 3, Article III of the Constitution provide:

Sec. 2. The right of the people to be secure in their persons, houses, papers and effects against unreasonable searches and seizures of whatever nature and for any purpose shall be inviolable, and no search warrant or warrant of arrest shall issue except upon probable cause to be determined personally by the judge after examination under oath or affirmation of the complainant and the witnesses he may produce, and particularly describing the place to be searched and the persons or things to be seized.

Sec. 3. (1) The privacy of communication and correspondence shall be inviolable except upon lawful order of the court, or when public safety or order requires otherwise as prescribed by law.

Page 276: Warrantless Arrests, Searches and Seizures and Privacy of Communication

276

(2) Any evidence obtained in violation of this or the preceding section shall be inadmissible for any purpose in any proceeding.

Our present constitutional provision on the guarantee against unreasonable search and seizure had its origin in the 1935 Charter which, worded as follows:

The right of the people to be secure in their persons, houses, papers and effects against unreasonable searches and seizures shall not be violated, and no warrants shall issue but upon probable cause, to be determined by the judge after examination under oath or affirmation of the complainant and the witnesses he may produce, and particularly describing the place to be searched, and the persons or things to be seized. (Sec. 1 [3], Article III)

was in turn derived almost verbatim from the Fourth Amendment ** to the United States Constitution. As such, the Court may turn to the pronouncements of the United States Federal Supreme Court and State Appellate Courts which are considered doctrinal in this jurisdiction.

Thus, following the exclusionary rule laid down in Mapp v. Ohio by the US Federal Supreme Court (367 US 643, 81 S.Ct. 1684, 6 L.Ed. 1081 [1961]), this Court, in Stonehill v. Diokno (20 SCRA 383 [1967]), declared as inadmissible any evidence obtained by virtue of a defective search and seizure warrant, abandoning in the process the ruling earlier adopted in Moncado v. People's Court (80 Phil. 1 [1948]) wherein the admissibility of evidence was not affected by the illegality of its seizure. The 1973 Charter (Sec. 4 [2], Art. IV) constitutionalized the Stonehill ruling and is carried over up to the present with the advent of the 1987 Constitution.

In a number of cases, the Court strictly adhered to the exclusionary rule and has struck down the admissibility of evidence obtained in violation of the constitutional safeguard against unreasonable searches and seizures. (Bache & Co., (Phil.), Inc., v. Ruiz, 37 SCRA 823 [1971]; Lim v. Ponce de Leon, 66 SCRA 299 [1975]; People v. Burgos, 144 SCRA 1 [1986]; Roan v. Gonzales, 145 SCRA 687 [1987]; See also Salazar v. Hon. Achacoso, et al., GR No. 81510, March 14, 1990).

It must be noted, however, that in all those cases adverted to, the evidence so obtained were invariably procured by the State acting through the medium of its law enforcers or other authorized government agencies.

On the other hand, the case at bar assumes a peculiar character since the evidence sought to be excluded was primarily discovered and obtained by a private person, acting in a private capacity and without the intervention and participation of State authorities. Under the circumstances, can accused/appellant validly claim that his constitutional right against unreasonable searches and seizure has been violated? Stated otherwise, may an act of a private individual, allegedly in violation of appellant's constitutional rights, be invoked against the State?

We hold in the negative. In the absence of governmental interference, the liberties guaranteed by the Constitution cannot be invoked against the State.

As this Court held in Villanueva v. Querubin (48 SCRA 345 [1972]:

1. This constitutional right (against unreasonable search and seizure) refers to the immunity of one's person, whether citizen or alien, from interference by government, included in which is his residence, his papers, and other possessions. . . .

. . . There the state, however powerful, does not as such have the access except under the circumstances above noted, for in the traditional formulation, his house, however

Page 277: Warrantless Arrests, Searches and Seizures and Privacy of Communication

277

humble, is his castle. Thus is outlawed any unwarranted intrusion by government, which is called upon to refrain from any invasion of his dwelling and to respect the privacies of his life. . . . (Cf. Schermerber v. California, 384 US 757 [1966] and Boyd v. United States, 116 US 616 [1886]; Emphasis supplied).

In Burdeau v. McDowell (256 US 465 (1921), 41 S Ct. 547; 65 L.Ed. 1048), the Court there in construing the right against unreasonable searches and seizures declared that:

(t)he Fourth Amendment gives protection against unlawful searches and seizures, and as shown in previous cases, its protection applies to governmental action. Its origin and history clearly show that it was intended as a restraint upon the activities of sovereign authority, and was not intended to be a limitation upon other than governmental agencies; as against such authority it was the purpose of the Fourth Amendment to secure the citizen in the right of unmolested occupation of his dwelling and the possession of his property, subject to the right of seizure by process duly served.

The above ruling was reiterated in State v. Bryan (457 P.2d 661 [1968]) where a parking attendant who searched the automobile to ascertain the owner thereof found marijuana instead, without the knowledge and participation of police authorities, was declared admissible in prosecution for illegal possession of narcotics.

And again in the 1969 case of Walker v. State (429 S.W.2d 121), it was held that the search and seizure clauses are restraints upon the government and its agents, not upon private individuals (citing People v. Potter, 240 Cal. App.2d 621, 49 Cap. Rptr, 892 (1966); State v. Brown, Mo., 391 S.W.2d 903 (1965); State v. Olsen, Or., 317 P.2d 938 (1957).

Likewise appropos is the case of Bernas v. US (373 F.2d 517 (1967). The Court there said:

The search of which appellant complains, however, was made by a private citizen — the owner of a motel in which appellant stayed overnight and in which he left behind a travel case containing the evidence*** complained of. The search was made on the motel owner's own initiative. Because of it, he became suspicious, called the local police, informed them of the bag's contents, and made it available to the authorities.

The fourth amendment and the case law applying it do not require exclusion of evidence obtained through a search by a private citizen. Rather, the amendment only proscribes governmental action."

The contraband in the case at bar having come into possession of the Government without the latter transgressing appellant's rights against unreasonable search and seizure, the Court sees no cogent reason why the same should not be admitted against him in the prosecution of the offense charged.

Appellant, however, would like this court to believe that NBI agents made an illegal search and seizure of the evidence later on used in prosecuting the case which resulted in his conviction.

The postulate advanced by accused/appellant needs to be clarified in two days. In both instances, the argument stands to fall on its own weight, or the lack of it.

First, the factual considerations of the case at bar readily foreclose the proposition that NBI agents conducted an illegal search and seizure of the prohibited merchandise. Records of the case clearly indicate that it was Mr. Job Reyes, the proprietor of the forwarding agency, who made search/inspection of the packages. Said inspection was reasonable and a standard operating procedure on the part of Mr.

Page 278: Warrantless Arrests, Searches and Seizures and Privacy of Communication

278

Reyes as a precautionary measure before delivery of packages to the Bureau of Customs or the Bureau of Posts (TSN, October 6 & 7, 1987, pp. 15-18; pp. 7-8; Original Records, pp. 119-122; 167-168).

It will be recalled that after Reyes opened the box containing the illicit cargo, he took samples of the same to the NBI and later summoned the agents to his place of business. Thereafter, he opened the parcel containing the rest of the shipment and entrusted the care and custody thereof to the NBI agents. Clearly, the NBI agents made no search and seizure, much less an illegal one, contrary to the postulate of accused/appellant.

Second, the mere presence of the NBI agents did not convert the reasonable search effected by Reyes into a warrantless search and seizure proscribed by the Constitution. Merely to observe and look at that which is in plain sight is not a search. Having observed that which is open, where no trespass has been committed in aid thereof, is not search (Chadwick v. State, 429 SW2d 135). Where the contraband articles are identified without a trespass on the part of the arresting officer, there is not the search that is prohibited by the constitution (US v. Lee 274 US 559, 71 L.Ed. 1202 [1927]; Ker v. State of California 374 US 23, 10 L.Ed.2d. 726 [1963]; Moore v. State, 429 SW2d 122 [1968]).

In Gandy v. Watkins (237 F. Supp. 266 [1964]), it was likewise held that where the property was taken into custody of the police at the specific request of the manager and where the search was initially made by the owner there is no unreasonable search and seizure within the constitutional meaning of the term.

That the Bill of Rights embodied in the Constitution is not meant to be invoked against acts of private individuals finds support in the deliberations of the Constitutional Commission. True, the liberties guaranteed by the fundamental law of the land must always be subject to protection. But protection against whom? Commissioner Bernas in his sponsorship speech in the Bill of Rights answers the query which he himself posed, as follows:

First, the general reflections. The protection of fundamental liberties in the essence of constitutional democracy. Protection against whom? Protection against the state. The Bill of Rights governs the relationship between the individual and the state. Its concern is not the relation between individuals, between a private individual and other individuals. What the Bill of Rights does is to declare some forbidden zones in the private sphere inaccessible to any power holder. (Sponsorship Speech of Commissioner Bernas , Record of the Constitutional Commission, Vol. 1, p. 674; July 17, 1986; Emphasis supplied)

The constitutional proscription against unlawful searches and seizures therefore applies as a restraint directed only against the government and its agencies tasked with the enforcement of the law. Thus, it could only be invoked against the State to whom the restraint against arbitrary and unreasonable exercise of power is imposed.

If the search is made upon the request of law enforcers, a warrant must generally be first secured if it is to pass the test of constitutionality. However, if the search is made at the behest or initiative of the proprietor of a private establishment for its own and private purposes, as in the case at bar, and without the intervention of police authorities, the right against unreasonable search and seizure cannot be invoked for only the act of private individual, not the law enforcers, is involved. In sum, the protection against unreasonable searches and seizures cannot be extended to acts committed by private individuals so as to bring it within the ambit of alleged unlawful intrusion by the government.

Appellant argues, however, that since the provisions of the 1935 Constitution has been modified by the present phraseology found in the 1987 Charter, expressly declaring as inadmissible any evidence obtained in violation of the constitutional prohibition against illegal search and seizure, it matters not whether the evidence was procured by police authorities or private individuals (Appellant's Brief, p. 8, Rollo, p. 62).

Page 279: Warrantless Arrests, Searches and Seizures and Privacy of Communication

279

The argument is untenable. For one thing, the constitution, in laying down the principles of the government and fundamental liberties of the people, does not govern relationships between individuals. Moreover, it must be emphasized that the modifications introduced in the 1987 Constitution (re: Sec. 2, Art. III) relate to the issuance of either a search warrant or warrant of arrest vis-a-vis the responsibility of the judge in the issuance thereof (SeeSoliven v. Makasiar, 167 SCRA 393 [1988]; Circular No. 13 [October 1, 1985] and Circular No. 12 [June 30, 1987]. The modifications introduced deviate in no manner as to whom the restriction or inhibition against unreasonable search and seizure is directed against. The restraint stayed with the State and did not shift to anyone else.

Corolarilly, alleged violations against unreasonable search and seizure may only be invoked against the State by an individual unjustly traduced by the exercise of sovereign authority. To agree with appellant that an act of a private individual in violation of the Bill of Rights should also be construed as an act of the State would result in serious legal complications and an absurd interpretation of the constitution.

Similarly, the admissibility of the evidence procured by an individual effected through private seizure equally applies, in pari passu, to the alleged violation, non-governmental as it is, of appellant's constitutional rights to privacy and communication.

2. In his second assignment of error, appellant contends that the lower court erred in convicting him despite the undisputed fact that his rights under the constitution while under custodial investigation were not observed.

Again, the contention is without merit, We have carefully examined the records of the case and found nothing to indicate, as an "undisputed fact", that appellant was not informed of his constitutional rights or that he gave statements without the assistance of counsel. The law enforcers testified that accused/appellant was informed of his constitutional rights. It is presumed that they have regularly performed their duties (See. 5(m), Rule 131) and their testimonies should be given full faith and credence, there being no evidence to the contrary. What is clear from the records, on the other hand, is that appellant refused to give any written statement while under investigation as testified by Atty. Lastimoso of the NBI, Thus:

Fiscal Formoso:

You said that you investigated Mr. and Mrs. Job Reyes. What about the accused here, did you investigate the accused together with the girl?

WITNESS:

Yes, we have interviewed the accused together with the girl but the accused availed of his constitutional right not to give any written statement, sir. (TSN, October 8, 1987, p. 62; Original Records, p. 240)

The above testimony of the witness for the prosecution was not contradicted by the defense on cross-examination. As borne out by the records, neither was there any proof by the defense that appellant gave uncounselled confession while being investigated. What is more, we have examined the assailed judgment of the trial court and nowhere is there any reference made to the testimony of appellant while under custodial investigation which was utilized in the finding of conviction. Appellant's second assignment of error is therefore misplaced.

3. Coming now to appellant's third assignment of error, appellant would like us to believe that he was not the owner of the packages which contained prohibited drugs but rather a certain Michael, a German national, whom appellant met in a pub along Ermita, Manila: that in the course of their 30-minute conversation, Michael requested him to ship the packages and gave him P2,000.00 for the cost of the

Page 280: Warrantless Arrests, Searches and Seizures and Privacy of Communication

280

shipment since the German national was about to leave the country the next day (October 15, 1987, TSN, pp. 2-10).

Rather than give the appearance of veracity, we find appellant's disclaimer as incredulous, self-serving and contrary to human experience. It can easily be fabricated. An acquaintance with a complete stranger struck in half an hour could not have pushed a man to entrust the shipment of four (4) parcels and shell out P2,000.00 for the purpose and for appellant to readily accede to comply with the undertaking without first ascertaining its contents. As stated by the trial court, "(a) person would not simply entrust contraband and of considerable value at that as the marijuana flowering tops, and the cash amount of P2,000.00 to a complete stranger like the Accused. The Accused, on the other hand, would not simply accept such undertaking to take custody of the packages and ship the same from a complete stranger on his mere say-so" (Decision, p. 19, Rollo, p. 91). As to why he readily agreed to do the errand, appellant failed to explain. Denials, if unsubstantiated by clear and convincing evidence, are negative self-serving evidence which deserve no weight in law and cannot be given greater evidentiary weight than the testimony of credible witnesses who testify on affirmative matters (People v. Esquillo, 171 SCRA 571 [1989]; People vs. Sariol, 174 SCRA 237 [1989]).

Appellant's bare denial is even made more suspect considering that, as per records of the Interpol, he was previously convicted of possession of hashish by the Kleve Court in the Federal Republic of Germany on January 1, 1982 and that the consignee of the frustrated shipment, Walter Fierz, also a Swiss national, was likewise convicted for drug abuse and is just about an hour's drive from appellant's residence in Zurich, Switzerland (TSN, October 8, 1987, p. 66; Original Records, p. 244; Decision, p. 21; Rollo, p. 93).

Evidence to be believed, must not only proceed from the mouth of a credible witness, but it must be credible in itself such as the common experience and observation of mankind can approve as probable under the circumstances (People v. Alto, 26 SCRA 342 [1968], citing Daggers v. Van Dyke, 37 N.J. Eg. 130; see alsoPeople v. Sarda, 172 SCRA 651 [1989]; People v. Sunga, 123 SCRA 327 [1983]); Castañares v. CA, 92 SCRA 567 [1979]). As records further show, appellant did not even bother to ask Michael's full name, his complete address or passport number. Furthermore, if indeed, the German national was the owner of the merchandise, appellant should have so indicated in the contract of shipment (Exh. "B", Original Records, p. 40). On the contrary, appellant signed the contract as the owner and shipper thereof giving more weight to the presumption that things which a person possesses, or exercises acts of ownership over, are owned by him (Sec. 5 [j], Rule 131). At this point, appellant is therefore estopped to claim otherwise.

Premises considered, we see no error committed by the trial court in rendering the assailed judgment.

WHEREFORE, the judgment of conviction finding appellant guilty beyond reasonable doubt of the crime charged is hereby AFFIRMED. No costs.

SO ORDERED.

Fernan, C.J., Gutierrez, Jr. and Feliciano, JJ., concur.

[G.R. No.  138881.  December 18, 2000]

THE PEOPLE OF THE PHILIPPINES, plaintiff-appellee, vs.LEILA JOHNSON Y REYES, accused-appellant.

D E C I S I O N

MENDOZA, J.:

Page 281: Warrantless Arrests, Searches and Seizures and Privacy of Communication

281

This is an appeal from the decision,[1] dated May 14, 1999, of the Regional Trial Court, Branch 110, Pasay City, finding accused-appellant Leila Johnson y Reyesguilty of violation of §16 of R.A. No. 6425 (Dangerous Drugs Act), as amended by R.A. No. 7659, and sentencing her to suffer the penalty of reclusion perpetua and to pay a fine of P500,000.00 and the costs of the suit.

The information against accused-appellant alleged:

That on June 26, 1998 inside the Ninoy Aquino International Airport, and within the jurisdiction of this Honorable Court, the above-named Accused did then and there willfully, unlawfully and feloniously possess three plastic bags of methamphetamine hydrochloride, a regulated drug, each bag weighing:

#1 ONE HUNDRED EIGHTY SEVEN POINT FIVE (187.5) grams;

#2  ONE HUNDRED NINETY EIGHT POINT ZERO (198.0) grams; and

#3 ONE HUNDRED NINETY FOUR POINT SEVEN (194.7) grams, respectively,

or a total of FIVE HUNDRED EIGHTY POINT TWO (580.2) grams of methamphetamine hydrochloride.

That the above-named accused does not have the corresponding

 license or prescription to possess or use said regulated drug.

CONTRARY TO LAW.[2]

Upon being arraigned, accused-appellant pleaded not guilty,[3] whereupon trial was held.

The prosecution presented four witnesses, namely, NBI Forensic Chemist George de Lara, SPO4 Reynaldo Embile, duty frisker Olivia Ramirez, and SPO1 Rizalina Bernal.   The defense presented accused-appellant who testified in her own behalf.

The facts are as follows:

Accused-appellant Leila Reyes Johnson was, at the time of the incident, 58 years old, a widow, and a resident of Ocean Side, California, U.S.A.  She is a former Filipino citizen who was naturalized as an American on June 16, 1968 and had since been working as a registered nurse, taking care of geriatric patients and those with Alzheimer’s disease, in convalescent homes in the United States.[4]

On June 16, 1998, she arrived in the Philippines to visit her son’s family in Calamba, Laguna.   She was due to fly back to the United States on July 26.  On July 25, she checked in at the Philippine Village Hotel to avoid the traffic on the way to the Ninoy Aquino International Airport (NAIA) and checked out at 5:30 p.m. the next day, June 26, 1998.[5]

At around 7:30 p.m. of that day, Olivia Ramirez was on duty as a lady frisker at Gate 16 of the   NAIA departure area.  Her duty was to frisk departing passengers, employees, and crew and check for weapons, bombs, prohibited drugs, contraband goods, and explosives.[6]

When she frisked accused-appellant Leila Johnson, a departing passenger bound for the United States via Continental Airlines CS-912, she felt something hard on the latter’s abdominal area.  Upon inquiry, Mrs. Johnson explained she needed to wear two panty girdles as she had just undergone an operation as a result of an ectopic pregnancy.[7]

Not satisfied with the explanation, Ramirez reported the matter to her superior, SPO4 Reynaldo Embile, saying “Sir, hindi po ako naniniwalang panty lang po iyon.” (“Sir, I do not believe that it is just a panty.”) She was directed to take accused-appellant to the nearest women’s room for

Page 282: Warrantless Arrests, Searches and Seizures and Privacy of Communication

282

inspection.  Ramirez took accused-appellant to the rest room, accompanied by SPO1 Rizalina Bernal.  Embile stayed outside.[8]

Inside the women’s room, accused-appellant was asked again by Ramirez what  the hard object on her stomach was and accused-appellant gave the same answer she had previously given.  Ramirez then asked her “to bring out the thing under her girdle.” Accused-appellant brought out three plastic packs, which Ramirez then turned over to Embile, outside the women’s room.[9]

The confiscated packs, marked as Exhibits C-1, C-2 and C-3, contained a total of 580.2 grams of a substance which was found by NBI Chemist George de Lara to be methamphetamine hydrochloride or “shabu.”[10]

Embile took accused-appellant and the plastic packs to the 1st Regional Aviation and Security Office (1st RASO) at the arrival area of the NAIA, where accused-appellant’s passport and ticket were taken and her luggage opened.  Pictures were taken and her personal belongings were itemized.[11]

In her defense, accused-appellant alleged that she was standing in line at the last boarding gate when she was approached by Embile and two female officers. She claimed she was handcuffed and taken to the women’s room.  There, she was asked to undress and was then subjected to a body search.  She insisted that nothing was found on her person.  She was later taken to a room filled with boxes, garbage, and a chair.  Her passport and her purse containing $850.00 and some change were taken from her, for which no receipt was issued to her.  After two hours, she said, she was transferred to the office of a certain Col. Castillo.[12]

After another two hours, Col. Castillo and about eight security guards came in and threw two white packages on the table.  They told her to admit that the packages were hers.  But she denied knowledge and ownership of the packages.  She was detained at the 1st RASO office until noon of June 28, 1999 when she was taken before a fiscal for inquest. [13] She claimed that throughout the period of her detention, from the night of June 26 until June 28, she was never allowed to talk to counsel nor was she allowed to call the U.S. Embassy or any of her relatives in the Philippines.[14]

On May 14, 1999, the trial court rendered a decision, the dispositive portion of which reads: [15]

WHEREFORE, judgment is hereby rendered finding the accused LEILA JOHNSON Y REYES, GUILTY beyond reasonable doubt of the offense of Violation of Section 16 of Republic Act 6425 as amended and hereby imposes on her the penalty of RECLUSION PERPETUA and condemns said accused to pay a fine of FIVE HUNDRED THOUSAND PESOS (P500,000.00) without subsidiary imprisonment in case of insolvency and to pay the costs of suit.

The Methamphetamine Hydrochloride (shabu) having a total net weight of 580.2 grams (Exhibits “G”, “C-2” and “C-3”) are hereby confiscated in favor of the government and the Branch Clerk of Court is hereby ordered to cause the transportation thereof to the Dangerous Drugs Board for disposition in accordance with law.

The accused shall be credited in full for the period of her detention at the City Jail of Pasay City during the pendency of this case provided that she agreed in writing to abide by and comply strictly with the rules and regulations of the City Jail.

SO ORDERED.

Accused-appellant contends that the trial court convicted her:  (1) “despite failure of the prosecution in proving the negative allegation in the information;” (2) “despite failure of the prosecution in proving the quantity of methamphetamine hydrochloride;” (3) “despite violation of her constitutional rights;” and (4) “when guilt was not proven beyond reasonable doubt.”[16]

First.  Accused-appellant claims that she was arrested and detained in gross violation of her constitutional rights.  She argues that the “shabu” confiscated from her is inadmissible against her

Page 283: Warrantless Arrests, Searches and Seizures and Privacy of Communication

283

because she was forced to affix her signature on the plastic bags while she was detained at the 1 st RASO office, without the assistance of counsel and without having been informed of her constitutional rights. Hence, she argues, the methamphetamine hydrochloride, or “shabu,” should have been excluded from the evidence.[17]

The contention has no merit.  No statement, if any, was taken from accused-appellant during her detention and used in evidence against her.  There is, therefore, no basis for accused-appellant’s invocation of Art. III, §12(1) and (3).  On the other hand, what is involved in this case is an arrest in flagrante delicto pursuant to a valid search made on her person.

The trial court held:

The constitutional right of the accused was not violated as she was never placed under custodial investigation but was validly arrested without warrant pursuant to the provisions of Section 5, Rule 113 of the 1985 Rules of Criminal Procedure which provides:

Sec. 5.  Arrest without warrant; when lawful.  A peace officer or a private person may, without a warrant, arrest a person:

(a)               when in his presence, the person to be arrested has committed, is actually committing, or is attempting to commit an offense;

(b)               when an offense has in fact just been committed, and he has personal knowledge of facts indicating that the person to be arrested has committed it; and

(Underscoring supplied)

x x x x

A custodial investigation has been defined in People. v. Ayson 175 SCRA 230 as “the questioning initiated by law enforcement officers after a person has been taken [in] custody or otherwise deprived of his freedom in any significant way.  This presupposes that he is suspected of having committed an offense and that the investigator is trying to elicit information or [a] confession from him."

The circumstances surrounding the arrest of the accused above falls in either paragraph (a) or (b) of the Rule above cited, hence the allegation that she has been subjected to custodial investigation is far from being accurate.[18]

The methamphetamine hydrochloride seized from her  during the routine frisk at the airport was acquired legitimately pursuant to airport security procedures.

Persons may lose the protection of the search and seizure clause by exposure of their persons or property to the public in a manner reflecting a lack of subjective expectation of privacy, which expectation society is prepared to recognize as reasonable.[19] Such recognition is implicit in airport security procedures.  With increased concern over airplane hijacking and terrorism has come increased security at the nation’s airports.  Passengers attempting to board an aircraft routinely pass through metal detectors; their carry-on baggage as well as checked luggage are routinely subjected to x-ray scans.  Should these procedures suggest the presence of suspicious objects, physical searches are conducted to determine what the objects are.  There is little question that such searches are reasonable, given their minimal intrusiveness, the gravity of the safety interests involved, and the reduced privacy expectations associated with airline travel.[20] Indeed, travelers are often notified through airport public address systems, signs, and notices in their airline tickets that they are subject to search and, if any prohibited materials or substances are found, such would be subject to seizure.  These announcements place passengers on notice that ordinary constitutional protections against warrantless searches and seizures do not apply to routine airport procedures.

Page 284: Warrantless Arrests, Searches and Seizures and Privacy of Communication

284

The packs of methamphetamine hydrochloride having thus been obtained through a valid warrantless search, they are admissible in evidence against the accused-appellant herein.  Corollarily, her subsequent arrest, although likewise without warrant, was justified since it was effected upon the discovery and recovery of “shabu” in her person in flagrante delicto.

Anent her allegation that her signature on the said packs (Exhibits C-1, C-2 and C-3 herein) had been obtained while she was in the custody of the airport authorities without the assistance of counsel, the Solicitor General correctly points out that nowhere in the records is it indicated that accused-appellant was required to affix her signature to the packs.  In fact, only the signatures of Embile and Ramirez thereon, along with their testimony to that effect, were presented by the prosecution in proving its case.

There is, however, no justification for the confiscation of accused-appellant’s passport, airline ticket, luggage, and other personal effects.   The pictures taken during that time are also inadmissible, as are the girdle taken from her, and her signature thereon.  Rule 126, §2 of the Revised Rules of Criminal Procedure authorizes the search and seizure only of the following:

Personal property to be seized. ¾  A search warrant may be issued for the search and seizure of personal property:

(a)      Subject of the offense;

(b)      Stolen or embezzled and other proceeds or fruits of the offense; and

(c)      Used or intended to be used as the means of committing an offense.

Accordingly, the above items seized from accused-appellant should be returned to her.

Second.  Accused-appellant argues that the prosecution failed to fully ascertain the quantity of methamphetamine hydrochloride to justify the imposition of the penalty of reclusion perpetua.

Section 20 of R.A. No. 6425, as amended by R.A. No. 7659, states:

Section 20 - Application Of Penalties, Confiscation And Forfeiture Of The Proceeds or Instrument Of The Crime – The penalties for offenses under Section 3, 4, 7, 8 and 9 of Article II and Sections 14, 14-A, 15 and 16 of Article III of this Act, shall be applied if the dangerous drugs involved is in any of the following quantities:

1.  40 grams or more of opium;

2.  40 grams or more of morphine;

3.  200 grams or more of shabu, or methylamphetamine hydrochloride;

4.  40 grams or more of heroin;

5.  750 grams or more of indian hemp of marijuana;

6.  50 grams of marijuana resin or marijuana resin oil;

7.  40 grams or more of cocaine or cocaine hydrochloride; or

8.  In case of other dangerous drugs, the quantity of which is far beyond therapeutic requirements as determined and promulgated by the Dangerous Drugs Board, after public consultation/hearings conducted for the purpose.

Page 285: Warrantless Arrests, Searches and Seizures and Privacy of Communication

285

Otherwise, if the quantity involved is less than the foregoing quantities, the penalty shall range from prision correccional to reclusion perpetua depending upon the quantity.

Under this provision, accused-appellant therefore stands to suffer the penalty of  reclusion perpetua to death for her possession of 580.2 grams of shabu.

Accused-appellant attempts to distinguish between a quantitative and a qualitative examination of the substance contained in Exhibits C-1, C-2 and C-3.  She argues that the examination conducted by the NBI forensic chemist was a qualitative one which merely yielded positive findings for shabu, but failed to establish its purity; hence, its exact quantity remains indeterminate and unproved.

This contention is likewise without merit.

The expert witness, George De Lara, stated that the tests conducted would have indicated the presence of impurities if there were any.  He testified:

PROS. VELASCO  By mixing it twice, Mr. Witness, if there are any adulterants or impurities, it will be discovered by just mixing it?

WITNESS   If some drugs or additives were present, it will appear in a thin layer chromatographic examination.

PROS. VELASCO  Did other drugs or other additives appear Mr. Witness?

WITNESS   In my thin layer chromatographic plate, it only appears one spot which resembles or the same as the Methamphetamine Hydrochloride sample

. . . .

PROS. VELASCO  So, Mr. Witness, if there are any adulterants present in the chemicals you have examined, in chemical examination, what color it will register, if any?

WITNESS   In sample, it contained a potassium aluminum sulfate, it will not react with the reagent, therefore it will not dissolve.  In my examination, all the specimens reacted on the re-agents, sir.

PROS. VELASCO  And what is potassium aluminum sulfate in layman’s term?

WITNESS   It is only a tawas.

. . . .

COURT       In this particular case, did you find any aluminum sulfate or tawas in the specimen?

WITNESS   None, your Honor.

. . . .

ATTY. AGOOT       I will cite an example, supposing ten grams of Methamphetamine Hydrochloride is mixed with 200 grams of tawas, you will submit that to qualitative examination, what will be your findings, negative or positive, Mr. Witness?

WITNESS   It will give a positive result for Methamphetamine Hydrochloride.

ATTY. AGOOT       That is qualitative examination.

WITNESS   And also positive for aluminum sulfate.[21]

A qualitative determination relates to the identity of the material, whereas a quantitative analysis requires the determination of the percentage combination of the components of a mixture. Hence, a qualitative identification of a powder may reveal the presence of heroin and quinine, for instance, whereas a quantitative analysis may conclude the presence of 10 percent heroin and 90 percent quinine. [22]

De Lara testified that he used a chromatography test to determine the contents of Exhibits C-1, C-2 and C-3.  Chromatography is a means of separating and tentatively identifying the components of a

Page 286: Warrantless Arrests, Searches and Seizures and Privacy of Communication

286

mixture.  It is particularly useful for analyzing the multicomponent specimens that are frequently received in a crime lab.  For example, illicit drugs sold on the street may be diluted with practically any material that is at the disposal of the drug dealer to increase the quantity of the product that is made available to prospective customers.  Hence, the task of identifying an illicit drug preparation would be an arduous one without the aid of chromatographic methods to first separate the mixture into its components. [23]

The testimony of De Lara established not only that the tests were thorough, but also that the scientifically correct method of obtaining an accurate representative sample had been obtained. [24] At any rate, as the Solicitor-General has pointed out, if accused-appellant was not satisfied with the results, it would have been a simple matter for her to ask for an independent examination of the substance by another chemist.  This she did not do.

Third.  Accused-appellant argues that the prosecution failed to prove the negative allegation in the information that she did not have a license to possess or use methamphetamine hydrochloride or “shabu.”

Art. III of Republic Act No. 6425, as amended by Republic Act No. 7659 provides:

SEC. 16.  Possession or Use of Regulated Drugs.  - The penalty of reclusion perpetua to death and a fine ranging from five hundred thousand pesos to ten million pesos shall be imposed upon any person who shall possess or use any regulated drug without the corresponding license or prescription, subject to the provisions of Section 20 hereof.

Accused-appellant claims that possession or use of methamphetamine hydrochloride or “shabu,”a regulated drug, is not unlawful unless the possessor or user does not have the required license or prescription.  She points out that since the prosecution failed to present any certification that she is not authorized to possess or use regulated drugs, it therefore falls short of the quantum of proof needed to sustain a conviction.

The contention has no merit.

The question raised in this case is similar to that raised in United States v. Chan Toco.[25] The accused in that case was charged with smoking opium without being duly registered.  He demurred to the information on the ground that it failed to allege that the use of opium had not been prescribed as a medicine by a duly licensed and practicing physician.

This Court denied the motion and said:

The evident interest and purpose of the statute is to prohibit and to penalize generally the smoking of opium in these Islands.  But the legislator desired to withdraw from the operation of the statute a limited class of smokers who smoked under the advice and by prescription of a licensed and practicing physician . . . . Hence where one is charged with a violation of the general provisions of the Opium Law,  it is more logical as well as more practical and convenient, if he did in fact smoke opium under the advice of a physician, that he should set up this fact by way of defense, than that the prosecution should be called upon to prove that every smoker, charged with a violation of the law, does so without such advice or prescription.  Indeed, when it is considered that under the law any person may, in case of need and at any time, procure the advice of a physician to use opium or some of its derivatives, and that in the nature of things no public record of prescriptions of this kind is or can be required to be kept, it is manifest that it would be wholly impracticable and absurd to impose on the prosecution the burden of alleging and proving the fact that one using opium does so without the advice of a physician.  To prove beyond a reasonable doubt, in a particular case, that one using opium does so without the advice or prescription of a physician would be in most cases a practical impossibility without the aid of the defendant himself, while a defendant charged with the illegal use of opium should find little difficulty in establishing the fact that he used it under the advice and on the prescription of a physician, if in fact he did so.[26]

An accused person sometimes owes a duty to himself if not to the State.   If he does not perform that duty he may not always expect the State to perform it for him.  If he fails to meet the obligation which he owes to himself, when to meet it is an easy thing for him to do, he has no one but himself to blame.

Page 287: Warrantless Arrests, Searches and Seizures and Privacy of Communication

287

Moreover, as correctly pointed out by the Solicitor General, there is nothing in R.A. No. 6425 or the Dangerous Drugs Act, as amended, which requires the prosecution to present a certification that accused-appellant has no license or permit to possess shabu.  Mere possession of the prohibited substance is a crime per se and the burden of proof is upon accused-appellant to show that she has a license or permit under the law to possess the prohibited drug.

Fourth. Lastly, accused-appellant contends that the evidence presented by the prosecution is not sufficient to support a finding that she is guilty of the crime charged.

This contention must likewise be rejected.

Credence was properly accorded to the testimonies of the prosecution witnesses, who are law enforcers.  When police officers have no motive to testify falsely against the accused, courts are inclined to uphold this presumption.  In this case, no evidence has been presented to suggest any improper motive on the part of the police enforcers in arresting accused-appellant.  This Court accords great respect to the findings of the trial court on the matter of credibility of the witnesses in the absence of any palpable error or arbitrariness in its findings.[27]

It is noteworthy that, aside from the denial of accused-appellant, no other witness was presented in her behalf.  Her denial cannot prevail over the positive testimonies of the prosecution witnesses. [28] As has been held, denial as a rule is a weak form of defense, particularly when it is   not  substantiated by clear and convincing  evidence.  The defense of denial or frame-up, like alibi, has been invariably viewed by the courts with disfavor for it can just as easily be concocted and is a common and standard defense ploy in most prosecutions for violation of the Dangerous Drugs Act.[29]

The Court is convinced that the requirements of the law in order that a person may be validly charged with and convicted of illegal possession of a dangerous drug in violation of R.A. No. 6425, as amended, have been complied with by the prosecution in this case.  The decision of the trial court must accordingly be upheld.

As regards the fine imposed by the trial court, it has been held that courts may fix any amount within the limits established by law.[30] Considering that five hundred eighty point two (580.2) grams of shabu were confiscated from accused-appellant, the fine imposed by the trial court may properly be reduced to  P50,000.00.

WHEREFORE, the decision of the Regional Trial Court of Pasay City, Branch 110, finding accused-appellant guilty of violation of §16 of R.A. No. 6425, as amended, and imposing upon her the penalty of reclusion perpetua is hereby AFFIRMED with the MODIFICATION that the fine imposed on accused-appellant is reduced to P50,000.00.  Costs against appellant.

The passport, airline ticket, luggage, girdle and other personal effects not yet returned to the accused-appellant are hereby ordered returned to her.

SO ORDERED.

Bellosillo, (Chairman), Quisumbing, Buena, and De Leon, Jr., JJ., concur.

[G.R. No. 148825.  December 27, 2002]

PEOPLE OF THE PHILIPPINES, appellee, vs. SUSAN CANTON, appellant.

Page 288: Warrantless Arrests, Searches and Seizures and Privacy of Communication

288

D E C I S I O N

DAVIDE, JR., C.J.:

Appellant Susan Canton (hereafter SUSAN) was charged before the Regional Trial Court of Pasay City with the violation of Section 16 of Article III of the Dangerous Drugs Act of 1972 (Republic Act No. 6425), as amended, under an Information[1] whose accusatory portion reads as follows:

That on February 12, 1998 at the Ninoy Aquino International Airport, and within the jurisdiction of this Honorable Court, the above named accused did then and there willfully, unlawfully and feloniously has in her possession NINE HUNDRED NINETY EIGHT POINT TWO EIGHT HUNDRED ZERO NINE (998.2809) GRAMS of methamphetamine hydrochloride, a regulated drug, without the corresponding prescription or license.

CONTRARY TO LAW.

The case was docketed as Criminal Case No. 98-0189 and raffled to Branch 110 of said court.

SUSAN entered a plea of not guilty upon her arraignment.

At the trial, the prosecution presented as witnesses Forensic Chemist Julieta Flores, lady frisker Mylene Cabunoc, and SPO4 Victorio de los Reyes.

For its part, the defense presented SPO2 Jerome Cause as its witness and had prosecution witness Mylene Cabunoc recalled to be presented as hostile witness. It opted not to let SUSAN take the witness stand.

The evidence for the prosecution established that on 12 February 1998, at about 1:30 p.m., SUSAN was at the Ninoy Aquino International Airport (NAIA), being a departing passenger bound for Saigon, Vietnam.[2] When she passed through the metal detector booth, a beeping sound was emitted.  Consequently, Mylene Cabunoc, a civilian employee of the National Action Committee on Hijacking and Terrorism (NACHT) and the frisker on duty at that time, called her attention, saying “Excuse me ma’am, can I search you?”[3]  Upon frisking SUSAN, Mylene felt something bulging at her abdominal area.  Mylene inserted her hand under the skirt of SUSAN, pinched the package several times and noticed that the package contained what felt like rice granules. [4] When Mylene passed her hand, she felt similar packages in front of SUSAN’s genital area and thighs.  She asked SUSAN to bring out the packages, but the latter refused and said: “Money, money only.” Mylene forthwith reported the matter to SPO4 Victorio de los Reyes, her supervisor on duty.[5]

SPO4 De los Reyes instructed Mylene to call Customs Examiner Lorna Jalac and bring SUSAN to a comfort room for a thorough physical examination.  Upon further frisking in the ladies’ room, Mylene touched something in front of SUSAN’s sex organ.  She directed SUSAN to remove her skirt, girdles and panty.  SUSAN obliged.  Mylene and Lorna discovered three packages individually wrapped and sealed in gray colored packing tape, which SUSAN voluntarily handed to them. [6] The first was taken from SUSAN’s abdominal area; the second, from in front of her genital area; and the third, from her right thigh.[7]  Mylene turned over the packages to SPO4 De los Reyes. [8] The latter forthwith informed his superior officer Police Superintendent Daniel Santos about the incident.  Together with SUSAN, they brought the gray plastic packs to the customs examination table, opened the same and found that they contained white crystalline substances[9] which, when submitted for laboratory examination, yielded positive results for methamphetamine hydrochloride or shabu, a regulated drug.[10]

For the defense, SPO2 Jerome Cause, an investigator of the First Regional Aviation Office, testified that no investigation was ever conducted on SUSAN.[11]However, SUSAN signed a receipt of the following articles seized from her: (1) three bags of methamphetamine hydrochloride or shabu approximately 1,100 grams; (2) one American passport bearing Number 700389994; (3) one Continental Micronesia plane ticket with stock control number 0414381077; and (4) two panty girdles. [12]He said that he informed SUSAN of her constitutional rights but admitted that she did not have a counsel when she signed the receipt.[13]  Yet he told her that she had the option to sign or not to sign the receipt.[14]

Page 289: Warrantless Arrests, Searches and Seizures and Privacy of Communication

289

When recalled as witness for the defense, Mylene merely reiterated the circumstances surrounding the arrest and search of SUSAN and the seizure of the prohibited items found on her person. [15]

After consideration of the evidence presented, the trial court rendered a decision [16] finding SUSAN guilty beyond reasonable doubt of the offense of violation of Section 16 of Article III of Republic Act No. 6425, as amended, and sentencing her to suffer the penalty of reclusion perpetua and to pay a fine of P1 million.

SUSAN filed a Motion for Reconsideration and/or New Trial,[17] alleging therein that the trial judge erred in (1) giving weight to the medical certificate executed by a certain Dr. Ma. Bernadette Arcena because it was not presented in court nor marked or admitted, and is therefore hearsay evidence; (2) upholding the presumption of regularity in the performance of duty of police officers, since lady frisker Mylene Cabunoc is not even a police officer; (3) making statements which gave the impression that the burden of proof was shifted to the accused; and (4) deliberately ignoring the decisive issue of how the evidence was secured.  SUSAN also assailed the propriety of the search and seizure without warrant on the ground that the seized items were not in plain view.  Furthermore, alleging bias and prejudice on the part of the trial judge, SUSAN filed a motion to inhibit Judge Porfirio G. Macaraeg from resolving the Motion for Reconsideration and/or New Trial.[18]

After conducting a hearing on 24 November 2000 to resolve appellant’s Motion for Reconsideration and/or New Trial, as well as the Motion to Inhibit the Judge, the trial court issued an order [19] on 26 November 2001 denying the motions.  According to the trial judge (1) he explained to SUSAN’s counsel the effects of the filing of a motion for reconsideration, but the latter chose to magnify the judge’s statement which was uttered in jest; (2) SUSAN’s conviction was not based on the medical report which was not presented in court; (3) there was no violation of SUSAN’s  constitutional rights because she was never interrogated during her detention without counsel; and (4) the specimens seized from her were found after a routine frisk at the airport and were therefore acquired legitimately pursuant to airport security procedures.

Unsatisfied with the decision of the trial court, SUSAN seasonably appealed to us, imputing to the trial court the following errors: (1) in justifying the warrantless search against her based on the alleged existence of probable cause; (2) in holding that she was caught flagrante  delicto and that the warrantless search was incidental to a lawful arrest; (3) in not ruling that the frisker went beyond the limits of the “Terry search” doctrine; (4) in not ruling that SUSAN was under custodial investigation without counsel; (5) in admitting to the records of the case the report of Dr. Ma. Bernadette Arcena, which was not testified on or offered in evidence, and using the same in determining her guilt; (6) in justifying under the rule on judicial notice its cognizance of the medical report that has not been offered in evidence; and (7) in applying the ruling in People v. Johnson.[20]

For assigned errors nos. 1 and 2, SUSAN asserts that the strip search conducted on her in the ladies’ room was constitutionally infirmed because it was not “incidental to an arrest.”  The arrest could not be said to have been made before the search because at the time of the strip search, the arresting officers could not have known what was inside the plastic containers hidden on her body, which were wrapped and sealed with gray tape.  At that point then, they could not have determined whether SUSAN was actually committing a crime.  The strip search was therefore nothing but a fishing expedition.  Verily, it is erroneous to say that she was caught flagrante delicto and that the warrantless search was incidental to a lawful arrest.

For assigned error no. 3, SUSAN maintains that, following the doctrine enunciated in Terry v. Ohio,[21] such stop and frisk search should have been limited to the patting of her outer garments in order to determine whether she was armed or dangerous and therefore a threat to the security of the aircraft.

For assigned error no. 4, SUSAN alleges that from the moment frisker Mylene felt a package at her abdominal area, started inquiring about the contents thereof, detained her, and decided to submit her to a strip search in the ladies’ room, she was under custodial investigation without counsel, which was violative of Section 12, Article III of the Constitution.

Page 290: Warrantless Arrests, Searches and Seizures and Privacy of Communication

290

For assigned errors nos. 5 and 6, SUSAN assails the propriety of the admission of the medical report executed by Dr. Ma. Bernadette Arcena on the ground that it was neither testified on nor offered in evidence.

Lastly, SUSAN questions the application of People v. Johnson[22] because of its sweeping statement allowing searches and seizures of departing passengers in airports in view of the gravity of the safety interests involved.  She stresses that the pertinent case should have been Katz v. United States,[23] which upholds the Fourth Amendment of the United States of America that “protects people and not places.”

In its Appellant’s Brief, the Office of the Solicitor General (OSG) declares that SUSAN was found flagrante delicto in possession of a regulated drug without being authorized by law.  Thus, the case falls squarely within the exception, being a warrantless search incidental to a lawful arrest.  Moreover, SUSAN voluntarily submitted herself to the search and seizure when she allowed herself to be frisked and brought to the comfort room for further inspection by airport security personnel.  It likewise maintains that the methamphetamine hydrochloride seized from SUSAN during the routine frisk at the airport was acquired legitimately pursuant to airport security procedures.

Anent the admission of the medical certificate issued by Dr. Ma. Bernadette Arcena, the OSG argues that SUSAN’s conviction was not solely based on the questioned document but also on the fact that she was caught flagrante delicto in possession of a regulated drug without being authorized by law.  Consequently, it supports SUSAN’s conviction but recommends the reduction of the fine from P1 million to P100,000.

We affirm SUSAN’s conviction.

We do not agree that the warrantless search and subsequent seizure of the regulated drugs, as well as the arrest of SUSAN, were violative of her constitutional rights.

Sections 2 and 3(2) of Article III of the 1987 Constitution provides:

Sec. 2.  The right of the people to be secure in their persons, houses, papers and effects against unreasonable searches and seizures of whatever nature and for any purpose shall be inviolable, and no search warrant or warrant of arrest shall issue except upon probable cause to be determined personally by the judge after examination under oath or affirmation of the complainant and the witnesses he may produce, and particularly describing the place to be searched and the persons or things to be seized.

Sec. 3….

(2)  Any evidence obtained in violation of this or the preceding section shall be inadmissible for any purpose in any proceeding.

What constitutes a reasonable or unreasonable search in any particular case is a judicial question, determinable from a consideration of the circumstances involved.  The rule is that the Constitution bars State intrusions to a person's body, personal effects or residence except if conducted by virtue of a valid search warrant issued in compliance with the procedure outlined in the Constitution and reiterated in the Rules of Court. [24]

The interdiction against warrantless searches and seizures is not absolute.  The recognized exceptions established by jurisprudence are (1) search of moving vehicles; (2) seizure in plain view; (3) customs searches; (4) waiver or consented searches; (5) stop and frisk situations (Terry search); and (6) search incidental to a lawful arrest.[25]

I.  The search conducted on SUSAN was not incidental to a lawful arrest.

We do not agree with the trial court and the OSG that the search and seizure conducted in this case were incidental to a lawful arrest.  SUSAN’s arrest did not precede the search.  When the metal detector

Page 291: Warrantless Arrests, Searches and Seizures and Privacy of Communication

291

alarmed while SUSAN was passing through it, the lady frisker on duty forthwith made a pat down search on the former. In the process, the latter felt a bulge on SUSAN’s abdomen.  The strip search that followed was for the purpose of ascertaining what were the packages concealed on SUSAN’s body.  If ever at the time SUSAN was deprived of her will and liberty, such restraint did not amount to an arrest.  Under Section 1 of Rule 113 of the Revised Rules of Criminal Procedure, as amended, arrest is the “taking of a person into custody in order that he may be bound to answer for the commission of an offense.”

As pointed out by the appellant, prior to the strip search in the ladies’ room, the airport security personnel had no knowledge yet of what were hidden on SUSAN’s body; hence, they did not know yet whether a crime was being committed.  It was only after the strip search upon the discovery by the police officers of the white crystalline substances inside the packages, which they believed to be shabu, that SUSAN was arrested.  The search cannot, therefore, be said to have been done incidental to a lawful arrest.  In a search incidental to a lawful arrest, the law requires that there be first a lawful arrest before a search can be made; the process cannot be reversed.[26]

II. The scope of a search pursuant to airport security procedure is not confined only to search for weapons under the “Terry search” doctrine.

The Terry search or the “stop and frisk” situation refers to a case where a police officer approaches a person who is acting suspiciously, for purposes of investigating possibly criminal behavior in line with the general interest of effective crime prevention and detection.  To assure himself that the person with whom he is dealing is not armed with a weapon that could unexpectedly and fatally be used against him, he could validly conduct a carefully limited search of the outer clothing of such person to discover weapons which might be used to assault him.[27]

In the present case, the search was made pursuant to routine airport security procedure, which is allowed under Section 9 of Republic Act No. 6235 reading as follows:

SEC. 9. Every ticket issued to a passenger by the airline or air carrier concerned shall contain among others the following condition printed thereon: “Holder hereof and his hand-carried luggage(s) are subject to search for , and seizure of, prohibited materials or substances.  Holder refusing to be searched shall not be allowed to board the aircraft,” which shall constitute a part of the contract between the passenger and the air carrier.

This constitutes another exception to the proscription against warrantless searches and seizures.  As admitted by SUSAN and shown in Annex “D” of her Brief, the afore-quoted provision is stated in the “Notice to All Passengers” located at the final security checkpoint at the departure lounge.   From the said provision, it is clear that the search, unlike in the Terry search, is not limited to weapons.  Passengers are also subject to search for prohibited materials or substances.

In this case, after the metal detector alarmed SUSAN consented to be frisked, which resulted in the discovery of packages on her body.  It was too late in the day for her to refuse to be further searched because the discovery of the packages whose contents felt like rice granules, coupled by her apprehensiveness and her obviously false statement that the packages contained only money, aroused the suspicion of the frisker that SUSAN was hiding something illegal.   It must be repeated that R.A. No. 6235 authorizes search for prohibited materials or substances.  To limit the action of the airport security personnel to simply refusing her entry into the aircraft and sending her home (as suggested by appellant), and thereby depriving them of “the ability and facility to act accordingly, including to further search without warrant, in light of such circumstances, would be to sanction impotence and ineffectivity in law enforcement, to the detriment of society.”[28] Thus, the strip search in the ladies’ room was justified under the circumstances.

III.  The ruling in People v. Johnson is applicable to the instant case.

The case of People v. Johnson, which involves similar facts and issues, finds application to the present case.  That case involves accused-appellant Leila Johnson, who was also a departing passenger bound for the United States via Continental Airlines CS-912.  Olivia Ramirez was then the frisker on duty, whose task was to frisk departing passengers, employees and crew to check for weapons, bombs,

Page 292: Warrantless Arrests, Searches and Seizures and Privacy of Communication

292

prohibited drugs, contraband goods and explosives.  When Olivia frisked Leila, the former felt something hard on the latter’s abdominal area.  Upon inquiry, Leila explained that she needed to wear two panty girdles, as she had just undergone an operation as a result of an ectopic pregnancy.   Not satisfied with the explanation, Olivia reported the matter to her superior, who then directed her to take Leila to the nearest women’s room for inspection.  In the comfort room, Leila was asked “to bring out the thing under her girdle.”  She acceded and brought out three plastic packs which contained a total of 580.2 grams of methamphetamine hydrochloride or shabu.  This Court ruled that the packs of “methamphetamine hydrochloride” seized during the routine frisk at the airport was acquired legitimately pursuant to airport security procedures and are therefore admissible in evidence against Leila.  Corollarily, her subsequent arrest, although likewise without warrant, was justified, since it was effected upon the discovery and recovery of shabu in her person flagrante delicto.  The Court held in this wise:

Persons may lose the protection of the search and seizure clause by exposure of their persons or property to the public in a manner reflecting a lack of subjective expectation of privacy, which expectation society is prepared to recognize as reasonable.  Such recognition is implicit in airport security procedures.  With increased concern over airplane hijacking and terrorism has come increased security at the nation’s airports.  Passengers attempting to board an aircraft routinely pass through metal detectors; their carry-on baggage as well as checked luggage are routinely subjected to x-ray scans.  Should these procedures suggest the presence of suspicious objects, physical searches are conducted to determine what the objects are.  There is little question that such searches are reasonable, given their minimal intrusiveness, the gravity of the safety interests involved, and the reduced privacy expectations associated with airline travel.  Indeed, travelers are often notified through airport public address systems, signs, and notices in their airline tickets that they are subject to search and, if any prohibited materials or substances are found, such would be subject to seizure.  These announcements place passengers on notice that ordinary constitutional protections against warrantless searches and seizures do not apply to routine airport procedures.

SUSAN’s reliance on Katz v. U.S.[29] is misplaced.  The facts and circumstances of that case are entirely different from the case at bar.  In that case, the accused was convicted in the United States District Court for the Southern District of California of transmitting wagering information by telephone.  During the trial, the government was permitted, over the accused’s objection, to introduce evidence of accused’s end of telephone conversations, which was overheard by FBI agents who had attached an electronic listening and recording device to the outside of the public telephone booth from which he placed his calls.  The Court of Appeals for the Ninth Circuit affirmed the conviction.  On certiorari, however, the Supreme Court of the United States of America reversed the decision, ruling that antecedent judicial authorization, which was not given in the instant case, was a constitutional precondition of the kind of electronic surveillance involved.  It ruled that what a person knowingly exposes to the public, even in his own house or office, is not a subject the Fourth Amendment protection, but what he seeks to preserve as private, even in an area accessible to the public, may be constitutionally protected.

The maxim – stare decisis et non quieta movere – invokes adherence to precedents and mandates not to unsettle things which are established.  When the court has once laid down a principle of law as applicable to a certain state of facts, it must adhere to that principle and apply it to all future cases where the facts are substantially the same.[30] There being a disparity in the factual milieu of Katz v. U.S. and the instant case, we cannot apply to this case the ruling in Katz.

IV.  The appellant, having been caught flagrante delicto, was lawfully arrested without a warrant.

Section 5, Rule 113 of the Rules of Court, as amended, provides:

SEC. 5. Arrest without warrant; when lawful. -- A peace officer or a private person may, without a warrant, arrest a person:

(a) When, in his presence, the person to be arrested has committed, is actually committing, or is attempting to commit an offense;

Page 293: Warrantless Arrests, Searches and Seizures and Privacy of Communication

293

(b) When an offense has just been committed and he has probable cause to believe based on personal knowledge of facts or circumstances that the person to be arrested has committed it; and

(c) When the person to be arrested is a prisoner who has escaped from a penal establishment or place where he is serving final judgment or is temporarily confined while his case is pending, or has escaped while being transferred from one confinement to another.

In cases falling under paragraphs (a) and (b) above, the person arrested without a warrant shall be forthwith delivered to the nearest police station or jail and shall be proceeded against in accordance with section 7 of Rule 112.

The present case falls under paragraph (a) of the afore-quoted Section.  The search conducted on SUSAN resulted in the discovery and recovery of three packages containing white crystalline substances, which upon examination yielded positive results for methamphetamine hydrochloride or shabu.  As discussed earlier, such warrantless search and seizure were legal.  Armed with the knowledge that SUSAN was committing a crime, the airport security personnel and police authorities were duty-bound to arrest her.  As held in People v. Johnson, her subsequent arrest without a warrant was justified, since it was effected upon the discovery and recovery of shabu in her person flagrante delicto.

V. The constitutional right to counsel afforded an accused under custodial investigation was not violated.

Entrenched is the rule that the rights provided in Section 12, Article III of the Constitution may be invoked only when a person is under “custodial investigation” or is “in custody interrogation.” [31] Custodial investigation refers to the “questioning initiated by law enforcement officers after a person has been taken into custody or otherwise deprived of his freedom of action in any significant way.” [32] This presupposes that he is suspected of having committed a crime and that the investigator is trying to elicit information or a confession from him.[33] And the right to counsel attaches upon the start of such investigation. [34] The objective is to prohibit “incommunicado” interrogation of individuals in a police-dominated atmosphere, resulting in self-incriminating statements without full warnings of constitutional rights.[35]

In this case, as testified to by the lone witness for the defense, SPO2 Jerome Cause, no custodial investigation was conducted after SUSAN’s arrest.  She affixed her signature to the receipt of the articles seized from her, but before she did so, she was told that she had the option to sign or not to sign it.   In any event, her signature to the packages was not relied upon by the prosecution to prove its case.  Moreover, no statement was taken from her during her detention and used in evidence against her.[36] Hence, her claim of violation of her right to counsel has no leg to stand on.

VI.  The admission of the medical report was erroneous.

SUSAN assails, on the ground of violation of the hearsay rule, the admission of the medical report on the physical and medical examination conducted upon appellant’s request, which contained the following:

On subsequent examinations, she was seen behaved and cooperative.  She related that she was an illegitimate daughter, married, but divorced in 1995.  She verbalized, “I gamble like an addict.  I gambled since I was young and I lost control of myself when I played cards.  When I lost control, I want my money back.  I owe other people lots of money.  I lost all the cash of my husband.  This is the first time I carried shabu.  I need the money.”  She denied having any morbid thoughts and perceptual disturbances. (Emphasis supplied).

This argument is meritorious.  The admission of the questioned document was erroneous because it was not properly identified.  Nevertheless, even without the medical report, appellant’s conviction will stand, as the court’s finding of guilt was not based on that document.

VII. SUSAN’s conviction and the penalty imposed on her are correct.

Page 294: Warrantless Arrests, Searches and Seizures and Privacy of Communication

294

Having found the warrantless search and seizure conducted in this case to be valid, we do not hesitate to rule that that the three packages of shabu recovered from SUSAN are admissible in evidence against her.  Supported by this evidence and the testimonies of the prosecution witnesses, her conviction must inevitably be sustained.

Sections 16 and 20 of Article III of the Dangerous Drugs Act of 1972 (Republic Act No. 6425), as amended, provides:

SEC. 16.  Possession or Use of Regulated Drugs.--The penalty of reclusion perpetua to death and a fine ranging from five hundred thousand pesos to ten million pesos shall be imposed upon any person who shall possess or use any regulated drug without the corresponding license or prescription, subject to the provisions of Section 20 hereof.

SEC. 20.  Application of Penalties, confiscation and Forfeiture of the Proceeds or Instruments of the Crime.--The  penalties for offenses under Section 3,4,7, 8, and 9 of Article II and Sections 14, 14-A, 15 and 16 of Article III of this Act shall be applied if the dangerous drugs involved [are] in any of the following quantities:

3.  200 grams or more of shabu or methylamphetamine hydrochloride….

There being no aggravating nor mitigating circumstance, the proper penalty is reclusion perpetua pursuant to Article 63(2) of the Revised Penal Code.

As regards the fine, courts may fix any amount within the limits established by law.  For possession of regulated drugs, the law fixes the range of the fine fromP500,000 to P10 million.  In view of the net weight of methamphetamine hydrochloride found in the possession of SUSAN, the trial court’s imposition of fine in the amount of P1 million is well within the range prescribed by law.

VIII. The other items seized from the appellant should be returned to her.

Section 3 of Rule 126 of the Revised Rules of Criminal Procedure authorizes the confiscation of the following:

SEC. 3.  Personal property to be seized. – A search warrant may be issued for the search and seizure of personal property:

(a) Subject of the offense;

(b) Stolen or embezzled and other proceeds, or fruits of the offense; or

(c) Used or intended to be used as the means of committing an offense.

Clearly, the seizure of SUSAN’s passport, plane tickets, and girdles exceeded the limits of the afore-quoted provision.  They, therefore, have to be returned to her.[37]

IN VIEW OF ALL THE FOREGOING, the judgment of the Regional Trial Court of Pasay City, Branch 110, in Criminal Case No. 98-0189 finding appellant SUSAN CANTON guilty beyond reasonable doubt of the violation of Section 16, Article III of the Dangerous Act of 1972 (Republic Act No. 6425), as amended, and sentencing her to suffer the penalty of reclusion perpetua and to pay a fine of One Million Pesos (P1,000,000) and the costs is hereby AFFIRMED.  The appellant’s passport, plane tickets, and girdles are hereby ordered to be returned to her.

Page 295: Warrantless Arrests, Searches and Seizures and Privacy of Communication

295

Costs de oficio.

SO ORDERED.

Vitug, Ynares-Santiago, Carpio, and Azcuna, JJ., concur.

G.R. No. 96177 January 27, 1993

PEOPLE OF THE PHILIPPINES, plaintiff-appellee, vs.MARI MUSA y HANTATALU, accused-appellant.

The Solicitor General for plaintiff-appellee.

Pablo L. Murillo for accused-appellant.

 

ROMERO, J.:

The appellant, Mari Musa, seeks, in this appeal, the reversal of the decision, dated August 31, 1990,  1 of the Regional Trial Court (RTC) of Zamboanga City, Branch XII, finding him guilty of selling marijuana in violation of Article II, Section 4 of Republic Act No. 6425, as amended, otherwise known as the Dangerous Drugs Act of 1972.

The information filed on December 15, 1989 against the appellant reads:

That on or about December 14, 1989, in the City of Zamboanga, Philippines, and within the jurisdiction of this Honorable Court, theabove-named accused, not being authorized by law, did then and there, wilfully, unlawfully and feloniously sell to one SGT. AMADO ANI, two (2) wrappers containing dried marijuana leaves, knowing the same to be a prohibited drug.

CONTRARY TO LAW. 2

Upon his arraignment on January 11, 1990, the appellant pleaded not guilty. 3

At the trial, the prosecution presented three (3) witnesses, namely: (1) Sgt. Amado Ani, Jr. of the 9th Narcotics Command (NARCOM) of Zamboanga City, who acted as poseur-buyer in the buy-bust operation made against the appellant; (2) T/Sgt. Jesus Belarga, also of the 9th Narcotics Command of Zamboanga City, who was the NARCOM team leader of the buy-bust operation; and (3) Athena Elisa P. Anderson, the Document Examiner and Forensic Chemist of PC-INP Crime Laboratory of Regional Command (RECOM) 9. The evidence of the prosecution was summarized by the trial court as follows:

Prosecution evidence shows that in the morning of December 13, 1989, T/Sgt. Jesus Belarga, leader of a NARCOTICS COMMAND (NARCOM) team based at Calarian, Zamboanga City, instructed Sgt. Amado Ani to conduct surveillance and test buy on a

Page 296: Warrantless Arrests, Searches and Seizures and Privacy of Communication

296

certain Mari Musa of Suterville, Zamboanga City. Information received from civilian informer was that this Mari Musa was engaged in selling marijuana in said place. So Sgt. Amado Ani, another NARCOM agent, proceeded to Suterville, in company with a NARCOM civilian informer, to the house of Mari Musa to which house the civilian informer had guided him. The same civilian informer had also described to him the appearance of Mari Musa. Amado Ani was able to buy one newspaper-wrapped dried marijuana (Exh. "E") for P10.00. Sgt. Ani returned to the NARCOM office and turned over the newspaper-wrapped marijuana to T/Sgt. Jesus Belarga. Sgt. Belarga inspected the stuff turned over to him and found it to be marijuana.

The next day, December 14, 1989, about 1:30 P.M., a buy-bust was planned. Sgt. Amado Ani was assigned as the poseur buyer for which purpose he was given P20.00 (with SN GA955883) by Belarga. Thebuy-bust money had been taken by T/Sgt. Jesus Belarga from M/Sgt. Noh Sali Mihasun, Chief of Investigation Section, and for which Belarga signed a receipt (Exh. "L" & "L-l" ) The team under Sgt. Foncargas was assigned as back-up security. A pre-arranged signal was arranged consisting of Sgt. Ani's raising his right hand, after he had succeeded to buy the marijuana. The two NARCOM teams proceeded to the target site in two civilian vehicles. Belarga's team was composed of Sgt. Belarga, team leader, Sgt. Amado Ani, poseur buyer, Sgt. Lego and Sgt. Biong.

Arriving at the target site, Sgt. Ani proceeded to the house of Mari Musa, while the rest of the NARCOM group positioned themselves at strategic places about 90 to 100 meters from Mari Musa's house. T/Sgt. Belarga could see what went on between Ani and suspect Mari Musa from where he was. Ani approached Mari Musa, who came out of his house, and asked Ani what he wanted. Ani said he wanted some more stuff. Ani gave Mari Musa the P20.00 marked money. After receiving the money, Mari Musa went back to his house and came back and gave Amado Ani two newspaper wrappers containing dried marijuana. Ani opened the two wrappers and inspected the contents. Convinced that the contents were marijuana, Ani walked back towards his companions and raised his right hand. The two NARCOM teams, riding the two civilian vehicles, sped towards Sgt. Ani. Ani joined Belarga's team and returned to the house.

At the time Sgt. Ani first approached Mari Musa, there were four persons inside his house: Mari Musa, another boy, and two women, one of whom Ani and Belarga later came to know to be Mari Musa's wife. The second time, Ani with the NARCOM team returned to Mari Musa's house, the woman, who was later known as Mari Musa's wife, slipped away from the house. Sgt. Belarga frisked Mari Musa but could not find the P20.00 marked money with him. Mari Musa was then asked where the P20.00 was and he told the NARCOM team he has given the money to his wife (who had slipped away). Sgt. Belarga also found a plastic bag containing dried marijuana inside it somewhere in the kitchen. Mari Musa was then placed under arrest and brought to the NARCOM office. At Suterville, Sgt. Ani turned over to Sgt. Belarga the two newspaper-wrapped marijuana he had earlier bought from Mari Musa (Exhs. "C" & "D").

In the NARCOM office, Mari Musa first gave his name as Hussin Musa. Later on, Mari Musa gave his true name — Mari Musa. T/Sgt. Jesus Belarga turned over the two newspaper-wrapped marijuana (bought at the buy-bust), the one newspaper-wrapped marijuana (bought at the test-buy) and the plastic bag containing more marijuana (which had been taken by Sgt. Lego inside the kitchen of Mari Musa) to the PC Crime Laboratory, Zamboanga City, for laboratory examination. The turnover of the marijuana specimen to the PC Crime Laboratory was by way of a letter-request, dated December 14, 1989 (Exh. "B"), which was stamped "RECEIVED" by the PC Crime Laboratory (Exh. "B-1") on the same day.

Page 297: Warrantless Arrests, Searches and Seizures and Privacy of Communication

297

Mrs. Athena Elisa P. Anderson, the Forensic Chemist of the PC Crime Laboratory, examined the marijuana specimens subjecting the same to her three tests. All submitted specimens she examined gave positive results for the presence of marijuana. Mrs. Anderson reported the results of her examination in her Chemistry Report D-100-89, dated December 14, 1989, (Exh. "J", "J-1", "J-2", "J-3", "J-4" and "J-5"). Mrs. Anderson identified in court the two newspaper wrapped marijuana bought at thebuy-bust on December 14, 1989, through her initial and the weight of each specimen written with red ink on each wrapper (Exhs. "C-1" and "D-1"). She also identified the one newspaper-wrapped marijuana bought at the test-buy on December 13, 1989, through her markings (Exh. "E-1"). Mrs. Anderson also identified her Chemistry Report (Exh. "J" & sub-markings.)

T. Sgt. Belarga identified the two buy-bust newspaper wrapped marijuana through his initial, the words "buy-bust" and the words "December 14, 1989, 2:45 P.M." (written on Exhs. "C" and "D"). Belarga also identified the receipt of the P20 marked money (with SN GA955883) (Exh. "L"), dated December 14, 1989, and his signature thereon (Exh."L-1"). He also identified the letter-request, dated December 14, 1989, addressed to the PC Crime Laboratory (Exh. "B") and his signature thereon (Exh. "B-2") and the stamp of the PC Crime Laboratory marked "RECEIVED" (Exh. "B-1"). 4

For the defense, the following testified as witnesses: (1) the accused-appellant Mari H. Musa; and (2) Ahara R. Musa, his wife. The trial court summarized the version of the defense, thus:

[O]n December 14, 1989, at about 1:30 in the afternoon, Mari Musa was in his house at Suterville, Zamboanga City. With him were his wife, Ahara Musa, known as Ara, his one-year old child, a woman manicurist, and a male cousin named Abdul Musa. About 1:30 that afternoon, while he was being manicured at one hand, his wife was inside the one room of their house, putting their child to sleep. Three NARCOM agents, who introduced themselves as NARCOM agents, dressed in civilian clothes, got inside Mari Musa's house whose door was open. The NARCOM agents did not ask permission to enter the house but simply announced that they were NARCOM agents. The NARCOM agents searched Mari Musa's house and Mari Musa asked them if they had a search warrant. The NARCOM agents were just silent. The NARCOM agents found a red plastic bag whose contents, Mari Musa said, he did not know. He also did not know if the plastic bag belonged to his brother, Faisal, who was living with him, or his father, who was living in another house about ten arms-length away. Mari Musa, then, was handcuffed and when Mari Musa asked why, the NARCOM agents told him for clarification.

Mari Musa was brought in a pick-up, his wife joining him to the NARCOM Office at Calarian, Zamboanga City. Inside the NARCOM Office, Mari Musa was investigated by one NARCOM agent which investigation was reduced into writing. The writing or document was interpreted to Mari Musa in Tagalog. The document stated that the marijuana belonged to Mari Musa and Mari Musa was asked to sign it. But Mari Musa refused to sign because the marijuana did not belong to him. Mari Musa said he was not told that he was entitled to the assistance of counsel, although he himself told the NARCOM agents he wanted to be assisted by counsel.

Mari Musa said four bullets were then placed between the fingers of his right hand and his fingers were pressed which felt very painful. The NARCOM agents boxed him and Mari Musa lost consciousness. While Mari Musa was maltreated, he said his wife was outside the NARCOM building. The very day he was arrested (on cross-examination Mari Musa said it was on the next day), Mari Musa was brought to the Fiscal's Office by three NARCOM agents. The fiscal asked him if the marijuana was owned by him and he said "not." After that single question, Mari Musa was brought to the City Jail. Mari Musa said

Page 298: Warrantless Arrests, Searches and Seizures and Privacy of Communication

298

he did not tell the fiscal that he had been maltreated by the NARCOM agents because he was afraid he might be maltreated in the fiscal's office.

Mari Musa denied the NARCOM agents' charge that he had sold two wrappers of marijuana to them; that he had received from them a P20.00 bill which he had given to his wife. He did not sell marijuana because he was afraid that was against the law and that the person selling marijuana was caught by the authorities; and he had a wife and a very small child to support. Mari Musa said he had not been arrested for selling marijuana before. 5

After trial, the trial court rendered the assailed decision with the following disposition:

WHEREFORE, finding accused Mari Musa y Hantatalu guilty beyond reasonable doubt of selling marijuana and pursuant to Sec. 4, Art II of Rep. Act No. 6425, he is sentenced to life imprisonment and to pay the fine of P20,000.00, the latter imposed without subsidiary imprisonment. 6

In this appeal, the appellant contends that his guilt was not proved beyond reasonable doubt and impugns the credibility of the prosecution witnesses.

The appellant claims that the testimony of Sgt. Ani, the poseur-buyer, is not credible because: (1) prior to the buy-bust operation, neither Sgt. Ani nor the other NARCOM agents were personally known by the appellant or vice-versa; and (2) there was no witness to the alleged giving of the two wrappers of marijuana by the appellant to Sgt. Ani.

Sgt. Ani testified that on December 13, 1989, upon instruction by T/Sgt. Jesus Belarga, he conducted a test-buy operation on the appellant whereby he bought one wrapper of marijuana for P15.00 from the latter. 7 He reported the successful operation to T/Sgt. Belarga on the same day. 8 Whereupon, T/Sgt. Belarga conducted a conference to organize a buy-bust operation for the following day.  9

On December 14, 1989, at 1:30 p.m., two NARCOM teams in separate vehicles headed by T/Sgt. Belarga and a certain Sgt. Foncardas went to the place of operation, which was the appellant's house located in Laquian Compound, Suterville, Zamboanga City. Sgt. Ani was with the team of T/Sgt. Belarga, whose other members were Sgts. Lego and Biong. 10 Sgt. Ani was given a marked P20.00 bill by T/Sgt. Belarga, which was to be used in the operation.

Upon reaching the place, the NARCOM agents positioned themselves at strategic places.  11 Sgt. Ani approached the house. Outside the house, the appellant asked Sgt. Ani what he wanted. Sgt. Ani asked him for some more marijuana. 12 Sgt. Ani gave him the marked P20.00 bill and the appellant went inside the house and brought back two paper wrappers containing marijuana which he handed to Sgt. Ani. 13 From his position, Sgt. Ani could see that there were other people in the house. 14

After the exchange, Sgt. Ani approached the other NARCOM agents and made the pre-arranged signal of raising his right hand. 15 The NARCOM agents, accompanied by Sgt. Ani, went inside the house and made the arrest. The agents searched the appellant and unable to find the marked money, they asked him where it was. The appellant said that he gave it to his wife. 16

The Court, after a careful reading of the record, finds the testimony of Sgt. Ani regarding the buy-bust operation, which resulted in the apprehension, prosecution and subsequent conviction of the appellant, to be direct, lucid and forthright. Being totally untainted by contradictions in any of the material points, it deserves credence.

Page 299: Warrantless Arrests, Searches and Seizures and Privacy of Communication

299

The contention that the appellant could not have transacted with Sgt. Ani because they do not know each other is without merit. The day before thebuy-bust operation, Sgt. Ani conducted a test-buy and he successfully bought a wrapper of marijuana from the appellant. Through this previous transaction, Sgt. Ani was able to gain the appellant's confidence for the latter to sell more marijuana to Sgt. Ani the following day, during the buy-bust operation. Moreover, the Court has held that what matters is not an existing familiarity between the buyer and the seller, for quite often, the parties to the transaction may be strangers, but their agreement and the acts constituting the sale and delivery of the marijuana. 17

The appellant, again to cast doubt on the credibility of Sgt. Ani, argues that it was impossible for the appellant to sell marijuana while his wife, cousin and manicurist were present. But the place of the commission of the crime of selling prohibited drugs has been held to be not crucial 18 and the presence of other people apart from the buyer and seller will not necessarily prevent the consummation of the illegal sale. As the Court observed in People v. Paco, 19 these factors may sometimes camouflage the commission of the crime. In the instant case, the fact that the other people inside the appellant's house are known to the appellant may have given him some assurance that these people will not report him to the authorities.

The appellant, besides assailing Sgt. Ani's credibility, also questions the credibility of T/Sgt. Belarga. The appellant submits that since T/Sgt. Belarga admitted that he was about 90 meters away from Sgt. Ani and the appellant, he could not have possibly witnessed the sale. The appellant invokes People v.Ale 20 where the Court observed that from a distance of 10-15 meters, a policeman cannot distinguish between marijuana cigarette from ordinary ones by the type of rolling done on the cigarette sticks. And since T/Sgt. Belarga allegedly did not see the sale, the appellant contends that the uncorroborated testimony of Sgt. Ani can not stand as basis for his conviction.

People v. Ale does not apply here because the policeman in that case testified that he and his companion were certain that the appellant therein handed marijuana cigarettes to the poseur-buyer based on the appearance of the cigarette sticks. The Court rejected this claim, stating that:

This Court cannot give full credit to the testimonies of the prosecution witnesses marked as they are with contradictions and tainted with inaccuracies.

Biñan testified that they were able to tell that the four cigarettes were marijuana cigarettes because according to him, the rolling of ordinary cigarettes are different from those of marijuana cigarettes. (tsn, November 13, 1984, p. 10).

It is however, incredible to believe that they could discern the type of rolling done on those cigarettes from the distance where they were observing the alleged sale of more or less 10 to 15 meters. 21

In the case at bar, however, T/Sgt. Belarga did not positively claim that he saw the appellant hand over marijuana to Sgt. Ani. What he said was that there was an exchange of certain articles between the two. The relevant portion of T/Sgt. Belarga's testimony reads: 22

Q Now, do you remember whether Sgt. Ani was able to reach the house of Mari Musa?

A Yes, ma'am.

Q After reaching Mari Musa, did you see what happened (sic)?

A Yes, ma'am.

Page 300: Warrantless Arrests, Searches and Seizures and Privacy of Communication

300

Q Could you please tell us?

A From our vehicle the stainless owner type jeep where Sgt. Lego, Sgt. Biong were boarded, I saw that Sgt. Ani proceeded to the house near the road and he was met by one person and later known as Mari Musa who was at the time wearing short pants and later on I saw that Sgt. Ani handed something to him, thereafter received by Mari Musa and went inside the house and came back later and handed something to Sgt. Ani.

Contrary to the contention of the appellant, it was not impossible for T/Sgt. Belarga to have seen, from a distance of 90-100 meters, Sgt. Ani hand to the appellant "something" and for the latter to give to the former "something."

Notwithstanding the fact that T/Sgt. Belarga could not have been certain that what Sgt. Ani received from the appellant was marijuana because of the distance, his testimony, nevertheless, corroborated the direct evidence, which the Court earlier ruled to be convincing, presented by Sgt. Ani on the following material points: (1) T/Sgt. Belarga instructed Sgt. Ani to conduct a surveillance and test-buy operation on the appellant at Suterville, Zamboanga City on December 13, 1989; 23 (2) later that same day, Sgt. Ani went back to their office and reported a successful operation and turned over to T/Sgt. Belarga one wrapper of marijuana; 24 (3) T/Sgt. Belarga then organized a team to conduct a buy-bust operation the following day; 25 (4) on December 14, 1989, T/Sgt. Belarga led a team of NARCOM agents who went to Suterville, Zamboanga City; 26 (5) T/Sgt. Belarga gave a P20.00 marked bill to Sgt. Ani which was to be used in the buy-bust operation; 27 (6) upon the arrival of the NARCOM agents in Suterville, Zamboanga City, Sgt. Ani proceeded to the house of the appellant while some agents stayed in the vehicles and others positioned themselves in strategic places; 28 the appellant met Sgt. Ani and an exchange of articles took place. 29

The corroborative testimony of T/Sgt. Belarga strengthens the direct evidence given by Sgt. Ani. Additionally, the Court has ruled that the fact that the police officers who accompanied the poseur-buyer were unable to see exactly what the appellant gave the poseur-buyer because of their distance or position will not be fatal to the prosecution's case 30 provided there exists other evidence, direct or circumstantial, e.g., the testimony of the poseur-buyer, which is sufficient to prove the consummation of the sale of the prohibited drug

The appellant next assails the seizure and admission as evidence of a plastic bag containing marijuana which the NARCOM agents found in the appellant's kitchen. It appears that after Sgt. Ani gave the pre-arranged signal to the other NARCOM agents, the latter moved in and arrested the appellant inside the house. They searched him to retrieve the marked money but didn't find it. Upon being questioned, the appellant said that he gave the marked money to his wife. 31 Thereafter, T/Sgt. Belarga and Sgt. Lego went to the kitchen and noticed what T/Sgt. Belarga described as a "cellophane colored white and stripe hanging at the corner of the kitchen." 32 They asked the appellant about its contents but failing to get a response, they opened it and found dried marijuana leaves. At the trial, the appellant questioned the admissibility of the plastic bag and the marijuana it contains but the trial court issued an Order ruling that these are admissible in evidence. 33

Built into the Constitution are guarantees on the freedom of every individual against unreasonable searches and seizures by providing in Article III, Section 2, the following:

The right of the people to be secure in their persons, houses, papers, and effects against unreasonable searches and seizures of whatever nature and for any purpose shall be inviolable, and no search warrant or warrant of arrest shall issue except upon probable cause to be determined personally by the judge after examination under oath or affirmation of the complainant and the witness he may produce, and particularly describing the place to be searched and the persons or things to be seized.

Page 301: Warrantless Arrests, Searches and Seizures and Privacy of Communication

301

Furthermore, the Constitution, in conformity with the doctrine laid down in Stonehill v. Diokno, 34 declares inadmissible, any evidence obtained in violation of the freedom from unreasonable searches and seizures. 35

While a valid search warrant is generally necessary before a search and seizure may be effected, exceptions to this rule are recognized. Thus, in Alvero v. Dizon, 36 the Court stated that. "[t]he most important exception to the necessity for a search warrant is the right of search and seizure as an incident to a lawful arrest." 37

Rule 126, Section 12 of the Rules of Court expressly authorizes a warrantless search and seizure incident to a lawful arrest, thus:

Sec. 12. Search incident to lawful arrest. — A person lawfully arrested may be searched for dangerous weapons or anything which may be used as proof of the commission of an offense, without a search warrant.

There is no doubt that the warrantless search incidental to a lawful arrest authorizes the arresting officer to make a search upon the person of the person arrested. As early as 1909, the Court has ruled that "[a]n officer making an arrest may take from the person arrested any money or property found upon his person which was used in the commission of the crime or was the fruit of the crime or which might furnish the prisoner with the means of committingviolence or of escaping, or which may be used as evidence in the trial of the cause . . . "  38 Hence, in a buy-bust operation conducted to entrap a drug-pusher, the law enforcement agents may seize the marked money found on the personof the pusher immediately after the arrest even without arrest and search warrants. 39

In the case at bar, the NARCOM agents searched the person of the appellant after arresting him in his house but found nothing. They then searched the entire house and, in the kitchen, found and seized a plastic bag hanging in a corner.

The warrantless search and seizure, as an incident to a suspect's lawful arrest, may extend beyond the person of the one arrested to include the premises or surroundings under his immediate control. 40 Objects in the "plain view" of an officer who has the right to be in the position to have that view are subject to seizure and may be presented as evidence. 41

In Ker v. California 42 police officers, without securing a search warrant but having information that the defendant husband was selling marijuana from his apartment, obtained from the building manager a passkey to defendants' apartment, and entered it. There they found the defendant husband in the living room. The defendant wife emerged from the kitchen, and one of the officers, after identifying himself, observed through the open doorway of the kitchen, a small scale atop the kitchen sink, upon which lay a brick-shaped package containing green leafy substance which he recognized as marijuana. The package of marijuana was used as evidence in prosecuting defendants for violation of the Narcotic Law. The admissibility of the package was challenged before the U.S. Supreme Court, which held, after observing that it was not unreasonable for the officer to walk to the doorway of the adjacent kitchen on seeing the defendant wife emerge therefrom, that "the discovery of the brick of marijuana did not constitute a search, since the officer merely saw what was placed before him in full view. 43 The U.S. Supreme Court ruled that the warrantless seizure of the marijuana was legal on the basis of the "plain view" doctrine and upheld the admissibility of the seized drugs as part of the prosecution's evidence. 44

The "plain view" doctrine may not, however, be used to launch unbridled searches and indiscriminate seizures nor to extend a general exploratory search made solely to find evidence of defendant's guilt. The "plain view" doctrine is usually applied where a police officer is not searching for evidence against the accused, but nonetheless inadvertently comes across an incriminating object. 45 Furthermore, the U.S. Supreme Court stated the following limitations on the application of the doctrine:

Page 302: Warrantless Arrests, Searches and Seizures and Privacy of Communication

302

What the "plain view" cases have in common is that the police officer in each of them had a prior justification for an intrusion in the course of which he came inadvertently across a piece of evidence incriminating the accused. The doctrine serves to supplement the prior justification — whether it be a warrant for another object, hot pursuit, search incident to lawful arrest, or some other legitimate reason for being present unconnected with a search directed against the accused — and permits the warrantless seizure. Of course, the extension of the original justification is legitimate only where it is immediately apparent to the police that they have evidence before them; the "plain view" doctrine may not be used to extend a general exploratory search from one object to another until something incriminating at last emerges. 46

It has also been suggested that even if an object is observed in "plain view," the "plain view" doctrine will not justify the seizure of the object where the incriminating nature of the object is not apparent from the "plain view" of the object. 47 Stated differently, it must be immediately apparent to the police that the items that they observe may be evidence of a crime, contraband, or otherwise subject to seizure.

In the instant case, the appellant was arrested and his person searched in the living room. Failing to retrieve the marked money which they hoped to find, the NARCOM agents searched the whole house and found the plastic bag in the kitchen. The plastic bag was, therefore, not within their "plain view" when they arrested the appellant as to justify its seizure. The NARCOM agents had to move from one portion of the house to another before they sighted the plastic bag. Unlike Ker vs. California, where the police officer had reason to walk to the doorway of the adjacent kitchen and from which position he saw the marijuana, the NARCOM agents in this case went from room to room with the obvious intention of fishing for more evidence.

Moreover, when the NARCOM agents saw the plastic bag hanging in one corner of the kitchen, they had no clue as to its contents. They had to ask the appellant what the bag contained. When the appellant refused to respond, they opened it and found the marijuana. Unlike Ker v. California, where the marijuana was visible to the police officer's eyes, the NARCOM agents in this case could not have discovered the inculpatory nature of the contents of the bag had they not forcibly opened it. Even assuming then, that the NARCOM agents inadvertently came across the plastic bag because it was within their "plain view," what may be said to be the object in their "plain view" was just the plastic bag and not the marijuana. The incriminating nature of the contents of the plastic bag was not immediately apparent from the "plain view" of said object. It cannot be claimed that the plastic bag clearly betrayed its contents, whether by its distinctive configuration, its transprarency, or otherwise, that its contents are obvious to an observer.  48

We, therefore, hold that under the circumstances of the case, the "plain view" doctrine does not apply and the marijuana contained in the plastic bag was seized illegally and cannot be presented in evidence pursuant to Article III, Section 3(2) of the Constitution.

The exclusion of this particular evidence does not, however, diminish, in any way, the damaging effect of the other pieces of evidence presented by the prosecution to prove that the appellant sold marijuana, in violation of Article II, Section 4 of the Dangerous Drugs Act of 1972. We hold that by virtue of the testimonies of Sgt. Ani and T/Sgt. Belarga and the two wrappings of marijuana sold by the appellant to Sgt. Ani, among other pieces of evidence, the guilt of the appellant of the crime charged has been proved beyond reasonable doubt.

WHEREFORE, the appeal is DISMISSED and the judgment of the Regional Trial Court AFFIRMED.

SO ORDERED.

Gutierrez, Jr., Bidin, Davide, Jr. and Melo, JJ., concur.

Page 303: Warrantless Arrests, Searches and Seizures and Privacy of Communication

303

[G.R. Nos. 138539-40.  January 21, 2003]

PEOPLE OF THE PHILIPPINES, appellee, vs. ANTONIO C. ESTELLA, appellant.

D E C I S I O N

PANGANIBAN, J.:

The Constitution bars the admission of evidence gathered in violation of the right against unreasonable search and seizure.  In the present case, the illegal drug was searched for and found in a hut that has not been proven to be owned, controlled, or used by appellant for residential or any other purpose.  Hence, he cannot be held guilty of illegal possession of the illegal drug found therein.

The Case

Antonio C. Estella appeals the August 25, 1998 Decision [1] of the Regional Trial Court (RTC) of Iba, Zambales (Branch 69) in Criminal Case No. RTC 2143-I.  The trial court found him guilty of violating Section 8, Article II of RA 6425, as amended by RA 7659, and sentenced him to reclusion perpetua as follows:

“WHEREFORE, foregoing considered, in Criminal Case No. RTC 2143-I, accused Antonio C. Estella is found GUILTY beyond reasonable doubt for Violation of Section 8, Article II of R.A. 6425 as amended by R.A. 7659 and is sentenced to suffer the penalty of reclusion perpetua.

Page 304: Warrantless Arrests, Searches and Seizures and Privacy of Communication

304

“The 8.320 kilograms of dried marijuana is ordered confiscated in favor of the government.  The Sheriff is directed to deliver the subject marijuana to the Dangerous Drugs Board for its proper disposition.

“In Criminal Case No. RTC 2144-I, accused Antonio C. Estella is ACQUITTED and the Information dated 07 January 1997 filed against him for violation of P.D. 1866 is dismissed with costs de oficio.

“The .38 caliber revolver without serial number and four (4) live ammunitions, subject of the offense, are ordered delivered to any authorized representative of the Philippine National Police, Firearms and Explosives Division, Camp Crame, Quezon City.”[2]

The Information dated January 7, 1997, charged appellant thus:

“That on or about the 20th day of November, 1996 at about 11:15 o’clock in the morning, at Purok Yakal, Barangay Baloganon, in the Municipality of Masinloc, Province of Zambales, Philippines, and within the jurisdiction of this Honorable Court, said accused, did then and there, wil[l]fully, unlawfully and feloniously have in his possession, custody and control, [o]ne (1) tin can labeled ‘CLASSIC’ containing twenty (20) small bricks of dried marijuana fruiting tops having a total weight of 589.270 grams each wrapped with a piece of reading material; [o]ne (1) tin can labeled ‘CLASSIC’ containing dried marijuana fruiting tops weighing 41.126 grams; [t]wo (2) white sando plastic bag each containing one (1) [brick] of dried marijuana fruiting tops having a total weight of 1.710 kilograms each wrapped with a piece of newspaper; [o]ne (1) white sando plastic bag containing two (2) bricks of dried marijuana fruiting tops having a total weight of 1.820 kilograms each wrapped with a piece of newspaper, all in the total of 8.320 kilograms of dried marijuana, without any authority to possess the same.”[3]

After the Information had been read to him in Filipino, a language he fully understood, [4] appellant, assisted by his counsel de parte,[5] pleaded not guilty when arraigned on March 11, 1997.  After due trial, the RTC convicted appellant of illegal possession of dangerous drugs (marijuana), but acquitted him of illegal possession of firearms.  On November 4, 1998, his counsel filed a Notice of Appeal.[6]

The Facts

Version of the Prosecution

In its Brief,[7] the Office of the Solicitor General (OSG) presents the prosecution’s version of the facts as follows:

“Prior to November 20, 1996, Executive Judge Romulo Estrada of the Regional Trial Court of Zambales issued a warrant for the conduct of a search and seizure in the residence of appellant at Purok Yakal, Barangay Baloganon, Masinloc, Zambales.

“In the morning of November 20, 1996, Senior Police Officer 1 (SPO1) Antonio Bulor[o]n, then Intelligence and Investigation Officer, together with SPO1 Jose Arca and several other members of the Provincial Special Operation Group based in Burgos, San Marcelino, Zambales proceeded to Masinloc.  They coordinated with the members of the Philippine National Police (PNP) in Masinloc and sought the assistance of Barangay Captain Rey Barnachea of Baloganon, Masinloc for the enforcement of the search warrant.  Barangay Captain Barnaceha accompanied the police officers to Purok Yakal, Barangay Baloganon, Masinloc, the place mentioned in the search warrant.

“On their way to Purok Yakal, SPO1 Buloron saw appellant sitting on a rocking chair located about two (2) meters away from a hut owned by Narding Estella, brother of appellant, and being rented by appellant’s

Page 305: Warrantless Arrests, Searches and Seizures and Privacy of Communication

305

live-in partner, named Eva.  They approached appellant and introduced themselves as police officers.  They showed appellant the search warrant and explained the contents to him.  SPO1 Buloron asked appellant if indeed he had in his possession prohibited drug and if so, to surrender the same so he would deserve a lesser penalty.

“While inside the hut, appellant surrendered to the team two cans containing dried marijuana fruiting tops.  One can contained twenty (20) bricks of fruiting tops.  The team searched the hut in the presence of appellant and his live-in partner.  They found a plastic container under the kitchen table, which contained four (4) big bricks of dried marijuana leaves and a .38 caliber revolver with four live ammunitions.  The team seized the prohibited drug, the revolver and ammunitions.  The team seized and signed a receipt for the seized items. Barangay Captain Barnachea and SPO1 Edgar Bermudez of the Masinloc Police Station also signed the receipt as witnesses.  SPO1 Buloron and his companions arrested appellant and brought him to San Marcelino, Zambales.

“At their office in San Marcelino, Zambales, SPO1 Buloron and SPO1 Arca placed their markings on the seized items for purposes of identification.  SPO1 Arca kept the seized items under his custody.  The next day, SPO1 Buloron and SPO1 Arca brought the seized items to San Antonio, Zambales, where Police Senior Inspector Florencio Sahagun examined the suspected marijuana dried leaves.  Inspector Sahagun prepared a certification of field test.

“On November 29, 1996, the suspected marijuana dried leaves were delivered to the PNP Crime Laboratory at Camp Olivas for further examination.  Senior Inspector Daisy Babor, a forensic chemist, examined the suspected marijuana dried leaves and issued Chemistry Report No. D-768-96 stating that the specimens are positive for marijuana, a prohibited drug. Specimen A weighed 1.710 kilograms, while Specimen D weighed 1.820 kilograms.”[8] (Citations omitted)

Version of the Defense

For his version of the facts, appellant merely reproduced the narration in the assailed RTC Decision as follows:

“Accused Antonio C. Estella [I]s married to Gloria Atrero Estella.  They have three (3) children, namely: Carmen Estella (8 years old), Antonio Estella, Jr. (5 years old) and Roen Estella (3 years old).  Since 1982, Antonio Estella has been [a] resident of Barangay Baloganon, Masinloc, Zambales.

“On 20 November 1996 between 10:30 o’clock and 11:00 o’clock in the morning, while accused was talking with his friends Rael Tapado and Victor de Leon at a vacant lot just outside the house of Camillo Torres and about 70 meters away from his house, a group of men approached them.  The group introduced themselves as policemen and told them that they were looking for Antonio Estella because they have a search warrant issued against him.  Accused identified himself to them.  The policemen inquired from the accused as to where his house is located and accused told them that his house is located across the road.  The police did not believe him and insisted that accused’s house (according to their asset) is that house located about 5-8 meters away from them.  Accused told the policemen to inquire from the Barangay Captain Barnachea as to where his house is and heard the latter telling the policemen that his house is located near the Abokabar junk shop.  After about half an hour, the policemen went inside the house nearby and when they came out, they had with them a bulk of plastic and had it shown to the accused.  They photographed the accused and brought him to their office at San Marcelino, Zambales.  Accused Antonio Estella was investigated a[t] San Marcelino, Zambales where he informed the police officers of the fact that the house they searched was occupied by Spouses Vicente and Fely Bakdangan.

Page 306: Warrantless Arrests, Searches and Seizures and Privacy of Communication

306

“Accused denied having surrendered to policeman Buloron tin cans containing marijuana and likewise having any firearm.

“Miguel Buccat, who personally knew the accused for about ten (10) years, identified the house depicted on a photograph as that house belonging to the accused.”[9] (Citations omitted)

Ruling of the Trial Court

In finding appellant guilty of violating the Dangerous Drugs Act, the court a quo relied heavily on the testimony of the prosecution’s principal witness, Intelligence and Investigation Officer SPO1 Antonio Buloron.  He was among the members of the police team that searched appellant’s alleged house.  Since the defense failed to present proof of any intent on the part of SPO1 Buloron to falsely impute to appellant such a serious crime, the trial court accorded full faith and credence to the police officer’s testimony.

Moreover, the RTC held that no less than the barangay captain of the place named in the search warrant led the police to the house.  Thus, appellant could not deny that he owned it.

As to the charge of illegal possession of firearms, the lower court ruled that the search warrant did not cover the seized firearm, making it inadmissible against appellant.  He was thus acquitted of the charge.

Hence, this recourse.[10]

The Issues

In his appeal, appellant assigns the following alleged errors for our consideration:

“A.     The trial court erred in convicting the accused based on the conjectural and conflicting testimonies of the prosecution witnesses;

“B.     The trial court gravely failed to consider the serious contradictions in the facts and evidences adduced by the prosecution;

“C.     The trial court gravely erred in finding that the guilt of the accused-appellant for the crime charged has been prove[n] beyond reasonable doubt, instead of judgment of acquittal demanded by the constitutional presumption of innocence[.]”[11]

Though not clearly articulated by appellant, the pivotal issue here is the legality of the police search undertaken in the hut where the subject marijuana was seized.

The Court’s Ruling

The appeal is meritorious.

Main Issue:

Page 307: Warrantless Arrests, Searches and Seizures and Privacy of Communication

307

Legality of the Search Undertaken

Once again, this Court is confronted with a situation that involves a well-enshrined dogma in our Constitution: the inviolable right of the people to be secure in their persons and properties against unreasonable searches and seizures.[12]  The exclusionary rule prescribed by Section 3(2), Article III of the Constitution, bars the admission of evidence obtained in violation of this right.[13]

The conviction or the acquittal of appellant hinges primarily on the validity of the police officers’ search and seizure, as well as the admissibility of the evidence obtained by virtue thereof.  Without that evidence, the prosecution would not be able to prove his guilt beyond reasonable doubt.

Ownership of the Subject House

Appellant claims that the hut,[14] which was searched by the police and where the subject marijuana was recovered, does not belong to him.  He points to another house[15] as his real residence.  To support his claim, he presents a document[16] that shows that the subject hut was sold to his brother Leonardo C. Estella by one Odilon Eclarinal.  The OSG, on the other hand, argues that just because “appellant has another house in a place away from the hut that was searched does not necessarily mean that the hut is not occupied by him or under his full control.” [17] The prosecution cites the testimony of Rey Barnachea, the barangay captain of that place, to show that the hut in question belongs to appellant.

The only link that can be made between appellant and the subject hut is that it was bought by his brother Leonardo a.k.a. “Narding” Estella.[18] We cannot sustain the OSG’s supposition that since it was being rented by the alleged live-in partner of appellant, it follows that he was also occupying it or was in full control of it.  In the first place, other than SPO1 Buloron’s uncorroborated testimony, no other evidence was presented by the prosecution to prove that the person renting the hut was indeed the live-in partner of appellant -- if he indeed had any.  Moreover, the testimony of Barnachea serves to undermine, not advance, the position of the prosecution.  We quote from his testimony:

“Q   Do you know who is the owner of that house?

A     What I know is that Narding Estella bought that house, sir.

Q    Who is that Narding Estella?

A     The brother of Tony Estella, sir.

Q    And you know that that has been rent[ed] to people?

A     Yes, sir.

Q    Now, so far how many people [rented]  that place or that house?

A     I do not have any information about that[,] sir.

Q    Why did you know that that place was rented?

A     Because when I asked Eva she replied that they [were] only renting that house, sir.

Q    How long has Eva been renting that house?

A     I do not have any information about that[,] sir.

Q    Do you know who was living with Eva?

A     No, sir.

Q    So, what you know is that Eva lives alone in that house?

Page 308: Warrantless Arrests, Searches and Seizures and Privacy of Communication

308

A     Yes, sir.

Q    And you do not know anybody who is renting that house?

A     I have no information, sir.

Q    And you do not know if the accused was renting [it] or not?

A     I don’t have any information, sir.”[19]

At most, the testimony shows that the subject hut was bought by Narding Estella and rented by someone named Eva.  The attempt to make it appear that appellant occupied it, or that it was under his full control, is merely conjectural and speculative.  We have often ruled that courts do not rely on evidence that arouses mere suspicion or conjecture.[20] To lead to conviction, evidence must do more than raise the mere possibility or even probability of guilt.[21] It must engender moral certainty.

Neither do we find merit in the OSG’s argument that appellant cannot deny ownership or control of the hut, since he was found in front of it, sitting on a rocking chair and drinking coffee. [22] Indeed, to uphold this proposition would be to stretch our imagination to the extreme.

The OSG maintains that when appellant was “shown the search warrant and asked about the existence of prohibited drug in his possession, appellant went inside the hut, took his stock of marijuana and turned it [over] to the police officers.” [23] This, according to the prosecution, clearly showed that he was not only occupying the hut, but was in fact using it to store the prohibited drug.[24]

It is well-settled that this Court is not precluded from assessing the probative value of witnesses’ testimonies on the basis of the transcript of stenographic notes (TSNs).[25]

In the case at bar, we believe that the trial court erred in adopting the prosecution’s dubious story.  It failed to see patent inconsistencies in the prosecution witnesses’ testimonies about the search undertaken.

A review of the TSNs shows that SPO1 Buloron, the prosecution's principal witness, testified that appellant had allegedly gone inside the hut; and that the latter had done so to get his stock of illegal drugs, which he turned over to the police.  Ironically, Captain Barnachea, who was purposely presented by the prosecution to corroborate SPO1 Buloron's story, belied it when he testified thus:

“PROS. QUINTILLAN:

Q    When the police officer showed that search warrant what did Antonio Estella said, if any, if you hear[d]?

A     What I saw is that Tony Estella is sitting in the rocking chair outside the house drinking coffee, sir.

Q    And you saw him and then the search warrant was presented, isn’t it?

A     Yes, sir.

Q    And when it was presented what did Tony Estella do?

A     What they did they show to Tony the search warrant and I also read the contents of the search warrant, sir.

Q    And when Tony was shown that search warrant what did he do immediately after being shown that search warrant?

A     He just [sat] and then he stood up, sir.

Q    And when he stood up what else did he do?

A     Nothing, sir.  The NARCOM g[o]t inside the house, sir.

Q    And where did Antonio Estella go when the police entered the house?

Page 309: Warrantless Arrests, Searches and Seizures and Privacy of Communication

309

A     He was just outside the house, sir.

Q    And how far is that house from Antonio Estella?

INTERPRETER:

Witness estimating the distance of about five (5) meters.

COURT:

Do the prosecution and defense agree to 5 meters?

BOTH COUNSEL:

Yes, Your Honor.

PROS. QUINTILLAN:

Q    And when the police entered the house did not Tony go with them?

A     I did not notice, sir.”[26]

It is undisputed that even before arriving at the hut, the police officers were already being assisted by Barangay Captain Barnachea.  Thus, it was highly improbable for him not to see personally appellant’s alleged voluntary surrender of the prohibited drug to the authorities.  And yet, his testimony completely contradicted the policemen’s version of the events.  He testified that appellant, after being served the search warrant, remained outside the hut and did nothing.  In fact, the former categorically stated that when the police officers had gone inside the hut to conduct the search, appellant remained seated on a rocking chair outside.[27] Barnachea’s statements sow doubts as to the veracity of SPO1 Buloron’s claim that, after being apprised of the contents of the search warrant, appellant voluntarily surrendered the prohibited drug to the police.[28]

Apart from the testimony of Barnachea -- which contradicted rather than validated the story of SPO1 Buloron -- no other evidence was presented to corroborate the latter’s narration of the events.  Without any independent or corroborative proof, it has little or no probative value at all.

In a criminal prosecution, the court is always guided by evidence that is tangible, verifiable, and in harmony with the usual course of human experience -- not by mere conjecture or speculation. [29] While the guilty should not escape, the innocent should not suffer.[30]

Search Incident to Lawful Arrest

The OSG argues that “[e]ven assuming that appellant was not the occupant of the hut, the fact remains that he voluntarily surrendered the marijuana to the police officers.   After appellant had surrendered the prohibited stuff, the police had a right to arrest him even without a warrant and to conduct a search of the immediate vicinity of the arrestee for weapons and other unlawful objects as an incident to the lawful arrest.”[31]

The above argument assumes that the prosecution was able to prove that appellant had voluntarily surrendered the marijuana to the police officers.  As earlier adverted to, there is no convincing proof that he indeed surrendered the prohibited drug, whether voluntarily or otherwise.  In fact, the testimony of Prosecution Witness Barnachea clouds rather than clarifies the prosecution’s story.

Given this backdrop, the police authorities cannot claim that the search was incident to a lawful arrest.  Such a search presupposes a lawful or valid arrest and can only be invoked through Section 5, Rule 113 of the Revised Rules on Criminal Procedure, which we quote:

“SEC. 5. Arrest without warrant; when lawful - A peace officer or a private person may, without a warrant, arrest a person:

Page 310: Warrantless Arrests, Searches and Seizures and Privacy of Communication

310

“(a) When, in his presence, the person to be arrested has committed, is actually committing, or is attempting to commit an offense;

“(b)    When an offense has just been committed and he has probable cause to believe based on personal knowledge of facts or circumstances that the person to be arrested has committed it; and

“(c)    When the person to be arrested is a prisoner who has escaped from a penal establishment or place where he is serving final judgment or is temporarily confined while his case is pending, or has escaped while being transferred from one confinement to another.

“In cases falling under paragraphs (a) and (b) above, the person arrested without a warrant shall be forthwith delivered to the nearest police station or jail and shall be proceeded against in accordance with Section 7 Rule 112.”

Never was it proven that appellant, who was the person to be arrested, was in possession of the subject prohibited drug during the search.  It follows, therefore, that there was no way of knowing if he had committed or was actually committing an offense in the presence of the arresting officers.   Without that knowledge, there could have been no search incident to a lawful arrest.

Assuming arguendo that appellant was indeed committing an offense in the presence of the arresting officers, and that the arrest without a warrant was lawful, it still cannot be said that the search conducted was within the confines of the law.  Searches and seizures incident to lawful arrests are governed by Section 12, Rule 126 of the Revised Rules of Criminal Procedure, which reads:

“Section 12.   Search incident to lawful arrest. – A person lawfully arrested may be searched for dangerous weapons or anything which may have been used or constitute proof in the commission of an offense without a search warrant.”

However, the scope of the search should be limited to the area within which the person to be arrested can reach for a weapon or for evidence that he or she can destroy. [32]  The prevailing rule is that the arresting officer may take from the arrested individual any money or property found upon the latter’s person -- that which was used in the commission of the crime or was the fruit of the crime, or which may provide the prisoner with the means of committing violence or escaping, or which may be used in evidence in the trial of the case.[33]

In the leading case Chimel v. California,[34] the Supreme Court of the United States of America laid down this rule:

“When an arrest is made, it is reasonable for the arresting officer to search the person arrested in order to remove any weapons that the latter might seek to use in order to resist arrest or effect his escape.  Otherwise, the officer’s safety might well be endangered, and the arrest itself frustrated.  In addition, it is entirely reasonable for the arresting officer to search for and seize any evidence on the arrestee’s person in order to prevent its concealment or destruction.  And the area into which an arrestee might reach in order to grab a weapon or evidentiary items must, of course, be governed by a like rule.  A gun on a table or in a drawer in front of one who is arrested can be as dangerous to the arresting officer as one concealed in the clothing of the person arrested. There is ample justification, therefore, for a search of the arrestee’s person and the area ‘within his immediate control’ – construing that phrase to mean the area from within which he might gain possession of a weapon or destructible evidence.

“There is no comparable justification, however, for routinely searching any room other than that in which an arrest occurs – or, for that matter, for searching through all the desk drawers or other closed or concealed areas in that room itself.”[35]

The purpose of the exception is to protect the arresting officer from being harmed by the person being arrested, who might be armed with a concealed weapon, and to prevent the latter from destroying

Page 311: Warrantless Arrests, Searches and Seizures and Privacy of Communication

311

evidence within reach.  The exception, therefore, should not be strained beyond what is needed to serve its purpose.[36]

In the case before us, searched was the entire hut, which cannot be said to have been within appellant’s immediate control.  Thus, the search exceeded the bounds of that which may be considered to be incident to a lawful arrest.

The Presence of the Accused or the

Witnesses During the Search

Having ruled that the prosecution failed to prove appellant’s ownership, control of or residence in the subject hut, we hold that the presence of appellant or of witnesses during the search now becomes moot and academic.

Obviously, appellant need not have been present during the search if he was neither the owner nor the lawful occupant of the premises in question.  Besides, as we have noted, the testimonies of the prosecution witnesses regarding these crucial circumstances were contradictory.  They erode SPO1 Buloron’s credibility as a prosecution witness and raise serious doubts concerning the prosecution’s evidence.  This Court is thus constrained to view his testimony with caution and care.

With the failure of the prosecution to establish the propriety of the search undertaken -- during which the incriminating evidence was allegedly recovered -- we hold that the search was illegal.  Without the badge of legality, any evidence obtained therein becomes ipso facto inadmissible.

Objections to the

Legality of the Search

Finally, the OSG argues that appellant is deemed to have waived his right to object to the legality of the search and the admissibility of the evidence seized through that search because, during the trial, he did not raise these issues.

On the contrary, during the trial, appellant constantly questioned the legality of the search.  In fact, when SPO1 Buloron was presented as a prosecution witness, the former’s counsel objected to the offer of the latter’s testimony on items allegedly confiscated during the search.  Appellant’s counsel argued that these items, which consisted of the marijuana and the firearm, had been seized illegally and were therefore inadmissible.[37]

Further, in his Comments and Objections to Formal Offer of Exhibits,[38] appellant once again questioned the legality of the search conducted by the police, a search that had yielded the evidence being used against him.

Finally, on October 21, 1997, he filed a Demurrer to Evidence [39] reiterating his objection to the search and to the eventual use against him of the evidence procured therefrom.

All told, without sufficient admissible evidence against appellant, the prosecution failed to establish his guilt with moral certainty.[40]  Not only did its evidence fall short of the quantum of proof required for a conviction, it has also failed to present any evidence at all.  Under our Bill of Rights, among the fundamental rights of the accused is to be presumed innocent until the contrary is proved. [41]  To overcome such presumption, the prosecution must establish guilt beyond reasonable doubt. Our criminal

Page 312: Warrantless Arrests, Searches and Seizures and Privacy of Communication

312

justice system dictates that if the prosecution fails to do so, it becomes not only the right of the accused to be set free, but also the constitutional duty of the court to set them free. [42]  This principle leaves this Court no option but to acquit Appellant Antonio C. Estella for insufficiency of evidence.

WHEREFORE, the appealed Decision is SET ASIDE.  Antonio C. Estella is ACQUITTED and ordered immediately RELEASED from custody, unless he is being held for some other lawful cause.

The director of the Bureau of Corrections is ORDERED to implement this Decision forthwith and to INFORM this Court, within five (5) days from receipt hereof, of the date appellant was actually released from confinement.  Costs de oficio.

SO ORDERED.

Puno, (chairman), Sandoval-Gutierrez, Corona and Carpio-Morales, JJ., concur.

G.R. No. 81561 January 18, 1991

PEOPLE OF THE PHILIPPINES, plaintiff-appelleevs.ANDRE MARTI, accused-appellant.

The Solicitor General for plaintiff-appellee.

Reynaldo B. Tatoy and Abelardo E. Rogacion for accused-appellant.

 

BIDIN, J.:p

This is an appeal from a decision * rendered by the Special Criminal Court of Manila (Regional Trial Court, Branch XLIX) convicting accused-appellant of violation of Section 21 (b), Article IV in relation to Section 4, Article 11 and Section 2 (e) (i), Article 1 of Republic Act 6425, as amended, otherwise known as the Dangerous Drugs Act.

The facts as summarized in the brief of the prosecution are as follows:

On August 14, 1987, between 10:00 and 11:00 a.m., the appellant and his common-law wife, Shirley Reyes, went to the booth of the "Manila Packing and Export Forwarders" in the Pistang Pilipino Complex, Ermita, Manila, carrying with them four (4) gift wrapped packages. Anita Reyes (the proprietress and no relation to Shirley Reyes) attended to them. The appellant informed Anita Reyes that he was sending the packages to a friend in Zurich, Switzerland. Appellant filled up the contract necessary for the transaction, writing therein his name, passport number, the date of shipment and the name and address of the consignee, namely, "WALTER FIERZ, Mattacketr II, 8052 Zurich, Switzerland" (Decision, p. 6)

Anita Reyes then asked the appellant if she could examine and inspect the packages. Appellant, however, refused, assuring her that the packages simply contained books, cigars, and gloves and were gifts to his friend in Zurich. In view of appellant's

Page 313: Warrantless Arrests, Searches and Seizures and Privacy of Communication

313

representation, Anita Reyes no longer insisted on inspecting the packages. The four (4) packages were then placed inside a brown corrugated box one by two feet in size (1' x 2'). Styro-foam was placed at the bottom and on top of the packages before the box was sealed with masking tape, thus making the box ready for shipment (Decision, p. 8).

Before delivery of appellant's box to the Bureau of Customs and/or Bureau of Posts, Mr. Job Reyes (proprietor) and husband of Anita (Reyes), following standard operating procedure, opened the boxes for final inspection. When he opened appellant's box, a peculiar odor emitted therefrom. His curiousity aroused, he squeezed one of the bundles allegedly containing gloves and felt dried leaves inside. Opening one of the bundles, he pulled out a cellophane wrapper protruding from the opening of one of the gloves. He made an opening on one of the cellophane wrappers and took several grams of the contents thereof (tsn, pp. 29-30, October 6, 1987; Emphasis supplied).

Job Reyes forthwith prepared a letter reporting the shipment to the NBI and requesting a laboratory examination of the samples he extracted from the cellophane wrapper (tsn, pp. 5-6, October 6, 1987).

He brought the letter and a sample of appellant's shipment to the Narcotics Section of the National Bureau of Investigation (NBI), at about 1:30 o'clock in the afternoon of that date, i.e., August 14, 1987. He was interviewed by the Chief of Narcotics Section. Job Reyes informed the NBI that the rest of the shipment was still in his office. Therefore, Job Reyes and three (3) NBI agents, and a photographer, went to the Reyes' office at Ermita, Manila (tsn, p. 30, October 6, 1987).

Job Reyes brought out the box in which appellant's packages were placed and, in the presence of the NBI agents, opened the top flaps, removed the styro-foam and took out the cellophane wrappers from inside the gloves. Dried marijuana leaves were found to have been contained inside the cellophane wrappers (tsn, p. 38, October 6, 1987; Emphasis supplied).

The package which allegedly contained books was likewise opened by Job Reyes. He discovered that the package contained bricks or cake-like dried marijuana leaves. The package which allegedly contained tabacalera cigars was also opened. It turned out that dried marijuana leaves were neatly stocked underneath the cigars (tsn, p. 39, October 6, 1987).

The NBI agents made an inventory and took charge of the box and of the contents thereof, after signing a "Receipt" acknowledging custody of the said effects (tsn, pp. 2-3, October 7, 1987).

Thereupon, the NBI agents tried to locate appellant but to no avail. Appellant's stated address in his passport being the Manila Central Post Office, the agents requested assistance from the latter's Chief Security. On August 27, 1987, appellant, while claiming his mail at the Central Post Office, was invited by the NBI to shed light on the attempted shipment of the seized dried leaves. On the same day the Narcotics Section of the NBI submitted the dried leaves to the Forensic Chemistry Section for laboratory examination. It turned out that the dried leaves were marijuana flowering tops as certified by the forensic chemist. (Appellee's Brief, pp. 9-11, Rollo, pp. 132-134).

Thereafter, an Information was filed against appellant for violation of RA 6425, otherwise known as the Dangerous Drugs Act.

After trial, the court a quo rendered the assailed decision.

Page 314: Warrantless Arrests, Searches and Seizures and Privacy of Communication

314

In this appeal, accused/appellant assigns the following errors, to wit:

THE LOWER COURT ERRED IN ADMITTING IN EVIDENCE THE ILLEGALLY SEARCHED AND SEIZED OBJECTS CONTAINED IN THE FOUR PARCELS.

THE LOWER COURT ERRED IN CONVICTING APPELLANT DESPITE THE UNDISPUTED FACT THAT HIS RIGHTS UNDER THE CONSTITUTION WHILE UNDER CUSTODIAL PROCEEDINGS WERE NOT OBSERVED.

THE LOWER COURT ERRED IN NOT GIVING CREDENCE TO THE EXPLANATION OF THE APPELLANT ON HOW THE FOUR PARCELS CAME INTO HIS POSSESSION (Appellant's Brief, p. 1; Rollo, p. 55)

1. Appellant contends that the evidence subject of the imputed offense had been obtained in violation of his constitutional rights against unreasonable search and seizure and privacy of communication (Sec. 2 and 3, Art. III, Constitution) and therefore argues that the same should be held inadmissible in evidence (Sec. 3 (2), Art. III).

Sections 2 and 3, Article III of the Constitution provide:

Sec. 2. The right of the people to be secure in their persons, houses, papers and effects against unreasonable searches and seizures of whatever nature and for any purpose shall be inviolable, and no search warrant or warrant of arrest shall issue except upon probable cause to be determined personally by the judge after examination under oath or affirmation of the complainant and the witnesses he may produce, and particularly describing the place to be searched and the persons or things to be seized.

Sec. 3. (1) The privacy of communication and correspondence shall be inviolable except upon lawful order of the court, or when public safety or order requires otherwise as prescribed by law.

(2) Any evidence obtained in violation of this or the preceding section shall be inadmissible for any purpose in any proceeding.

Our present constitutional provision on the guarantee against unreasonable search and seizure had its origin in the 1935 Charter which, worded as follows:

The right of the people to be secure in their persons, houses, papers and effects against unreasonable searches and seizures shall not be violated, and no warrants shall issue but upon probable cause, to be determined by the judge after examination under oath or affirmation of the complainant and the witnesses he may produce, and particularly describing the place to be searched, and the persons or things to be seized. (Sec. 1 [3], Article III)

was in turn derived almost verbatim from the Fourth Amendment ** to the United States Constitution. As such, the Court may turn to the pronouncements of the United States Federal Supreme Court and State Appellate Courts which are considered doctrinal in this jurisdiction.

Thus, following the exclusionary rule laid down in Mapp v. Ohio by the US Federal Supreme Court (367 US 643, 81 S.Ct. 1684, 6 L.Ed. 1081 [1961]), this Court, in Stonehill v. Diokno (20 SCRA 383 [1967]), declared as inadmissible any evidence obtained by virtue of a defective search and seizure warrant, abandoning in the process the ruling earlier adopted in Moncado v. People's Court (80 Phil. 1 [1948]) wherein the admissibility of evidence was not affected by the illegality of its seizure. The 1973 Charter

Page 315: Warrantless Arrests, Searches and Seizures and Privacy of Communication

315

(Sec. 4 [2], Art. IV) constitutionalized the Stonehill ruling and is carried over up to the present with the advent of the 1987 Constitution.

In a number of cases, the Court strictly adhered to the exclusionary rule and has struck down the admissibility of evidence obtained in violation of the constitutional safeguard against unreasonable searches and seizures. (Bache & Co., (Phil.), Inc., v. Ruiz, 37 SCRA 823 [1971]; Lim v. Ponce de Leon, 66 SCRA 299 [1975]; People v. Burgos, 144 SCRA 1 [1986]; Roan v. Gonzales, 145 SCRA 687 [1987]; See also Salazar v. Hon. Achacoso, et al., GR No. 81510, March 14, 1990).

It must be noted, however, that in all those cases adverted to, the evidence so obtained were invariably procured by the State acting through the medium of its law enforcers or other authorized government agencies.

On the other hand, the case at bar assumes a peculiar character since the evidence sought to be excluded was primarily discovered and obtained by a private person, acting in a private capacity and without the intervention and participation of State authorities. Under the circumstances, can accused/appellant validly claim that his constitutional right against unreasonable searches and seizure has been violated? Stated otherwise, may an act of a private individual, allegedly in violation of appellant's constitutional rights, be invoked against the State?

We hold in the negative. In the absence of governmental interference, the liberties guaranteed by the Constitution cannot be invoked against the State.

As this Court held in Villanueva v. Querubin (48 SCRA 345 [1972]:

1. This constitutional right (against unreasonable search and seizure) refers to the immunity of one's person, whether citizen or alien, from interference by government, included in which is his residence, his papers, and other possessions. . . .

. . . There the state, however powerful, does not as such have the access except under the circumstances above noted, for in the traditional formulation, his house, however humble, is his castle. Thus is outlawed any unwarranted intrusion by government, which is called upon to refrain from any invasion of his dwelling and to respect the privacies of his life. . . . (Cf. Schermerber v. California, 384 US 757 [1966] and Boyd v. United States, 116 US 616 [1886]; Emphasis supplied).

In Burdeau v. McDowell (256 US 465 (1921), 41 S Ct. 547; 65 L.Ed. 1048), the Court there in construing the right against unreasonable searches and seizures declared that:

(t)he Fourth Amendment gives protection against unlawful searches and seizures, and as shown in previous cases, its protection applies to governmental action. Its origin and history clearly show that it was intended as a restraint upon the activities of sovereign authority, and was not intended to be a limitation upon other than governmental agencies; as against such authority it was the purpose of the Fourth Amendment to secure the citizen in the right of unmolested occupation of his dwelling and the possession of his property, subject to the right of seizure by process duly served.

The above ruling was reiterated in State v. Bryan (457 P.2d 661 [1968]) where a parking attendant who searched the automobile to ascertain the owner thereof found marijuana instead, without the knowledge and participation of police authorities, was declared admissible in prosecution for illegal possession of narcotics.

Page 316: Warrantless Arrests, Searches and Seizures and Privacy of Communication

316

And again in the 1969 case of Walker v. State (429 S.W.2d 121), it was held that the search and seizure clauses are restraints upon the government and its agents, not upon private individuals (citing People v. Potter, 240 Cal. App.2d 621, 49 Cap. Rptr, 892 (1966); State v. Brown, Mo., 391 S.W.2d 903 (1965); State v. Olsen, Or., 317 P.2d 938 (1957).

Likewise appropos is the case of Bernas v. US (373 F.2d 517 (1967). The Court there said:

The search of which appellant complains, however, was made by a private citizen — the owner of a motel in which appellant stayed overnight and in which he left behind a travel case containing the evidence*** complained of. The search was made on the motel owner's own initiative. Because of it, he became suspicious, called the local police, informed them of the bag's contents, and made it available to the authorities.

The fourth amendment and the case law applying it do not require exclusion of evidence obtained through a search by a private citizen. Rather, the amendment only proscribes governmental action."

The contraband in the case at bar having come into possession of the Government without the latter transgressing appellant's rights against unreasonable search and seizure, the Court sees no cogent reason why the same should not be admitted against him in the prosecution of the offense charged.

Appellant, however, would like this court to believe that NBI agents made an illegal search and seizure of the evidence later on used in prosecuting the case which resulted in his conviction.

The postulate advanced by accused/appellant needs to be clarified in two days. In both instances, the argument stands to fall on its own weight, or the lack of it.

First, the factual considerations of the case at bar readily foreclose the proposition that NBI agents conducted an illegal search and seizure of the prohibited merchandise. Records of the case clearly indicate that it was Mr. Job Reyes, the proprietor of the forwarding agency, who made search/inspection of the packages. Said inspection was reasonable and a standard operating procedure on the part of Mr. Reyes as a precautionary measure before delivery of packages to the Bureau of Customs or the Bureau of Posts (TSN, October 6 & 7, 1987, pp. 15-18; pp. 7-8; Original Records, pp. 119-122; 167-168).

It will be recalled that after Reyes opened the box containing the illicit cargo, he took samples of the same to the NBI and later summoned the agents to his place of business. Thereafter, he opened the parcel containing the rest of the shipment and entrusted the care and custody thereof to the NBI agents. Clearly, the NBI agents made no search and seizure, much less an illegal one, contrary to the postulate of accused/appellant.

Second, the mere presence of the NBI agents did not convert the reasonable search effected by Reyes into a warrantless search and seizure proscribed by the Constitution. Merely to observe and look at that which is in plain sight is not a search. Having observed that which is open, where no trespass has been committed in aid thereof, is not search (Chadwick v. State, 429 SW2d 135). Where the contraband articles are identified without a trespass on the part of the arresting officer, there is not the search that is prohibited by the constitution (US v. Lee 274 US 559, 71 L.Ed. 1202 [1927]; Ker v. State of California 374 US 23, 10 L.Ed.2d. 726 [1963]; Moore v. State, 429 SW2d 122 [1968]).

In Gandy v. Watkins (237 F. Supp. 266 [1964]), it was likewise held that where the property was taken into custody of the police at the specific request of the manager and where the search was initially made by the owner there is no unreasonable search and seizure within the constitutional meaning of the term.

Page 317: Warrantless Arrests, Searches and Seizures and Privacy of Communication

317

That the Bill of Rights embodied in the Constitution is not meant to be invoked against acts of private individuals finds support in the deliberations of the Constitutional Commission. True, the liberties guaranteed by the fundamental law of the land must always be subject to protection. But protection against whom? Commissioner Bernas in his sponsorship speech in the Bill of Rights answers the query which he himself posed, as follows:

First, the general reflections. The protection of fundamental liberties in the essence of constitutional democracy. Protection against whom? Protection against the state. The Bill of Rights governs the relationship between the individual and the state. Its concern is not the relation between individuals, between a private individual and other individuals. What the Bill of Rights does is to declare some forbidden zones in the private sphere inaccessible to any power holder. (Sponsorship Speech of Commissioner Bernas , Record of the Constitutional Commission, Vol. 1, p. 674; July 17, 1986; Emphasis supplied)

The constitutional proscription against unlawful searches and seizures therefore applies as a restraint directed only against the government and its agencies tasked with the enforcement of the law. Thus, it could only be invoked against the State to whom the restraint against arbitrary and unreasonable exercise of power is imposed.

If the search is made upon the request of law enforcers, a warrant must generally be first secured if it is to pass the test of constitutionality. However, if the search is made at the behest or initiative of the proprietor of a private establishment for its own and private purposes, as in the case at bar, and without the intervention of police authorities, the right against unreasonable search and seizure cannot be invoked for only the act of private individual, not the law enforcers, is involved. In sum, the protection against unreasonable searches and seizures cannot be extended to acts committed by private individuals so as to bring it within the ambit of alleged unlawful intrusion by the government.

Appellant argues, however, that since the provisions of the 1935 Constitution has been modified by the present phraseology found in the 1987 Charter, expressly declaring as inadmissible any evidence obtained in violation of the constitutional prohibition against illegal search and seizure, it matters not whether the evidence was procured by police authorities or private individuals (Appellant's Brief, p. 8, Rollo, p. 62).

The argument is untenable. For one thing, the constitution, in laying down the principles of the government and fundamental liberties of the people, does not govern relationships between individuals. Moreover, it must be emphasized that the modifications introduced in the 1987 Constitution (re: Sec. 2, Art. III) relate to the issuance of either a search warrant or warrant of arrest vis-a-vis the responsibility of the judge in the issuance thereof (SeeSoliven v. Makasiar, 167 SCRA 393 [1988]; Circular No. 13 [October 1, 1985] and Circular No. 12 [June 30, 1987]. The modifications introduced deviate in no manner as to whom the restriction or inhibition against unreasonable search and seizure is directed against. The restraint stayed with the State and did not shift to anyone else.

Corolarilly, alleged violations against unreasonable search and seizure may only be invoked against the State by an individual unjustly traduced by the exercise of sovereign authority. To agree with appellant that an act of a private individual in violation of the Bill of Rights should also be construed as an act of the State would result in serious legal complications and an absurd interpretation of the constitution.

Similarly, the admissibility of the evidence procured by an individual effected through private seizure equally applies, in pari passu, to the alleged violation, non-governmental as it is, of appellant's constitutional rights to privacy and communication.

Page 318: Warrantless Arrests, Searches and Seizures and Privacy of Communication

318

2. In his second assignment of error, appellant contends that the lower court erred in convicting him despite the undisputed fact that his rights under the constitution while under custodial investigation were not observed.

Again, the contention is without merit, We have carefully examined the records of the case and found nothing to indicate, as an "undisputed fact", that appellant was not informed of his constitutional rights or that he gave statements without the assistance of counsel. The law enforcers testified that accused/appellant was informed of his constitutional rights. It is presumed that they have regularly performed their duties (See. 5(m), Rule 131) and their testimonies should be given full faith and credence, there being no evidence to the contrary. What is clear from the records, on the other hand, is that appellant refused to give any written statement while under investigation as testified by Atty. Lastimoso of the NBI, Thus:

Fiscal Formoso:

You said that you investigated Mr. and Mrs. Job Reyes. What about the accused here, did you investigate the accused together with the girl?

WITNESS:

Yes, we have interviewed the accused together with the girl but the accused availed of his constitutional right not to give any written statement, sir. (TSN, October 8, 1987, p. 62; Original Records, p. 240)

The above testimony of the witness for the prosecution was not contradicted by the defense on cross-examination. As borne out by the records, neither was there any proof by the defense that appellant gave uncounselled confession while being investigated. What is more, we have examined the assailed judgment of the trial court and nowhere is there any reference made to the testimony of appellant while under custodial investigation which was utilized in the finding of conviction. Appellant's second assignment of error is therefore misplaced.

3. Coming now to appellant's third assignment of error, appellant would like us to believe that he was not the owner of the packages which contained prohibited drugs but rather a certain Michael, a German national, whom appellant met in a pub along Ermita, Manila: that in the course of their 30-minute conversation, Michael requested him to ship the packages and gave him P2,000.00 for the cost of the shipment since the German national was about to leave the country the next day (October 15, 1987, TSN, pp. 2-10).

Rather than give the appearance of veracity, we find appellant's disclaimer as incredulous, self-serving and contrary to human experience. It can easily be fabricated. An acquaintance with a complete stranger struck in half an hour could not have pushed a man to entrust the shipment of four (4) parcels and shell out P2,000.00 for the purpose and for appellant to readily accede to comply with the undertaking without first ascertaining its contents. As stated by the trial court, "(a) person would not simply entrust contraband and of considerable value at that as the marijuana flowering tops, and the cash amount of P2,000.00 to a complete stranger like the Accused. The Accused, on the other hand, would not simply accept such undertaking to take custody of the packages and ship the same from a complete stranger on his mere say-so" (Decision, p. 19, Rollo, p. 91). As to why he readily agreed to do the errand, appellant failed to explain. Denials, if unsubstantiated by clear and convincing evidence, are negative self-serving evidence which deserve no weight in law and cannot be given greater evidentiary weight than the testimony of credible witnesses who testify on affirmative matters (People v. Esquillo, 171 SCRA 571 [1989]; People vs. Sariol, 174 SCRA 237 [1989]).

Appellant's bare denial is even made more suspect considering that, as per records of the Interpol, he was previously convicted of possession of hashish by the Kleve Court in the Federal Republic of

Page 319: Warrantless Arrests, Searches and Seizures and Privacy of Communication

319

Germany on January 1, 1982 and that the consignee of the frustrated shipment, Walter Fierz, also a Swiss national, was likewise convicted for drug abuse and is just about an hour's drive from appellant's residence in Zurich, Switzerland (TSN, October 8, 1987, p. 66; Original Records, p. 244; Decision, p. 21; Rollo, p. 93).

Evidence to be believed, must not only proceed from the mouth of a credible witness, but it must be credible in itself such as the common experience and observation of mankind can approve as probable under the circumstances (People v. Alto, 26 SCRA 342 [1968], citing Daggers v. Van Dyke, 37 N.J. Eg. 130; see alsoPeople v. Sarda, 172 SCRA 651 [1989]; People v. Sunga, 123 SCRA 327 [1983]); Castañares v. CA, 92 SCRA 567 [1979]). As records further show, appellant did not even bother to ask Michael's full name, his complete address or passport number. Furthermore, if indeed, the German national was the owner of the merchandise, appellant should have so indicated in the contract of shipment (Exh. "B", Original Records, p. 40). On the contrary, appellant signed the contract as the owner and shipper thereof giving more weight to the presumption that things which a person possesses, or exercises acts of ownership over, are owned by him (Sec. 5 [j], Rule 131). At this point, appellant is therefore estopped to claim otherwise.

Premises considered, we see no error committed by the trial court in rendering the assailed judgment.

WHEREFORE, the judgment of conviction finding appellant guilty beyond reasonable doubt of the crime charged is hereby AFFIRMED. No costs.

SO ORDERED.

Fernan, C.J., Gutierrez, Jr. and Feliciano, JJ., concur.

G.R. No. 93833 September 28, 1995

SOCORRO D. RAMIREZ, petitioner, vs.HONORABLE COURT OF APPEALS, and ESTER S. GARCIA, respondents.

 

KAPUNAN, J.:

A civil case damages was filed by petitioner Socorro D. Ramirez in the Regional Trial Court of Quezon City alleging that the private respondent, Ester S. Garcia, in a confrontation in the latter's office, allegedly vexed, insulted and humiliated her in a "hostile and furious mood" and in a manner offensive to petitioner's dignity and personality," contrary to morals, good customs and public policy." 1

In support of her claim, petitioner produced a verbatim transcript of the event and sought moral damages, attorney's fees and other expenses of litigation in the amount of P610,000.00, in addition to costs, interests and other reliefs awardable at the trial court's discretion. The transcript on which the civil case was based was culled from a tape recording of the confrontation made by petitioner. 2 The transcript reads as follows:

Plaintiff Soccoro D. Ramirez (Chuchi) — Good Afternoon M'am.

Defendant Ester S. Garcia (ESG) — Ano ba ang nangyari sa 'yo, nakalimot ka na kung paano ka napunta rito, porke member ka na, magsumbong ka kung ano ang gagawin ko sa 'yo.

Page 320: Warrantless Arrests, Searches and Seizures and Privacy of Communication

320

CHUCHI — Kasi, naka duty ako noon.

ESG — Tapos iniwan no. (Sic)

CHUCHI — Hindi m'am, pero ilan beses na nila akong binalikan, sabing ganoon —

ESG — Ito and (sic) masasabi ko sa 'yo, ayaw kung (sic) mag explain ka, kasi hanggang 10:00 p.m., kinabukasan hindi ka na pumasok. Ngayon ako ang babalik sa 'yo, nag-aaply ka sa States, nag-aaply ka sa review mo, kung kakailanganin ang certification mo, kalimutan mo na kasi hindi ka sa akin makakahingi.

CHUCHI — Hindi M'am. Kasi ang ano ko talaga noon i-cocontinue ko up to 10:00 p.m.

ESG — Bastos ka, nakalimutan mo na kung paano ka pumasok dito sa hotel. Magsumbong ka sa Union kung gusto mo. Nakalimutan mo na kung paano ka nakapasok dito "Do you think that on your own makakapasok ka kung hindi ako. Panunumbyoyan na kita (Sinusumbatan na kita).

CHUCHI — Itutuloy ko na M'am sana ang duty ko.

ESG — Kaso ilang beses na akong binabalikan doon ng mga no (sic) ko.

ESG — Nakalimutan mo na ba kung paano ka pumasok sa hotel, kung on your own merit alam ko naman kung gaano ka "ka bobo" mo. Marami ang nag-aaply alam kong hindi ka papasa.

CHUCHI — Kumuha kami ng exam noon.

ESG — Oo, pero hindi ka papasa.

CHUCHI — Eh, bakit ako ang nakuha ni Dr. Tamayo

ESG — Kukunin ka kasi ako.

CHUCHI — Eh, di sana —

ESG — Huwag mong ipagmalaki na may utak ka kasi wala kang utak. Akala mo ba makukuha ka dito kung hindi ako.

CHUCHI — Mag-eexplain ako.

ESG — Huwag na, hindi ako mag-papa-explain sa 'yo, makaalala ka kung paano ka puma-rito. "Putang-ina" sasabi-sabihin mo kamag-anak ng nanay at tatay mo ang mga magulang ko.

ESG — Wala na akong pakialam, dahil nandito ka sa loob, nasa labas ka puwede ka ng hindi pumasok, okey yan nasaloob ka umalis ka doon.

Page 321: Warrantless Arrests, Searches and Seizures and Privacy of Communication

321

CHUCHI — Kasi M'am, binbalikan ako ng mga taga Union.

ESG — Nandiyan na rin ako, pero huwag mong kalimutan na hindi ka makakapasok kung hindi ako. Kung hindi mo kinikilala yan okey lang sa akin, dahil tapos ka na.

CHUCHI — Ina-ano ko m'am na utang na loob.

ESG — Huwag na lang, hindi mo utang na loob, kasi kung baga sa no, nilapastangan mo ako.

CHUCHI — Paano kita nilapastanganan?

ESG — Mabuti pa lumabas ka na. Hindi na ako makikipagusap sa 'yo. Lumabas ka na. Magsumbong ka. 3

As a result of petitioner's recording of the event and alleging that the said act of secretly taping the confrontation was illegal, private respondent filed a criminal case before the Regional Trial Court of Pasay City for violation of Republic Act 4200, entitled "An Act to prohibit and penalize wire tapping and other related violations of private communication, and other purposes." An information charging petitioner of violation of the said Act, dated October 6, 1988 is quoted herewith:

INFORMATION

The Undersigned Assistant City Fiscal Accusses Socorro D. Ramirez of Violation of Republic Act No. 4200, committed as follows:

That on or about the 22nd day of February, 1988, in Pasay City Metro Manila, Philippines, and within the jurisdiction of this honorable court, the above-named accused, Socorro D. Ramirez not being authorized by Ester S. Garcia to record the latter's conversation with said accused, did then and there willfully, unlawfully and feloniously, with the use of a tape recorder secretly record the said conversation and thereafter communicate in writing the contents of the said recording to other person.

Contrary to law.

Pasay City, Metro Manila, September 16, 1988.

MARIANO M. CUNETAAsst. City Fiscal

Upon arraignment, in lieu of a plea, petitioner filed a Motion to Quash the Information on the ground that the facts charged do not constitute an offense, particularly a violation of R.A. 4200. In an order May 3, 1989, the trial court granted the Motion to Quash, agreeing with petitioner that 1) the facts charged do not constitute an offense under R.A. 4200; and that 2) the violation punished by R.A. 4200 refers to a the taping of a communication by a person other than a participant to the communication. 4

From the trial court's Order, the private respondent filed a Petition for Review on Certiorari with this Court, which forthwith referred the case to the Court of Appeals in a Resolution (by the First Division) of June 19, 1989.

Page 322: Warrantless Arrests, Searches and Seizures and Privacy of Communication

322

On February 9, 1990, respondent Court of Appeals promulgated its assailed Decision declaring the trial court's order of May 3, 1989 null and void, and holding that:

[T]he allegations sufficiently constitute an offense punishable under Section 1 of R.A. 4200. In thus quashing the information based on the ground that the facts alleged do not constitute an offense, the respondent judge acted in grave abuse of discretion correctible by certiorari. 5

Consequently, on February 21, 1990, petitioner filed a Motion for Reconsideration which respondent Court of Appeals denied in its Resolution 6 dated June 19, 1990. Hence, the instant petition.

Petitioner vigorously argues, as her "main and principal issue" 7 that the applicable provision of Republic Act 4200 does not apply to the taping of a private conversation by one of the parties to the conversation. She contends that the provision merely refers to the unauthorized taping of a private conversation by a party other than those involved in the communication. 8 In relation to this, petitioner avers that the substance or content of the conversation must be alleged in the Information, otherwise the facts charged would not constitute a violation of R.A. 4200. 9 Finally, petitioner agues that R.A. 4200 penalizes the taping of a "private communication," not a "private conversation" and that consequently, her act of secretly taping her conversation with private respondent was not illegal under the said act. 10

We disagree.

First, legislative intent is determined principally from the language of a statute. Where the language of a statute is clear and unambiguous, the law is applied according to its express terms, and interpretation would be resorted to only where a literal interpretation would be either impossible 11 or absurb or would lead to an injustice. 12

Section 1 of R.A. 4200 entitled, " An Act to Prohibit and Penalized Wire Tapping and Other Related Violations of Private Communication and Other Purposes," provides:

Sec. 1. It shall be unlawfull for any person, not being authorized by all the parties to any private communication or spoken word, to tap any wire or cable, or by using any other device or arrangement, to secretly overhear, intercept, or record such communication or spoken word by using a device commonly known as a dictaphone or dictagraph or detectaphone or walkie-talkie or tape recorder, or however otherwise described.

The aforestated provision clearly and unequivocally makes it illegal for any person, not authorized by all the parties to any private communication to secretly record such communication by means of a tape recorder. The law makes no distinction as to whether the party sought to be penalized by the statute ought to be a party other than or different from those involved in the private communication. The statute's intent to penalize all persons unauthorized to make such recording is underscored by the use of the qualifier "any". Consequently, as respondent Court of Appeals correctly concluded, "even a (person) privy to a communication who records his private conversation with another without the knowledge of the latter (will) qualify as a violator" 13 under this provision of R.A. 4200.

A perusal of the Senate Congressional Records, moreover, supports the respondent court's conclusion that in enacting R.A. 4200 our lawmakers indeed contemplated to make illegal, unauthorized tape recording of private conversations or communications taken either by the parties themselves or by third persons. Thus:

xxx xxx xxx

Senator Tañada: That qualified only "overhear".

Page 323: Warrantless Arrests, Searches and Seizures and Privacy of Communication

323

Senator Padilla: So that when it is intercepted or recorded, the element of secrecy would not appear to be material. Now, suppose, Your Honor, the recording is not made by all the parties but by some parties and involved not criminal cases that would be mentioned under section 3 but would cover, for example civil cases or special proceedings whereby a recording is made not necessarily by all the parties but perhaps by some in an effort to show the intent of the parties because the actuation of the parties prior, simultaneous even subsequent to the contract or the act may be indicative of their intention. Suppose there is such a recording, would you say, Your Honor, that the intention is to cover it within the purview of this bill or outside?

Senator Tañada: That is covered by the purview of this bill, Your Honor.

Senator Padilla: Even if the record should be used not in the prosecution of offense but as evidence to be used in Civil Cases or special proceedings?

Senator Tañada: That is right. This is a complete ban on tape recorded conversations taken without the authorization of all the parties.

Senator Padilla: Now, would that be reasonable, your Honor?

Senator Tañada: I believe it is reasonable because it is not sporting to record the observation of one without his knowing it and then using it against him. It is not fair, it is not sportsmanlike. If the purpose; Your honor, is to record the intention of the parties. I believe that all the parties should know that the observations are being recorded.

Senator Padilla: This might reduce the utility of recorders.

Senator Tañada: Well no. For example, I was to say that in meetings of the board of directors where a tape recording is taken, there is no objection to this if all the parties know. It is but fair that the people whose remarks and observations are being made should know that the observations are being recorded.

Senator Padilla: Now, I can understand.

Senator Tañada: That is why when we take statements of persons, we say: "Please be informed that whatever you say here may be used against you." That is fairness and that is what we demand. Now, in spite of that warning, he makes damaging statements against his own interest, well, he cannot complain any more. But if you are going to take a recording of the observations and remarks of a person without him knowing that it is being taped or recorded, without him knowing that what is being recorded may be used against him, I think it is unfair.

xxx xxx xxx

(Congression Record, Vol. III, No. 31, p. 584, March 12, 1964)

Senator Diokno: Do you understand, Mr. Senator, that under Section 1 of the bill as now worded, if a party secretly records a public speech, he would be penalized under Section 1? Because the speech is public, but the recording is done secretly.

Senator Tañada: Well, that particular aspect is not contemplated by the bill. It is the communication between one person and another person — not between a speaker and a public.

Page 324: Warrantless Arrests, Searches and Seizures and Privacy of Communication

324

xxx xxx xxx

(Congressional Record, Vol. III, No. 33, p. 626, March 12, 1964)

xxx xxx xxx

The unambiguity of the express words of the provision, taken together with the above-quoted deliberations from the Congressional Record, therefore plainly supports the view held by the respondent court that the provision seeks to penalize even those privy to the private communications. Where the law makes no distinctions, one does not distinguish.

Second, the nature of the conversations is immaterial to a violation of the statute. The substance of the same need not be specifically alleged in the information. What R.A. 4200 penalizes are the acts of secretly overhearing, intercepting or recording private communications by means of the devices enumerated therein. The mere allegation that an individual made a secret recording of a private communication by means of a tape recorder would suffice to constitute an offense under Section 1 of R.A. 4200. As the Solicitor General pointed out in his COMMENT before the respondent court: "Nowhere (in the said law) is it required that before one can be regarded as a violator, the nature of the conversation, as well as its communication to a third person should be professed." 14

Finally, petitioner's contention that the phrase "private communication" in Section 1 of R.A. 4200 does not include "private conversations" narrows the ordinary meaning of the word "communication" to a point of absurdity. The word communicate comes from the latin word communicare, meaning "to share or to impart." In its ordinary signification, communication connotes the act of sharing or imparting signification, communication connotes the act of sharing or imparting, as in a conversation, 15 or signifies the "process by which meanings or thoughts are shared between individuals through a common system of symbols (as language signs or gestures)" 16 These definitions are broad enough to include verbal or non-verbal, written or expressive communications of "meanings or thoughts" which are likely to include the emotionally-charged exchange, on February 22, 1988, between petitioner and private respondent, in the privacy of the latter's office. Any doubts about the legislative body's meaning of the phrase "private communication" are, furthermore, put to rest by the fact that the terms "conversation" and "communication" were interchangeably used by Senator Tañada in his Explanatory Note to the bill quoted below:

It has been said that innocent people have nothing to fear from their conversations being overheard. But this statement ignores the usual nature of conversations as well the undeniable fact that most, if not all, civilized people have some aspects of their lives they do not wish to expose. Free conversations are often characterized by exaggerations, obscenity, agreeable falsehoods, and the expression of anti-social desires of views not intended to be taken seriously. The right to the privacy of communication, among others, has expressly been assured by our Constitution. Needless to state here, the framers of our Constitution must have recognized the nature ofconversations between individuals and the significance of man's spiritual nature, of his feelings and of his intellect. They must have known that part of the pleasures and satisfactions of life are to be found in the unaudited, and free exchange of communication between individuals — free from every unjustifiable intrusion by whatever means. 17

In Gaanan vs. Intermediate Appellate Court, 18 a case which dealt with the issue of telephone wiretapping, we held that the use of a telephone extension for the purpose of overhearing a private conversation without authorization did not violate R.A. 4200 because a telephone extension devise was neither among those "device(s) or arrangement(s)" enumerated therein, 19 following the principle that "penal statutes must be construed strictly in favor of the accused." 20 The instant case turns on a different note, because the applicable facts and circumstances pointing to a violation of R.A. 4200 suffer from no ambiguity, and

Page 325: Warrantless Arrests, Searches and Seizures and Privacy of Communication

325

the statute itself explicitly mentions the unauthorized "recording" of private communications with the use of tape-recorders as among the acts punishable.

WHEREFORE, because the law, as applied to the case at bench is clear and unambiguous and leaves us with no discretion, the instant petition is hereby DENIED. The decision appealed from is AFFIRMED. Costs against petitioner.

SO ORDERED.

Padilla, Davide, Jr. and Bellosillo JJ., concur.

Hermosisima, Jr., J., is on leave.

G.R. No. L-69809 October 16, 1986

EDGARDO A. GAANAN, petitioner, vs.INTERMEDIATE APPELLATE COURT and PEOPLE OF THE PHILIPPINES, respondents.

 

GUTIERREZ, JR., J.:

This petition for certiorari asks for an interpretation of Republic Act (RA) No. 4200, otherwise known as the Anti-Wiretapping Act, on the issue of whether or not an extension telephone is among the prohibited devices in Section 1 of the Act, such that its use to overhear a private conversation would constitute unlawful interception of communications between the two parties using a telephone line.

The facts presented by the People and narrated in the respondent court's decision are not disputed by the petitioner.

Page 326: Warrantless Arrests, Searches and Seizures and Privacy of Communication

326

In the morning of October 22, 1975, complainant Atty. Tito Pintor and his client Manuel Montebon were in the living room of complainant's residence discussing the terms for the withdrawal of the complaint for direct assault which they filed with the Office of the City Fiscal of Cebu against Leonardo Laconico. After they had decided on the proposed conditions, complainant made a telephone call to Laconico (tsn, August 26, 1981, pp. 3-5).

That same morning, Laconico telephoned appellant, who is a lawyer, to come to his office and advise him on the settlement of the direct assault case because his regular lawyer, Atty. Leon Gonzaga, went on a business trip. According to the request, appellant went to the office of Laconico where he was briefed about the problem. (Exhibit 'D', tsn, April 22, 1982, pp. 4-5).

When complainant called up, Laconico requested appellant to secretly listen to the telephone conversation through a telephone extension so as to hear personally the proposed conditions for the settlement. Appellant heard complainant enumerate the following conditions for withdrawal of the complaint for direct assault.

(a) the P5,000.00 was no longer acceptable, and that the figure had been increased to P8,000.00. A breakdown of the P8,000.00 had been made together with other demands, to wit: (a) P5,000.00 no longer for the teacher Manuel Montebon, but for Atty. Pintor himself in persuading his client to withdraw the case for Direct Assault against Atty. Laconico before the Cebu City Fiscal's Office;

(b) Public apology to be made by Atty. Laconico before the students of Don Bosco Technical High School;

(c) Pl,000.00 to be given to the Don Bosco Faculty club;

(d) transfer of son of Atty. Laconico to another school or another section of Don Bosco Technical High School;

(e) Affidavit of desistance by Atty. Laconico on the Maltreatment case earlier filed against Manuel Montebon at the Cebu City Fiscal's Office, whereas Montebon's affidavit of desistance on the Direct Assault Case against Atty. Laconico to be filed later;

(f) Allow Manuel Montebon to continue teaching at the Don Bosco Technical School;

(g) Not to divulge the truth about the settlement of the Direct Assault Case to the mass media;

(h) P2,000.00 attorney s fees for Atty. Pintor. (tsn, August 26, 1981, pp. 47-48).

Twenty minutes later, complainant called up again to ask Laconico if he was agreeable to the conditions. Laconico answered 'Yes'. Complainant then told Laconico to wait for instructions on where to deliver the money. (tsn, March 10, 1983, pp. 2-12).

Complainant called up again and instructed Laconico to give the money to his wife at the office of the then Department of Public Highways. Laconico who earlier alerted his friend Colonel Zulueta of the Criminal Investigation Service of the Philippine Constabulary, insisted that complainant himself should receive the money. (tsn, March 10, 1982, pp. 26-33). When he received the money at the Igloo Restaurant, complainant was arrested by agents of the Philippine Constabulary.

Page 327: Warrantless Arrests, Searches and Seizures and Privacy of Communication

327

Appellant executed on the following day an affidavit stating that he heard complainant demand P8,000.00 for the withdrawal of the case for direct assault. Laconico attached the affidavit of appellant to the complainant for robbery/extortion which he filed against complainant. Since appellant listened to the telephone conversation without complainant's consent, complainant charged appellant and Laconico with violation of the Anti-Wiretapping Act.

After trial on the merits, the lower court, in a decision dated November 22, 1982, found both Gaanan and Laconico guilty of violating Section 1 of Republic Act No. 4200. The two were each sentenced to one (1) year imprisonment with costs. Not satisfied with the decision, the petitioner appealed to the appellate court.

On August 16, 1984, the Intermediate Appellate Court affirmed the decision of the trial court, holding that the communication between the complainant and accused Laconico was private in nature and, therefore, covered by Rep. Act No. 4200; that the petitioner overheard such communication without the knowledge and consent of the complainant; and that the extension telephone which was used by the petitioner to overhear the telephone conversation between complainant and Laconico is covered in the term "device' as provided in Rep. Act No. 4200.

In this petition for certiorari, the petitioner assails the decision of the appellate court and raises the following issues; (a) whether or not the telephone conversation between the complainant and accused Laconico was private in nature; (b) whether or not an extension telephone is covered by the term "device or arrangement" under Rep. Act No. 4200; (c) whether or not the petitioner had authority to listen or overhear said telephone conversation and (d) whether or not Rep. Act No. 4200 is ambiguous and, therefore, should be construed in favor of the petitioner.

Section 1 of Rep. Act No. 4200 provides:

Section 1. It shall be unlawful for any person, not being authorized by all the parties to any private communication or spoken word, to tap any wire or cable or by using any other device or arrangement, to secretly overhear, intercept, or record such communication or spoken word by using a device commonly known as a dictaphone or dictagraph or detectaphone or walkie-talkie or tape-recorder, or however otherwise described:

It shall be unlawful for any person, be he a participant or not in the act or acts penalized in the next preceeding sentence, to knowingly possess any tape record, wire record, disc record, or any other such record, or copies thereof, of any communication or spoken word secured either before or after the effective date of this Act in the manner prohibited by this law; or to replay the same for any other person or persons; or to communicate the contents thereof, either verbally or in writing, or to furnish transcriptions thereof, whether complete or partial, to any other person: Provided, that the use of such record or any copies thereof as evidence in any civil, criminal investigation or trial of offenses mentioned in Section 3 hereof, shall not be covered by this prohibition.

We rule for the petitioner.

We are confronted in this case with the interpretation of a penal statute and not a rule of evidence. The issue is not the admissibility of evidence secured over an extension line of a telephone by a third party. The issue is whether or not the person called over the telephone and his lawyer listening to the conversation on an extension line should both face prison sentences simply because the extension was used to enable them to both listen to an alleged attempt at extortion.

There is no question that the telephone conversation between complainant Atty. Pintor and accused Atty. Laconico was "private" in the sense that the words uttered were made between one person and another

Page 328: Warrantless Arrests, Searches and Seizures and Privacy of Communication

328

as distinguished from words between a speaker and a public. It is also undisputed that only one of the parties gave the petitioner the authority to listen to and overhear the caller's message with the use of an extension telephone line. Obviously, complainant Pintor, a member of the Philippine bar, would not have discussed the alleged demand for an P8,000.00 consideration in order to have his client withdraw a direct assault charge against Atty. Laconico filed with the Cebu City Fiscal's Office if he knew that another lawyer was also listening. We have to consider, however, that affirmance of the criminal conviction would, in effect, mean that a caller by merely using a telephone line can force the listener to secrecy no matter how obscene, criminal, or annoying the call may be. It would be the word of the caller against the listener's.

Because of technical problems caused by the sensitive nature of electronic equipment and the extra heavy loads which telephone cables are made to carry in certain areas, telephone users often encounter what are called "crossed lines". An unwary citizzen who happens to pick up his telephone and who overhears the details of a crime might hesitate to inform police authorities if he knows that he could be accused under Rep. Act 4200 of using his own telephone to secretly overhear the private communications of the would be criminals. Surely the law was never intended for such mischievous results.

The main issue in the resolution of this petition, however, revolves around the meaning of the phrase "any other device or arrangement." Is an extension of a telephone unit such a device or arrangement as would subject the user to imprisonment ranging from six months to six years with the accessory penalty of perpetual absolute disqualification for a public officer or deportation for an alien? Private secretaries with extension lines to their bosses' telephones are sometimes asked to use answering or recording devices to record business conversations between a boss and another businessman. Would transcribing a recorded message for the use of the boss be a proscribed offense? or for that matter, would a "party line" be a device or arrangement under the law?

The petitioner contends that telephones or extension telephones are not included in the enumeration of "commonly known" listening or recording devices, nor do they belong to the same class of enumerated electronic devices contemplated by law. He maintains that in 1964, when Senate Bill No. 9 (later Rep. Act No. 4200) was being considered in the Senate, telephones and extension telephones were already widely used instruments, probably the most popularly known communication device.

Whether or not listening over a telephone party line would be punishable was discussed on the floor of the Senate. Yet, when the bill was finalized into a statute, no mention was made of telephones in the enumeration of devices "commonly known as a dictaphone or dictagraph, detectaphone or walkie talkie or tape recorder or however otherwise described." The omission was not a mere oversight. Telephone party lines were intentionally deleted from the provisions of the Act.

The respondent People argue that an extension telephone is embraced and covered by the term "device" within the context of the aforementioned law because it is not a part or portion of a complete set of a telephone apparatus. It is a separate device and distinct set of a movable apparatus consisting of a wire and a set of telephone receiver not forming part of a main telephone set which can be detached or removed and can be transferred away from one place to another and to be plugged or attached to a main telephone line to get the desired communication corning from the other party or end.

The law refers to a "tap" of a wire or cable or the use of a "device or arrangement" for the purpose of secretly overhearing, intercepting, or recording the communication. There must be either a physical interruption through a wiretap or the deliberate installation of a device or arrangement in order to overhear, intercept, or record the spoken words.

An extension telephone cannot be placed in the same category as a dictaphone, dictagraph or the other devices enumerated in Section 1 of RA No. 4200 as the use thereof cannot be considered as "tapping" the wire or cable of a telephone line. The telephone extension in this case was not installed for that purpose. It just happened to be there for ordinary office use. It is a rule in statutory construction that in

Page 329: Warrantless Arrests, Searches and Seizures and Privacy of Communication

329

order to determine the true intent of the legislature, the particular clauses and phrases of the statute should not be taken as detached and isolated expressions, but the whole and every part thereof must be considered in fixing the meaning of any of its parts. (see Commissioner of Customs v. Esso Estandard Eastern, Inc., 66 SCRA 113,120).

In the case of Empire Insurance Com any v. Rufino (90 SCRA 437, 443-444), we ruled:

Likewise, Article 1372 of the Civil Code stipulates that 'however general the terms of a contract may be, they shall not be understood to comprehend things that are distinct and cases that are different from those upon which the parties intended to agree.' Similarly, Article 1374 of the same Code provides that 'the various stipulations of a contract shall be interpreted together, attributing to the doubtful ones that sense which may result from all of them taken jointly.

xxx xxx xxx

Consequently, the phrase 'all liabilities or obligations of the decedent' used in paragraph 5(c) and 7(d) should be then restricted only to those listed in the Inventory and should not be construed as to comprehend all other obligations of the decedent. The rule that 'particularization followed by a general expression will ordinarily be restricted to the former' is based on the fact in human experience that usually the minds of parties are addressed specially to the particularization, and that the generalities, though broad enough to comprehend other fields if they stood alone, are used in contemplation of that upon which the minds of the parties are centered. (Hoffman v. Eastern Wisconsin R., etc., Co., 134 Wis. 603, 607, 115 NW 383, cited in Francisco, Revised Rules of Court (Evidence), 1973 ed, pp. 180-181).

Hence, the phrase "device or arrangement" in Section 1 of RA No. 4200, although not exclusive to that enumerated therein, should be construed to comprehend instruments of the same or similar nature, that is, instruments the use of which would be tantamount to tapping the main line of a telephone. It refers to instruments whose installation or presence cannot be presumed by the party or parties being overheard because, by their very nature, they are not of common usage and their purpose is precisely for tapping, intercepting or recording a telephone conversation.

An extension telephone is an instrument which is very common especially now when the extended unit does not have to be connected by wire to the main telephone but can be moved from place ' to place within a radius of a kilometer or more. A person should safely presume that the party he is calling at the other end of the line probably has an extension telephone and he runs the risk of a third party listening as in the case of a party line or a telephone unit which shares its line with another. As was held in the case of Rathbun v. United States (355, U.S. 107, 2 L Ed 2d 137-138):

Common experience tells us that a call to a particular telephone number may cause the bell to ring in more than one ordinarily used instrument. Each party to a telephone conversation takes the risk that the other party may have an extension telephone and may allow another to overhear the conversation. When such takes place there has been no violation of any privacy of which the parties may complain. Consequently, one element of 605, interception, has not occurred.

In the same case, the Court further ruled that the conduct of the party would differ in no way if instead of repeating the message he held out his hand-set so that another could hear out of it and that there is no distinction between that sort of action and permitting an outsider to use an extension telephone for the same purpose.

Page 330: Warrantless Arrests, Searches and Seizures and Privacy of Communication

330

Furthermore, it is a general rule that penal statutes must be construed strictly in favor of the accused. Thus, in case of doubt as in the case at bar, on whether or not an extension telephone is included in the phrase "device or arrangement", the penal statute must be construed as not including an extension telephone. In the case ofPeople v. Purisima, 86 SCRA 542, 562, we explained the rationale behind the rule:

American jurisprudence sets down the reason for this rule to be the tenderness of the law of the rights of individuals; the object is to establish a certain rule by conformity to which mankind would be safe, and the discretion of the court limited. (United States v. Harris, 177 US 305, 44 L Ed 780, 20 S Ct 609; Braffith v. Virgin Islands (CA3) 26 F2d 646; Caudill v. State, 224 Ind 531, 69 NE2d; Jennings v. Commonwealth, 109 VA 821,63 SE 1080, all cited in 73 Am Jur 2d 452). The purpose is not to enable a guilty person to escape punishment through a technicality but to provide a precise definition of forbidden acts." (State v. Zazzaro, 20 A 2d 737, quoted in Martin's Handbook on Statutory Construction, Rev. Ed. pp. 183-184).

In the same case of Purisima, we also ruled that on the construction or interpretation of a legislative measure, the primary rule is to search for and determine the intent and spirit of the law. A perusal of the Senate Congressional Records will show that not only did our lawmakers not contemplate the inclusion of an extension telephone as a prohibited device or arrangement" but of greater importance, they were more concerned with penalizing the act of recording than the act of merely listening to a telephone conversation.

xxx xxx xxx

Senator Tañada. Another possible objection to that is entrapment which is certainly objectionable. It is made possible by special amendment which Your Honor may introduce.

Senator Diokno.Your Honor, I would feel that entrapment would be less possible with the amendment than without it, because with the amendment the evidence of entrapment would only consist of government testimony as against the testimony of the defendant. With this amendment, they would have the right, and the government officials and the person in fact would have the right to tape record their conversation.

Senator Tañada. In case of entrapment, it would be the government.

Senator Diokno. In the same way, under this provision, neither party could record and, therefore, the court would be limited to saying: "Okay, who is more credible, the police officers or the defendant?" In these cases, as experienced lawyers, we know that the Court go with the peace offices.

(Congressional Record, Vol. 111, No. 33, p. 628, March 12, 1964).

xxx xxx xxx

Senator Diokno. The point I have in mind is that under these conditions, with an agent outside listening in, he could falsify the testimony and there is no way of checking it. But if you allow him to record or make a recording in any form of what is happening, then the chances of falsifying the evidence is not very much.

Page 331: Warrantless Arrests, Searches and Seizures and Privacy of Communication

331

Senator Tañada. Your Honor, this bill is not intended to prevent the presentation of false testimony. If we could devise a way by which we could prevent the presentation of false testimony, it would be wonderful. But what this bill intends to prohibit is the use of tape record and other electronic devices to intercept private conversations which later on will be used in court.

(Congressional Record, Vol. III, No. 33, March 12, 1964, p. 629).

It can be readily seen that our lawmakers intended to discourage, through punishment, persons such as government authorities or representatives of organized groups from installing devices in order to gather evidence for use in court or to intimidate, blackmail or gain some unwarranted advantage over the telephone users. Consequently, the mere act of listening, in order to be punishable must strictly be with the use of the enumerated devices in RA No. 4200 or others of similar nature. We are of the view that an extension telephone is not among such devices or arrangements.

WHEREFORE, the petition is GRANTED. The decision of the then Intermediate Appellate Court dated August 16, 1984 is ANNULLED and SET ASIDE. The petitioner is hereby ACQUITTED of the crime of violation of Rep. Act No. 4200, otherwise known as the Anti-Wiretapping Act.

SO ORDERED.

Feria (Chairman), Fernan, Alampay and Paras, JJ., concur.

G.R. No. L-68635 March 12, 1987

IN THE MATTER OF PROCEEDINGS FOR DISCIPLINARY ACTION AGAINST ATTY. WENCESLAO LAURETA, AND OF CONTEMPT PROCEEDINGS AGAINST EVA MARAVILLA-ILUSTRE in G.R. NO. 68635, entitled "EVA MARAVILLA-ILUSTRE vs. HON. INTERMEDIATE APPELLATE COURT, ET AL.

R E S O L U T I O N

PER CURIAM:

In almost identical letters dated 20 October 1986, personally sent to Justices Andres R. Narvasa, Ameurfina M. Herrera, and Isagani A. Cruz, and a fourth letter, dated 22 October 1986 addressed to Justice Florentino P. Feliciano, all members of the First Division of this COURT, (mcorporated herein by reference), in feigned ignorance of the Constitutional requirement that the Court's Divisions are composed of, and must act through, at least five (5) members, and in a stance of dangling threats to effect a change of the Court's adverse resolution, petitioner Eva Maravilla Ilustre wrote in part:

Page 332: Warrantless Arrests, Searches and Seizures and Privacy of Communication

332

Please forgive us for taking the Liberty of addressing you this letter which we do hope you will read very carefully.

It is important to call your attention to the dismissal of Case No. G.R. 68635 entitled Eva Maravilla Ilustre vs. Hon. Intermediate Appellate Court, et al. by an untenable minute-resolution although an extended one, dated 14 May 1986 which we consider as an just resolution deliberately and knowingly promulgated by the First Division of the Supreme Court of which you are a member.

xxx xxx xxx

We consider the three minute-resolution: the first dated 14 May 1986; the second, dated 9 July 1986; and the third, 3 September 1986, railroaded with such hurry/ promptitude unequaled in the entire history of the Supreme Court under circumstances that have gone beyond the limits of legal and judicial ethic.

xxx xxx xxx

Your attention is called to minute-resolution of 9 July 1986 which writes finish to our case before the Supreme Court (... THIS IS FINAL.) There is nothing final in this world We assure you that this case is far from finished by a long shot For at the proper time, we shall so act and bring this case before another forum where the members of the Court can no longer deny our action with minute resolutions that are not only unjust but are knowingly and deuberat only promulgated. The people deserve to know how the members of the highest tribunal of the land perform in the task of decision making by affixing their respective signed on judgments that they render on petitions that they themselves give due course.

Please understand that we are pursuing further remedies in our quest for justice under the law. We intend to hold responsible members of the First Division who participated in the promulgation of these three minute- resolutions in question. For the members thereof cannot claim immunity when their action runs afoul with penal sanctions, even in the performance of official functions; like others, none of the division members are above the law.

In our quest for justice, we wish to avoid doing injustice to anyone, particularly the members of the First Division, providing that they had no hand in the promulgation of the resolution in question. That is why we are requesting you to inform us your participation in the promulgation of these resolutions in question. Even we who are poor are also capable of playing fair even to those who take advantage of our poverty by sheer power and influence. We shall then wait for your reply. If, however, we do not hear from you after a week, then we will consider your silence that you supported the dismissal of our petition. We will then be guided accordingly. (Emphasis supplied).

The letter also attacked the participation in the case of Justice Pedro L. Yap, Chairman of the First Division in this wise:

As Division Chairman, Associate Justice Pedro Yap, as a copy of Resolution dated 14 May 1986 we received indicate, did not even have the elementary courtesy of putting on record that he voluntarily inhibited himself from participating in the promulgation of this minute-resolution, although an extended one, which he should have done consistent with judicial decorum and the Canons of Judicial Ethics. After he is the law partner of Atty. Sedfrey A. Ordonez counsel for respondents, now the distinguished Solicitor General ... indicative that even at this stage of the proceeding in point of time, the Supreme Court

Page 333: Warrantless Arrests, Searches and Seizures and Privacy of Communication

333

still recognizes Atty. Sedfrey A. Ordonez as counsel for respondents, even as he is already the Solicitor General. For not withdrawing from the case formally Atty. Ordonez has manifested his unmitigated arrogance that he does not respect the Canons of Professional Ethics, to the actuation of his law partner, Associate Justice Pedro Yap, Chairman of the First Division of the Supreme Court, an act that further aggravates the growing wrinkles in the domain of judicial statesmanship, impressed as it is, with very serious and dangerous implications.

(9) By 11 April 1986, date of the reorganization of the First Division, Atty. Sedfrey A. Ordonez already became the Solicitor General. With such amazingly magical coincidence, Dr. Pedro Yap, law partner of Atty. Sedfrey A. Ordonez in the law firm Salonga, Ordonez Yap, Padian became the Chairman of the Division.

xxx xxx xxx

(11) So we see that on 11 August 1986 to 14 May 1986 when some members of the Division were still busy putting their respective offices in order and had possibly have no Idea about the Maravilla case.

Was it possible for Chairman Yap to have convinced the Division members that Maravilla petition is without merit and since the members — the new ones knew nothing about the case, readily agreed to the dismissal of the petition by a minute. resolution — an extended one. After all, this was the case of the Solicitor General. If this is what happened, then we are sorry to say that you were deliberately "had"

After ala the 14 May 1986 untenable minute resolution although an extended one, does not bear the signatures of the Division members. The members should have signed the resolution, after ala the Supreme Court had given the petition due course, indicating whether they concur, dissent or otherwise abstain from voting.

The letter to Justice Herrera went on to state:

We assume, of course, that you had studied the case thoroughly since you were with the original First Division under the chairmanship of then Justice Claudio Teehankee. We assure you that we will bring this case before another forum to hold responsible the members of the Division who participated in the dismissal of the case by the unjust minute-resolutions, knowingly rendered for intended objective that your conscience you are aware.

xxx xxx xxx

We leave the next move to you by informing us your participation in the promulgation of the minute-resolutions in question Please do not take this matter rightly for we know justice in the end will prevail. For if we do not hear from you within a week, we will consider your silence as your admission that you supported the dismissal of the petition. In this way, we shall then be guided accordingly. The moment we take action in the plans we are completing, we will then call a press conference with TV and radio coverage. Arrangements in this regard are being done. The people should or ought to know why we were thwarted in our quest for plain justice.

xxx xxx xxx

Page 334: Warrantless Arrests, Searches and Seizures and Privacy of Communication

334

Finally, in view of action that we are prepared to take in this case, that will no doubt cause nationwide attention, and there should be anyone that will cause me harm personally, may we request you to show this letter to the authorities concerned so that they will know where to look, when it becomes necessary. (Emphasis supplied)

The aforesaid letters were included in the Agenda of the First Division of 22 October 1986, were "Noted," and referred en consulta to the Court en banc.

On 28 October 1986, the Court en banc took up the background and history of the case, found no reason to take any further action, and referred the case back to the First Division 11 as set forth in the latter's resolution of October 27, 1986. " In this Resolution, the First Division traced the history of the case, clarified that Justice Yap assumed his position in this Court only on 2 May 1986; that when the resolution of dismissal was issued on 14 May 1986, Justice Abad Santos was the incumbent Chairman of the First Division, and that Justice Yap was unaware that Atty. Ordonez was private respondents' counsel; that upon realization thereof, Justice Yap inhibited himself from further participation in the case; and that Justice Yap was designated Chairman of the First Division only on 14 July 1986, after the compulsory retirement of Justice Vicente Abad Santos on 12 July 1986. The Resolution of the First Division (incorporated herein by reference) concluded thus:

The dispositions in this case were arrived at after careful study. Because a case is resolved against the interests of a party, does not mean that it is an "unjust decision" or that it has been "railroaded".

This Division declares without hesitation that it has consistently rendered justice without fear or favor. YAP, J., took no part.

On 3 November 1986, petitioner again addressed similar letters to Justices Narvasa, Herrera, and Cruz, (incorporated herein by reference), excerpts from which follow:

It is rather amazing that when we wrote you our previous letter, we never dreamed that you would rush, as you did rush for assistance en consults with the Honorable Court en banc. The unfortunate part of it all is the fact that the Court en banc had to promulgate its resolution dated 28 October 1986 which to us when considered in its entirety, is just as untenable as the First Division extended and unsigned minute-resolution of 14 May 1986.

Evidently you misunderstood our point of inquiry, to wit: 'Did you or did you not approve the dismissal of our petition under

1) The l4 May l986 minute resolution? Yes or No.

2) The 9 July l986 minute resolution? Yes or No.

3) The 3 Sept. 1986 minute resolution? Yes or No.

That was we asked. The other matters contained in our letter were intended merely to give you the highlights of our case. This is what we wanted to know to properly guide us when we finally bring our case to the other forum of justice.

Did it ever occur to you that when you and the other members of the First Division referred our letters to the Honorable Court en banc en consults it was all your fault that the Court en banc had to promulgate its unsigned extended minute-resolution that unfortunately exposed the distinguished members of the newly reorganized Supreme

Page 335: Warrantless Arrests, Searches and Seizures and Privacy of Communication

335

Court and, at the same time, convicted themselves as guilty of distorting facts involved in our petition?

This, we are sure, will come as a shock to you. We will show you why.

xxx xxx xxx

This is just a sample of what we will expose to the nation before the other forum of justice where we will soon bring this case beyond the reach of the newly reorganized Supreme Court We are prepared to expose many more of this kind of judicial performance readily constituting truvesty of justice Ponder upon this well because it is our very firm conviction that the people deserve to know how the distinguished members of the highest tribunal of the land perform in duties in this most sensitive area of decision making.

Anyhow, whether you referred our letter to the Court en banc consults) or not, the situation remains the same. At the proper time, as we said, we will bring this case before another forum of justice where the members of the First Division, in fact the Honorable Court en banc may no longer deny our action by mere untenable and unjust minute resolutions. Better believe it that we intend to hold responsible members of the First Division who took part in the promulgation of the untenable and unjust extended minute-resolution that is not even signed by any of those who promulgated it; therefore, to us, is clearly bereft of judicial integrity from its very inception on 14 May 1986.

xxx xxx xxx

Thus, we will bring this case before another forum of justice as Eva Maravilla Ilustre against the distinguished members of the First Division, in fact against the entire membership of the newly organized Supreme Court (because of its en banc unsigned extended minute-resolution that iswithout judicial integrity, dated 28 October 1986). But do not be mislead (sic) for we are not alone in this fight. Other lawyers, not just by their mere sympathy for me personally and my case, but by their firm conviction that judicial statesmanship must be maintained at nines in the highest tribunal of justice in the land, that they have offered their free legal services when the legal confrontation begins.

xxx xxx xxx

Paragraph 4, found on page 3 of the en banc resolution projects the most fantastic, most unbelievable picture of Division Chairman Justice Yap. It states —

...When the resolution of dismissal on May 14, 1986, Justice Yap was unaware that Atty. Sedfrey A. Ordonez was private respondent's counsel.

The Honorable Court en banc must think everybody stupid to swallow this statement hook, line and sinker For Justice Yap we say. Tell that to the marines. But more than this, we leave this matter to the conscience of Justice Yap.

Ignoramus that we are, unschooled in the domain of law and procedure, but we are learning a few as we promulgated our case within legitimate that we state here that both resolutions that promulgated by the Court en banc of 28 October 1986 and that promulgated by the First Division dated 27 October 1986, are nothing but a desperate attempt when both are considered in their respective entirety, to maneuver without

Page 336: Warrantless Arrests, Searches and Seizures and Privacy of Communication

336

success, some semblance of justification on the untenable and unjust 14 May 1986 extended and unsigned minute-resolution that is bereft of judicial integrity.

xxx xxx xxx

Thus, if the members of the First Division and those of the Honorable Court en banc think for one minute that because of their respective 4-page minute but extended resolutions apparently impressive for their lack of merit deliberately unsigned that exposed their lack of judicial integrity, that we win now give up the fight, just forget it. Ignoramus that we are, better believe it when we say we are prepared to carry the fight before another forum of justice. When we do, we shall call for a press conference with TV and radio coverage, so that we can present to the entire nation our quest for justice against the steam-roller of power and influence and, at the same time, to call the attention of the people to the manner in which the members of the highest tribunal of the land perform their respective individual and collective functions in the domain of this most sensitive area of decision making.

Allow us to restate our previous and now, our present inquiry, to wit:

Did you or did you not approve the dismissal of our petition under —

a) The l4 May l986 minute resolution? Yes or No.

b) The 9 July l986 minute resolution? Yes or No.

c) The 3 Sept. 1986 minute resolution? Yes or No. (Emphasis supplied).

True to her threats, after having lost her case before this Court, petitioner filed on 16 December 1986 an Affidavit-Complaint before the Tanodbayan, totally disregarding the facts and circumstances and legal considerations set forth in this Court's aforecited Resolutions of the First Division and en banc. Some Members of this Court were maliciously charged with having knowingly and deliberately rendered, with bad faith, an unjust, extended Minute Resolution "making" her opponents the "illegal owners" of vast estates. Some Justices of the Court of Appeals were similarly maliciously charged with knowingly rendering their "unjust resolution" of 20 January 1984 "through manifest and evident bad faith," when their Resolution had in fact and law been upheld by this Court. Additionally, Solicitor General Sedfrey A. Ordonez and Justice Pedro Yap of this Court were also maliciously charged with having used their power and influence in persuading and inducing the members of the First Division of this Court into promulgating their "unjust extended Minute Resolution of 14 May 1986."

All the foregoing, in complete disregard of the Resolutions of this Court, as the tribunal of last resort, 1) upholding the challenged judgment of the Court of Appeals; 2) dismissing the Petition on the ground that the doctrine of res judicata was clearly applicable not only as to the probate of the Will of the decedent but also as to the heirship of petitioner, among others, and their right to intervene and participate in the proceedings; and 3) finding that there was no attempt whatsoever on the part of Justice Yap nor Solicitor General Ordonez to unduly influence the members of the First Division.

The Complaint before the Tanodbayan (incorporated herein by reference) was allegedly filed "in my quest for justice, something that has been closed to me by the Supreme Court forever" and specifically charged:

CHARGE NO. ONE —

Atty. Sedfrey A. Ordonez and Justice Pedro Yap of 1) "persuading, inducing, influencing the members of the newly organized First Division ... into promulgating their

Page 337: Warrantless Arrests, Searches and Seizures and Privacy of Communication

337

unjust, extended minute RESOLUTION of 14 May 1986, knowingly with deliberate intent with such unusual hurry promptitude unequaled in the entire history of the Supreme Court base on insignificant issues and deliberately evading/prevaricating the more important substantial ones raised in my petition, in violation of Section 3, sub-letter (a) of Republic Act No. 3019, as amended, ... ; and

(2) Under the same Section 3, sub-letter (e) of the same Republic Act ... for causing me and the other heirs of Ponciano Maravilla undue injury by using their power and influence as Solicitor-General and Associate Justice, respectively. ...

CHARGE NO. TWO —

Associate Justices Luis Javellana, Vicente Mendoza and Serafin Cuevas, members of the then FOURTH SPECIAL CASES DIVISION, Intermediate Appellate Court —

1) For knowingly rendering their unjust RESOLUTION dated 20 January 1984 in the exercise of their functions through manifest and evident bad faith in CA-G.R. No. SP-13680, entitled "Francisco Q. Maravilla, et al. v. Hon. Antonia Corpus Macandog, et al." in violation of article 204 of the revised Penal Code,-

2) For causing me and the other heirs such undue injury' by deliberately, knowingly rendering their unjust RESOLUTION dated 20 January 1984 ... in violation of Republic Act No. 3019, as amended, Section 3 (e) thereof.

CHARGE NO THREE —

Associate Justice Vicente Abad Santos (retired) then Chairman of the First Division of the Supreme Court as of 14 May 1986, and Associate Justice Isagani Cruz, Andres Narvasa, Ameurfina M. Herrera and Pedro Yap, ... —

1) For knowingly and deliberately rendering their unjust extended MINUTE RESOLUTION of 14 May 1986 dismissing my petition in G.R. No. 68635, ... with manifest and evident bad faith to make the clients of Atty. Sedfrey A. Ordonez now the distinguished Solicitor General the 'illegal owners' of the vast estates of my aunt Digna Maravilla ...;

2) Under Section 3, sub-letter (e) Republic Act No. 3019, as amended, ... for deliberately causing us heirs of Ponciano Maravilla undue injury by depriving us of our rights over my aunt's vast estates because of their manifest and evident bad faith in knowingly promulgating their unjust extended minute RESOLUTION of 14 May 1986, deliberately intended to make the clients of Atty. Sedfrey A. Ordonez, now the Solicitor General the "illegal owners" of my aunt Digna Maravilla's estates when under the law, these Ordonez clients are not entitled to own these vast properties whether under testate or intestate succession or mixed succession (Emphasis supplied).

Atty. Laureta himself reportedly circulated copies of the Complaint to the press, which was widely publicized in almost all dailies on 23 December 1986, without any copy furnished this Court nor the members who were charged. The issue of the Daily Express of 23 December 1986 published a banner headline reading —

ORDONEZ, 8 JUSTICES FACE GRAFT CHARGES

Page 338: Warrantless Arrests, Searches and Seizures and Privacy of Communication

338

thereby making it unjustly appear that the Justices of this Court and the other respondents were charged with "graft and corruption" when the Complaint was actually filed by a disgruntled litigant and her counsel after having lost her case thrice in this Court.

On 26 December 1986, the Tanodbayan (Ombudsman) dismissed petitioner's Complaint and decreed in the dispositive portion of his Resolution (herein incorporated by reference) that:

WHEREFORE, all the premises considered, this Office resolves to dismiss the complaint against Justices Pedro Yap, Isagani Cruz, Andres Narvasa, Ameurfina Melencio-Herrera, Vicente Abad Santos, and will continue evaluating the complaint against Justices Serafin Cuevas, Luis Javellana and Vicente Mendoza, Solicitor General Sedfrey Ordonez and the private respondents.

The aforestated Resolution indicated at the bottom of the last page:

Copy Furnished:

DEAN WENCESLAO LAURETA

Counsel for the Complainant

919 Prudencio Street

Sampaloc, Manila

In the Resolution of this Court en banc, dated January 29, 1986, it required:

(1) Petitioner Eva Maravilla Ilustre to show cause, with in ten (10) days from notice, why she should not be held in contempt for her aforecited statements, conduct, acts and charges against the Supreme Court and/or official actions of the Justices concerned, which statements, unless satisfactorily explained, transcend the permissible bounds of propriety and undermine and degrade the administration of justice; and

(2) Atty. Wenceslao Laureta, as an officer of the Court, to show cause, within ten (10) days from notice, why no disciplinary action should be taken against him for the aforecited statements, conduct, acts and charges against the Supreme Court and the official actions of the Justices concerned, and for hiding therefrom in anonymity behind his client's name, in an alleged quest for justice but with the manifest intent to bring the Justices into disrepute and to subvert public confidence in the Courts and the orderly administration of justice. (pp. 383-384, Rollo).

(1)

In her Compliance-Answer filed on February 9, 1987, wherein Eva Maravilla Ilustre prays that the contempt proceedings against her be dismissed, she contends, in essence, that: (1) "there was no intention to affront the honor and dignity" of the Court; (2) the letters addressed to the individual Justices were private in character and were never meant for anybody, much less the Supreme Court en banc, "there (being) a constitutional mandate affording protection to privacy of communications;" (3) if her statements in those letters were really contemptuous, the Court "should have immediately taken disciplinary proceedings" against her, and not having done so, the Court has "forfeited" that right and is now "estopped" from doing so; this citation for contempt is a "vindictive reprisal" for her having filed the complaint before the Tanodbayan, "an action that lacks sincerity, taken not in the spirit of judicial statemanship;" (4) she instituted the complaint before the Tanodbayan "in my honest behalf that I lost my

Page 339: Warrantless Arrests, Searches and Seizures and Privacy of Communication

339

case before the Supreme Court not because of lack of merit or of its own merits, assisted by attorneys who offered their services in the prosecution of my case;" (5) the newspaper publicity of this case "was no fault of mine; neither is it the fault of my former counsel Dean Wenceslao Laureta, " who prevailed upon her to call off the press conference with TV and radio coverage; that she is not a "disgruntled litigant" who thrice lost before the Court, rather, she has challenged the validity of the resolutions of the Court "containing distortion of facts, conjectures and mistaken inferences" particularly, in that (a) there is no res judicata, (b) the Court of Appeals in its decision declared that the judgment of the trial Court had long attained finality, so that it can no longer be set aside, (c) her 11 opponents," clients of Atty. Ordonez, are not entitled to own her aunt's "vast properties" whether under the law of testate or intestate succession or mixed succession," (d) that the statement in this Court's Resolution that the Court of Appeals had denied intervention is an "unadulterated distortion of the facts;" (b) the statement in the en banc Resolution that some Justices of the Court of Appeals were similarly maliciously charged with knowingly rendering their "unjust resolution" of 20 January 1984 is a bit "premature, a pre-judgment over a case over which this Court does not have jurisdiction;" (7) Atty. Laureta is not her counsel in the case before the Tanodbayan; (8) before the latter body, she has "established not only probable cause but has also proved the collective culpability (of the Justices concerned) as charged;" (9) and that her 53 page Motion for Reconsideration before the Tanodbayan is made an integral part of her Answer.

(2)

In his own Answer, Atty. Laureta maintains substantially that: (1) he is not respondent Ilustre's counsel before the Tanodbayan and that she has consulted and/or engaged the services of other attorneys in the course of the prosecution of her case, fike Atty. Edgardo M. Salandanan and Atty. Vedastro B. Gesmundo; that he just learned from other sources that respondent llustre was planning to bring her case to the Tanodbayan with the assistance of other lawyers who offered her their legal services; (2) it was he who dissuaded her from calling her intended press conference and from circulating copies of her complaint "not only in the performance of duty as an officer of the court, but also as a former president of Manila III Chapter of the Integrated Bar of the Philippines and as a professional lecturer in Legal and Judicial Ethics in some Manila law schools in his desire to protect and uphold the honor and dignity of the Supreme Court as the highest tribunal of the land." He should, therefore, be given "a little bit of credit for what he did" instead of taking this disciplinary proceeding against him; that Ms. Ilustre is not a "disgruntled litigant" who "lost her case thrice in this Court;" (3) he did not prepare respondent Ilustre's letters to the individual Justices, I appearances to the contrary notwithstanding," that these letters were "never, at any time, considered as constituting contempt of court" in the resolutions of this Court, otherwise, "it would have taken immediate disciplinary action as it is doing now;" the Court has lost its right to consider the statements in the letters as constituting contempt and it is now "estopped" from proceeding with this disciplinary action; (4) by doing so, this Court has "unmistakably revealed the intent and character that underlie its present action as a vindictive judicial vengeance, inconsistent with the spirit of judicial statesmanship by hiding behind the well-recognized fact that the Supreme Court is supreme in the domain of the administration of justice;" (5) "there was no disregard intended to the Resolution of the Honorable Court, as the tribunal of last resort, relative to its upholding the judgment of the Court of Appeals;" he is just doing "his duty as an officer of the court to put the records in this regard in their proper light;" particularly (a) that the judgment of the trial court had attained its finality long ago, (b) the doctrine of res judicata is inapplicable, otherwise, this Court would not have remanded the case to the Court of Appeals for review, (c) the observation in the First Division's extended Resolution of 14 July 1986 that Justice Yap was unaware that Atty. Ordonez was private respondents' counsel "defies every vestige of human understanding," that Justice Yap had forthwith inhibited himself from participating in the case is not borne out by the record of this case. Justice Yap had "never voluntarily entered on the record his inhibition" when he should have done so when respondent Ilustre's petition was taken up; Justice Yap's partner, Atty. Ordonez, continued to be recognized by this Court as counsel for private respondents even as he was the Solicitor General; (b) finally, "appearances to the contrary notwithstanding, he has not colted acts unworthy of his profession. The truth of the matter is, he should at least be credited in whatever small way for his acts and efforts taken by him to protect and uphold the honor and dignity of the Honorable Court.

Page 340: Warrantless Arrests, Searches and Seizures and Privacy of Communication

340

We find the explanations of both Ms. Ilustre and Atty. Laureta unsatisfactory. Their claims that they had done nothing that could constitute an affront to the honor and dignity of this Court dissipate in the face of attendant facts and circumstances and "defy every vestige of human understanding," to use their own language. Indeed, they should not "think that they will win a hearing by the sheer multiplication of words." (Mathew 6:7).

Respondents' reliance on the "privacy of communication" is misplaced. Letters addressed to individual Justices, in connection with the performance of their judicial functions become part of the judicial record and are a matter of concern for the entire Court. The contumacious character of those letters constrained the First Division to refer the same to the Court en banc, en consults and so that the Court en banc could pass upon the judicial acts of the Division. It was only in the exercise of forbearance by the Court that it refrained from issuing immediately a show cause order in the expectancy that after having read the Resolution of the Court en banc of October 28, 1986, respondents would realize the unjustness and unfairness of their accusations.

The Court is far from "estopped" in initiating these proceedings. The Chief Justice had promptly announced his Statement, dated December 23, 1986, that "the Supreme Court will take appropriate steps on the matter upon its resumption of sessions on the first working day of the year. "

There is no vindicative reprisal involved. The Court's authority and duty under the premises is unmistakable. It must act to preserve its honor and dignity from the scurrilous attacks of an irate lawyer, mouthed by his client, and to safeguard the morals and ethics of the legal profession.

We are not convinced that Atty. Laureta had nothing to do with respondent Ilustre's letters to the individual Justices, nor with the com plaint filed before the Tanodbayan. In the Motion for Reconsideration, dated June 11, 1986, filed by Atty. Laureta in the main petition, he stressed:

10. The composition of the First Division was reduced to five members. Strangely enough, about one month later, the Honorable Court promulgated its extended resolution with such promptitude in the entire history of the Supreme Court, unequaled in a manner of speaking, ...

In the Manifestation and Motion, dated June 25, 1986, filed by Atty. Laureta (p. 311, Rollo), the same phrases were incanted:

the promptitude with which the Resolution of 14 May 1986 was promulgated (par. 9, Motion for Reconsideration, p. 5) unequaled in the entire history of the Supreme Court in so far as petitions given due course is concerned ... (Emphasis given)

Those same terms are reproduced verbatim in the letters ostensibly authored by respondent Ilustre addressed to the individual Justices whom respondents have charged. Thus:

We consider the three minute resolutions ... railroaded with such hurry/promptitude unequalled in the entire history of the Supreme Court under circumstances that have gone beyond the limits of legal and judicial ethics" ltr., to Justice Narvasa, p. 2; Itr., to Justice Herrera, p. 2; Itr., to Justice Cruz, p. 2).

xxx xxx xxx

with such unusual hurry/promptitude unequalled in the entire history of the Supreme Court (Ltr., to Justice Narvasa, p. 5; Itr., to Justice Herrera, p. 5; Itr., to Justice Cruz, p. 5).

Page 341: Warrantless Arrests, Searches and Seizures and Privacy of Communication

341

The same terminologies are reiterated in the Complaint and in the Motion for Reconsideration filed before the Tanodbayan (p. 2).

Further, in his Manifestation & Motion, dated June 25, 1986, Atty. Laureta stated:

counsel for petitioner personally inquired from Division Clerk of Court Corazon Served the following:

(1) When was the above-entitled case deliberated by the First Division?

(2) Are there recorded minutes of such deliberation?

(3) Who among the members of the Division voted for dismissal of the petition to be promulgated by resolution and who did not, if any?

(4) Who prepared the Resolution? (p.312, Rollo).

Atty. Laureta's obsession to receive the answer to his queries surfaces again in the second letters dated November 3, 1986 to the individual Justices under the supposed signatures of respondent Ilustre, thus:

Evidently you misunderstood our point of in our first letter. It is a very simple inquiry, to wit Did you or did you not approve the dismissal of our petition under

1) The l4 May l986 minute resoluTion? Yes or No

2) The 9 July l986 minute resoluTion? Yes or No

3) The 3 Sept. 1986 minute resolution? Yes or No. (Emphasis supplied) ltr., to Justice Narvasa, p. 1; to Justice Herrera, p. 1; to Justice Cruz, p. 1)

Additionally, the disparaging remarks like: exertion of "undue" and "powerful influence" by Atty. Ordonez and Justice Yap; "distortion of facts, conjectures and mistaken references"; "untenable minute resolution although extend. "unjust minute resolution" repeated by Atty. Laureta in his several pleadings, echoed and re-echoed in the individual letters to the Justices, as well as in the Complaint and the Motion for Reconsideration before the Tanodbayan, reveal the not-too-hidden hand of Atty. Laureta.

The foregoing is bolstered by the reports received by the members of the Court that copies of the complaint filed with the Tanodbayan were distributed to the editors of the metropolitan newspapers in envelopes bearing the name of respondent Laureta, who was heard over the radio speaking on the same complaint, and that he was following up the complaint and the motion for reconsideration of the order of dismissal of the Tanodbayan.

Furthermore, respondent Laureta as his co-respondent Ilustre's lawyer had control of the proceedings. As stressed by this Court in an early case, as such lawyer, "Whatever steps his client takes should be within his knowledge and responsibility. Indeed, Canon 16 of the Canons of Legal E times should be reminded him that '(a) lawyer should use his best efforts to restrain and to prevent his clients from doing those things which the lawyer himself ought not to do, particularly with reference to their conduct towards courts, judicial officers, jurors, witnesses and suitors. If a client pursuits in such wrongdoing the lawyer should terminate their relation.' " (In Re: Contempt Proceedings in Surigao Mineral Reservation Board vs. Cloribel, 31 SCRA 1, 23) Respondent Laureta manifestly failed to discharge such responsibility. For all intents and purposes, he appears to have encouraged and abetted his client in denigrating the members of the First Division of this Court, by baselessly charging them with rendering an "unjust" resolution with "deliberate bad faith," because of his stubborn insistence on his untenable arguments which had been

Page 342: Warrantless Arrests, Searches and Seizures and Privacy of Communication

342

rejected as without merit by the Court's First Division, whose Resolution was upheld by the Court en banc. Worse, the dissemination in the print and broadcast media in bold captions falsely depicting the Justices as "FAC(ING) GRAFT CHARGES" instead of the baseless rantings of a disgruntled litigant appear to have been timed to place them in a bad light at the height of the Christmas season.

We come now to the specific accusations of respondents.

They charge Associate Justices Vicente Abad Santos (retired) then Chairman of the First Division of the Supreme Court as of May 14,1986, Andres Narvasa, Ameurfina M. Herrera, and Pedro Yap for knowingly and deliberately rendering their "unjust, extended Resolution of May 14, 1986" dismissing their petition in this case with manifest and evident bad faith to make the clients of Atty. Sedfrey A. Ordonez (now the Solicitor General) the "illegal owners" of the estates of Digna Maravilla, thereby causing the heirs of Ponciano Maravilla (Digna's eldest brother) undue injury by depriving them of their rights over the estates of Digna Maravilla (Charge No. Three before the Tanodbayan). They further charge Justice Yap (and Atty. Sedfrey Ordonez) of having 11 persuade(ed), inducted(ed) and influence(ed) the members of the newly organized First Division into promulgating their "unjust, extended minute Resolution of 14 May 1986" (Charge No. One before the Tanodbayan), which Resolution, (the "Division Resolution, " for short) is herewith attached as Annex "A ".

Preliminarily, respondents deny that respondent Ilustre lost three times in this Court. It cannot be denied, however, that, as stated in the Resolution of October 28, 1986 of the Court en banc, this is the third time (in fact, the fourth, if we include Fernandez, et al. vs. Maravilla, L-18799, 10 SCRA 589 [1964]) that a controversy involving the estate of the late Digna Maravilla is elevated to this Court. The first was in G.R. No. L-23225 (37 SCRA 672 [1971], where this Court ruled:

IN VIEW OF THE FOREGOING, the decree of the court below denying probate of the 1944 will of Digna Maravilla (Exhibit "A") is reversed and the said testament is hereby ordered probated. Let the records be returned to the Court of origin for further proceedings conformable to law. ...

As stated in the en banc Resolution of October 28, 1986 (hereto attached as Annex " B ", and hereinafter referred to as the "Banc Decision") while respondent Ilustre was not a party in that case, upon remand of the case to the probate Court, she and other children of the deceased brothers and sisters of the testatrix filed two Motions for Intervention. Respondent Ilustre's participation in the state involved, therefore, harks back to that first case.

The Court of Appeals resolved the issue of intervention in CA-G.R. No. 05394, entitled "Heirs of Pastor Maravilla, et al. vs. Hon. Ernesto S. Tengco, et al." in a Decision penned by Justice Venicio E scolin (hereinafter referred to as the "Escolin Decision") wherein it was categorically ruled that there was no point to allowing intervention on the part of respondent Ilustre, et al., "for failure to show any right or interest in the estate in question. " Thus:

(2) As heretofore stated, private respondents, in their counter-petition for mandamus, seek this Court's resolution on the petitioners' motion for intervention in Sp. Proc. No. 4977. In their respective pleadings and memoranda, the parties have lengthily discussed the issue of whether or not petitioners may be allowed to intervene; and the same may as well be determined in the present case, if only 'to avoid or, at least, minimize further protracted controversy' between the parties (PCIB vs. Hon. Escolin, 56 SCRA 266). A resolution of this issue should render moot and academic the question anent the disqualification of respondent Judge.

We agree with private respondents that petitioners' motions for intervention are devoid of merit, for failure on their part to show any right or interest in the estate in question. There is no dispute that the last will and testament of the late Digna Maravilla had already been

Page 343: Warrantless Arrests, Searches and Seizures and Privacy of Communication

343

admitted to probate in a final judgment which the Supreme Court promulgated on March 2, 1971 (G.R. No. L-23225). In the said will Digna instituted her husband Herminio Maravilla as —

xxx xxx xxx

The above testamentary provision for the universal heirship of Herminio Maravilla over the residue of the decedent's present and future property legally and completely excluded the petitioners, as collateral relatives of the testatrix, from inheriting any part of the latter's estate through intestate succession or mixed succession. Having no forced or compulsory heirs, except her husband, the testatrix had the absolute freedom to institute the latter as her sole, universal heir, and such freedom is recognized by Article 842 of the Civil Code which provides:

ART. 842. One who has no compulsory heirs may dispose by will of all his estate or any part of it in favor of any person having capacity to succeed.

One who has compulsory heirs may dispose of his estate provided he does not contravene the provisions of this Code with regard to the legitimate of said heirs.

There is therefore no point in allowing the petitioners, who clearly appear to have no interest in the estate, to intervene in the proceedings involving the settlement thereof.

xxx xxx xxx

The aforesaid Decision was affirmed by this Court in G.R. No. L-46155 on November 9, 1977 and has become final. That was the second case involving the estate filed before this Court.

Respondents' contention, therefore, that the statement in the Banc Resolution "that the Court of Appeals had denied intervention" is an "unadulterated distortion of the facts" is obviously erroneous and intended to mislead.

The "Escolin Decision" (in CA-G.R. No. 05394-R), which had become final, also finally foreclosed any claim that respondent Ilustre, and those who sought to intervene with her, may have had on the estate of Digna Maravilla. In unmistakable terms, what the Court of Appeals held in that Decision, affirmed by this Court, bears repeating:

The above testamentary provision for the universal heirship of Herminio Maravilla over the residue of the decedent's present and future property legally and completely excluded the petitioners, as collateral relatives of the testatrix, from inheriting any part of the latter's estate through intestate succession or mixed succession. ...

To circumvent that judgment, however, two years later, or on February 29, 1979, respondent Ilustre, with respondent Laureta as counsel, filed a complaint for partition of Digna Maravilla's estate and for damages against the heirs of Digna Maravilla's husband, who had then passed away (docketed as Civil Case No. X-404), before the Court of First Instance of Negros Occidental, San Carlos City, Branch X, presided over by Judge Antonia Corpuz Macandog. That Court, after declaring defendants therein (private respondents in the petition under review) in default, ordered "all properties of Digna Maravilla mentioned in this case to go back to their trunk of origin, the plaintiffs herein who are represented by Eva Maravilla Ilustre and Eva Maravilla Ilustre herself" (hereinafter referred to as the "Macandog Decision"). In addition, the judgment

Page 344: Warrantless Arrests, Searches and Seizures and Privacy of Communication

344

awarded damages to the respondent Ilustre, et al., (the plaintiffs therein), and the sum of P100,000.00 to their counsel, respondent Laureta.

A special civil action for certiorari was filed by the defeated parties (private respondents in the petition under review) before this Court, docketed as G.R. No. L-58014, praying that the lower Court's declaration of default in Civil Case No. X-404 and all other actions or decisions taken thereafter be declared null and void and that the dismissal of the complaint be ordered. On January 21, 1982, this Court resolved to refer the case to the Court of Appeals in aid of its appellate jurisdiction, questions of fact being involved.

In a Decision dated January 14, 1983, the Court of Appeals (Fourth Division)', 1 in AC-G.R. SP No. 13680 (hereafter called the Busran Decision"), dismissed the petition and denied certiorari stating in one breath that "the judgment subject of assail had long become final" (at p. 13), and in another "for all we know, the judgment below had already attained finality long ago." The reason relied upon was that petitioners therein had the remedy of appeal but instead availed of Certiorari, which is not a substitute therefor.

On motion for reconsideration, however, filed by petitioners (private respondents in the petition under review), in that appealed case AC-GR SP No. 13680), the same Court of Appeals (Fourth Special Cases Division) 2 in its Resolution of January 20, 1984 (the "Javellana Resolution"), reconsidered and set aside the BusRan Decision" and entered another one:

1. Annulling the order of default of the Hon. respondent Court dated 29 April 1980 and its decision dated 11 August 1981; and

2. Dismissing private respondents' complaint in Civil Case No. X-404 and ordering the Hon. respondent Court not to take further action therein.

Respondent Ilustre challenged that reversal in the present Petition for Review filed on October 22, 1984. This is the third case brought before this Court involving the same estate. Review was denied in an extended Resolution by the First Division of this Court in the challenged Resolution of May 14, 1986, for the following reasons:

The appealed Decision stands on firm legal grounds.

(1) The Order of Default of the Trial Court was issued in grave abuse of discretion. The Answer was only one day late besides the fact that when so filed, the Order of default had not yet been issued by the Trial Court.

(2) While appeal is, indeed, the remedy from a judgment by default, certiorari may be resorted to when a party has been illegally declared in 4 default Omico Mining & Industrial Corporation vs. Vallejos 63 SCRA 300-301 [19751), or where it is necessary to restore order to proceedings in the Court below (Lim Tanhu vs. Ramolete, 66 SCRA 462-463 [19751).

(3) More importantly, the judgment of the Trial Court, in Civil Case No. X-404 declaring that the Testatrix's collateral relatives have a rightful claim to her estate to the exclusion of the husband who was designated her sole and universal heir, nullifies the Will already probated by final judgment and overturns the pronouncements of both the Appellate Court and this Court on the case.

There being former judgments on the issues which have become final rendered by Courts having jurisdiction of the subject matter and the parties, the said judgments having been rendered on the merits, and there being between the prior and subsequent action Identity of parties, subject matter and substantial Identity of cause of action, it is clear that

Page 345: Warrantless Arrests, Searches and Seizures and Privacy of Communication

345

the complaint below in Civil Case X-404 is barred by the principle of res adjudicata, and whatever transpired therein are nun and void ab initio and without any legal effect.

To rule otherwise would upset the fundamental issue on which res judicata rests that parties ought not to be permitted to litigate the same issue more than once, that when a right or fact has been judicially determined, the judgment of the Court, so long as it remains unreversed, should be conclusive upon the parties and those in privity with them in law or estate (Sarabia vs. Sec. of Agriculture and Natural Resources, 2 SCRA 54 [1961]).

ACCORDINGLY, the review sought for is denied and respondent Court's judgment in CA-G.R. SP No. 13080 is hereby affirmed.

SO ORDERED.

Respondents decry the fact that the First Division set aside the due course Order and denied review in an extended Minute Resolution instead of in a signed Decision. They allege that said Resolution was "railroaded with such hurry/promptitude unequalled in the entire history of the Supreme Court under circumstances that have gone beyond the limits of legal and judicial ethics," unduly "persuaded, induced and influenced" by Solicitor General Ordonez and Justice Pedro Yap.

Nothing is farthest from the truth. As explained in the "Banc Resolution"

The petition for review was assigned to the then First Division of seven Justices, which initially gave it due course because the resolution of the Intermediate Appellate Court had reversed a decision originally rendered by the then Court of Appeals, and in order to have more time for further study.

Pleadings were submitted, the last being on May 3, 1985, which can be considered as the date when this case was submitted for resolution.

The First Division of seven (7) was not able to act on the case up to the February, 1986 political upheaval The last incident in the case was a motion for the early release of decision filed by petitioner on November 19, 1985.

When this Court was reorganized in April of 1986, the membership of the First Division was reduced to five (5) Justices. Taking account of the motion of petitioner for early release of decision, the new First Division, then chairmanned by Justice Abad Santos, realizing that the doctrine of res judicata was clearly applicable not only as to the probate of the will but also as to the heirship of petitioner, among others, and their right to intervene and participate in the proceedings — resolved, on May 14, 1986 to dismiss the petition through an extended resolution which at the same time recalled the due course order. The new Division of 5 acted unanimously.

The recall of a due course Order after a review of the records of the case is a common occurrence in the Court. Respondents speak as if it were only their petition which has been subjected to such recall. They have lost all objectivity in this regard. They are hardly qualified, and cannot presume to speak of the I entire history" of the Supreme Court.

As to the participation of Justice Yap in the ease, the "Banc Resolution" stated:

Justice Yap clarified that he was an official mission to Switzerland for the Presidential Commission on Good Government after his appointment to the Supreme Court an April

Page 346: Warrantless Arrests, Searches and Seizures and Privacy of Communication

346

11, 1986 and did not assume his position in the Supreme Court his return on May 2, 1986. When the resolution of dismissal on May 14, 1986 was issued, Justice Yap was unaware that Atty. Sedfrey Ordonez was private respondent's counsel.

On June 11, 1986, petitioner filed a motion for reconsideration, which was taken up by the First Division on July 9, 1986 with Justice Abad Santos still the Chairman. This time, Justice Yap, realizing that his former partner, Atty. Ordonez, had submitted the pleadings for petitioner, inhibited himself and Justice Edgardo L. Paras was designated under Special Order No. 21, dated July 9, 1986, to sit in the Division in his place. The motion for reconsideration was denied with finality on July 9, 1986.

Justice Yap was designated Chairman of the First Division on July 14, 1986.

On August 7, 1986, petitioner asked leave to file a second motion for reconsideration, which was denied on September 3, 1986, entry of judgment of the May 14, 1986 resolution having been made on July 28, 1986. Justice Yap again took no part in the deliberation of the case.

But respondents continue to claim derisively that Justice Yap could not have been "unaware" of the appearance of Atty. Sedfrey Ordonez. They reacted by saying "ten it to the marines" (Letters of November 3, 1986 to Justices Narvasa, Herrera, and Cruz, at p. 8, respectively). But that was the true and untarnished fact. With so many cases being handled by the Court, the appearances of lawyers during deliberative sessions very often escape attention, concentration being centered on the issues to be resolved.

Respondents also fault the Court for "still recogniz(ing) Atty. Ordonez as counsel" for their opponents in the case. In the same " Banc Resolution," it was clarified:

A copy of the resolution, dated May 14, 1986, was sent by the Releasing Clerks to Atty. Sedfrey A. Ordonez as his name still appears on the cover page of the Rollo. It was not necessarily because the Supreme Court still recognizes him as counsel for respondents (at p. 4)

The fact of the matter is that even Atty. Laureta continued to recognize Atty. Ordonez as counsel as shown by his pleadings filed before the Court, which inevitably contained the notation "copy furnished Atty. Sedfrey Ordonez." No withdrawal of appearance having been presented by Atty. Ordonez in the main petition, his name continues to be in the Rollo of the case and the personnel concerned continue to furnish him with copies of Resolutions of this Court.

In respect of the charge that the Resolutions of the First Division of May 14, 1986, July 9, 1986 denying the Motion for Reconsideration with finality, and September 3, 1986 denying leave to file a second motion for reconsideration since entry of judgment of the May 14, 1986 Resolution had been made on July 28, 1986, were "unjust" and were "railroaded," the Banc Resolution, adopting the Division Resolution, explained:

The aforesaid resolutions were by no means 'railroaded.' The pleadings filed by the parties, as in any other case, were included in the Agenda of the First Division as soon as feasible. The Division acts promptly on all Agenda items, and the minutes of its deliberations are released as soon as possible after Agenda day.

xxx xxx xxx

Page 347: Warrantless Arrests, Searches and Seizures and Privacy of Communication

347

The dispositions in this case were arrived at after careful study. Because a case is resolved against the interests of a party, does not mean that it is an 'unjust decision;' or that it has been "railroaded."

This Division declares without hesitation that it has consistently rendered justice without fear or favor. (at p. 4)

Respondents insist that the doctrine of "res judicata" is inapplicable. In their own words "the ordered probate of the 1944 Will of Digna Maravilla by judgment of the Supreme Court in G.R. No. L-23225 is conclusive only as to the genuineness and due execution of said will but not upon the validity of testamentary provision, particularly with the invalid designation of Herminio Maravilla as sole and universal heir of Digna Maravilla."

On this point, the "Javellana Resolution," in reversing the Busran Decision" AC-GR SP No. 13680), aptly held:

The then Court of Appeals held that the questioned decision does not run counter to the decision of the Hon. Supreme Court in G.R. No. L-23225 admitting the will of Digna Maravilla to probate because the latter refers to the extrinsic validity of the will while the former concerns its intrinsic validity. We cannot agree with this observation because it is quite clear from the questioned decision that the will was in effect declared not to have been freely and voluntarily executed by the deceased Digna Maravilla but was the result of the evil and fraudulent machinations of her husband, Herminio Maravilla, and sets aside said will The declaration that private respondents, as collateral relatives of the deceased Digna Maravilla, are entitled to her estate, is an indication that the Hon. respondent Court has nullified the will. Private respondents are not compulsory heirs and, in the absence of their being named legatees or devisees in the will, they could only lay claim to the estate of Digna Maravilla if the latter died without a will, pursuant to Art. 1003 of the New Civil Code, to wit:

Art. 1003. If there are no descendants ascendants, illegitimate children or a surviving spouse, the collateral relatives shall succeed to the entire estate of the deceased in accordance with the following articles.'

However, assuming arguendo, that the matter complain d of by private respondents referred only to the intrinsic validity of the will, still, it was improper for them to have instituted a separate action in a court other than that in which the probate proceeding was pending.

xxx xxx xxx

It seems clear from these provisions of the law that while the estate is being settled in the Court of First Instance in a special proceeding, no ordinary action can be maintained in that court or in any other court by a person claiming to be the heir, against the executor or against other persons claiming to be heirs, for the purpose of having the rights of the plaintiffs in the estate determined The very purpose of the trial or hearing provided for in section 753 is to settle and determine those questions, and until they are settled and determined in that proceeding and under that section no action such as the present one can be maintained.

Considering that the "Escolin Decision, " as affirmed by this Court on November 9, 1977 in G.R. No. L-46155, had become final, the "Javellana Resolution" aptly observed:

Page 348: Warrantless Arrests, Searches and Seizures and Privacy of Communication

348

3. The questioned decision of the Hon. respondent Court dated 12 August 1981 (referring to the "Macandog Decision") unsettles and reviews issues which had long been laid to rest by the Hon. Supreme Court and the then Court of Appeals.

But respondents ask: if res judicata were applicable, why did this Court, in G.R. No. L-50814, refer the case to the Court of Appeals? The answer is simple. The issue of whether the remedy of petitioners' in that case was appeal and not certiorari had to be resolved. If certiorari were proper, then the "Macandog Decision" had not become final. If appeal, its finality would be the consequence. The "Javellana Resolution," which -reversed the Busran Decision," held that certiorari was proper when a party has been illegally declared in default. It follows that the "Macandog Decision" had not attained finality.

Still undaunted, respondents claim that the Court of Appeals "deliberately evaded divaricated" two important issues: (1) that the judgment of the Trial Court (in CC No. X-404) had attained finality as in fact the Court of Appeals had held that the "judgment of assail had long become final," and (2) that Digna Maravilla's husband could not be instituted as the sole and universal heir of the wife on indestructible ground of moral impossibility and could not inherit wife's vast estate on the ground of utter unworthiness.

The penchant of respondents for making misleading statements is again obvious. It was not in the "Javellana Resolution" that the Court of Appeals held that "the judgment of assail (referring to the 'Macandog Decision') had long become final." That was in the BurRan Decision," which was precisely reversed by the " Javellana Resolution."

As to the alleged unworthiness of the husband to inherit from his wife, the "Javellana Resolution" pointedly observed:

The last will and testament of Digna Maravilla which instituted her husband, Herminio Maravilla, as her sole and universal heir, was admitted to probate, pursuant to a final judgment of the Hon. Supreme Court in G.R. No. L-23225, 27 February 1971. This probate foreclosed all questions as to the age and mental capacity of the testator, the signing of the document by the testator, or by someone in his behalf, and the acknowledgment of the instrument by him in the presence of the required member of witnesses who affix their signatures to the will to attest the act. In re Estate of Johnson, 39 Phil. 156, 168). Yet, more than ten years later, the Hon. respondent Court would nullify the effects of the probate by declaring that Digna Maravilla did not voluntarily and sanely execute the probated last will and testament, unifying the institution of Herminio Maravilla as her sole and universal heir, and ordering the return of the properties of Digna Maravilla to the trunk of origin.

The soundness of the legal conclusions arrived at in the "Escolin Decision" and "Javellana Resolution" commends itself. Only a disgruntled litigant and a defeated lawyer would claim that those judgments were accepted "hook, line and sinker" by this Court. The doctrine of res judicata is inescapably applicable. Thus it was that the First Division, in its challenged Resolution of May 14, 1986, found it unnecessary, after further study, to have a signed Decision and, instead, recalled the due course Order, which it had previously issued to give it "more time for further study" (p. 2, Banc Resolution, October 28, 1986). Contrary to respondents' claim, the Court is not "duty bound" to render signed Decisions all the time. It has ample discretion to formulate Decisions and/or minute Resolutions, provided a legal basis is given, depending on its evaluation of a case.

But obdurately enough, respondents have seen fit to take their case to the Tanodbayan charging the members of the First Division of this Court collectively with having knowingly and deliberately rendered an "unjust extended minute Resolution" with deliberate bad faith in violation of Article 204 of the Revised Penal Code 3 and for deliberately causing "undue injury" to respondent Ilustre and her co-heirs because of the 11 unjust Resolution" promulgated, in violation of the AntiGraft and Corrupt Practices Act.  4

Page 349: Warrantless Arrests, Searches and Seizures and Privacy of Communication

349

Respondents' action is brazenly unjustifiable. Nor can they plead ignorance. As aptly declared in the Chief Justice's Statement of December 24, 1986, which the Court hereby adopts in toto, "(I)t is elementary that the Supreme Court is supreme the third great department of government entrusted exclusively with the judicial power to adjudicate with finality all justiciable disputes, public and private. No other department or agency may pass upon its judgments or declare them "unjust." " It is elementary that "(A)s has ever been stressed since the early case of Arnedo vs. Llorente (18 Phil. 257, 263 [1911]) "controlling and irresistible reasons of public policy and of sound practice in the courts demand that at the risk of occasional error, judgments of courts determining controversies submitted to them should become final at some definite time fixed by law, or by a rule of practice recognized by law, so as to be thereafter beyond the control even of the court which rendered them for the purpose of correcting errors of fact or of law, into which, in the opinion of the court it may have fallen. The very purpose for which the courts are organized is to put an end to controversy, to decide the questions submitted to the litigants, and to determine the respective rights of the parties." (Luzon Brokerage Co., Inc. vs. Maritime Bldg., Co., Inc., 86 SCRA 305, 316-317)

Respondents should know that the provisions of Article 204 of the Revised Penal Code as to "rendering knowingly unjust judgment" refer to an individual judge who does so "in any case submitted to him for decision" and even then, it is not the prosecutor who would pass judgment on the "unjustness" of the decision rendered by him but the proper appellate court with jurisdiction to review the same, either the Court of Appeals and/or the Supreme Court. Respondents should likewise know that said penal article has no application to the members of a collegiate court such as this Court or its Divisions who reach their conclusions in consultation and accordingly render their collective judgment after due deliberation. It also follows, consequently, that a charge of violation of the AntiGraft and Corrupt Practices Act on the ground that such a collective decision is "unjust" cannot prosper.

The Chief Justice's Statement of the supremacy of the Supreme Court's judicial power is by no means a "display of arrogance" as per respondents' puerile contention, but a restatement of the fundamental principle of separation of powers and checks and balances under a republican form of government such as ours, viz. that the three co-equal branches of government, the executive, legislative and judicial, are each supreme and independent within the limits of its own sphere Neither one can interfere with the performance of the duties of the other. (Forbes vs. Chuoco 16 Phil. 534 [1910]). As restated by the late Justice Jose P. Laurel in the 1936 landmark case of Angara vs. Electoral Commission (63 Phil. 134), our Constitution "as a definition of the powers of government"placed upon the judiciary the great burden of "determining the nature, scope and extent of such powers" and 'when the judiciary mediates to allocate constitutional boundaries, it does not assert any superiority over the other departments . . . but only asserts the solemn and sacred obligation entrusted to it by the Constitution to determine conflicting claims of authority under the Constitution and to establish for the parties in an actual controversy the rights which the instrument secures and guarantees to them.' "

As an officer of the Court, respondent Laureta, should realize that the cardinal principle he would grossly impair and violate is that of the independence of the judiciary, which the members of the bar are called upon to defend and preserve. The independence of the judiciary is the indispensable means for enforcing the supremacy of the Constitution and the rule of law.

To subject to the threat and ordeal of investigation and prosecution, a judge, more so a member of the Supreme Court for official acts done by him in good faith and in the regular exercise of official duty and judicial functions is to subvert and undermine that very independence of the judiciary, and subordinate the judiciary to the executive. "For it is a general principle of the highest importance to the proper administration of justice that a judicial officer in exercising the authority vested in him, shall be free to act upon his own convictions, without apprehension of personal consequences to himself. Liability to answer to everyone who might feel himself aggrieved by the action of the judge would be inconsistent with the possession of this freedom, and would destroy that independence without which no judiciary can be either respectable or useful." (Bradley vs. Fisher, 80 U.S. 335).

Page 350: Warrantless Arrests, Searches and Seizures and Privacy of Communication

350

Indeed, resolutions of the Supreme Court as a collegiate court, whether en banc or division, speak for themselves and are entitled to full faith and credence and are beyond investigation or inquiry under the same principle of conclusiveness of enrolled bills of the legislature. (U.S. vs. Pons 34 Phil 729; Gardiner, et al. vs. Parades, et al., 61 Phil. 118; Mabanag vs. Lopez Vito, 78 Phil. 1) The Supreme Court's pronouncement of the doctrine that "(I)t is well settled that the enrolled bill . . . is conclusive upon the courts as regards the tenor of the measure passed by Congress and approved by the President. If there has been any mistake in the printing of the bill before it was certified by the officers of Congress and approved by the Executive [as claimed by petitioner-importer who unsuccessfully sought refund of margin fees] — on which we cannot speculate, without jeopardizing the principle of separation of powers and undermining one of the cornerstones of our democratic system — the remedy is by amendment or curative legislation, not by judicial decree" is fully and reciprocally applicable to Supreme Court orders, resolutions and decisions, mutatis mutandis (Casco Phil. Chemical Co., Inc. vs. Gimenez, 7 SCRA 347, 350. (Citing Primicias vs. Paredes, 61 Phil. 118, 120; Mabanag vs. Lopez Vito, 78 Phil. 1; Macias vs. Comelec, 3 SCRA 1)

The Court has consistently stressed that "the doctrine of separation of powers calls for the executive, legislative and judicial departments being left alone to discharge their duties as they see fit (Tan vs. Macapagal, 43 SCRA 677). It has thus maintained in the same way that the judiciary has a right to expect that neither the President nor Congress would cast doubt on the mainspring of its orders or decisions, it should refrain from speculating as to alleged hidden forces at work that could have impelled either coordinate branch into acting the way it did. The concept of separation of powers presupposes mutual respect by and between the three departments of the government. (Tecson vs. Salas, 34 SCRA 275, 286-287)

To allow litigants to go beyond the Court's resolution and claim that the members acted "with deliberate bad faith" and rendered and "unjust resolution" in disregard or violation of the duty of their high office to act upon their own independent consideration and judgment of the matter at hand would be to destroy the authenticity, integrity and conclusiveness of such collegiate acts and resolutions and to disregard utterly the presumption of regular performance of official duty. To allow such collateral attack would destroy the separation of powers and undermine the role of the Supreme Court as the final arbiter of all justiciable disputes.

Dissatisfied litigants and/or their counsels cannot without violating the separation of powers mandated by the Constitution relitigate in another forum the final judgment of this Court on legal issues submitted by them and their adversaries for final determination to and by the Supreme Court and which fall within the judicial power to determine and adjudicate exclusively vested by the Constitution in the Supreme Court and in such inferior courts as may be established by law.

In resume, we find that respondent Ilustre has transcended the permissible bounds of fair comment and criticism to the detriment of the orderly administration of justice in her letters addressed to the individual Justices quoted in the show-cause Resolution of this Court en banc, particularly the underlined portions thereof; in the language of the charges she filed before the Tanodbayan quoted and underscored in the same Resolution; in her statements, conduct, acts and charges against the Supreme Court and/or the official actions of the Justices concerned and her ascription of improper motives to them; and in her unjustified outburst that she can no longer expect justice from this Court. The fact that said letters are not technically considered pleadings, nor the fact that they were submitted after the main petition had been finally resolved does not detract from the gravity of the contempt committed. The constitutional right of freedom of speech or right to privacy cannot be used as a shield for contemptuous acts against the Court.

We likewise find that Atty. Laureta has committed acts unbecoming an officer of the Court for his stance of dangling threats of bringing the matter to the "proper forum" to effect a change of the Court's adverse Resolution; for his lack of respect for and exposing to public ridicule, the two highest Courts of the land by challenging in bad faith their integrity and claiming that they knowingly rendered unjust judgments (Montecillo vs. Gica 60 SCRA 234 [1974]); for authoring, or at the very least, assisting and/or abetting and/or not preventing the contemptuous statements, conduct, acts and malicious charges of his client,

Page 351: Warrantless Arrests, Searches and Seizures and Privacy of Communication

351

respondent Ilustre, notwithstanding his disclaimer that he had absolutely nothing to do with them, which we find disputed by the facts and circumstances of record as above stated; for totally disregarding the facts and circumstances and legal considerations set forth in this Court's Resolutions of the First Division and en banc, as the Tribunal of last resort; for making it appear that the Justices of this Court and other respondents before the Tanodbayan are charged with "graft and corruption" when the complaint before the Tanodbayan, in essence, is a tirade from a disgruntled litigant and a defeated counsel in a case that has been brought thrice before this Court, and who would readily accept anything but the soundness of the judgments of the Courts concerned, all with the manifest intent to bring the Justices of this Court and of the Court of Appeals into disrepute and to subvert public confidence in the Courts.

Atty. Laureta should be reminded that his first duty is not to his client but to the administration of justice; to that end, his client's success is wholly subordinate; and his conduct ought to and must always be scrupulously observant of law and ethics. For like the Court itself, "a lawyer is an instrument or agency to advance the ends of justice." (Surigao Mineral Conservation Board vs. Cloribel, 31 SCRA 1 [1970]; Castaneda vs. Ago, 65 SCRA 505 [1975[).

In assessing the penalty on respondent Laureta, the Court notes that "disciplinary proceedings against lawyers are sui generis. Neither purely civil nor purely criminal, they do not involve a trial of an action or a suit, but are rather investigations by the Court into the conduct of one of its officers. Not being intended to inflict punishment, it is in no sense a criminal prosecution. Accordingly, there is neither a plaint nor a prosecutor therein. It may be initiated by the Court motu proprio. Public interest is its primary objective, and the real question for determination is whether or not the attorney is still a fit person to be allowed the privileges as such. Hence, in the exercise of its disciplinary powers, the Court merely calls upon a member of the Bar to account for his actions as an officer of the Court with the end in view of preserving the purity of the legal profession and the proper and honest administration of justice by purging the profession of members who by their misconduct have proved themselves no longer worthy to be entrusted with the duties and responsibilities pertaining to the office of an attorney." Viewed in the light of the demonstrated persistence of grave misconduct and undermining public confidence in the honor and integrity of the Court and its members (at a time when the Court is exerting every effort to regain public confidence in our courts after the trauma and debacle undergone by them in the past regime), the Court shall impose upon him an indefinite suspension, leaving it to him to prove at some future and opportune time, that he shag have once again regained the fitness to be allowed to resume the practice of law as an officer of the Courts. (In re: Almacen, 31 SCRA 562)

ACCORDINGLY, (1) respondent Eva Maravilla Ilustre is hereby held in contempt, and is hereby fined in the amount of P1,000.00 only, mindful that the power of contempt should be exercised on the preservative and not on the vindictive principle of punishment; and

(2) Atty. Wenceslao Laureta is found guilty of grave professional misconduct, rendering him unfit to continue to be entrusted with the duties and responsibilities belonging to the office of an attorney, and is hereby suspended from the practice of law until further Orders, the suspension to take effect immediately.

Let copies of this Resolution be circulated to all Courts of the country for their information and guidance, and spread in the personal record of Atty. Wenceslao Laureta.

SO ORDERED.

Teehankee, C.J., Fernan, Narvasa, Melencio-Herrera, Alampay, Gutierrez, Jr., Cruz, Paras, Feliciano, Gancayco, Padilla, Bidin, Sarmiento and Cortes, JJ., concur.

Yap, J., took no part.

Page 352: Warrantless Arrests, Searches and Seizures and Privacy of Communication

352

G.R. No. L-69377               July 20, 1987

PEOPLE OF THE PHILIPPINES, plaintiff-appellee, vs.ALEXANDER ALBOFERA and ROMEO LAWI-AN, accused-appellants.

MELENCIO-HERRERA, J.:

Mandatory review of the Decision of the Regional Trial Court, Branch XVIII, Digos, Davao del Sur, in Criminal Case No. 184.* which convicted accused Alexander Albofera and Romeo Lawi-an of Murder, inflicted on them the capital punishment, and ordered them to indemnify the heirs of the victim in the amount of P35,000.00, "by way of moral as well as actual damages."

Page 353: Warrantless Arrests, Searches and Seizures and Privacy of Communication

353

There is no direct evidence linking both accused to the crime charged, their alleged participation therein having been found by the Trial Court to have been proved by circumstantial evidence adduced by the prosecution as follows:

Sometime in June or July 1980, at about 4:30 o'clock in the afternoon. Rodrigo Esma was tending his onion farm located in Upper Bagong Silang, Managa, Bansalan, Davao del Sur, near the place of the accused Romeo Lawi-an, a long time acquaintance, when accused Alexander Albofera, whom Esma also knew for years and likewise a resident of the same place, called him and informed him they would ran after somebody. Esma acceded (tsn, October 20, 1982, pp. 38-42). Together, Albofera and Esma proceeded at once to the house of accused Lawi-an. There Lawi-an told Albofera that the forester was around making a list of people engaged in "caingin" (tsn, October 20, 1982, p. 43). Whereupon, Albofera asked Esma to join him in going after the forester. The two were able to overtake the forester, a certain Teodoro Carancio, at the lower portion of the road (tsn, October 20, 1982, p. 44). Albofera at once put his arm on the shoulder of Carancio and asked him to go with them to the upper portion because they will do something there. Carancio was taken to the house of accused Lawi-an where several persons were already gathered, among whom were accused Lawi-an, a certain alias Jun, Boy Lawi-an, and Joel Maldan. Once inside and seated, Albofera began questioning Carancio about his purpose in the place. Carancio replied that he was there to inspect the "caingin" as a forester Albofera resented this, telling Carancio that his acts hurt the poor people who were making a plain and simple living. Carancio answered that he was only complying with the orders of the government. Albofera then asked Carancio should he be set free not to come back anymore. Carancio, however, replied that he would still come back and bring his nephew who is an army man. Thereupon, Albofera intimated to Carancio that he is a member of the NPA and that the NPA's were against the forestry personnel. Thereafter, the persons gathered decided to kin Carancio. Right then and there, Albofera tied Carancio's hands at the back. Carancio pleaded for mercy. Unheeding, Albofera, Romeo Lawi-an, alias Jun, Boy Lawi-an, and Joel Maldan decided to bring, and they indeed brought, Carancio to the forest some 200 meters away from Lawi-an's house. Esma did not join the group but remained in the house of Lawi-an (tsn, October 20, 1982, pp. 44-51). Not long after, the group returned to Lawi-an's house, but without Carancio. Albofera's hands, as wen as alias Jun's hands were bloodied. After washing their hands, Albofera announced that they had already finished the killing. He also warned everyone, particularly Esma, against revealing or saying anything to any person or the military, otherwise he (Albofera) would hold him accountable. After that, E smaller went home (tsn, October 20, 1982, pp. 52-54).

Meanwhile, at about the same time Efren Sisneros and his wife were weeding their farm in Barangay Buenavista, Bansalan, which is adjacent to Bagong Silang, Managa, also in Bansalan, when the son of accused Lawi-an, who is his compadre, arrived and informed him that his father (the accused) wanted him (Sisneros) in his house. So, Sisneros went with Lawi-an's son (tsn, September 16, 1982, pp. 3-7, 11). On reaching the front yard of the Lawi-ans, Sisneros saw the accused Lawi-an at the window. A lot of people were likewise in the house, and he recognized Boy Lawi-an and a certain Jun Menez among others. Sisneros called for accused Lawi-an. The latter went down and they talked downstairs. Accused Lawi-an explained that he had Sisneros fetched, because the people inside the house were discussing on what to do with somebody — a Bureau of Forestry employee-later on Identified as Teodoro Carancio — who was also inside the house at the time, and that they were inclined to kill that person who, according to Lawi-an, was a hindrance to the farmers, because he (the forester) had caused Lawi-an's uncle and brother-in-law to be put in jail and fined for cutting trees in the forest. Shocked, Sisneros could only say "do not do that because killing a person is great sin toward God." Thereafter, accused Lawi-an went upstairs. Sisneros who was left downstairs went home (tsn, September 16, 1982, pp. 11-16, 21).

The following day, at about 9:00 o'clock in the morning, Sisneros was at his farm when accused Lawi-an and Jun Menez passed by and called him. When Sisneros got near the two, accused Lawi-an told him that the forester was already killed and warned him not to reveal this matter to anybody otherwise he would be killed (tsn, September 16, 1982, pp. 16-20.)

Page 354: Warrantless Arrests, Searches and Seizures and Privacy of Communication

354

The threat to his life caused Sisneros to be cautious in not reporting at once the matter to the authorities. However, in June 1981, Sisneros finally reported the killing of that forester to his brother Margarito, a CHDF member in Bansalan. Margarito then accompanied him to the municipal hall to see the Chief of Police, P/Sgt. Arnulfo Gohol. Sisneros related the killing to Sgt. Gohol. That forester must have already been reported missing, for Sgt. Gohol told Sisneros that the slain forester was Teodoro Carancio. Sisneros asked that his Identity be kept secret in the meantime pending the arrest of Albofera and Lawi-an. Sgt. Gohol acceded (tsn. September 16, 1982, pp. 19-20, 21-22).

The police authorities arrested accused Albofera on July 2, 1981. ...

x x x           x x x          x x x

Accused Romeo Lawi-an was subsequently arrested on July 4, 1981 (pp. 12, 15, Record).

Also in July, 1981, the two accused, shortly after their arrest, led the police authorities to the place in Bagong Silang where they buried the slain forester, specifically in a hilly portion near the forest where the trees were not quite big besides a coffee plantation (tsn, January 6, 1983, pp, 84-87). And on the very spot pointed to by the two accused, the authorities dug and recovered the cadaver, together with the clothings, namely: a maroon sweater, a semi-green trousers and fatigue briefs worn by the victim, still intact. After placing these in a sack, the group left at about 6:00 o'clock p.m. and returned to the municipal building at around 10:00 p.m. (tsn, January 6, 1983, pp. 87-89, 94-101).

x x x           x x x          x x x 1

On July 2, 1981, Albofera executed an extra-judicial confession before the Municipal Circuit Judge. He stated therein that he was forced to join the NPA movement for fear of his life; that said group had ordered the "arrest" of the victim, Carancio, a Forest Guard in the National Park, because he was "a very strict employee of the government who arrested several kaingeros already in the National Park and Romeo Lawi-an being one of his victims before, got mad of his actuations prompting the latter to report said person to the NPA for possible punishments;" and that the group "sentenced him (the victim) to die by stabbing." Albofera further declared:

Q. 21- Was said Carancio killed by the group?

A. Yes, sir, Carancio was stabbed to death by alias John, Romeo Lawi-an, alias "Dolly" Fred, Albert and myself in succession.

Q. 22- Do you mean to say that you have participated in stabbing Carancio to death?

A. I was the last to thrust said bolo but I know that said victim was already dead when I did it.

Accused for their part, maintain:

That in or about the middle of 1980, both the accused-appellants ALEXANDER ALBOFERA and ROMEO LAWI-AN, who were farmer-residents at that time of Sitio Balutakay, Barangay Mansaga, a remote settlement in the Municipality of Bansalan, Davao del Sur, were fetched from their farm houses by four (4) persons, known to be NPA elements operating in their locality and Identified with their aliases "Fred", "Dolly", "John", and "Albert". Both accused together with Efren Sisneros and Rodrigo Esma were brought by the said four (4) NPA's to a secluded forested area in order to witness the execution of a man, whom the prosecution later claimed to be one Teodoro Carancio, an employee of the Bureau of Forest Development. Thereafter, both the

Page 355: Warrantless Arrests, Searches and Seizures and Privacy of Communication

355

accused, Efren Sisneros and Rodrigo Esma were ordered by these armed NPA to bury the remains of the victim. Afterwards, they were warned, with threat to their lives, not to reveal or report to the government authorities. 2

In the course of the trial, the prosecution presented a letter written in the Visayan dialect by accused Alexander Albofera, while under detention, to witness Rodrigo Esma several days before the latter testified on October 20, 1982, which was translated into English by the Trial Court interpreter and reads as follows:

10-5-82

Dear Odeng,

Ding first of all how are you are in good health. As to me if you will also ask me I am here suffering from hardship, so that Ding, help me that I can get out in this difficult condition because your affidavit is the one that has weight. Ding, you go with Mining to my lawyer so that I can study your reason because I will ride if what is your affidavit. If you will not appear before the lawyer it will mean that you will pushed us. Ding, you know about this incident and that I do not want this to happen but you were the one persisting in fact I asked you and you acceded so that that happened. But now you are going to put us down will you not pity the uncle of your wife and furthermore you were not threatened by me we have agreed about this thing now you will free yourself. Ding you must bear in mind that you are a part of this if that will happen to me I will include you so that we will be together in jail anyway your affidavit is there that you are one of those who apprehended the forestry and Noy Roming will testify that no one threatened us and also according to him that he will declare that the two of us apprehended so that the three of us win be convicted. Ding why is it that we are not going to understand one another so that we will not be hard-up we have still a way that will be taught by my lawyer in which we have nothing to fear each one of us. Anyway you can still be a witness of the other side my lawyer wants to know only your reason so that he can study this in order that I ran prepare and ride on it so that you will not be included and I can also get out from this case because if you will not make any arrangement our reasons will contradict with each other even though we have exculpated you we will instead be together here if how many years will be my sentence yours will also be the same because I will include you anyway you were present in the incident nobody force us nobody can witness that you were force by me because that is not the truth. And Ding, I have not squeal because it's difficult the other side no jail it’s better for the government because we will just be imprisoned you are the one who win know how to understand we win help one another in order that no hazard of both of us anyway you will not be imprisoned of this you will only help me in order that I can get out here. This is our agreement, is it not.

I hope you will remember our being together before we are very close but now because we have a misunderstanding but our complainant you do not even know him will they look back at you after this.

Your mercy

(Sgd.) Alex Albofera. 3

Rodrigo Esma's Affidavit referred to in the letter taken on July 21, 1981, mentioned accused "Albofera and "alias Jun" "as having killed the victim.

After trial, the lower Court found the circumstantial evidence sufficient to warrant conviction beyond reasonable doubt of both accused for the crime charged, and sentenced them to death in its Decision of October 5, 1984, now before us.

The accused raise the following errors:

Page 356: Warrantless Arrests, Searches and Seizures and Privacy of Communication

356

I

That the Regional Trial Court of Davao del Sur gravely erred in finding both accused guilty of murder, as charged in the information, and ordaining a sentence of death, instead of dismissing the charge anchor absolving them as the entire proceedings on the case in the Municipal Court of Bansalan up to rendition of judgment in the CFI of Davao del Sur are void ab initio and a nullity for being tainted with serious illegalities and jurisdictional infirmities as from the inception of appellant's illegal arrest, tortures, and detention without bait their fundamental constitutional and human rights were blatantly violated, brazenly trampled upon and utterly reduced to naught.

II

That the Regional Trial Court of Davao del Sur gravely erred-in failing to consider at all the prosecution's evidence on record, which reasonably raises doubt upon the conclusiveness of the bases as to the supposed victim's (1) Identity; (2) his alleged fact of death as stated in the certificate of death [Exh. D]; (3) the place of death and, the approximate or credible date of death; and-consequently, in not finding that on the basis of the above-factual hiatus, the evidence has failed to establish the guilt of both accused beyond that quantum of reasonable doubt as zealously mandated by the constitution.

III

That the Court a quo erred in holding that evidence adduced against accused-appellants conceded to be merely circumstantial in character and confirmed as such in the appealed decision, has attained such degree of proof and weight of moral persuasion as to leave no vestige of reasonable doubt on the guilt of both accused.

IV

That the Court a quo erred in appreciating as competent evidence the letter written by accused Alexander Albofera to Rodrigo Esma (Exh. B), the admissibility thereof being specifically excluded under Sec. 4, Art. IV of the 1973 Constitution.

V

That the Regional Trial Court of Davao del Sur gravely erred in admitting and considering as competent evidence the illegally extracted extra-judicial confession of accused Alexander Albofera (Exh. C) in violation of, and contrary to Sec. 20, Art. IV of the Philippine Constitution of 1973 and the Supreme Court's judicial precedents in point.

VI

That the Trial Court gravely erred (1) in sustaining the prosecution's theory that both accused-appellants were responsible and culpable for the killing of the alleged victim; (2) in according credence to the testimonies of prosecution's witnesses Efren Sisneros and Rodrigo K. Esma; (3) in failing to sustain the defense theory; and (4) in convicting and sentencing both accused-appellants.

VII

Furthermore, the lower court gravely incurred the following patent reversible errors: (1) in finding aggravating and qualifying circumstances in the alleged commission of murder, and (2) in not absolving the two accused-appellants, and awarding damages. 4

Page 357: Warrantless Arrests, Searches and Seizures and Privacy of Communication

357

Succintly stated, the essential issues posed are:

1) Whether or not "serious illegalities and jurisdictional infirmities," in fact, attended the proceedings below and "constitutional and human rights of the accused brazenly trampled upon."

2) Whether or not the extrjudicial confession of accused Alexander Albofera, and his letter to Rodrigo Esma are admissible in evidence;

3) Whether or not the Identity of the victim and the fact of his death were duly proved;

4) Whether or not the circumstantial evidence adduced is sufficient to warrant conviction; and

5) Whether or not qualifying and aggravating circumstances were duly proved.

On Irregularities alleged:

The charge of illegalities and infirmities is absolutely without basis. There was nothing illegal in the accused's detention without bail. They were charged with and held for the crime of murder, a capital offense and, therefore, were not entitled to bail where the evidence of guilt was strong. That was for the Trial Court to evaluate. The preliminary investigation was far from being "hasty and farcical." If the second stage thereof was not held it was because the accused had waived the same and prayed for the transmittal of the case to the then Court of First Instance for trial on the merits; it was not because they were deprived of the right. Much less has due process been denied the accused. They were duly informed of the charge against them and they were given fun opportunity to interpose and prove their defense.

On the Admissibility of the Extra-Judicial Confession of Accused A Alexander Albofera:

The preliminary questions addressed to said accused when his Sworn Statement was taken read:

P R E L I M I N A R Y:

Mr. Albofera, I am reminding you that you are now under investigation in connection with the commission of an offense, but before I will proceed in it, I would like to inform you that under the Constitution of the Philippines it is so provided that you have the right to remain silent, the right to counsel of your own choice to be present with you while being investigated, the right to self-incrimination and the right to due process, do you understand this:

Answer — Yes sir, I do.

I would like to inform you further, that the manner in which this investigation will be conducted in English, but however, the contents of the same will be interpreted to you in dialect you fully understand and speak, and if you choose to answer one of the question or questions propounded to you, your answer will be reduced into writing and the same will be used in evidence against you or to your favor in any court of justice in the country, do you understand what I am explaining to you?

Answer — Yes. sir, I understand it because you explained it to me clearly.

Do you need then the assistance of counsel to assist you while investigated?

Page 358: Warrantless Arrests, Searches and Seizures and Privacy of Communication

358

Answer — I think I do not need any yet this time because I know what I am going to declare here it being the truth of the matter, sir.

Since you do not (have) any lawyer yet, are you willing to proceed with this investigation and submit yourself freely into it?

Answer — I wish that this investigation will be continued because lawyer is not necessary yet.

Are you willing to swear and sign this statement of yours freely to justify that your submission into the said investigation is free and voluntary?

Answer — Yes sir, I will sign it if only to prove that all what I have stated are true and to the best of my knowledge and ability.

(Sgd.) Alex Albofera

(Exhibits "C", "C-1").

Judicial precedents5 have laid down the rule that the foregoing form of questioning, does not satisfy the Constitutional requirement that an accused be apprised of his constitutional rights to remain silent and to counsel. It is, at best, ceremonial and perfunctory, with the answers being mere formalisms put into the mouth of the affiant. What is contemplated is the transmission of meaningful information, comprehended by the person under investigation, not a mere recitation of the Constitutional mandates.

More, the extra-judicial confession was extracted without the assistance of counsel contrary to the rulings of this Court in Morales, Jr. vs. Enrile, No. L-61016, April 26, 1983, 121 SCRA 538, affirmed in People vs. Galit, No. L-51770, March 20, 1985, 135 SCRA 465, People vs. Burgos, L-68955, September 4, 1986, 144 SCRA 1, that "no custodial investigation shall be conducted unless it be in the presence of counsel engaged by the person arrested, by any person on his behalf, or appointed by the court upon petition either of the detainee himself or by anyone on his behalf."

While Albofera seemingly waived his right to counsel, which he is allowed to do, aside from the fact that we are not convinced that the waiver was voluntary, knowing and intelligent, the waiver was not valid because it was made without the assistance of counsel.6 That principle is now enshrined in the 1987 Constitution, which explicitly requires that the waiver be in writing and in the presence of counsel.7

For failure to meet such exacting standards, the extrajudicial confession of accused Albofera must be stricken out and held inadmissible in evidence against him.8

On the admissibility of Albofera's Letter.

Accused Albofera contends that his letter to prosecution witness, Rodrigo Esma (Exhibit "B"), is inadmissible in evidence against him under the exclusionary provisions of Section 4, Article IV of the 1973 Constitution (substantially reproduced in Section 3, Article III of the 1987 Constitution), which provides:

Sec. 4 (1) The privacy of communication and correspondence shall be inviolable except upon lawful order of the Court, or when public safety and order require otherwise.

2) Any evidence obtained in violation of this or the preceding section shall be inadmissible for any purpose in any proceeding.

Page 359: Warrantless Arrests, Searches and Seizures and Privacy of Communication

359

The submission is untenable. The foregoing provision implements another Constitutional provision on the security of a citizen against unreasonable search and seizure. The production of that letter by the prosecution was not the result of an unlawful search and seizure nor was it through unwarranted intrusion or invasion into Albofera's privacy. Albofera admitted having sent the letter and it was its recipient, Rodrigo Esma himself, who produced and Identified the same in the course of his testimony in Court. Besides, there is nothing really self-incriminatory in the letter. Albofera mainly pleaded that Esma change his declaration in his Affidavit and testify in his (Albofera's) favor. Furthermore, nothing Albofera stated in his letter is being taken against him in arriving at a determination of his culpability.

On the Identity of the Victim and the Fact of Death:

Accused argue that corpus delicti had not been established as the body of the victim, Teodoro Carancio, was not Identified, nor the fact, place and approximate date of his death established.

The term corpus delicti stands for the substance of the crime, the fact that a crime has actually been committed.9The evidence adduced in this case sufficiently proved the commission of the crime. In fact, the accused themselves pointed to the grave where the body of a person, allegedly slain in their presence, had been dumped and which, when dug, produced human remains, which turned out to be those of the victim.

The skeletal remains of the victim were Identified by his brother, Benjamin Carancio, through the victim's front teeth whose "base seemed rusty" and which bore resemblance to his own, as well as through the victim's clothes, fatigue briefs, maroon sweater and trousers, which Benjamin recognized.10 Prosecution witness Esma also Identified the victim from a photograph which was presented to him.11

On the Circumstantial Evidence:

Circumstantial evidence is admissible in the absence of an eyewitness to the commission of a crime, and it is sufficient for conviction if: (1) there is more than one circumstance; (2) the facts from which the inferences are derived are proven; and (3) the combination of all the circumstances is such as to produce conviction beyond reasonable doubt.12

The circumstances testified to by prosecution witnesses meet the foregoing criteria. Even disregarding Albofera's extra-judicial confession, the combination of circumstances sufficiently point to his guilt. The presence of both accused at the scene of the incident is admitted by them. They also admit that they witnessed the execution of the victim, although they claim that they were merely compelled to do so. The foregoing version, however, is negated by Rodrigo Esma's testimony from which it is clear that it was Albofera, his long-time friend, who had fetched witness Esma and informed the latter that they would run after somebody. Together, they proceeded to the house of accused Lawi-an who informed Albofera that the victim was around making a list of "kaingeros." Albofera asked Esma to join him in going after the forester. Overtaking the latter, Albofera took him to Lawi-an's house where a group had already congregated and a discussion followed as to the victim's fate. Albofera resented the victim's determination to do his duty and the latter's statement that he was bringing an army man to help him. Sufficient motive was provided to do away with the victim. Albofera tied the victim's hands and, with Lawi-an and three others, took the victim to the forest. When the group returned not long after, the victim was no longer with them. Witness Esma noticed Albofera's and "alias Jun's" hands bloodied. After they had washed their hands, Albofera announced to everyone present at Lawi-an's house that the victim had been done away with and warned everyone not to reveal the incident to anyone including the military.

Apparent from the foregoing narrated circumstances is the fact that it was Albofera who was "calling the shots;" that it was he who was the leader of the group and not "alias Jun" as he would want this Court to believe.

Page 360: Warrantless Arrests, Searches and Seizures and Privacy of Communication

360

Esma's testimony is worthy of credence. He was a friend of long standing of Albofera.lawphi1 There was no reason for him to attribute to Albofera the commission of such a serious crime as Murder, if such not the truth.

Accused Lawi-an must be held equally culpable. That he was part of the criminal design from its initial stages until its culmination is revealed through the circumstances brought out by prosecution witness, Sisneros who testified that while he was weeding his farm, Lawi-an, his "compadre," sent his son to fetch him (Sisneros). With the son, they proceeded to Lawi-an's house where Sisneros saw many people. Lawi-an went down the house and explained to Sisneros that they were discussing what to do with the victim, and that they were inclined to kill him. Sisneros advised against it and went home. The following morning, Lawi-an passed by Sisneros farm and informed the latter that the victim had already been killed with the warning to Sisneros not to reveal the incident to anyone.

Prosecution witness, Esma, further buttressed the fact of Lawi-an's participation in the criminal plot when he testified that it was Lawi-an who informed Albofera that the victim was around making a list of "kaingeros;" that it was on the strength of that information that Albofera coaxed Esma into joining him to search for the victim; that Lawi-an was with Albofera and three others who, starting from Lawi-an's house, took the victim to the forest and then returned thereafter without the victim, obviously because the latter had been done away with.

While the degree of actual participation by Lawi-an in committing the offense is not described with accuracy, Lawi-an's conduct before and after the commission of the crime shows that he acted in concert with his co-accused Albofera. He indubitably cooperated with the latter and three other persons in bringing about the death of the victim goaded by resentment against the latter for his strict enforcement of forestry laws, which led to the incarceration of Lawi-an's uncle and brother-in-law and the imposition of fines against them. The circumstances proven sufficiently establish a community of purpose-a conspiracy among the perpetrators — such that the crime committed in furtherance thereof must be held to be the act of all regardless of the extent and character of an accused's active participation.13

On the Attendance of Qualifying and Aggravating Circumstances:

No reversible error was committed by the Trial Court in appreciating the presence of qualifying and aggravating circumstances.

The killing of the victim was committed treacherously, his hands having been tied behind his back so that he was totally helpless and defenseless, and in no position to resist nor fight back. The accused employed means which tended directly to insure the execution of the crime without risk to themselves arising from the defense which the victim might have made.

Evident premeditation was likewise present as both accused and their co-conspirators had deliberately planned to commit the crime and had persistently and continuously followed it notwithstanding that they had ample time to reflect and allow their conscience to overcome their resolution to kill.14

The accused likewise took advantage of superior strength although this cannot be appreciated separately as it is deemed absorbed in treachery.15

The killing of the victim because of his strictness and the resentment against him as a forester constitutes the aggravating circumstance of disregard of the respect due the offended party on account of his rank,16 and not because the victim was engaged in the discharge of his duties under Article 14 (5) of the Revised Penal Code as found by the Trial Court.

With the attendance of the qualifying circumstance of treachery and two (2) generic aggravating circumstances with no mitigating circumstance to offset them, the crime committed is Murder and the death penalty imposed by the Trial Court is proper. However, with the abolition of the death penalty under

Page 361: Warrantless Arrests, Searches and Seizures and Privacy of Communication

361

Section 19(l), Article III of the 1987 Constitution, and as mandated therein the death penalty imposed by the Trial Court should be reduced toreclusion perpetua.

WHEREFORE, the judgment of conviction is hereby affirmed with modification that the accused Alexander Albofera and Romeo Lawi-an are hereby sentenced to suffer the penalty of reclusion perpetua, to indemnify the heirs of the victim, Teodoro Carancio, in the amount of P30,000.00, and each to pay one-half (1/2) of the costs.

SO ORDERED.

Teehankee, C.J., Yap, Fernando, Narvasa, Gutierrez, Jr., Cruz, Paras, Feliciano, Gancayco, Padilla, Bidin, Sarmiento and Cortes, JJ., concur.

[G.R. No. 107383. February 20, 1996.]

CECILIA ZULUETA, petitioner, vs. COURT OF APPEALS and ALFREDO MARTIN, respondents.

D E C I S I O N

MENDOZA, J.:

This is a petition to review the decision of the Court of Appeals, affirming the decision of the Regional Trial Court of Manila (Branch X) which ordered petitioner to return documents and papers taken by her from private respondent’s clinic without the latter’s knowledge and consent.

The facts are as follows:

Petitioner Cecilia Zulueta is the wife of private respondent Alfredo Martin. On March 26, 1982, petitioner entered the clinic of her husband, a doctor of medicine, and in the presence of her mother, a driver and private respondent’s secretary, forcibly opened the drawers and cabinet in her husband’s clinic and took 157 documents consisting of private correspondence between Dr. Martin and his alleged paramours, greetings cards, cancelled checks, diaries, Dr. Martin’s passport, and photographs. The

Page 362: Warrantless Arrests, Searches and Seizures and Privacy of Communication

362

documents and papers were seized for use in evidence in a case for legal separation and for disqualification from the practice of medicine which petitioner had filed against her husband.

Dr. Martin brought this action below for recovery of the documents and papers and for damages against petitioner. The case was filed with the Regional Trial Court of Manila, Branch X, which, after trial, rendered judgment for private respondent, Dr. Alfredo Martin, declaring him “the capital/exclusive owner of the properties described in paragraph 3 of plaintiff’s Complaint or those further described in the Motion to Return and Suppress” and ordering Cecilia Zulueta and any person acting in her behalf to immediately return the properties to Dr. Martin and to pay him P5,000.00, as nominal damages; P5,000.00, as moral damages and attorney’s fees; and to pay the costs of the suit. The writ of preliminary injunction earlier issued was made final and petitioner Cecilia Zulueta and her attorneys and representatives were enjoined from “using or submitting/admitting as evidence” the documents and papers in question. On appeal, the Court of Appeals affirmed the decision of the Regional Trial Court. Hence this petition.

There is no question that the documents and papers in question belong to private respondent, Dr. Alfredo Martin, and that they were taken by his wife, the herein petitioner, without his knowledge and consent. For that reason, the trial court declared the documents and papers to be properties of private respondent, ordered petitioner to return them to private respondent and enjoined her from using them in evidence. In appealing from the decision of the Court of Appeals affirming the trial court’s decision, petitioner’s only ground is that in Alfredo Martin v. Alfonso Felix, Jr.,1 this Court ruled that the documents and papers (marked as Annexes A-i to J-7 of respondent’s comment in that case) were admissible in evidence and, therefore, their use by petitioner’s attorney, Alfonso Felix, Jr., did not constitute malpractice or gross misconduct. For this reason it is contended that the Court of Appeals erred in affirming the decision of the trial court instead of dismissing private respondent’s complaint.

Petitioner’s contention has no merit. The case against Atty. Felix, Jr. was for disbarment. Among other things, private respondent, Dr. Alfredo Martin, as complainant in that case, charged that in using the documents in evidence, Atty. Felix, Jr. committed malpractice or gross misconduct because of the injunctive order of the trial court. In dismissing the complaint against Atty. Felix, Jr., this Court took note of the following defense of Atty. Felix, Jr. which it found to be “impressed with merit:”2

On the alleged malpractice or gross misconduct of respondent [Alfonso Felix, Jr.], he maintains that:

       xxx                                  xxx                                  xxx

4. When respondent refiled Cecilia’s case for legal separation before the Pasig Regional Trial Court, there was admittedly an order of the Manila Regional Trial Court prohibiting Cecilia from using the documents Annex “A-I to J-7.” On September 6, 1983, however having appealed the said order to this Court on a petition for certiorari, this Court issued a restraining order on aforesaid date which order temporarily set aside the order of the trial court. Hence, during the enforceability of this Court’s order, respondent’s request for petitioner to admit the genuineness and authenticity of the subject annexes cannot be looked upon as malpractice. Notably, petitioner Dr. Martin finally admitted the truth and authenticity of the questioned annexes. At that point in time, would it have been malpractice for respondent to use petitioner’s admission as evidence against him in the legal separation case pending in the Regional Trial Court of Makati? Respondent submits it is- not malpractice.

Significantly, petitioner’s admission was done not thru his counsel but by Dr. Martin himself under oath. Such verified admission constitutes an affidavit, and, therefore, receivable in evidence against him. Petitioner became bound by his admission. For Cecilia to avail herself of her husband’s admission and use the same in her action for legal separation cannot be treated as malpractice.

Thus, the acquittal of Atty. Felix, Jr. in the administrative case amounts to no more than a declaration that his use of the documents and papers for the purpose of securing Dr. Martin’s admission as to their genuiness and authenticity did not constitute a violation of the injunctive order of the trial court.   By no means does the decision in that case establish the admissibility of the documents and papers in question.

It cannot be overemphasized that if Atty. Felix, Jr. was acquitted of the charge of violating the writ of preliminary injunction issued by the trial court, it was only because, at the time he used the documents

Page 363: Warrantless Arrests, Searches and Seizures and Privacy of Communication

363

and papers, enforcement of the order of the trial court was temporarily restrained by this Court. The TRO issued by this Court was eventually lifted as the petition for certiorari filed by petitioner against the trial court’s order was dismissed and, therefore, the prohibition against the further use of the documents and papers became effective again.

Indeed the documents and papers in question are inadmissible in evidence. The constitutional injunction declaring “the privacy of communication and correspondence [to be] inviolable”3 is no less applicable simply because it is the wife (who thinks herself aggrieved by her husband’s infidelity) who is the party against whom the constitutional provision is to be enforced. The only exception to the prohibition in the Constitution is if there is a “lawful order [from a] court or when public safety or order requires otherwise, as prescribed by law.”4 Any violation of this provision renders the evidence obtained inadmissible “for any purpose in any proceeding.”5

The intimacies between husband and wife do not justify any one of them in breaking the drawers and cabinets of the other and in ransacking them for any telltale evidence of marital infidelity. A person, by contracting marriage, does not shed his/her integrity or his right to privacy as an individual and the constitutional protection is ever available to him or to her.

The law insures absolute freedom of communication between the spouses by making it privileged. Neither husband nor wife may testify for or against the other without the consent of the affected spouse while the marriage subsists.6 Neither may be examined without the consent of the other as to any communication received in confidence by one from the other during the marriage, save for specified exceptions.7 But one thing is freedom of communication; quite another is a compulsion for each one to share what one knows with the other. And this has nothing to do with the duty of fidelity that each owes to the other.

WHEREFORE, the petition for review is DENIED for lack of merit.

SO ORDERED.

Regalado (Chairman), Romero, and Puno, JJ., concur.

[G.R. No. 113271.  October 16, 1997]

WATEROUS DRUG CORPORATION and MS. EMMA CO, petitioners, vs. NATIONAL LABOR RELATIONS COMMISSION and ANTONIA MELODIA CATOLICO, respondents.

D E C I S I O N

DAVIDE, JR. J.:

“Nor is he a true Servant [who] buys dear to share in the Profit with the Seller.” [1]

This petition for certiorari under Rule 65 of the Rules of Court seeks to declare private respondent Antonia Melodia Catolico (hereafter Catolico) not a “true Servant,” thereby assailing the 30 September 1993 decision[2] and 2 December 1993 Resolution[3] of the National Labor Relations Commission (NLRC) in NLRC-NCR CA No. 005160-93, which sustained the reinstatement and monetary awards in favor of private respondent[4] and denied the petitioners’ motion for reconsideration.[5]

The facts are as follows:

Catolico was hired as a pharmacist by petitioner Waterous Drug Corporation (hereafter WATEROUS) on 15 August 1988.

Page 364: Warrantless Arrests, Searches and Seizures and Privacy of Communication

364

On 31 July 1989, Catolico received a memorandum[6] from WATEROUS Vice President-General Manager Emma R. Co warning her not to dispense medicine to employees chargeable to the latter’s accounts because the same was a prohibited practice.  On the same date, Co issued another memorandum[7] to Catolico warning her not to negotiate with suppliers of medicine without consulting the Purchasing Department, as this would impair the company’s control of purchases and, besides she was not authorized to deal directly with the suppliers.

As regards the first memorandum, Catolico did not deny her responsibility but explained that her act was “due to negligence,” since fellow employee Irene Soliven “obtained the medicines in bad faith and through misrepresentation when she claimed that she was given a charge slip by the Admitting Dept.”  Catolico then asked the company to look into the fraudulent activities of Soliven.[8]

In a memorandum[9] dated 21 November 1989, WATEROUS Supervisor Luzviminda E. Bautro warned Catolico against the “rush delivery of medicines without the proper documents.”

On 29 January 1990, WATEROUS Control Clerk Eugenio Valdez informed Co that he noticed an irregularity involving Catolico and Yung Shin Pharmaceuticals, Inc. (hereafter YSP), which he described as follows:

… A case in point is medicine purchased under our Purchase Order (P.O.) No. 19045 with YSP Sales Invoice No. 266 representing purchase of ten (10) bottles of Voren tablets atP384.00 per unit. Previous P.O.s issued to YSP, Inc. showed that the price per bottle is P320.00 while P.O. No. 19045 is priced at P384.00 or an over price of P64.00 per bottle (or total of P640.00).  WDRC paid the amount of P3,840.00 thru MBTC Check No. 222832 dated December 15, 1988. Verification was made to YSP, Inc. to determine the discrepancy and it was found that the cost per bottle was indeed overpriced. YSP, Inc. Accounting Department (Ms. Estelita Reyes) confirmed that the difference represents refund of jack-up price of ten bottles of Voren tablets per sales invoice no. 266 as per their check voucher no. 629552 (shown to the undersigned), which was  paid to Ms. Catolico through China Bank check no. 892068 dated November 9, 1989....

The undersigned talked to Ms. Catolico regarding the check but she denied having received it and that she is unaware of the overprice. However, upon conversation with Ms. Saldana, EDRC Espana Pharmacy Clerk, she confirmed that the check amounting to P640.00 was actually received by Ms. Catolico.  As a matter of fact, Ms. Catolico even asked Ms. Saldana if she opened the envelope containing the check but Ms. Saldana answered her “talagang ganyan, bukas.”  It appears that the amount in question (P640.00) had been pocketed by Ms. Catolico.[10]

Forthwith,  in her memorandum[11] dated 31 January 1990, Co asked Catolico to explain, within twenty-four hours, her side of the reported irregularity.  Catolico asked for additional time to give her explanation,[12] and she was granted a 48-hour extension from 1 to 3 February 1990.  However, on 2 February 1990, she was informed that effective 6 February 1990 to 7 March 1990, she would be placed on preventive suspension to protect the interests of the company.[13]

In a letter dated 2 February 1990, Catolico requested access to the file containing Sales Invoice No. 266 for her to be able to make a satisfactory explanation.  In said letter she protested Saldaña’s invasion of her privacy when Saldaña opened an envelope addressed to Catolico.[14]

In a letter[15] to Co dated 10 February 1990, Catolico, through her counsel, explained that the check she received from YSP was a Christmas gift and not a “refund of overprice.”  She also averred that the preventive suspension was ill-motivated, as it sprang from an earlier incident between her and Co’s secretary, Irene Soliven.

On 5 March 1990, WATEROUS Supervisor Luzviminda Bautro, issued a memorandum [16] notifying Catolico of her termination; thus:

Page 365: Warrantless Arrests, Searches and Seizures and Privacy of Communication

365

We received your letter of explanation and your lawyer's letter dated Feb. 2, 1990 and Feb. 10, 1990 respectively regarding our imposition of preventive suspension on you for acts of dishonesty.  However, said letters failed to rebut the evidences [sic] in our possession which clearly shows that as a Pharmacist stationed at Espana Branch, you actually made Purchase Orders at YSP Phils., Inc. for 10 bottles of Voren tablets at P384.00/bottle with previous price of P320.00/bottle only. A check which you received in the amount of P640.00 actually represents the refund of over price of said medicines and this was confirmed by Ms. Estelita Reyes, YSP Phils., Inc. Accounting Department.

Your actuation constitutes an act of dishonesty detrimental to the interest of the company.  Accordingly, you are hereby terminated effective March 8, 1990.

On 5 May 1990, Catolico filed before the Office of the Labor Arbiter a complaint for unfair labor practice, illegal dismissal, and illegal suspension.[17]

In his decision[18] of 10 May 1993, Labor Arbiter Alex Arcadio Lopez found no proof of unfair labor practice against petitioners.  Nevertheless, he decided in favor of Catolico because petitioners failed to “prove what [they] alleged as complainant’s dishonesty,” and to show that any investigation was conducted.  Hence, the dismissal was without just cause and due process. He thus declared the dismissal and suspension illegal but disallowed reinstatement, as it would not be to the best interest of the parties.  Accordingly, he awarded separation pay to Catolico computed at one-half month’s pay for every year of service; back wages for one year; and the additional sum of P2,000.00 for illegal suspension “representing 30 days work.”  Arbiter Lopez computed the award in favor of Catolico as follows:

30 days Preventive Suspension                           P 2,000.00

Backwages                                                             26,858.50

1/12 of P26,858.50                                                   2,238.21

Separation pay (3 years)                                             4,305.15

                                      TOTAL AWARD:           P 35,401.86

Petitioners seasonably appealed from the decision and urged the NLRC to set it aside because the Labor Arbiter erred in finding that Catolico was denied due process and that there was no just cause to terminate her services.

In its decision[19] of 30 September 1993, the NLRC affirmed the findings of the Labor Arbiter on the ground that petitioners were not able to prove a just cause for Catolico’s dismissal from her employment.  It found that petitioner’s evidence consisted only of the check of P640.00 drawn by YSP in favor of complainant, which her co-employee saw when the latter opened the envelope.  But, it declared that the check was inadmissible in evidence pursuant to Sections 2 and 3(1 and 2) of Article III of the Constitution.[20] It concluded:

With the smoking gun evidence of respondents being rendered inadmissible, by virtue of the constitutional right invoked by complainants, respondents’ case falls apart as it is bereft of evidence which cannot be used as a legal basis for complainant’s dismissal.

The NLRC then dismissed the appeal for lack of merit, but modified the dispositive portion of the appealed decision by deleting the award for illegal suspension as the same was already included in the computation of the aggregate of the awards in the amount of P35,401.86.

Their motion for reconsideration having been denied, petitioners filed this special civil action for certiorari, which is anchored on the following grounds:

I.  Public respondent committed grave abuse of discretion in its findings of facts.

Page 366: Warrantless Arrests, Searches and Seizures and Privacy of Communication

366

II.  Due process was duly accorded to private respondent.

III.  Public respondent gravely erred in applying Section 3, Article III of the 1987 Constitution.

As to the first and second grounds, petitioners insist that Catolico had been receiving “commissions” from YSP, or probably from other suppliers, and that the check issued to her on 9 November 1989 was not the first or the last.  They also maintained that Catolico occupied a confidential position and that Catolico’s receipt of YSP’s check, aggravated by her “propensity to violate company rules,” constituted breach of confidence.  And contrary to the findings of NLRC, Catolico was given ample opportunity to explain her side of the controversy.

Anent the third ground, petitioners submit that, in light of the decision in the People v. Marti,[21] the constitutional protection against unreasonable searches and seizures refers to the immunity of one’s person from interference by government and cannot be extended to acts committed by private individuals so as to bring it within the ambit of alleged unlawful intrusion by the government.

In its Manifestation in Lieu of Comment, the Office of the Solicitor General (OSG) disagreed with the NLRC's decision, as it was of the persuasion that (a) the conclusions reached by public respondent are inconsistent with its findings of fact; and (b) the incident involving the opening of envelope addressed to private respondent does not warrant the application of the constitutional provisions.  It observed that Catolico was given “several opportunities” to explain her side of the check controversy, and concluded that the opportunities granted her and her subsequent explanation “satisfy the requirements of just cause and due process.”  The OSG was also convinced that Catolico’s dismissal was based on just cause and that Catolico’s admission of the existence of the check, as well as her “lame excuse” that it was a Christmas gift from YSP, constituted substantial evidence of dishonesty.  Finally, the OSG echoed petitioners’ argument that there was no violation of the right of privacy of communication in this case,[22] adding that petitioner WATEROUS was justified in opening an envelope from one of its regular suppliers as it could assume that the letter was a business communication in which it had an interest.

In its Comment which we required to be filed in view of the adverse stand of the OSG, the NLRC contends that petitioners miserably failed to prove their claim that it committed grave abuse of discretion in its findings of fact.  It then prays that we dismiss this petition.

In her Comment, Catolico asserts that petitioners’ evidence is too “flimsy” to justify her dismissal.  The check in issue was given to her, and she had no duty to turn it over to her employer.  Company rules do not prohibit an employee from accepting gifts from clients, and there is no indication in the contentious check that it was meant as a refund for overpriced medicines.   Besides, the check was discovered in violation of the constitutional provision on the right to privacy and communication; hence, as correctly held by the NLRC, it was inadmissible in evidence.

Catolico likewise disputes petitioners’ claim that the audit report and her initial response that she never received a check were sufficient to justify her dismissal. When she denied having received a check from YSP, she meant that she did not receive any refund of overprice, consistent with her position that what she received was a token gift.  All that can be gathered from the audit report is that there was apparently an overcharge, with no basis to conclude that Catolico pocketed the amount in collusion with YSP.  She thus concluded that her dismissal was based on a mere suspicion.

Finally,  Catolico insists that she could not have breached the trust and confidence of WATEROUS because, being merely a pharmacist, she did not handle “confidential information or sensitive properties.”  She was doing the task of a saleslady: selling drugs and making requisitions when supplies were low.

A thorough review of the record leads us to no other conclusion than that, except as to the third ground, the instant petition must fail.

Concededly, Catolico was denied due process.  Procedural due process requires that an employee be apprised of the charge against him, given reasonable time to answer the charge, allowed ample opportunity to be heard and defend himself, and assisted by a representative if the employee so desires.

Page 367: Warrantless Arrests, Searches and Seizures and Privacy of Communication

367

[23] Ample opportunity connotes every kind of assistance that management must accord the employee to enable him to prepare adequately for his defense, including legal representation.[24]

In the case at bar, although Catolico was given an opportunity to explain her side, she was dismissed from the service in the memorandum of 5 March 1990 issued by her Supervisor after receipt of her letter and that of her counsel.  No hearing was ever conducted after the issues were joined through said letters.  The Supervisor’s memorandum spoke of “evidences [sic] in [WATEROUS] possession,” which were not, however, submitted.  What the “evidences” [sic] other than the sales invoice and the check were, only the Supervisor knew.

Catolico was also unjustly dismissed.  It is settled that the burden is on the employer to prove just and valid cause for dismissing an employee, and its failure to discharge that burden would result in a finding that the dismissal is unjustified.[25] Here, WATEROUS proved unequal to the task.

It is evident from the Supervisor’s memorandum that Catolico was dismissed because of an alleged anomalous transaction with YSP. Unfortunately for petitioners, their evidence does not establish that there was an overcharge.  Control Clerk Eugenio C. Valdez, who claims to have discovered Catolico’s inappropriate transaction, stated in his affidavit:[26]

4.  My findings revealed that on or before the month of July 31, 1989, Ms. Catolico in violation of the [company] procedure, made an under the table deal with YSP Phils. to supply WDRC needed medicines like Voren tablets at a jack-up price of P384.00 per bottle of 50 mg. which has a previous price of only P320.00;

5.  I verified the matter to YSP Phils. to determine the discrepancy and I found out that the cost per bottle was indeed overpriced. The Accounting Department of YSP Phils. through Ms. Estelita Reyes confirmed that there was really an overprice and she said that the difference was refunded through their check voucher no. 629552 which was shown to me and the payee is Melodia Catolico, through a China Bank Check No. 892068 dated November 9, 1989.

It clearly appears then that Catolico’s dismissal was based on hearsay  information.  Estelita Reyes never testified nor executed an affidavit relative to this case; thus, we have to reject the statements attributed to her by Valdez.  Hearsay evidence carries no probative value.[27]

Besides, it was never shown that petitioners paid for the Voren tablets.  While Valdez informed Co, through the former’s memorandum[28] of 29 January 1990, that WATEROUS paid YSP P3,840.00 “thru MBTC Check No. 222832,” the said check was never presented in evidence, nor was any receipt from YSP offered by petitioners.

Moreover, the two purchase orders for Voren tablets presented by petitioners do not indicate an overcharge.  The purchase order dated 16 August 1989[29] stated that the Voren tablets cost P320.00 per box, while the purchase order dated 5 October 1989[30] priced the Voren tablets at P384.00 per bottle.  The difference in price may then be attributed to the different packaging used in each purchase order.

Assuming that there was an overcharge, the two purchase orders for the Voren tablets were recommended by Director-MMG Mario R. Panuncio, verified by AVP-MNG Noli M. Lopez and approved by Vice President-General Manager Emma R. Co.  The purchase orders were silent as to Catolico’s participation in the purchase. If the price increase was objectionable to petitioners, they or their officers should have disapproved the transaction.  Consequently, petitioners had no one to blame for their predicament but themselves. This set of facts emphasizes the exceedingly incredible situation proposed by petitioners.  Despite the memorandum warning Catolico not to negotiate with suppliers of medicine, there was no proof that she ever transacted, or that she had the opportunity to transact, with the said suppliers.  Again, as the purchase orders indicate, Catolico was not at all involved in the sale of the Voren tablets.  There was no occasion for Catolico to initiate, much less benefit from, what Valdez called  an “under the table deal” with YSP.

Catolico’s dismissal then was obviously grounded on mere suspicion, which in no case can justify an employee’s dismissal.  Suspicion is not among the valid causes provided by the Labor Code for the termination of employment;[31] and even the dismissal of an employee for loss of trust and confidence

Page 368: Warrantless Arrests, Searches and Seizures and Privacy of Communication

368

must rest on substantial grounds and not on the employer’s arbitrariness, whims, caprices, or suspicion.[32] Besides, Catolico was not shown to be a managerial employee, to which class of employees the term “trust and confidence” is restricted.[33]

As regards the constitutional violation upon which the NLRC anchored its decision, we find no reason to revise the doctrine laid down in People vs. Marti[34] that the Bill of Rights does not protect citizens from unreasonable searches and seizures perpetrated by private individuals.  It is not true, as counsel for Catolico claims, that the citizens have no recourse against such assaults.  On the contrary, and as said counsel admits, such an invasion gives rise to both criminal and civil liabilities.

Finally, since it has been determined by the Labor Arbiter that Catolico’s reinstatement would not be to the best interest of the parties, he correctly awarded separation pay to Catolico.  Separation pay in lieu of reinstatement is computed at one month’s salary for every year of service. [35] In this case, however, Labor Arbiter Lopez computed the separation pay at one-half month’s salary for every year of service. Catolico did not oppose or raise an objection.  As such, we will uphold the award of separation pay as fixed by the Labor Arbiter.

WHEREFORE, the instant petition is hereby DISMISSED and the challenged decision and resolution of the National Labor Relations Commission dated 30 September 1993 and 2 December 1993, respectively, in NLRC-NCR CA No. 005160-93 are AFFIRMED, except as to its reason for upholding the Labor Arbiter’s decision, viz., that the evidence against private respondent was inadmissible for having been obtained in violation of her constitutional rights of privacy of communication and against unreasonable searches and seizures which is hereby set aside.

Costs against petitioners.

SO ORDERED.

Bellosillo, Vitug, Kapunan, and Hermosisima, Jr., JJ., concur.

G.R. No. L-1716             June 28, 1949

MATERIAL DISTRIBUTORS (PHIL.), INC., and HARRY LYONS, petitioner, vs.FELIPE NATIVIDAD, Judge of First Instance of Manila, and LOPE SARREAL, respondents.

Gibbs, Gibbs, Chuidian and Quasha for petitioner.Claro M. Recto for respondent Lope Sarreal.No appearance for respondent Judge.

PERFECTO, J.:

On March 24, 1947, Lope Sarreal filed a complaint (amended on April 10, 1947, to include Harry Lyons) seeking a money judgment against petitioners on three causes of action in the total of P1,256,229.30.

On May 27, 1947, Sarreal filed a motion for the production and inspection of the following documents:

I. Books or Papers of Material Distributors (Phil.) Inc.:

1. Cash Receipts Journal2. Cash Payments Journal3. All Individual Ledgers, specially of the following persons or entities

Page 369: Warrantless Arrests, Searches and Seizures and Privacy of Communication

369

(a) British-American Engineering Corporation(b) Philippine Refinery(c) Felipe Buencamino(d) Luzon Stevedoring(e) Standard Oil Company of New York(f) Philippine Exchange Co., Inc.(g) Manila Laundry Company(h) Filipino Businessmen's Syndicate(i) Material Distributors Inc., Wichita, Kansas(j) Harry Lyons

4. All letters exchanged between Material Distributors (Phil.) Inc., Material Distributors, Inc. of Wichita, Kansas and Harry Lyons, between October 9, 1946 and March 31, 1947.

5. All cablegrams exchanged between Material Distributors (Phil.), Inc., and Material Distributors, Inc., Wichita, Kansas, between October 9, 1946 to March 31, 1947.

II. Books and Papers of the defendant Harry Lyons.

1. Letters exchanged between Harry Lyons and Material Distributors, Inc., Wichita, Kansas between September 14, 1946 and March 24, 1947.2. Cablegrams exchanged between Harry Lyons and Material Distributors, Inc., Wichita, Kansas, between September 14, 1946 and March 24, 1947.3. Cash Receipts Journal.4. Cash Payments Journal.

On June 4, 1947, Sarreal filed a supplemental motion for the production and inspection of the originals of Annexes A and B of the complaint.

On June 12, 1947, petitioner filed a memorandum and opposition to Sarreal's above mentioned original and supplemental motion on the ground that he failed to show good cause and that the motion were evidently filed for the purpose of fishing evidence.

On July 16, 1947, respondent judge, granting both motions, required petitioners to produce the documents and annexes in question on July 24, 1947.

On account of the absence in the Philippines of Harry Lyons, petitioner moved, reserving whatever rights they have under the Rules of Court, to postpone the inspection of the documents and annexes in question and accordingly respondent judge postponed it to August 15, 1947.

On August 13, 1947, petitioners moved for the reconsideration of the order of July 16, on the following grounds:

"(a) Article 46 of the Code of Commerce which prohibits the delivery, communication and general examination of the correspondence of merchants, a substantial right, as well as the petitioners' right to the inviolability of their correspondence as guaranteed by the Constitution would be violated by the order requiring the production of the following documents:

BOOKS AND PAPERS OF DEFENDANT HARRY LYONS

(1) Letters exchange between Harry Lyons and Material Distributors, Inc., of Wichita, Kansas, between Sept. 14, 1946 and March 24,1947;

Page 370: Warrantless Arrests, Searches and Seizures and Privacy of Communication

370

(2) Cablegrams exchanged between Harry Lyons and Material Distributors, Inc., of Wichita, Kansas, between September 14, 1946 and March 24, 1947.

BOOKS AND PAPERS OF MATERIAL DISTRIBUTORS (PHIL.) INC.

(4-5) All letters and cablegrams exchanged between Material Distributors (Phil.), Inc., Material Distributors, Inc., of Wichita, Kansas, and Harry Lyons between October 9, 1946 and March 31, 1947.

"(b) That the production for the plaintiff's inspection of all the foregoing documents above enumerated, as well as of the following documents, would constitute a "fishing expedition," not allowed by Rule 21 of the Rules of Court, since their materiality or probable materiality is not shown by the pleadings of the parties except by movant's bare allegation which are disputed by your petitioners:

BOOKS AND PAPERS OF MATERIAL DISTRIBUTORS (PHIL.) INC.

1-2. Cash Receipts Journal and Cash Payments Journal.

3. All individual Ledgers, specially of the following persons or entities.

(b) Philippine Refinery.(c) Felipe Buencamino.(d) Luzon Stevedoring.(e) Standard Oil Company of New York.(f) Philippine Exchange Co., Inc.(g) Manila Laundry Company.

"(c) That plaintiff is not entitled to the production and inspection of the originals of Annexes A and B because his only purpose, as stated in his supplemental motion, Exhibit D, was to find out if a case of falsification has been made; that the issue between the parties in this regard is material only to your petitioners' affirmative defense, and if the plaintiff's purpose was as stated in said supplemental motion, then your petitioners claimed their privilege against self-incrimination. That this letter privilege was also claimed insofar as the production and inspection of the other documents were concerned by your petitioners in view of counsel for respondent Lope Sarreal's charge to the Honorable City Fiscal for the City of Manila that your petitioners were violating our Corporation Law."

On September 27, 1947, respondent judge denied the motion for reconsideration.

Petitioners impugn the validity of the orders of July 16 and September 27, 1947, as were issued by the respondent judge in excess of his jurisdiction or with grave abuse of his discretion, and prayed for the annulment or modification of the order of July 16, 1947.

Respondent Sarreal advanced the following reasons to show that the orders complained of were not issued in excess of the trial court's jurisdiction or with grave abuse of discretion:

(a) The motions of the respondent Lope Sarreal of May 27, 1947 and June 4, 1947 contain allegations of the ultimate fact that the books and papers mentioned in said motions constitute or contain evidence material to the matters involved in the case and are in the possession, custody or control of the petitioners herein, and allegation to this effect is adequate showing of good cause for the production and inspection of the documents mentioned therein, being an allegation in the very words used in Form 11 of the Appendix Forms of our Rules of Court, and therefore a

Page 371: Warrantless Arrests, Searches and Seizures and Privacy of Communication

371

sufficient compliance with said Rule (Go Tianco vs. Judge Diaz, G. R. L-7, January 22, 1946, reported in the June 1946 issue of the official Gazette).

(b) Article 46 of the Code of Commerce invoked by the petitioners does not apply to cases of production and inspection of books and papers belonging to a party to the action in which such production and inspection are sought(Decision of Supreme Court of Spain of March 30, 1894). At any rate, said Article of the Code of Commerce has been impliedly repealed by Act No. 190, pertinent portions of which are now embodied in our Rules of Court (3 Op. of Atty. Gen., 380).

(c) Neither would the inspection of books and papers of the petitioners amount to a violation of the inviolability of the correspondence under Sec. 1, No. 5, Article III of the Constitution of the Philippines, considering that the inspection of said books and papers are sought through proper order of the trial court, and the Constitutional provision invoked by the petitioners precisely allows inspection of communication and correspondence upon lawful order of the court. Moreover, this provision of our Constitution creates no new right, being merely a re-enforcement of the Constitutional prohibition against unreasonable searches and seizures (Sinco, Philippine Government and Political Law, 4th Edition, p. 632), and when the inspection of such books and papers was allowed "upon lawful order of the court" made through the respondent Judge, such inspection cannot be considered as unreasonable although such books and papers are private in character (First National Bank vs. Hughes, 6 Fed., C 737, 741, appeal dismissed for want of jurisdiction in 106 U.S., 523, 27 Law ed., 268, 1 Sup. Ct. Rep. 489; Johnson Steel Street-Rail Co.vs. North Branch Steel Co., 48 Fed., 191; Victor G. Beede Co. vs. Joseph Bancroft and Sons Co., 98 Fed., 175, affirmed in 52 L. R. A., 734, 45 C. C. A., 354, 106 Fed., 396, where this question was not involved; Burnham vs. Morrissey, 14 Gray, 226, 74 A. Dec., 676; United States vs. Terminal R. Assoc., 148 Fed., 486; Re Dunn, 9 Mo. App., 225; Elder and Bogardus, 1 Edm. Sel Cas., 110; Boston and M. R. Co. vs. States [N. H.], 77 Atl., 996; Hopkinson vs. Burghley, L. R. 2ch., 447; Groker-Wheeler Co. vs. Bullock [C. C.], 134 Fed., 241; Re Bolster, 110 Pac., 547.).

(d) The inspection of the said documents is not for the purpose of "fishing evidence" but with a view to enabling the respondent Lope Sarreal to designate with the particularity of the subpoena duces tecum to be obtained in connection with trial of the case on its merits the specific books and papers containing the entry of receipts and payments made by the petitioners, such books and papers being material to the case in view, among others, of the allegation in the amended complaint that the defendants, the petitioners herein, had been remitting all or the greater volume of the proceeds from the sales of equipment and materials of the defendants in Civil Case No. 2059 outside the jurisdiction of the trial court and had been disposing of their properties with the intention of defrauding their creditors. At any rate, "fishing expedition" is allowed and is precisely contemplated in Rule 21 of our Rules of Court as a weapon of discovery (XXVI Am. Bar. Ass. Jur. No. 1, Jan. 1940, 48; Golden vs. Arcadia Mutual Casualty Company, D. C. III., 1942, 3 F. R. D., 26; Leach vs. Griff Bros. Coop. Corp., D. C. Miss, 1942 2 F. R. D., 444; Civil Aeronautics Board of Aeronautics Authority vs. Canadian Colonial Airways, D. C., 1941, 41 F. S., 1006; Quemus Theatre Co. vs. Warner Bros. Pictures, D. C. N. J., 1940, 35 F. S., 949; United Mercantile Agency vs. Silver Fleet Motor Express, D. C. Ky., 1941, F. R. D., 709; Walling vs. Richmon Screw Anchor Company, D. C. N. Y., 1943,4 F. R. D., 265; Monarch Liquor Corp. vs. Schenley Distillers Corp., D. C. N.Y., 1941, 2 F. R. D., 51; Walshvs. Comm. Mutual Life Insurance Company of Hartford, Conn. [1939], 26 F. Supp., 556; Olson Transportation Company vs. Socony Vacuum Oil Company, 7 F. R. D., 234).

(e) The originals of Annexes A and B are relevant not only to the case of the defendants but also to that of the plaintiff in Civil Case No. 2059 here involved, in view of the issue of fact raised by the pleadings of the parties as to whether the originals of Annexes A and B have been falsified by the insertion therein of the names of Gil J. Puyat and Raymond Lehmann after said Annexes were signed by respondent Lope Sarreal and delivered to the petitioner Harry Lysons, in view of

Page 372: Warrantless Arrests, Searches and Seizures and Privacy of Communication

372

which respondent Lope Sarreal is entitled to the production and inspection thereof under the provisions of Rule 21 of our Rules of Court.

(f) Even if ocular inspection of said Annexes A and B may reveal falsification thereof by the petitioners amounting to a violation of the applicable provisions of our Revised Penal Code, the petitioners cannot exempt themselves from the production of said exhibits for mere inspection and copying, inasmuch as the Constitutional prohibition against self-incrimination has been extended in specific case only to the production of documents as evidence, and only when the person producing them is made to take the witness stand and identify them under oath, and not to the production of such documents for mere inspection (Comm. vs. Southern Express Co., 1914, 160 Ky., 1, 169 SW., 517, annotated cases 1916A, 373, L. R. A., 1915B, 913; U. S. vs. Hughes, 12 Blatchff, 553); the reason being that the Constitutional prohibition is one against compelling a person to be a "witness against himself", and this has been held to mean testimonial compulsion or extraction of admission form the person's own lips (4 Wigmore, 865, 2263; Wilson vs. U. S., 55 Law, ed., 776, citing cases).

(g) Moreover, the corporate records sought to be inspected are not covered by the Constitutional prohibition against self-incrimination, even though such documents may contain evidence tending to subject any or all of the officers of a corporation to a criminal indictment (Wilson vs. U. S., 221 U. S., 361, 51 Law. ed., 771; Oklahoma Press Pub. Co. vs. Walling, 327 U. S.,186, 90 Law. ed., 614, 627-629; U. S.vs. Baunch & Lamp Optical Co., 321U. S., 707, 88 Law. ed., 1024, 1037 [1944]; U. S. vs. White, 88 Law. ed., 1547).

(h) Production and inspection of documents have been allowed and sustained in decided cases, under Orders which were broader than those here assailed, for the production and inspection of all books of accounts, all memoranda and records, stocks book, ledger, journal, cash book, bank book, bank deposit slip, check book, voucher, contract, income tax return, booking record and correspondence (U. S. vs. Duoder, 1 F. R. S., 466, U. S. D. C. June 16, 1939; Orange Country Theater Corp. vs. League, 1 F. R. S., 448).

(i) The respondent Judge, before issuing the Orders complained of, gave the parties full opportunity, not only to discuss the question involved by repeated oral arguments but also by written memoranda, and in the exercise of his discretion issued the Orders complained of only after full consideration of all the questions of fact and law involved.

The production and inspection of documents and books here in question call for the interpretation and application of section 1 of Rule 21, which reads as follows:

SECTION 1. Motion for production or inspection; order. — Upon motion of any party showing good cause therefor and upon notice to all other parties, the court in which an action is pending may (a) order any party to produce and permit the inspection and copying or photographing, by or on behalf of the moving party, of any designated documents, papers, books, accounts, letters, photographs, objects or tangible things, not privileged, which constitute or contain evidence material to any matter involved in the action and which are in his possession, custody or control; or (b) order any party to permit entry upon designated land or other property in his possession or control for the purpose of inspecting, measuring, surveying, or photographing the property or any designated relevant object or operation thereon. The order shall specify the time, place and manner of making the inspection and taking copies and photographs, and may prescribe such terms and conditions as are just.

Petitioners contend that in filling his original and supplemental motions, Sarreal has failed to show good cause for the issuance of the requested order. It appears, however, in the original motion of May 27, 1947, that the books and papers therein mentioned "constitute or contain the evidence material to the matters involved in the above entitled case."

Page 373: Warrantless Arrests, Searches and Seizures and Privacy of Communication

373

In the supplemental motion of June 4, 1947, it is alleged that there is direct conflict between the allegations of the complaint and amended complaint and those of the answer and amended answer as to whether or not the names of Gil J. Puyat and Raymond W. Lehmann appear in any part of the originals of Annexes A and B of the complaint, and plaintiff Sarreal wanted the production and inspection of said originals to show that they did not contain the names of Gil J. Puyat and Raymond W. Lehmann, and that if said names should appear now typed in said Annexes A and B, said additional names must have been typed by direction of Harry Lysons without the knowledge or consent of Sarreal and after said originals were delivered by Harry Lysons and filed by the latter and that the changes so introduced are a forgery.

With these allegations in the original and supplemental motions Sarreal has fulfilled the requirements of showing good cause for the production and inspection of the books and documents in question under Rule 21.

Petitioners contained that the order of the trial judge violated petitioner's constitutional rights against self-incrimination.

We have considered carefully persons advanced by petitioners and memoranda in support of this allegation and we found nothing in them to show how, without the inspection of Annexes A and B of the complaint, petitioners may incriminate themselves. We have, therefore, to dismiss such contention.

(3) The right of the people to be secure in their persons, houses, papers, and effects against unreasonable searches and seizure shall not be violated, and no warrant shall issue but upon probable cause, to be determined by the judge after examination under oath or affirmation of the complainant and the witnesses he may produce, and particularly describing the place to be searched, and the persons or things to be seized." (Sec. 1. Art. III, Constitution of the Philippines.)

(5) The privacy of communication and correspondence shall be inviolable except upon lawful order of the court or when public safety and order require otherwise. (Sec. 1. Art. III, Constitution of the Philippines.)

The orders in question, issued in virtue of the provisions of Rule 21, pertain to a civil procedure that cannot be identified or confused with the unreasonable searches prohibited by the Constitution. But in the erroneous hypothesis that the production and inspection of books and documents in question is tantamount to a search warrant, the procedure outlined by Rule 21 and followed by respondent judge place them outside the realm of the prohibited unreasonable searches. There is no question that, upon the pleadings in the case, Sarreal has an interest in the books and documents in question, that they are material and important to the issues between him and petitioners, that justice will be better served if all the facts pertinent to the controversy are placed before the trial court.

The constitutional guarantee of privacy of communication and correspondence will not be violated, because the trial court has power and jurisdiction to issue the order for the production and inspection of the books and documents in question in virtue of the constitutional guarantee making an express exception in favor of the disclosure of communication and correspondence upon lawful order of a court of justice.

After a careful consideration of the legal question raised by petitioners, this Court has arrived at the conclusion that the trial judge, in issuing the order of July 16, 1947, has not exceed his jurisdiction or acted with grave abuse of discretion.

Petition denied with costs against petitioner.

Moran, C.J., Paras, Feria, Tuason, Montemayor, and Reyes, JJ., concur.MORAN, C.J.: Mr. Justice Pablo voted for this decision.

Page 374: Warrantless Arrests, Searches and Seizures and Privacy of Communication

374